2013 Frenchmans Hill Estate The Pooks Pinot Gris

American Society of Critical Care
Anesthesiologists
Residents’ Guide
To Learning in the
Intensive Care Unit
The information presented in these guidelines was obtained by the Committee on Resident Education.
The validity of the opinions presented, drug dosages, accuracy and completeness of contents
are not guaranteed by ASCCA.
Copyright В© 2009 by the American Society of Critical Care Anesthesiologists, Park Ridge, Illinois.
All rights reserved. Contents may not be reproduced without prior written permission of the publisher.
Third Edition.
American Society of Critical Care Anesthesiologists
520 N. Northwest Highway
Park Ridge, IL 60068-2573
$25.00
ASCCA ResidentКјs Guide to the ICU
Contents
Preface to Third Edition
vi
Preface to Second Edition
vii
Preface to First Edition
viii
General ICU Topics
1
2
3
4
5
6
7
8
9
10
11
12
The ICU Experience
Ruben J. Azocar, M.D.
1
ICU Management
Ruben J. Azocar, M.D.
3
Family Support and Ethics Issues
W. Christopher Croley, M.D., F.C.C.P., Karen J. Schwenzer, M.D., David
M. Rothenberg, M.D., F.C.C.M.
5
Cardiopulmonary Resuscitation (CPR)
Todd W. Sarge, M.D.
10
Transport of the Critically Ill
Daniel Castillo, M.D., Claudia Chidiac, M.D., Miguel A Cobas, M.D.
14
Sedation of the Critically Ill
Ruben J. Azocar, M.D.
19
Analgesia for the Critically Ill
Ruben J. Azocar, M.D.
23
Neuromuscular Blockade
Ruben J. Azocar, M.D.
29
Acid-Base Balance
Nathanael Slater, D.O., Khaldoun Faris, M.D.
33
Metabolic and Endocrine Abnormalities
Ruben J. Azocar, M.D.
41
Fluid Management
Ronald W. Pauldine, M.D.
46
Gastrointestinal Bleeding
Fahim Habib, M.D., Miguel A. Cobas, M.D., Juan Asensio, M.D.
51
ii
ASCCA ResidentКјs Guide to the ICU
13
14
15
Thromboembolic Disease
Gustavo Angaramo, M.D.
56
Bleeding in the ICU
Ruben J. Azocar, M.D.
60
Specialized Nutrition Support in Critically Ill Patients
Gustavo Angaramo, M.D.
65
Monitoring Topics
16
17
18
Routine Monitoring in the ICU
Daniel R. Brown, M.D., Ph.D.
70
Rational Use of the Pulmonary Artery Catheter (PAC)
Fanni Nhuch, M.D., Ricardo Martinez-Ruiz, M.D., Miguel A. Cobas, M.D.
73
Echocardiography in the ICU
Frank Rosemeier, M.D., Miguel A. Cobas, M.D.
78
Respiratory System Topics
19
20
21
22
23
24
Airway Management in the ICU
Todd W. Sarge, M.D.
84
Chronic Airway Management
Ruben J. Azocar, M.D.
89
Management of Mechanical Ventilation
Edward A. Bittner, M.D., Ph.D.
93
Side-Effects of Mechanical Ventilation
Jean Kwo, M.D., Luca Bigatello, M.D.
102
Lung Protection Strategies
Paul G. Jodka, M.D.
106
Weaning from Mechanical Ventilation
Stephen O. Heard, M.D.
109
Cardiovascular System Topics
25
Support of the Failing Circulation: Shock
Michael Wall, M.D., F.C.C.M
114
iii
ASCCA ResidentКјs Guide to the ICU
26
27
28
Diagnosis and Treatment of Dysrhythmias
Michael Wall, M.D., F.C.C.M.
119
Diagnosis and Treatment of Myocardial Ischemia
Ruben J. Azocar, M.D.
123
Valvular Heart Disease
Frank Rosemeier, M.D., Miguel A. Cobas, M.D.
129
Sepsis and Infections Diseases Topics
29
30
31
32
33
The Systemic Inflammatory Response Syndrome (SIRS), Sepsis and
Multiple Organ Dysfunction Syndrome (MODS)
Sean M. Quinn, M.D., Miguel A. Cobas, M.D.
135
Nosocomial Infections
Todd W. Sarge, M.D.
143
Infections and Antibiotic Therapy
Ronald W. Pauldine, M.D.
147
Antibiotic Prophylaxis
Ronald W. Pauldine, M.D.
156
Management of the Immunocompromised Patient
Zdravka D. Zafirova, M.D.
160
Neurologic Topics
34
35
36
Neurologic Critical Care
Khaldoun Faris, M.D., Faraz A. Syed, D.O.
167
Traumatic Brain Injury
Ruben J. Azocar, M.D.
175
Management of Increased Intracranial Pressure
Jonathan R. Jagid, M.D., Matthew M. Ruel, M.D., Leo Harris, P.A.,
M.P.H., Miguel A. Cobas, M.D.
183
Renal Topics
37
38
Renal Protection
Daniel R. Brown, M.D., Ph.D.
192
Renal Replacement Therapy
Ruben J. Azocar, M.D.
196
iv
ASCCA ResidentКјs Guide to the ICU
Miscellaneous Topics
39
40
41
42
43
44
Toxicology and Support of Patients with Drug Overdoses
Matthew D. Koff, M.D., M.S.
200
Solid Organ Transplantation
Zdravka D. Zafirova, M.D., Michael C. Woo, M.D.
205
Organ Donation and Procurement in the ICU
Jocelyn A. Park, M.D.
214
Trauma Management
Edgar J. Pierre, M.D., David A. Riesco, M.D., Miguel A. Cobas, M.D.
217
Burn Management
Edgar J. Pierre, M.D., Joseph Borau, M.D., Miguel A. Cobas, M.D.
225
Obstetric Critical Care
Gustavo Angaramo, M.D.
235
Answers
237
v
ASCCA ResidentКјs Guide to the ICU
Preface to the Third Edition
Intensivists have witnessed profound changes in the delivery of critical care medicine in
the last decade. A large number of randomized, prospective clinical trials or “before and
after” studies have impacted the way we practice critical care. Examples include the use
of drotrecogin alfa activated for the treatment of severe sepsis and septic shock; low
tidal volume ventilation in patients with acute lung injury and the acute respiratory
distress syndrome; implementation of ventilator and catheter “bundles” to reduce
nosocomial infections; and the use of alpha-2 agonists for sedation of the critically
patient. It was clear that the “Anesthesiology Residents’ Guide to Learning in the
Intensive Care Unit” was in need of revision. We and the American Society of Critical
Care Anesthesiologists (ASCCA) are pleased to provide this revised guide to
supplement the critical care reading material used by anesthesia residents and fellow.
The third edition has been expanded to include several important topics including
“Echocardiography”, “Traumatic Brain Injury” and “Organ Donation and Procurement in
the ICU”. Similar to the second edition, we have retained the short case presentations
and the self-study questions. The reading lists have been updated.
The majority of the authors who participated in the revision of this edition are new. We
would like to thank the previous authors who were either the original authors or who
helped revise the second edition. We have acknowledged their contributions at the end
of each chapter. The ASCCA is dedicated to timely revisions of this guide and plans are
already underway on the 4th edition.
Daniel Talmor, M.D.
Stephen O. Heard, M.D.
vi
ASCCA ResidentКјs Guide to the ICU
Preface to the Second Edition
Since the publication date of the first edition in 1995, the scope and practice of critical
care medicine have continued to change. Examples of new issues encountered during
this time include controversies over pulmonary artery catheter use and pressures over
efficient management of critical care services as managed care demands are realized.
Additionally, during this time period, academic departments have grappled with
conflicting demands of resident education and efficient patient care in the new medical
marketplace. A primary mission of the American Society of Critical Care
Anesthesiologists is to assist anesthesiology residency programs to educate the future
perioperative physicians, who are today’s anesthesia residents and fellows. The
hospitalized patients of today are older and sicker than ever before, and many will travel
to locations in hospital where the primary medical caregiver will be an anesthesiologist.
With proper skills and training, these anesthesiologists will be the most appropriate
caregivers for these critically ill patients, helping maintain complex homeostasis by
thorough evaluation and management preceding, during, and following surgical
procedures.
The curriculum of the second edition was modified somewhat from the first edition by
members of the Resident Education Task Force of the American Society of Critical Care
Anesthesiologists in an attempt to bring “up-to-date” information to the hands of
residents who are caring for critically ill and injured patients. To this purpose, several
new sections have been added to the topical outline (e.g., ICU Management, Rational
Use of PA Catheters, Lung Protective Strategies); the bulk of the topic outline remains
as a presentation of the most important concepts of critical care. Short case
presentations have been added to provoke interest and increase relevance. Self-study
questions remain (with annotated answer keys) to allow the resident to evaluate his or
her understanding. Reading lists have also been annotated to emphasize the relevance
of most references.
Lucy A. Weston, Ph.D., M.D.
Acknowledgements:
Without the help and support of the members of the Resident Education Task Force,
this endeavor would not be possible. Many special thanks to Doug Coursin, Tom
Fuhrman, Gary Hoormann, and especially, Charlie Durbin, who have provided official
(and unofficial) technical, editorial, and emotional assistance and trusted this editor with
such a monumental task. Thanks to all contributors, particularly those working on new
sections and/or with tight timelines.
Many thanks to Michelle Smith for her secretarial efforts. She has been my third and
fourth hands.
vii
ASCCA ResidentКјs Guide to the ICU
Preface to the First Edition
The field of critical care medicine continues to expand as new technologies are
developed and old technologies are refined. Anesthesiologists have been pioneers in
the development of critical care medicine. Changes in the health care environment have
led the American Society of Critical Care Anesthesiologists to define the
anesthesiologists as the perioperative physician. To achieve this role, anesthesia
residents must be competent in critical care medicine. Assessing critically ill patients
prior to operative procedures, management of intraoperative anesthetic care, treating
postoperative pain, and management of organ function after a surgical procedure is
best done by an anesthesiologist.
The members of the Resident Education Task Force of the American Society of Critical
Care Anesthesiologists have developed this curriculum guide to help residents achieve
competence in caring for the critically ill and injured patients. This guide is not meant to
be an exhaustive curriculum for an intensivist but useful for all residents completing
anesthesiology training. These were developed into outlines of the most important
concepts with a reading list for each. Additionally, self-study questions are included with
most sections to allow the resident to evaluate his or her current understanding.
This guide may also be used by residency program directors to evaluate or improve
their own curriculums. It is not our intention that all aspects of critical care mentioned in
the guide be part of the two month critical care rotation required by the American Board
of Anesthesiology. Other experiences, including cardiac anesthesia, consult services,
and postoperative recovery room would certainly contribute to mastery of these topics.
Charles G. Durbin, M.D., FCCM
viii
ASCCA ResidentКјs Guide to the ICU
Contributors
Gustavo Angaramo, M.D., Assistant Professor, Department of Anesthesiology/Critical
Care Medicine, University of Massachusetts Medical School, Boston, Massachusetts
Juan Asensio, M.D., Professor of Surgery and Critical Care, Miller School of Medicine,
University of Miami, Miami, Florida
Ruben J. Azocar, M.D., Assistant Professor, Residency Program Director, Director of
Clinical Simulator Center, Department of Anesthesiology, Boston University School of
Medicine, Boston, Massachusetts
Luca M. Bigatello, M.D., Department of Anesthesia and Critical Care, Massachusetts
General Hospital, Boston, Massachusetts
Edward A. Bittner, M.D., Ph.D., Fellowship Director, Critical Care Anesthesiology,
Massachusetts General Hospital, Boston Massachusetts
Joseph Borau, M.D., Resident Physician, Department of Clinical Anesthesiology, Miller
School of Medicine, University of Miami, Miami, Florida
Daniel R. Brown, M.D., Ph.D., F.C.C.M., Chair, Division of Critical Care Medicine
Department of Anesthesiology, Mayo Clinic, Rochester, Minnesota
Daniel Castillo, M.D., Assistant Professor of Clinical Anesthesiology, Miller School of
Medicine, University of Miami, Miami Florida
Claudia Chidiac, M.D., Clinical Fellow in Anesthesia, Jackson Memorial Hospital,
Miami Florida
Miguel A. Cobas, M.D., Assistant Professor of Clinical Anesthesiology, Miller School of
Medicine, University of Miami, Miami Florida
W. Christopher Croley, M.D., F.C.C.P., Assistant Professor of Anesthesiology and
Critical Care Medicine, Director, Surgical Intensive Care Unit, Co-Medical Director, Rush
University Simulation Lab, Associate Director of Resident Education,
Rush University Medical Center, Chicago, Illinois
Khaldoun Faris, M.D., Assistant Professor of Anesthesiology, Associate Director,
Surgical Critical Care Program Director: Anesthesiology Critical Care Medicine
Fellowship, Department of Anesthesiology, University of Massachusetts Medical School,
Worcester, Massachusetts
ix
ASCCA ResidentКјs Guide to the ICU
Fahim Habib, M.D., Assistant Professor of Surgery and Critical Care, Miller School of
Medicine, University of Miami, Miami, Florida
Leo Harris, P.A, M.P.H., Department of NeuroSurgery, Miller School of Medicine,
University of Miami, Miami, Florida
Stephen O. Heard, M.D., Professor and Chair, Department of Anesthesiology,
University of Massachusetts Medical School, Worcester, Massachusetts
Jonathan R. Jagid, M.D., Assistant Professor of Neurological Surgery, Miller School of
Medicine, University of Miami, Miami, Florida
Paul G. Jodka, M.D., Program Director, Critical Care Anesthesia Fellowship, Baystate
Medical Center, Springfield, Massachusetts
Matthew D. Koff, M.D. M.S., Assistant Professor of Anesthesiology, Department of
Anesthesiology and Critical Care, Dartmouth Medical School and Dartmouth-Hitchcock
Medical Center, Lebanon, New Hampshire
Jean Kwo, M.D., Instructor in Anesthesia, Harvard Medical School, Boston,
Massachusetts
Ricardo Martinez-Ruiz, M.D., Assistant Professor of Clinical Anesthesiology, Miller
School of Medicine, University of Miami, Miami, Florida
Fanni Nhuch, M.D., Clinical Fellow of Critical Care, Department of Anesthesiology,
Miller School of Medicine, University of Miami, Miami, Florida
Jocelyn A. Park, M.D., Critical Care Fellowship, Department of Anesthesiology,
Dartmouth-Hitchcock Medical Center, Lebanon, New Hampshire
Ronald W. Pauldine, M.D., Clinical Associate Professor, University of Washington
School of Medicine, Seattle, Washington
Edgar J. Pierre, M.D., Assistant Professor of Clinical Anesthesiology, Miller School of
Medicine. University of Miami, Miami, Florida
Sean M. Quinn, M.D., Clinical Fellow in Critical Care, Dept of Anesthesiology, Miller
School of Medicine, University of Miami, Miami, Florida
David A. Riesco, M.D., Anesthesiology Resident, Department of Anesthesiology,
University of Miami School of Medicine/Jackson Memorial Hospital, Miami, Florida
Frank Rosemeier, M.D., Assistant Professor of Anesthesiology, Miller School of
Medicine, University of Miami, Miami, Florida
x
ASCCA ResidentКјs Guide to the ICU
David M. Rothenberg, M.D., F.C.C.M., Professor of Anesthesiology, Associate Dean,
Associate Dean, Academic Affiliations, Rush University Medical Center, Chicago, Illinois
Matthew M. Ruel, M.D., Resident in Anesthesiology, Miller School of Medicine,
University of Miami, Miami, Florida
Todd W. Sarge, M.D., Instructor of Anesthesia, Harvard Medical School, Program
Director, Critical Care Fellowship Program, Department of Anesthesia, Critical Care and
Pain Medicine, Beth Israel Deaconess Medical Center, Boston, Massachusetts
Karen J. Schwenzer, M.D., Associate Professor of Anesthesiology and Critical Care
Medicine, Department of Anesthesiology, University of Virginia Health System,
Charlottesville, Virginia
Nathanael Slater, D.O., Critical Care Fellow, Department of Anesthesiology, University
of Massachusetts Medical School, Worcester, Massachusetts
Faraz A. Syed, D.O., Anesthesiology Resident: CA-3, University of Massachusetts,
Worcester, Massachusetts
Michael Wall, M.D., F.C.C.M., Associate Professor of Anesthesiology and
Cardiothoracic Surgery, Washington University School of Medicine, St. Louis, Missouri
Michael C. Woo, M.D., Assistant Professor, University of Chicago, Department of
Anesthesia and Critical Care, Chicago, Illinois
Zdravka D. Zafirova, M.D., Assistant Professor, Department of Anesthesia and Critical
Care, The University of Chicago, Chicago, Illinois
xi
ASCCA ResidentКјs Guide to the ICU
xii
ASCCA Residents Guide to the ICU
1. The ICU Experience
Ruben J. Azocar, M.D.
An 82-year-old female is admitted to the medical ward at a small community with abdominal pain on a
Friday afternoon. Her sign and symptoms correlate with small bowel obstruction. Her prior medical history
include CAD, type 2 diabetes and obesity. Her status deteriorates overnight and she is taking emergently
to the OR for an exploratory laparotomy which reveals ischemic bowel requiring a small bowel resection.
The surgeon is concerned about her postoperative course and requests for her to be admitted to the ICU.
Anesthesiologists played a major role in the creation of CCM, which is a logical extension of anesthesia
practice.1 However anesthesiologists represent a minority of the critical care providers in the US. In the
next several years the demand for critical care providers will increase not only as the population in the US
ages but as external forces demand intensivists to provide care to the critically ill. These factors will
potentially create a demand for intensivists and many of us believe it will provide the opportunity for
anesthesiologist to regain presence in the ICUs across the country.
The practice of critical care is a multidisciplinary and collaborative effort. The Society of Critical Care
Medicine (SCCM) maintains that the most appropriate care for the critically ill is best provided by an
integrated team of dedicated experts directed by a trained physician credentialed in critical care medicine.
2
These characteristics make the ICU a fertile environment for the development of key aspects of an
anesthesiology consultant. The six core competencies of medical education as defined by the ACGME
can be learned, developed and evaluated in the ICU.3 The resident should learn essentials of critical care
principles, learn and polish technical skills, provide compassionate patient care based on evidence and
develop impeccable communication skills during rounds, discussion with other member of the health team
and the patients or their families. Professional demeanor and adherence to ethical principles, a clear
understanding of the system in which each particular ICU exists and the opportunity to apply practice
base learning are present everyday in the ICU.
Traditionally ICUs have been classified as closed or open, depending on whether or not the ICU team has
“total control” of the care or they act as consultants with the primary physician being the one in “charge”.
These are obsolete concepts as the literature supports the team approach in which a collaborative effort
to provide the safe delivery of medical care is taken.4 This concept also introduces the idea of team
training with the goal of creating teams in which all members recognizes the skills and knowledge of each
member, all team members contribute to patient assessment and care plan development. The key for the
success of the team is based on mutual trust and respect, communication among all team members to
clarify goals and monitor progress. Finally, a team concept promotes morale, reduces stress, and
enhances ability to provide excellent care.
This team based-high intensity approach has been shown to improve outcomes in the ICU.5, 6
In addition, the benefits of assuring that the critically ill and injured patients receive care from integrated
teams of dedicated experts directed by trained intensivists are not limited to the clinical aspects of ICU.
Significant cost savings can be realized by the hospital.7 This observation is a key factor considering that
the ICU consumes a large part of the hospital resources.
The core of the ICU team consists of intensivists, critical care nurses, respiratory therapists, pharmacists/
pharmacologists, and patients and their families. In addition, primary care physicians, physician
assistants, social workers, dietitians, ethicists, consultant medical specialists, and other professionals are
also often part of the team.2 The objective of this team is to work collaboratively for the betterment of their
patients using knowledge, technology and compassion to provide timely, safe, effective, and efficient
patient-centered care.
1
ASCCA Residents Guide to the ICU
At this time there are not enough critical specialists to provide coverage to all critically ill patients in the
country. Alternatives such as telemedicine have been suggested to fill this void.8 This represents an
opportunity for our specialty provide care to the sickest patients in the hospital but this time outside the
wall of the operating room. Even if critical care is not your subspecialty career path, the experience while
in the ICU makes any anesthesiologist a better perioperative physician.
The chapter is a revision of the original chapter authored by Laureen L. Hill, M.D. and Ronald G. Pearl, Ph.D., M.D.
REFERENCES:
1. Hanson CW 3rd, Durbin CG Jr, Maccioli GA, et al The anesthesiologist in critical care medicine: past, present, and
future. Anesthesiology. 2001 ;95: 781-8. 2. http://www.sccm.org/SCCM/Professional+Development/Quality+Initiatives/Right+Care+Right+Now/
3. http://www.acgme.org/Outcome/
4. Pronovost PJ, Holzmuller CG, Clattenburg L et al: Team care: beyond open and closed intensive care units. Curr
Opin Crit Care. 2006; 12:604-8
5. Azocar RJ, Lisbon A. Captaining the ship during a storm: who should care for the critically ill? Chest. 2001 ;
120:694-6
6. Pronovost PJ, Angus DC, Dorman T, et al Physician staffing patterns and clinical outcomes in critically ill patients:
a systematic review JAMA 2002; 288: 2151-62
7. Pronovost PJ, Needham DM, Waters H, et al Intensive care unit physician staffing: financial modeling of the
Leapfrog Standard. Crit Care Med 2004; 32:1247-53
8. Rosenfeld BA, Dorman T, Breslow MJ, et al : Intensive care unit telemedicine: alternate paradigm for providing
continuous intensivist care. Crit Care Med 200; 28:3925-31
QUESTIONS:
1.1.
A.
B.
C.
D.
The best approach for ICU care is:
Closed ICU
Open ICU
Semi-closed
Team approach
A.
B.
C.
D.
The approach of a devoted team to provide ICU care provides all the following benefits EXCEPT:
Improves survival
Decreases cost
Decreases length of stay
Affects the resident learning negatively
A.
B.
C.
D.
The following statements regarding ICU are true EXCEPT:
Anesthesiologists were important in the inception of ICUs
There are too many ICU providers
Telemedicine is an alternative to provide continuous ICU care
External forces are demanding that critically ill patients be cared for by critical care professionals
1.2.
1.3.
1.4.
A.
B.
C.
D.
The ACGME has defined 6 core competencies for medical training. Which of the following is correct?
All can be learned and evaluated in the ICU
Communication skills are most important in the ICU provider
Ethical dilemmas in the ICU are key in the development of professionalism
It is difficult to lear about system-base practice as patient care is the fundamental aspect of the ICU
experience
2
ASCCA Residents Guide to the ICU
2. ICU Management
Ruben J. Azocar, M.D.
A 55-year-old male with no significant past medical history develops Transfusion Related Acute Lung
Injury after receiving 1 unit of PRBC. The transfusion was ordered to treat his acute drop in hemoglobin
secondary to upper GI bleeding. He improves rapidly and is extubated two days later. The day after the
extubation, the ICU team decides he can be transfer back to the wards. However, his primary care doctor
is worried and wants him to stay in the ICU for an extra day.
Patients in the ICU account for nearly 30% of the acute care hospital costs, yet these patients occupy
only 10% of the impatient beds.1 In addition a report in 1984 revealed that >20% of hospital budgets
were expended in the care of critically ill patients and that around 1% of the gross national product was
expended for ICU services. The number of patients requiring ICU services is likely to continue to rise as
the aging population in the US increases. In addition there are social and economic pressures by
managed care organizations, hospital administrators and even consortiums such as the Leap Frog
Group, that are constantly requesting efficiency and efficacy in the care of the critically ill.
It is the responsibility of ICU Director to determine and enforce admission and discharge criteria, examine
length of stay, measure quality of care parameters and maintain a quality improvement program. Those
are few of the aspects of “ICU Management”
Is there a best practice model for ICU?
There is not a defined “best model”; however, the literature has identified factors that are related to
improved outcomes, decreased mortality, improved efficiency, decreased length of stay and decreased
costs. Such factors include the following1:
•
Timely and personal intervention by an intensivist reduces mortality, reduces length of stay and
decreases cost of care.
•
If an intensivist assumes the administrative role in the ICU benchmarking, standardization of care
and clinical research is provided then decreases in length of stay, cost of care and treatment related
complications has been observed.
•
Drug related events and cost of care are decreased when a critical care pharmacist is present in the
ICU
•
Excessive nurse workload is associated with increased mortality in the ICU
•
The presence of full time respiratory care practitioners in the ICU decreases ventilator days, length
of stay and cost.
Based on these data, it seems safe to affirm that the concept of open versus closed units is a obsolete
concept and that the team model approach, lead by an intensivist, for delivery of ICU care reduces
mortality, ICU length of stay, hospital length of stay, and cost of care.2,3 By definition, the intensivist is a
physician who is trained through a primary specialty (internal medicine, pediatrics, surgery, emergency
medicine, anesthesiology) and who then completes an ACGME program in critical care medicine and
obtains a certificate of special qualification in critical care. 1
A key component of the team approach is the definition of the clinical roles of all of the members of the
team. There is an excellent review of the roles of the team members in the appendix section of the article
by Brilli et al. 1 Fulfillment of those roles will facilitate the flow of the work, standardize the care and
achieve the goals above mentioned.
It is important that criteria for triage, admission and discharge from the ICU and or intermediate care units
(step downs) is well delineated and that compliance is maintained.4,5 There are SCCM guidelines that
provide the ICU director with parameters to determine what variables should be measured. Overuse in
the ICU not only generates additional and unnecessary costs but also poses a risk for the patients as
staying in the ICU is not devoid of risks.
Another important aspect to the improvement in ICU care is the collection of data and /or benchmarking
to demonstrate improvement or maintenance of the best care possible to the critically ill. Monitoring for
3
ASCCA Residents Guide to the ICU
complications such as central line infections, development of DVP or GI bleeding, compliance with
preventive measures (i.e Head of bed at 30 degrees for prevention of VAP), adherence to specific
protocols such as strict glucose control or following the SCCM sepsis guidelines is fundamental. Likewise,
it is crucial to determine the acuity and predictors of outcome (APACHE, SOFA, ISS scores) of patients
admitted to any particular ICU and compare those values with actual outcome. If those parameters are
not measured, achievement of goals and maintenance of quality cannot be demonstrated.
In summary, convincing data suggest that merely having daily rounds led by an intensivist enhances
patient care significantly. Further improvements can be obtained by maintaining a nurse-to-patient ratio of
no greater than 1:2, adding critical care pharmacists, and providing dedicated respiratory therapists to the
ICU team.
This chapter is a revision of the original chapter authored by Edward C. Bratzke, M.D.
REFERENCES
1. Brill RJ, Spevetz A, Branson RD et al Critical Care delivery in the intensive care unit: Defining clinical roles and the
best practice model. Crit Care Med 2001;29: 2007-2019
2. Durbin CG Jr. Team model: advocating for the optimal method of care delivery in the intensive care unit. Crit Care
Med. 2006 Mar;34(3 Suppl):S12-7.
3. Azocar RJ, Lisbon A. Captaining the Ship during a Storm: Who should care for the Critically Ill? Chest 2001; 120:
694-696
4. Guidelines for ICU Admission, discharge and triage. Crit Care Med 1999;27:633-638
5. Guidelines on Admission and discharge for adults intermediate care units. Crit Care Med 1998;26:607-610
QUESTIONS:
A.
B.
C.
D.
The appropriate care for the patient in the clinical vignette is:
Keep the patient in the ICU
Send the patient to the ward
Discuss the patient care with his PCP and after clarifying the situation send to the ward
None of above
A.
B.
C.
D.
In a team approach, the responsibilities of the intensivist include the following EXCEPT:
Leading the team to provide integral ICU care
Maintaining a quality assurance program in the ICU
Being there for every single step of a weaning protocol
Delineating admission, discharge and triage parameters
A.
B.
C.
D.
In terms of costs, ICUs:
Consume over 50% of Hospital budgets
Consume 10% of the gross national product
Can be run more efficiently with an intensivist directing the ICU and providing patient care
Are likely to decrease as the US population ages
A.
B.
C.
D.
The literature supports the presence of the following members of an ICU team EXCEPT:
Critical Care pharmacist
1:4 nursing ratios
Respiratory therapist
Intensivist
A.
B.
C.
D.
APACHE
is a weaning protocol
is a severity of injury score for trauma patients only
predicts length of stay in the ICU
predicts mortality in critically ill patients
2.1.
2.2.
2.3.
2.4.
2.5.
4
ASCCA Residents Guide to the ICU
3. Family Support and Ethics Issues
W. Christopher Croley, M.D., FCCP; Karen J. Schwenzer, M.D.; David M. Rothenberg, M.D., FCCM
A 68-year-old female was brought to the emergency department after two days of worsening nausea,
vomiting and fatigue. During that time, the patient had attempted to contact her son without success.
Finally, she had to call an ambulance to bring her to the hospital emergency department.
She was found to have a small bowel obstruction and taken immediately to the operating room for an
emergent exploratory laparotomy. She was transferred to the surgical intensive care unit (SICU)
intubated, sedated and mechanically ventilated. The patient was hemodynamically stable, rapidly
weaned from mechanical ventilation and extubated six hours after admission. No family had been
present or available since the admission.
Shortly after extubation, the patient informed the nurse that her son rarely calls or visits, and she would
like a close friend, whom she described as her “adopted daughter”, listed as the emergency contact. She
informed the nurse that her friend had been on a cruise, but should be back now; she was certain her
friend would be worried when she realized she was hospitalized.
Intraoperatively, the surgeon noted a mass on the head of the pancreas with an area of necrotic tissue
around the mass. A biopsy was taken and sent for frozen section, which revealed inflammatory changes;
no malignant cells were identified. On postoperative day 1 (POD 1), the surgeon discussed his findings
with the patient who revealed that her brother died of pancreatic cancer, and that regardless of the
pathology she felt she had pancreatic cancer. The surgeon informed her they would await final biopsy
results and advised her not to worry. The patient reiterated her concern of pancreatic cancer and stated
that if it was in fact malignant, she would not want any additional treatment.
Later that day, the patient developed wheezing, shortness of breath, hypotension and required emergent
intubation. She was transferred back to the SICU and diagnosed with an acute pulmonary embolus.
Initially, the patient required mechanical ventilation as well as inotropic support, which was weaned
approximately 36 hours after the initial event. She continued to require ventilator support and the
discussion of a tracheostomy was initiated on POD 12 with the “adopted daughter” who had been present
daily since POD 7. At that time, the “adopted daughter” felt it was best to proceed with the tracheostomy
since the mass was found to be inflammatory and not malignant. However, since there was no power of
attorney on file, the next of kin is the patient’s son.
When the son was contacted, he refused to authorize permission to perform the tracheostomy. He stated
that he was not close to his mother, but was well aware of her wishes to not have any means of artificial
support to prolong her life. He disclosed that based on similar preoperative symptoms his uncle had
experienced there was a possibility this could be a pancreatic cancer, and wanted hospice care
consulted. The “adopted daughter” feels he is ready for her to die; she states that he never interacts with
her and is ready for his inheritance. She does not think he is making this decision based upon his
mother’s prior wishes and believes that we should move forward and provide aggressive care.
The principles of clinical ethics and end of life issues are an important part of the ICU environment.
Effective communication with patients and family is crucial, as misunderstandings will compound an
already emotionally charged situation.
I.
General Principles
A. Beneficence
1. Preserving life
2. Curing
3. Healing
4. Restoring or maintaining bodily functions and mental capacities
5. Preventing disease or injury
6. Relieving suffering
5
ASCCA Residents Guide to the ICU
B. Nonmaleficence
1. Minimize unnecessary procedures
2. Withdrawal of support that is only death delaying
C. Autonomy
1. Patient cultural and/or religious variations
2. Role of family to support, help, and care for the patient
3. Role of family as advisor to the patient
4. Role of family as surrogate decision makers
5. Parent-child relationships
D. Social Justice
1. Fiduciary responsibility
2. Trust of patient and trustworthiness of physician
3. Professional integrity of physician
4. Respectful treatment of patient
5. Futility
E. Relationship of managed care organization, physician, and patient
1. Monetary incentives and withholding of premiums affect changes in physicians’ behavior
2. Managed care organization conflicting with the traditional physician-patient relationship
a) Exclusionary contracts and restriction on choice of physician
b) Restriction to specific sites of care
c) Restriction on type of care
d) Restriction on referral pattern
3. Potential of managed care organization to limit amount and type of care at or near the end of
life
II. Ethical Considerations
A. Patient confidentiality and privacy
B. Truth-telling and disclosure
1. Patient self-determination and autonomy
2. Therapeutic benefit of open communication
3. Cultural differences and language barriers
4. Techniques of disclosure
a) Professional standard
b) Reasonable person standard
c) Differentiate between disclosure and information transfer
5. Disclosing uncertainty and futility
6. Disclosing therapeutic errors
a) Truthfulness and sensitivity
b) Benevolent deception
c) Omission and evasion of truth
C. Patient’s capacity to share in decision making
D. Decision making for incapacitated patients:
1. Identify a surrogate decision maker
a) Legally appointed guardian
b) Patient designated proxy
c) Family members
2. Sources of guidance for surrogate decision-making
a) Written advance directives–durable power of attorney for health care decisions, living
wills, health care proxy, legally appointed guardians
(1) Verbal advance directives
(2) Substituted judgment standard
(3) Best interest standard
(4) Physician based standard
E. Process of informed consent
1. Informational Elements
a) Disclosure of information
(1) Diagnosis
(2) Nature and purpose of proposed treatment
6
ASCCA Residents Guide to the ICU
(3) Risks of proposed treatment
(4) Alternative choices of treatment
(5) Risks and consequences of no treatment
b) Comprehension of information
2. Consent elements
a) Voluntary consent
b) Capacity to consent
F. Refusal of treatment including refusal based on religious grounds
G. Consent for human investigation and experimentation
H. Exceptions to requirement for informed consent:
1. Emergencies
2. Statutory exceptions for minors
a) Emancipated minors
b) Mature minors
c) Married minors
d) Minors who are parents
e) Minors in military service
3. Patient waiver
4. National and state waivers for informed consent
a) HIV testing
b) Vaccination programs
c) Newborn genetic screening
III. Pediatric Issues
A. Role of parent as surrogate decision maker
B. Withholding and withdrawing therapy from critically ill, malformed, or handicapped neonates and
critically ill older children
1. Local, state, and national statutes
2. Parental role
3. Jehovah’s Witness refusal of life-saving blood transfusion for minor child or fetus
C. Role of older child in decision making (assent)
IV. Mediating Disputes
A. Requests for medically futile or even harmful treatment
B. Requests for medically inappropriate CPR
C. Resolving disputes amongst the health care team
D. Resolving disputes amongst family members
E. Role of time-limited trials of treatment
F. Surrogate demands to withdraw beneficial treatment
G. Parental demands to withhold beneficial treatment from minor child
1. Model of shared decision making
2. Duty of physician to promote best interests of child
3. Legal counsel
H. Resolving disputes with managed care organization and patient advocacy
I. Mediation help
1. Hospital ethics committee
2. Clergy members
3. Family counselors
4. Social workers
5. Patient representatives
6. Psychologists and psychiatrists
7. Legal counsel
V. Death and Dying
A.
Discussion of end-of-life issues
1.
Ethically and legally valid advance directives
2.
Benefit and burden analysis within the confines of medical uncertainty
3.
Futility
7
ASCCA Residents Guide to the ICU
Emotional support to the dying patient and family
Permission for autopsy
Permission for organ donation
B.
Withholding and withdrawing treatment
1.
Local, state, and national statutes regarding end-of-life issues
2.
Mechanisms to obtain legal advice
3.
Do Not Resuscitate orders (DNR)
a)
Perioperative suspension vs. maintenance of DNR
4.
Techniques to discontinue life sustaining therapy
a)
Foregoing advanced life support (mechanical ventilation, vasoactive drug therapy,
hemodialysis, etc)
b)
Foregoing basic life support (intravenous hydration and artificial nutrition, etc)
C.
Physician-assisted suicide
1.
Local, state, and national statutes
2.
Requests for physician-assisted suicide
a)
Major depressive disorder
b)
Dementia
c)
Intractable pain
d)
Incurable disease
e)
Financial burdens
3.
Alternatives
a)
Outpatient and inpatient hospice
b)
Multidisciplinary palliative care service
c)
Pain management service
D.
Physician involvement with lethal injection
1.
Local, state, and national statutes
2.
Relief of suffering vs. violation of professional integrity
4.
5.
6.
REFERENCES/READING RECOMMENDATIONS:
1. Lautrette A, Ciroldi M, Ksibi H, Azoulay E. End-of-life family conferences: Rooted in the evidence. Crit Care Med.
2006;11(Suppl):S364-S372.
2. Rie MA, Kofke WA. Non-therapeutic quality improvement: The conflict of organizational ethics and societal rule of
law. Crit Care Med. 2007;2(Suppl):S66-S76.
3. Wlody GS. Nursing management and organizational ethics in the intensive care unit. . Crit Care Med. 2007;2
(Suppl):S29-S35.
4. Szalados JE. Discontinuation of Mechanical Ventilation at End-of-Life: The Ethical and Legal Boundaries of
Physician Conduct in Termination of Life Support. Crit Care Clin.;2007;23:317–337.
5. Curtis JR, Engelberg RA. Measuring success of interventions to improve the quality of end-of-life care in the
intensive care unit. Crit Care Med. 2006;11(Suppl):S341-S347.
6. Gavrin JR. Ethical considerations at the end of life in the intensive care unit. Crit Care Med. 2007;2(Suppl):S85-S94.
7. Giacomini M, Cook D, DeJean D, Shaw R, Gedge E. Decision tools for life support: A review and policy analysis.
Crit Care Med. 2006;3:864-870.
8. The Ethics Committee of the Society of Critical Care Medicine. Recommendations for end-of-life care in the
intensive care unit. Crit Care Med. 2001;12:2332-2348.
9. Buchman TG, Cassell J, Ray SE, Wax ML. Who should manage the dying patient? Rescue, Shame and the SICU
dilemma. J Am Coll Surg. 2002;5:853-854.
8
ASCCA Residents Guide to the ICU
QUESTIONS:
3.1.
A capable and informed patient’s decision to refuse a recommended treatment:
A. Is justified, but based on a different set of ethical guidelines than the process of obtaining
informed consent
B. Should be put to a more rigid standard than if the patient consents to treatment
C. Requires stronger moral and legal justification than if the patient consents to treatment
D. Is a function of the status of the patient’s insurance
E. Is the converse of the right to consent to treatment and is equally binding to health care
providers
A.
B.
C.
D.
E.
Critical care providers who choose to honor a competent adult Jehovah’s Witness refusal to
accept a life-saving blood transfusion, do so based on:
The principle of social justice
The principle of patient autonomy
Respect for the writings of the Old Testament
The principle of informed consent
The principle of therapeutic privilege
A.
B.
C.
D.
The Health Care Decisions Act of 1992 allows a patient the right:
To choose his or her own physician
To suspend life-sustaining interventions
To request what a physician considers futile treatment
To ask for physician assisted suicide
A.
B.
C.
D.
When caring for a terminally ill and suffering patient, pain control medications should be:
Limited because of fear of a possible dependency
Limited because of fear of causing a respiratory arrest
Limited to doses to relieve pain and promote comfort
Limited if patient has a history of illegal narcotic abuse
3.2.
3.3.
3.4.
3.5.
Which of the following statements about caring for terminally ill patients are true:
If burdens of treatment outweigh the benefits, critical care providers should cease all
treatment.
B. If patients or their surrogate decision-makers express a wish to discontinue treatment, critical
care providers must concur even if the patient risks death.
C. Physician-assisted suicide should be available and offered to terminally ill patients.
D. Discontinuing treatment for a terminally ill patient may be construed as euthanasia
A.
3.6.
A.
B.
C.
D.
E.
A 37-year-old woman with end-stage breast cancer is seriously impaired neurologically. She
has an identical twin who knows her sister’s wishes. The patient cannot participate in
decision-making about her treatment. Which of the following proxy decision makers is
favored by both ethics and law?
Her identical twin
Her mother
Her appointed durable power of attorney for health care decisions
Her father
A court-appointed guardian
9
ASCCA Residents Guide to the ICU
4. Cardiopulmonary Resuscitation (CPR)
Todd W. Sarge, M.D.
A 50-year-old woman with End-Stage Renal Failure is admitted with fever, chills and general malaise for
infection work-up. She has not had dialysis for several days due to her symptoms and was admitted to
the ICU for hypotension and tachycardia. She is on a norepinephrine infusion at 0.12 mcg/kg/min and
has a central venous line placed in the emergency department showing a central venous pressure of 19
mmHg. Shortly after arrival, she is noted to be increasingly tachypneic, with worsening hypotension. She
suddenly loses consciousness and a “code” is called overhead. Upon arrival to the bedside, the
physicians note a wide complex rhythm with a sinusoidal pattern.
INTRODUCTION: A major goal in the development of the modern ICU was the need for a special
environment to care for the patient at risk for or following a cardiopulmonary arrest. Physicians, nurses
and ancillary staff working in the intensive care unit should be certified and familiar with advanced
cardiopulmonary resuscitation protocols. Equipment should be readily available throughout the hospital
and intensive care units to include a defibrillator and other ACLS equipment and drugs. Physicians
should also be familiar with the physiological principles of artificial circulation, potential alternatives to
standard techniques, ethical issues surrounding CPR, and post-resuscitation care.
I.
Airway Management
A. Establish Unresponsiveness
1. Still important in a monitored ICU
a) See Defibrillation Section IV if monitored Ventricular Fibrillation / Tachycardic arrest with
hypotension and/or loss of consciousness
2. Call for help, e.g. “Code Blue”
3. Administer supplemental oxygen as soon as possible
B. Techniques for opening airway
1. Head tilt/chin lift
2. Jaw thrust
C. Adjuncts for airway control
1. Oropharyngeal/nasopharyngeal airways
2. Endotracheal intubation
3. Laryngeal Mask Airways
II. Breathing/Ventilation
A. Physiology of ventilation during CPR
1. Lung-thorax compliance
2. Esophageal sphincter opening pressure
B. Techniques of artificial respiration
1. Mouth-to-mouth/nose
2. Mouth-to-mask
3. Bag-valve-mask
4. Endotracheal tube
III. Circulation
A. Assess Pulse and Rhythm
1. Carotid pulse deemed most reliable
2. Establish rhythm via electrocardiogram (ECG)
a) Assume VF in pulseless state until proven otherwise
b) Assess rhythm early with “quick look” feature
c) Establish definitive ECG as soon as possible
d) Always correlate the rhythm with clinical condition / pulse
B. Physiology of circulation during closed chest compression
1. Cardiac compression theory vs. thoracic pump theory
2. Distribution of blood flow during CPR
10
ASCCA Residents Guide to the ICU
3. Gas transport during CPR
C. Technique of closed chest compression
1. Use sternum with heel of hand parallel to long axis of sternum
2. Keeping elbows locked, depress chest 1.5 to 2 inches
3. Allow equal time for compression and full recoil at a rate of 100 compressions per minute
D. Alternative methods of circulatory support
1. Closed Chest Compression Techniques
a) Simultaneous ventilation-compression CPR and abdominal binding
b) Interposed abdominal compression CPR
c) Pneumatic vest CPR
d) Active compression-decompression CPR
2. Invasive techniques
a) Open chest cardiac massage
b) Cardiopulmonary bypass
E. Assessing the adequacy of circulation during CPR
1. Pulse palpation
2. Aortic/arterial diastolic pressure
3. Myocardial perfusion pressure = aortic diastolic - right atrial diastolic pressure
4. Pupillary responsiveness
5. End-tidal CO2
IV. Defibrillation
A. Duration and pattern of ventricular fibrillation (VF)
1. Influence of time
a) May defibrillate prior to rescue breathing if monitoring and defibrillation immediately
available in ventricular fibrillation/tachycardia arrest
2. Importance for prognosis
a) For best prognosis in VF, CPR should be initiated with 4 minutes and defibrillation within
8 minutes
B. Defibrillators–Energy, current, and impedance
1. Energy vs. current
a) Determinants of transthoracic impedance
b) Energy requirements
C. Adverse effects
V. Pharmacologic therapy
A. Route of administration
1. Intravenous access preferred – central > peripheral
a) IV access permits volume expansion along with drug administration
2. Endotracheal - Some drugs safe for endotracheal administration
a) LANE-V = Lidocaine, Atropine, Naloxone, Epinephrine, Vasopressin
B. Sympathomimetics and vasopressors
1. Mechanism of action - Alpha vs. beta agonist
a) Agents
(1) Epinephrine – standard dose 1 mg every 5 minutes vs. “high dose”
(2) Norepinephrine – No great advantages over epinephrine in arrest
(3) Isoproterenol – pure beta agonist should not be used during arrest. Primary role is
a bridge to transvenous pacing in atropine resistant brady-arrhythmias
(4) Dopamine – exerts dopamine receptor activity at lower doses in addition to alpha
and beta
(5) Dobutamine – Potent beta agonist with less chronotropic effect (tachycardia) than
isoproterenol for use in heart failure
(6) Vasopressin – not a catechol, but a naturally occurring antidiuretic hormone. May
be used in place of epinephrine or as second agent at 40 Units IV bolus. May be
more effective in setting of acidosis
C. Anti-Arrhythmics
1. Amiodarone – demonstrated success in survival to hospital admission in out of hospital
cardiac arrest due to ventricular arrhythmias, now given class IIb recommendation for
11
ASCCA Residents Guide to the ICU
patients with ventricular arrhythmias after defibrillation
Lidocaine – indeterminate efficacy, but viable choice for hemodynamically compromising
premature ventricular complexes or stable VT
3. Procainamide – may be used for arrhythmias not treated with amiodarone or lidocaine,
however beware of hypotension and AV nodal block
4. Adenosine – useful for terminating reentrant tachyarrhythmias or diagnosing atrial fibrillation /
flutter. Short 5 second half-life.
5. Magnesium – should be given in ventricular arrhythmias due to hypomagnesemia or torsades
de pointes.
D. Other Agents
1. Atropine
a) Asystole or symptomatic bradycardia
2. Calcium
a) useful only in setting of known hypocalcemia, hyperkalemia or calcium channel blocker
toxicity
3. Sodium Bicarbonate
a) Theoretical advantages and disadvantages
b) Danger of excessive use
2.
VI. ACLS algorithms
A. Universal algorithm for unresponsive patient – ABC’s – avoid hyperventilation.
B. Algorithm for ventricular fibrillation and pulseless ventricular tachycardia
1. Electrical defibrillation is most important and should be performed ASAP
2. CPR should be continued while charging and immediately after first shock
C. Algorithm for pulseless electrical activity
1. Pharmacology, volume administration and cause determination are most important
2. Causes of PEA – “5 H’s and 5 T’s”
a) Hypovolemia, Hypoxia, Hydrogen ions (acidosis), Hypo-hyperkalemia, Hypoglycemia,
Hypothermia
b) Toxins, Tamponade, Tension pneumothorax, Thromboembolism, Trauma
D. Algorithm for asystole
1. Pacing is considered to be of little benefit
2. Epinephrine is still drug of choice, followed by atropine
VII. Post-resuscitation care
A. Brain-oriented support
1. Blood pressure management
2. Blood gas management
3. Blood volume and hemodilution
4. Blood glucose
B. Prognosis
1. Neurologic exam
2. CSF enzymes
VIII.Ethical considerations
A. Do Not Resuscitate / Do Not Intubate Orders
B. Medical futility vs. patient request
C. Disease transmission between rescuer and patient
This chapter is a revision of the original chapter authored by Charles W. Otto, M.D.
READING LIST:
1. ECC Committee, Subcommittees and Task Forces of the American Heart Association. 2005 AHA Guidelines for
Cardiopulmonary Resuscitation and Emergency Cardiac Care. Circulation 112 Supplement 24: IV1-203, 2005. This
is the most recent summary of the accepted guidelines for CPR in the United States.
2. Field JM. (editor), ACLS Subcommittee 2005-2006. American Heart Association- Advanced Cardiac Life Support
Provider Manual. Dallas (TX); 2006, pp. 1-150. The most recent manual of advanced life support.
12
ASCCA Residents Guide to the ICU
3. Ali B, Zafari AM. Narrative review: cardiopulmonary resuscitation and emergency cardiovascular care: review of
the current guidelines. Ann Intern Med. 2007 Aug 7;147(3):171-9. Clear and concise review and rationale of current
CPR techniques with references
QUESTIONS:
4.1.
4.2.
A.
B.
C.
D.
E.
In the ICU, the preferred first intervention for a patient developing ventricular fibrillation is:
Epinephrine
Intubation
Mouth-to-mouth ventilation
Defibrillation
Call 911
A.
B.
C.
D.
All of the following are true regarding the proper settings of a defibrillator EXCEPT:
Biphasic defibrillation requires less energy than monophasic defibrillation
The synchronous mode should be selected for cardioversion of atrial fibrillation
The asynchronous mode should be selected for shocking VF
The synchronous mode should be selected for shocking rapid pulseless VT > 150 b/min
A.
B.
C.
D.
E.
Sodium Bicarbonate administration is recommended by the American Heart Association
(AHA) during CPR for VF when:
Arterial pH falls below 7.1
CPR has continued for longer than 8 minutes
Serum lactate levels exceed 10 mmol/L
All of the above
None of the above
A.
B.
C.
D.
E.
Epinephrine improves outcome from CPR by:
Stimulating myocardial contraction
Improving success of defibrillation
Improving myocardial blood flow
None of the above
All of the above
4.3.
4.4.
4.5.
4.6.
Epinephrine administered endotracheally during CPR is less effective than when
administered intravenously.
A. True
B. False
A.
B.
C.
D.
E.
Calcium chloride is indicated during CPR:
When a known diltiazem overdose is present.
When pulseless wide-complex bradyarrhythmia is present.
When resuscitation has not occurred after 20 minutes of CPR.
When ventricular fibrillation has a low amplitude.
When a pulseless narrow-complex tachycardia is present.
13
ASCCA Residents Guide to the ICU
5. Transport of the Critically Ill
Daniel Castillo, M.D., Claudia Chidiac, M.D., Miguel A. Cobas, M.D.
A 36-year-old male was brought to the emergency room following a motor vehicle accident. On arrival, the
patient was awake and oriented. His vitals signs were: HR 122, BP 100/58, RR 32, Saturation 93%, temp
98.30 F. Patient was complaining of chest pain and dyspnea. Primary trauma care guidelines were
followed. His physical examination was notable for decreased breaths sounds on the left hemithorax.
Laboratory studies showed a HCT 40%, arterial blood gas showed a pH 7.48, pO2: 70 mm Hg, pCO2: 28
mmHg, base deficit: -3, O2 saturation: 92%. CXR was notable for a left pneumothorax and mediastinal
widening. Oxygen was administered, intravenous access was obtained and fluid administered, a chest
tube was placed. Repeated ABG showed now a pH 7.38, pO2 95 mm Hg, pCO2 35 mmHg, base deficit:
-2, O2 saturation 97%. Because of persistent chest pain and the mediastinal widening found on CXR, it
was decided to obtain a chest CT to rule out a dissecting aortic injury. His vital signs remained stable. The
CT scan showed a possibility of dissection involving the aortic arch. An arterial line was placed and
additional large bore intravenous access obtained. The decision was made to transfer the patient to a
tertiary care facility approximately 20 miles away, for final diagnosis and possible surgery. The family
arrived to the hospital and was made aware of the patient’s condition and the need for transfer. At the
time of transfer, the patient remains in stable condition.
Critically ill patients frequently need regionalization of care and specialized resources requiring their
transportation. Although most of these patients will drive significant benefits from such a transport, they
may also be at considerable risk at transport-related morbidity and mortality. Therefore the decision for
such a transport must be carefully analyzed. Alterations and interruptions in therapeutic support
measures are tolerated poorly by critically ill patients. These patients may require transport to various
sites within the institution or to tertiary care centers for diagnostic and/or therapeutic interventions. During
these periods of transfer, patients are at risk for untoward events resulting from altered levels of support.
An appreciation of the pitfalls and legal issues of patient transport are important in the practice of critical
care medicine.
TERMINOLOGY:
Primary transport refers to the transportation of a patient from the incident site to a medical facility.
Secondary transport (or interhospital): is when the patient is moved from one hospital to another to
receive an increased level of care not available in the initial facility.
Intrahospital transport: is the transportation of the patient within the hospital annexes or to a different
floor within the same hospital, usually for investigations or treatment unavailable at the ward or intensive
care location (CT scan, MRI…)
I.
Interhospital transportation–risks versus benefits
A. Key points:
1. The aim during transport should be to provide the same or better quality care than the patient
had before transport
2. The common “scoop and run” practice must be abandoned and replaced with goals oriented
measures.
3. Risk stratification refers to identification of high–risk patient, which is an important step in
patient preparation, since those patients could undergo additional resuscitation and/or be
accompanied by special trained transport personnel.
B. Responsibilities of referring physician
1. Establish the need for transfer. These patients require services that exceed resources of
referral institution (usual indication for adults is trauma or cardiovascular instability; for
children is respiratory, neurological, or trauma)
2. Identify referral center
3. Optimize pre-transfer stabilization following COBRA/EMTALA legislation – may achieve true
stabilization only at receiving hospital.
14
ASCCA Residents Guide to the ICU
Do no further harm– transfer should not compromise patient outcome, and should be a
continuation of ongoing primary management.
5. Facilitate communication between referring and accepting physicians and nurses -provide
medical record, laboratory data, and radiographs
6. Obtain informed consent from patient or patient’s representative which should include the risk
of transfer as such.
7. Arrange appropriate transport (mode of transportation, expertise of personnel, equipments.
8. Communication: it is essential that the transfer team establish clear communication with both
referring and receiving physicians.
Responsibilities of receiving physician
1. Concur with and accept transfer
2. Assist with transfer management
Mode of transport considerations (i.e., ground, helicopter, fixed wing)
1. Time/distance - patient acuity, patient weight
2. Safety - weather conditions
3. Trained personnel – need minimum of 2 persons capable of airway management, intravenous
therapy, interpretation and treatment of dysrhythmias, basic and advanced cardiac, and
trauma life support training.
4. Means to communicate with physician(s)
Potential difficulties during transportation
1. Decreased barometric pressure
a. Decreased oxygenation (may be difficult to support oxygenation at high altitudes if
oxygen requirements at ground level are FiO2 = 100%).
b. Volume expansion of air in trapped spaces (ETT cuff, pneumothorax, sinuses, bowel,
brain)
2. Decreased temperature and humidity
3. Adverse effects of acceleration and deceleration forces on hemodynamics and intracranial
pressure
4. Excessive noise level and inability to auscultate lungs, heart, blood pressure
5. Motion artifacts and electrical interference
6. Space limitations for personnel, equipment, drugs
Therapy/stabilization before transport
1. Airway – ETT/mechanical ventilation, supplemental oxygen, NG suction, chest tubes
2. Cardiovascular – heart rate and rhythm, blood pressure, control bleeding, intravenous access
3. Neurological –hyperventilation, mannitol/diuretics, steroids, immobilization, intracranial
pressure monitor
4. Diagnostic studies – radiographic, electrolytes, hematocrit, arterial blood gas, toxicology
screen, urinalysis
5. Wounds/fractures –clean, dressed, splint/traction, tetanus, antibiotics
Equipment and medications available during transport
1. Airway–bag, mask, ETT, intubation equipment (including difficult intubation equipment),
ventilator with alarms, oxygen, suction
2. Cardiovascular–ECG, blood pressure, defibrillator, intravenous fluids and infusion pumps
3. Drugs for treatment of cardiac/pulmonary arrest, physiologic derangements, dysrhythmias,
patient discomfort/agitation
4. Communication equipment for contact with receiving/referring hospital
Monitoring and interventions during transport
1. Airway–oxygen sensor, pulse oximeter, capnography, airway pressure
2. Cardiovascular–ECG, non-invasive blood pressure cuff, arterial line, central venous pressure,
pulmonary artery pressure, intracranial pressure
3. Pulmonary – Mechanical ventilation, ventilator settings (portable), bag ventilation, Chest
tubes to suction, periodic auscultation of breath sounds.
4. Neurological: Monitor Neuromuscular paralysis if paralytics are being administered with
concomitant adequate sedation. If paralysis is not used, monitor and document mental status
changes.
5. Medical record documenting status and interventions during transport: airway, cardiovascular,
neurological, intake and output of fluids, medications, tubes/lines/drains, special
interventions/procedures, monitors and mechanical support
4.
C.
D.
E.
F.
G.
H.
15
ASCCA Residents Guide to the ICU
I.
6. Continue ongoing therapy/modify as indicated or directed
Legal Issues–COBRA/EMTALA legislation
1. Hospitals must examine and stabilize all patients (regardless of ability to pay)
2. Physician must certify that benefits of transfer to outside facility outweigh risks
3. Receiving facility must have personnel/space and agree to accept transfer
4. Transferring facility must provide medical record
5. Transfer is effectuated through qualified personnel with appropriate equipment
6. Financial penalties to physician and/or hospital may occur for failure to comply
7. Physician and/or hospital can be sued for violations that result in injury; receiving institutions
that receive “dumped” patients can sue for damages/obtain injunctions
8. Transferring facility is responsible for emergency treatment and transfer decisions made by
physicians
9. Any transfer can result in litigation if the patient deteriorates during transfer
II. Intrahospital transportation:
A. General considerations:
1. Why? Is it necessary?
2. Where? What is the best route?
3. Who should act as escort?
4. What equipment?
5. Do they know we are coming?
6. Risks versus benefits analyzed?
B. Indications
1. Patients requiring tests/procedures usually are more severely ill
2. Reasons–50-60% for diagnostic tests, 40-50% for operation
C. Pre-transport
1. Coordinate services/personnel for patient transport
2. Coordinate timely transport and beginning of test/procedure
3. Arrange equipment, supplies, and medications for monitoring, ongoing therapy, and
emergencies.
4. The medical care of patients during transportation should follow the same standards of care
as in the ward where they are being transported from, including monitoring and medications.
D. Outcomes of intrahospital transportation.
1. Average transportation time–75-90 minutes
2. Costs–changes in nursing/physician coverage, disposable supplies, and transportation of
equipment.
3. Inability to maintain same level of care, less staff, small spaces, equipment, different levels of
care, procedures or diagnostic procedures done in sites of the hospital where certain
monitors, pumps, equipment can’t be used. (MRI suite).
E. Frequency of complications: approx. one-third of transports involve mishaps
1. Airway–decreased O2 saturation, hypocarbia, hypercarbia (less frequent than hypocarbia,
associated with bag ventilation), unplanned extubation with difficult re-intubation,
disconnection, plugging of ETT.
a) When muscle paralysis is necessary for the safety of the transportation or for medical
reasons, it is important to dose the paralytics appropriately and to continue twitch
monitoring.
b) Patients that are paralyzed have to be sedated appropriately during transportation.
2. Cardiovascular: hypotension more frequent than hypertension (may be related to respiratory
complication), dysrhythmias, loss of pacemaker or balloon pump, bleeding, interruption of
vasoactive drug infusions and accidental loss of central lines
3. Neurological–increased ICP, inadequate sedation, problems during emergence from
anesthesia, stabilization of spine before official “clearance”.
4. Equipment ECG lead displacement, loss/disconnect of iv lines/infusions, failure of monitoring
equipment/infusion pump, depletion of O2/medication supply, difficulty accommodating
monitors/equipment/traction
5. Morbidity and mortality of complications–with appropriate monitoring and preparation,
significant morbidity is unlikely
6. Patients factors: diagnosis unclear; those injured tend to be more severely ill; risk is likely
16
ASCCA Residents Guide to the ICU
7.
greater with more intensive support.
Problems with communication: sign outs are done to different medical teams and different
levels of care, things can be forgotten or missed.
This chapter is a revision of the original chapter authored by Debra Szeluga, M.D. and Alan Lisbon, M.D.
READING LIST:
1. Advanced Trauma Life Support: Instructor’s Manual. 1993;293-303. Best manual of life support procedures.
2. Braman SS, Dunn SM, Amico CA, Millman RP. Complications of intrahospital transport in critically ill patients.
Ann Intern Med 1987;107(4):469-73. Study identifying potential problems with patient transport outside the ICU
and the frequency of their occurrence.
3. Brink LW, Neuman B, Wynn J. Air transport. Pediatr Clin North Am 1993;40(2):439-57. Describes modes of
transport, personnel issues, and costs of patient transport.
4. Day SE. Intra-transport stabilization and management of the pediatric patient. Pediatr Clin North Am 1993;40(2):
263-74. Describes issues in pediatric transport.
5. Frew SA, Roush WR, LaGreca K. COBRA: implications for emergency medicine. Ann Emerg Med 1988;17(8):
835-7. Best description of legal issues associated with patient transfer.
6. Guidelines Committee, American College of Critical Care Medicine; Society for Critical Care Medicine and
American Association of Critical Care Nurses Transfer Guidelines Task Force. Guidelines for the transfer of
critically ill patients. Crit Care Med 1993;21(6):931-7.
7. Indeck M, Peterson S, Smith J, Brotman S. Risk, cost, and benefit of transporting ICU patients for special studies. J
Trauma 1988;28(7):1020-5. Risks and costs of intra-hospital transport.
8. Johnson CM, Gonyea MT. Subspecialty clinics: pediatrics–transport of the critically ill child. May Clin Proc
1993;68(Oct):982-7. Overview of the principles and operating procedures of pediatric transport teams.
9. Smith I, Fleming S, Cernaianu A. Mishaps during transport from the intensive care unit. Crit Care Med 1990;18(3):
278-81. Study to identify factors that influence occurrence of intra-hospital transport mishaps.
10. Szem JW, Hydo LJ, Fischer E, Kapur S, Klemperer J, Barie PS. High risk intrahospital transport of critically ill
patients: safety and outcome of the necessary road trip.” Crit Care Med 1995;23(10):16660-6. Study of transportrelated complications and their effect on morbidity and mortality.
11. Richard Orr, John, Kimberly Roth: Transport Medicine: Chapter 262 ----CRITICAL CARE BBOOKK
12. Eddy Fan, Russell D MacDonald, Neill KJ Adhikari, Damon C Scales, Randy S Wax, Thomas E Stewart and Niall
ED Ferguson: Crit. Care 2006;10(1): ----pages--- Outcome of interfacility critical care adult patients transport: a
systematic review
13. A Gray, S Bush, S Whiteley: Emerg Med J 2004; 21:281-285. Secondary transport of critically ill and injured adult.
14. Faculty of Intensive care of the Australian and New Zealand College of Anaesthetists and Australian College for
emergency medicine. Minimum standard for transport of the critically ill. [www.acem.org.au]
QUESTIONS:
5.1.
A.
B.
C.
D.
E.
5.2.
A.
B.
C.
D.
E.
The Secondary transport:
Can only be done by physicians.
Is conducted between different counties or a State line.
Can replace primary transport when the patient is stable.
Is conducted by the receiving emergency department.
Is when the patient is moved from one hospital to another to receive an increased level of care not
available in the initial facility.
During non-pressurized air transport all is true EXCEPT:
The requirements for supplemental Oxygen are different than in the ground.
Patients can achieve higher PO2 with less FIO2
Is not contraindicated for patients with ICDКјs.
Chest tubes should be connected to constant suction.
There is a limit for the amount of equipment that can travel as well as for the amount of personnel.
17
ASCCA Residents Guide to the ICU
5.3.
A.
B.
C.
D.
E.
5.4.
Regarding EMTALA (COBRA) legislation:
No litigation can be conducted if the patient deteriorates during transfer.
Antibiotics should never be given prior to transport as they will cause subsequent cultures to be
negative.
Private Hospitals must examine and stabilize all patients only if covered by insurance or self pay.
Financial penalties to physician and/or hospital may occur for failure to comply
Receiving facility is responsible for emergency treatment and transfer decisions made by physicians.
D.
Which of the following statement is true?
The practice of “scoop and run” should be applied depending on the patientʼs condition.
The aim during transport should be to provide the same or better care than before transfer.
Further harm is always a possibility when transferring a patient, because continuation of ongoing
primary management is not always possible.
Depending on patientКјs clinical situation, informed consent is not always required.
A.
B.
C.
D.
E.
Which of the following statement is true?
Risk of transfer does not need to be emphasized to the family
Pre-transfer stabilization following COBRA legislation is mandatory
Communication is sometimes not necessary between the teams
Arrangement for appropriate transport is the usual responsibility of the receiving physician
Risk stratification, depending on patientКјs condition, is not always possible
A.
B.
C.
5.5.
18
ASCCA Residents Guide to the ICU
6. Sedation of the Critically Ill
Ruben J. Azocar, M.D.
Sedation in the ICU has become one of the most important issues in the care of the critically ill
patient. Proper sedation is important to assure patient comfort in times of physiological stress and
to facilitate ongoing therapy such as mechanical ventilation. Furthermore as survival from sepsis
and ARDS is increasing, the issue of post-traumatic stress disorder after care in the ICU needs to
be considered. However, excessive sedation can also lead to prolonged ventilator days, longer
length of stay in the ICU and increases in costs. This discussion is based on the Society of
Critical Care Clinical Practice Guidelines for the Sustained Use of Sedatives and Analgesics in
the Critically Ill Adult.1 However, these guidelines are in the process of revision at this time as
more scientific evidence and understanding of the importance of sedation has been reported in
the literature.
It is important to differentiate among anxiety, agitation or delirium as a cause of changes in the
patient’s mental status. The causes of anxiety and agitation in critically ill patients are
multifactorial and are likely secondary to an inability to communicate amid continuous noise and
lighting, excessive stimulation, pain, sleep deprivation and metabolic derangements. When
patients exhibit signs of anxiety or agitation, the first priority is to identify and treat any underlying
physiological disturbances, such as hypoxemia, hypoglycemia, hypotension, pain, and withdrawal
from alcohol and other drugs. 1
An analgesic may be the appropriate initial therapy when pain is the suspected cause of acute
agitation, however it is important to emphasize that opioids may produce sedating effects but do
not have amnestic properties that would diminish awareness or provide amnesia for stressful
events. Not only should acute pain issues be considered but chronic issue must be assessed and
treated as well. A more detail discussion of pain management in the ICU will be covered in
Chapter 7. Adding a sedation agent may be necessary. However, in order to titrate sedatives to
the desire end-point, assessment of the degree of agitation or sedation is most important. Hence
the concept of goal directed sedation becomes important.
An ideal sedation scale should provide data that are simple to compute and record, accurately
describe the degree of sedation or agitation within well-defined categories, guide the titration of
therapy, and have validity and reliability in ICU patients. Many scales are available, but a true
gold-standard scale has not been established. The Ramsey scale, the Riker Sedation-Agitation
Scale (SAS),the Motor Activity Assessment Scale (MAAS) , the Vancouver Interaction and
Calmness Scale (VICS) have been used and validated for use in the ICU.1 However, the The
Richmond Agitation-Sedation Scale, a 10-level scale from +4 "combative" to -5 "unarousable” has
been validated and found to have an excellent inter-rater reliability.2 In addition, Ely et al
validated this scale for its ability to detect changes in sedation status over consecutive days of
ICU care, against constructs of level of consciousness and delirium, and correlated with the
administered dose of sedative and analgesic medications. 3
Recently the need to differentiate between “anxiety and agitation” and “delirium” has gained more
interest. As many as 80% of ICU patients have delirium, characterized by an acutely changing or
fluctuating mental status, inattention, disorganized thinking, and an altered level of consciousness
that may or may not be accompanied by agitation.1 Delirium has been found to be an
independent predictor of higher 6-month mortality and longer hospital stay even after adjusting for
relevant covariates including coma, sedatives, and analgesics in patients receiving mechanical
ventilation.4 Delirium may be associated with confusion and different motor subtypes: hypoactive,
hyperactive, or mixed. Hypoactive delirium, which is associated with the worst prognosis, is
characterized by psychomotor retardation manifested by a calm appearance, inattention,
decreased mobility, and obtundation in extreme cases and can be confused with a “properly
sedated” state . Hyperactive delirium is easily recognized by agitation, combative behaviors, lack
of orientation, and progressive confusion after sedative therapy.1 Assessment of delirium in the
ICU patient has been neglected because it is often difficult to communicate with the critically ill
particularly if receiving mechanical ventilation. Fortunately, delirium investigators have recently
19
ASCCA Residents Guide to the ICU
collaborated to develop and validate a rapid bedside instrument to accurately diagnose delirium in
ICU patients, who are often nonverbal because they are receiving mechanical ventilation.5 This
instrument is called the Confusion Assessment Method for the ICU (CAM-ICU). Critical care
nurses can complete delirium assessments with the CAM-ICU in an average of 2 minutes with an
accuracy of 98%, compared with a full DSM-IV assessment by a geriatric psychiatric expert
(which usually requires at least 30 minutes to complete). To complete the CAM-ICU, patients are
observed for the presence of an acute onset of mental status change or a fluctuating mental
status, inattention, disorganized thinking, or an altered level of consciousness. With the CAMICU, delirium was diagnosed in 87% of the ICU patients with an average onset on the second day
and a mean duration of 4.2 В± 1.7 days.5
Although promising, the use of modified EEG in the form of the Bispectral index (BIS) and/or the
Physiometrics scale index (PSI) have not found enough support in the literature to justify their
routine use. In addition, they do not seem able to detect delirium.6
An algorithm for sedation published in a clinical practice guidelines by the American College of
Critical Care Medicine can be found by following this link:
http://www.learnicu.org/Quick_Links/Pages/default.aspx
These SCCM guidelines provide a general guideline for the use of analgesic and sedatives. 1
Benzodiazepines are sedatives and hypnotics that block the acquisition and encoding of new
information and potentially unpleasant experiences (anterograde amnesia) but do not induce
retrograde amnesia. 1 Although they lack any analgesic properties, they have an opioid-sparing
effect by moderating the anticipatory pain response. They do not have significant hemodynamic
effects unless given at large bolus doses or in a hypovolemic patient. Midazolam is suggested in
acute events due to its rapid onset of action but short half life. Traditionally, lorazepam has been
used when sedation is anticipated for longer periods of time. Although the use of this medication
is widespread, recent data suggest that they can be a risk factor in the development of delirium. 7
In addition if large doses of lorazepam are need to achieve the desired level of sedation, there is
the risk of developing a hyperosmolar anion gap acidosis secondary to propylene glycol.8 Due to
these effects, lorazepam will likely play a lesser role in future guidelines and its use might be
limited to the treatment and prevention of alcohol withdrawal.
Propofol is an intravenous, non-barbiturate general anesthetic agent. However, sedative and
hypnotic properties can be demonstrated at lower doses. Compared with benzodiazepines,
propofol produces a similar degree of amnesia at “equi-sedative” doses in volunteer. Propofol
has a rapid onset and short duration of sedation once discontinued. Adverse effects most
commonly seen with propofol include hypotension, bradycardia, and pain upon peripheral venous
injection. The hypotension is dose related and is more frequent after bolus dose administration.
Propofol is available as an emulsion in a phospholipid vehicle, which provides 1.1 kcal/mL from
fat and should be counted as a caloric source. Elevation of pancreatic enzymes has been
reported during prolonged infusions of propofol . Pancreatitis has been reported following
anesthesia with propofol, although a causal relationship has not been established .In addition,
prolong infusions of high doses of propofol may cause the propofol infusion syndrome. This
syndrome is characterized by severe metabolic acidosis, rhabdomyolysis, acute renal failure,
refractory myocardial failure, and hyperlipidemia.9 Although the cases reported have occurred
mostly in children, there are some reports in adult patients. Propofol requires a dedicated I
catheter when administered as a continuous infusion because of the potential for drug
incompatibility and infection.
Although [alpha]-2 agonists are not mentioned in the SCCM algorithm, they can play an important
role in sedation in the ICU. Clonidine has been used to augment the effects of general
anesthetics and narcotics and to treat drug withdrawal syndromes in the ICU. 1 Placement of a
clonidine patch with 48 hrs of enteric administration (while the patch “kicks in”) is the method
used at our institution. The more selective [alpha]-2 agonist, dexmedetomidine, was recently
approved for use as a sedative with analgesic-sparing activity for short-term use (<24 hours) in
patients receiving mechanical ventilation. 1 More recent reports have suggested that longer
utilization is possible without increasing deleterious effects. Patients remain sedated when
20
ASCCA Residents Guide to the ICU
undisturbed, but arouse readily with gentle stimulation. Dexmetomidine reduces concurrent
analgesic and sedative requirements and produces anxiolytic effects comparable to
benzodiazepines. Reports of use of this drug in substance abuse patients to minimize
sympathetic “surges” in traumatic brain injury have been reported. Patients receiving
dexmedetomidine may develop bradycardia and hypotension, especially in the presence of
intravascular volume depletion or high sympathetic tone. Cost and the optimal duration of therapy
limit the role of this new agent in the sedation of ICU patients.
Haloperidol is one of the most common drugs used to treat patients with delirium.1 Haloperidol is
commonly given via intermittent i.v. injection. The optimal dose and regimen of haloperidol have
not been well defined. Haloperidol has a long half-life (18–54 hours) and loading regimens are
used to achieve a rapid response in acutely delirious patients. A loading regimen starting with a 2mg dose, followed by repeated doses (double the previous dose) every 15–20 minutes while
agitation persists, has been described. High doses of haloperidol (>400 mg per day) have been
reported, but QT prolongation may result.
The successful use of dexmedetomidine in delirium has been reported. Antipsychotic
medications such as quetipine are used to aid with sedation in the ICU. They can be provided via
the enteric route. However, caution should be use as these drugs can have multiple drug
interactions. It is interesting to mention that the first step in the treatment of delirium is nonpharmacological and includes but it is not limited to reorientation, removal of catheters and
restrains and change of environment.
In summary, sedation in the ICU remains a challenge. A goal oriented approach is recommended.
Therefore assessment of the patient and differentiation between “agitation” and hyperactive
delirium and “properly sedated” and hypoactive delirium is most important. The SCCM Guidelines
provides us with an algorithm that can be adapted to each institution.
This chapter is a revision of the original chapter authored by Debra Szeluga, M.D. and Alan Lisbon, M.D.
REFERENCES:
1. Jacobi J, Fraser GL, Coursin DB, et al. Clinical practice guidelines for the sustained use of sedatives and
analgesics in the critically ill adult. Critical Care Medicine. 2002; 30: 142-56
2. Sessler CN, Gosnell MS, Grap MJ, et al. The Richmond Agitation-Sedation Scale: Validation and
Reliability in Adult Intensive Care Unit Patients. Am J Respir Crit Care Med. 2002; 166: 1338-44.
3. Ely EW, Truman B, Shintani A, et al. Monitoring Sedation Status Over Time in ICU Patients: Reliability
and Validity of the Richmond Agitation-Sedation Scale (RASS). JAMA. Jun 2003; 289: 2983-91.
4. Ely EW, Shintani A, Truman B, et al Delirium as a Predictor of Mortality in Mechanically Ventilated
Patients in the Intensive Care Unit. JAMA. April 14, 2004;291: 1753-62.
5. Ely EW, Inouye SK, Bernard GR, et al. Delirium in Mechanically Ventilated Patients: Validation and
Reliability of the Confusion Assessment Method for the Intensive Care Unit (CAM-ICU). JAMA. Dec
2001;286: 2703-10.
6. Ely EW, Truman B, Manzi DJ, et al Consciousness monitoring in ventilated patients: bispectral EEG
monitors arousal not delirium. Intensive Care Med. 2004; 30(8): 1537-43.
7. Pandharipande P, Shintani A, Peterson J, et al. Lorazepam Is an Independent Risk Factor for Transitioning
to Delerium in Intensive Care Unit Patients. Anesthesiology. 2006 ;104(1):21-6.
8. Arroliga AC, Shehab N, McCarthy K, et al Relationship of continuous infusion lorazepam to serum
propylene glycol concentration in critically ill adults. Crit Care Med. 2004 Aug;32(8):1709-14.
9. Sabsovich I, Rehman Z, Yunen J, Coritsidis G. Propofol infusion syndrome: a case of increasing morbidity
with traumatic brain injury. Am J Crit Care. 2007 Jan;16(1):82-5.
10. Casserly B, O'Mahony E, Timm E, et al Propofol infusion syndrome: an unusual cause of renal failure.
Am J Kidney Dis. 2004 Dec;44(6): 98-101.
11. Patil N, Weinhouse GL. Randomized control trial of dexmetomidine to treat acute intensive care unit
delirium Crit Care Med 2006; 34 (12) (Abstract Supplement:) p. A7.
21
ASCCA Residents Guide to the ICU
QUESTIONS:
6.1.
A.
B.
C.
D.
E.
After 2 weeks of sedation with IV midazolam and fentanyl by continuous infusion, the patient
fails to wake up following 6 hours off both medications. The appropriate next intervention
should be:
Emergency CT scan to look for neurologic injury
Naloxone drip at 50 micrograms/min.
Flumazenil bolus
Continue monitoring for 24 hours
EEG to rule out non-convulsive status epilepticus.
A.
B.
C.
D.
E.
An agitated patient receiving bolus narcotics and benzodiazepines may:
Be in pain.
Be withdrawing from benzodiazepines.
Have an organic cause for agitation.
May be hypercarbic.
All of the above may be true.
A.
B.
C.
D.
E.
The use of potent inhalational anesthesia in the ICU:
Is indicated for some cases of severe bronchospasm.
Can be safely done by an ICU nurse or respiratory therapist.
Should never be done.
Causes an unacceptable incidence of renal failure after 24 hours.
Requires monitoring for atmospheric pollution.
A.
B.
C.
D.
E.
A patient is given a bolus of 500 mcg of fentanyl and ventilation by mask is found to be difficult.
The most likely cause of this problem is:
The patient has an upper airway obstruction.
The patient has developed non-convulsive status from the fentanyl.
The patient has the rigid chest syndrome from rapid opioid administration.
The patientКјs false teeth have occluded his pharynx.
The person providing mask ventilation has not enough experience managing the airway.
A.
B.
C.
D.
E.
Propofol
Should rarely be used in the ICU due to myocardial depression.
Results in rapid awakening after long-term infusion.
Is difficult to manage due to its short duration.
Must be used with a narcotic.
Results in increased costs and should not be used.
A.
B.
C.
D.
E.
A patient is receiving fentanyl and midazolam for sedation in the treatment room while a
peritoneal dialysis catheter is being placed. Which of the following can be said if this is done
safely?
If the patient is receiving 2 L/min oxygen by nasal cannula, pulse oximetry is not needed.
An ECG monitor should be attached to the patient.
An anesthesiologist must administer these potent medications.
A defibrillator must be in the room with the patient.
A second person will be monitoring the patient without other duties.
6.2.
6.3.
6.4.
6.5.
6.6.
22
ASCCA Residents Guide to the ICU
7. Analgesia in the ICU
Ruben J. Azocar, M.D.
A 25-year-old male sustained severe blunt chest trauma after colliding with a pole at high speed
in his motorcycle. He is awake and alert and initial CXR reveals multiple left rib fractures and a
fractured sternum. A chest CT confirms those findings with the addition of an area of
consolidation in the upper left lung that correlates with a pulmonary contusion. He is admitted to
the ICU for observation and he complains of severe pain on his left chest. On exam, he has
tachycardia, hypertension and tachypnea.
Pain is commonly encountered by intensive care unit patients. Pain is not limited to trauma or
surgery but is associated with common procedures in the ICU such suctioning and turning. The
presence of an endotracheal tube or other cannulae or catheters is also a source of pain. Puntillo
reported that procedural pain was often described as sharp, stinging, stabbing, shooting, and
awful and than less than 20% of patients received opiates before procedures.1 Furthermore,
patients with chronic pain syndromes may have their baseline pain aggravated by their position or
immobility while in the ICU.
Unrelieved pain may lead to physical and psychic suffering. Puntillo reported that 63% of patient
in the ICU rated their pain as being moderate to severe in intensity.2 Schelling et al reported that
40% of post ICU ARDS patients recall having pain while in the ICU and that those patients have a
higher frequency of chronic pain issues when compared with control. Furthermore, these ICU
patients had a higher post-traumatic stress disorder scores than controls.3
Pain is frequently the primary cause of agitation in the ICU patient and physiologic changes
associated with pain may adversely affect patients’ ICU course. Inadequate analgesia often
results in increased sympathetic tone, causing tachycardia, hypertension and increased systemic
vascular resistance. These changes may lead to increased myocardial oxygen demand and
myocardial ischemia if oxygen delivery cannot match the demands. Postoperative pulmonary
complications are also more likely in patients with inadequate pain control. "Splinting," or limiting
the depth of respiration as a result of pain after surgery causes a reduction in tidal volume and
functional residual capacity.
Paired with inhibition of cough, postoperative pulmonary
complications such as hypoxemia, atelectasis and respiratory infection are more likely. Patients
with head injury will experience increased intracranial pressure and therefore increase their
morbidity/mortality risk.
Deep venous thrombosis and pulmonary embolism may develop
secondary to immobility because of pain or the resulting need for increased sedation in patients
with poor pain control. Other consequences of inadequate analgesia are increased secretion of
certain hormones such as antidiuretic hormone, prolactin, and cortisol that can negatively impact
the patient’s metabolic state.
Similar to sedation, appropriate pain management starts with a systematic and consistent
assessment of pain. The most reliable indicator of pain, however, is the patient’s self-report.
Patients are often able to communicate the location, characteristics, aggravating/alleviating
factors, and the intensity of their pain. There are also a number of scales (visual analog scale,
verbal rating scale, and numeric rating scale) which have been employed in the assessment of
pain.
A Visual Analogue Scale (VAS) is a measurement instrument that tries to measure a
characteristic or attitude that is believed to range across a continuum of values and cannot easily
be directly measured. For instance, the amount of pain that a patient feels ranges across a
continuum from none to an extreme amount of pain. From the patient's perspective, this spectrum
appears continuous. The purpose of this scale is to capture this idea of an underlying continuum.
Operationally, a VAS is usually a horizontal line, 100 mm in length, anchored by word descriptors
at each end, as illustrated in Fig. 7.1
23
ASCCA Residents Guide to the ICU
No Pain________________________________________________________Worse Pain Ever
Figure 7.1: The patient marks on the line the point that they feel represents their perception of their current
state. The VAS score is determined by measuring in millimeters from the left hand end of the line to the
point that the patient marks.
A Verbal Rating Scale (VRS), involves asking the patient to verbally rate his or her level of
perceived pain intensity on a numerical scale from 0 to 10, with the zero representing one
extreme (e.g. no pain) and the 10 representing the other extreme (e.g. the worst pain possible).
The Numeric Rating Scale (NRS) is a zero to ten point scale that patients are given. The patient
then chooses the point on the scale that most correlates with their pain level. Figure 2 is an
example of a numeric rating scale.
Figure 7.2: Numeric Pain Rating Scale.
Unfortunately, the assessment of pain using the above scales becomes difficult in the critically ill
patient who is sedated, anesthetized, or receiving neuromuscular blockade. Therefore, based on
current guidelines from the SCCM Sedation and Analgesia Task Force,4 pain assessment and
response to therapy should be performed using a scale appropriate to the patient population. For
patients who cannot verbally communicate their pain level, subjective observation of the patient
may be useful. Patients experiencing pain often make certain facial expressions, body
movements, and posturing. Physiologic changes such as increased heart rate, blood pressure,
and respiratory rate are also good indicators of pain. However it is important that this assessment
is consistent across the observers to develop a standardized pain evaluation tool. The goal is to
have a tool that is easy replicable, considers the patient cultural differences and that can be
performed quickly.
Agents for Analgesia
Almost all critically ill patients, particularly those receiving mechanical ventilation, will receive an
analgesic agent. A wide variety of pharmacological agents are available for analgesia and, while
recommendations have been made regarding the �best’ analgesic for ICU patients, practice varies
widely between and within ICUs. The choice of agent can be based on many factors, including the
relative needs for analgesia, the pharmacodynamics and pharmacokinetics of the drug in
question, route and ease of administration, the tolerance profile and the cost. While many studies
have been conducted comparing the effectiveness of various agents, there is relatively little
published information on variations in analgesic drug use among units or across national and
international boundaries.
Currently, the mainstay of analgesic therapy in the ICU patient is intravenous opioids. Opioids
work by stimulating opiate receptors in the spinal cord and CNS, and possibly in the periphery.
Multiple subtypes have now been identified with some receptors more involved in analgesia than
24
ASCCA Residents Guide to the ICU
others. Different agents have been shown to stimulate these receptors to varying degrees. The
agents are classified as naturally occurring opium alkaloids, such as morphine and codeine,
semisynthetic derivatives, such as oxycodone, hydromorphone, and heroin, and the synthetic
opioids, such as fentanyl, meperidine, and methadone.
Side effects of opioids are varied, with the degree of side effects the limiting factor in therapy.
The dose-limiting side effect of most opioids is respiratory depression. However, large amounts
of opioids can be safely administered to critically ill patients who are mechanically ventilated if
weaning from mechanical intubation is not being considered.
Morphine
Morphine has both analgesic and sedative effects and can be conveniently administered by bolus
dose or continuous infusion in patients in whom prolonged analgesia and sedation are necessary.
Morphine can cause hypotension either as a consequence of drug-induced bradycardia or
histamine release. Morphine induced bradycardia occurs as a result of stimulation of the vagal
nucleus in the medulla. Systemic vasodilation as a consequence of morphine-induced histamine
release can be seen with doses as low as 0.1 to 0.2 mg per kg. This is more pronounced in
volume depleted patients. The depressant effect on the medullary respiratory center is difficult to
predict. Initially, respiratory rate slows more significantly than tidal volume is reduced, but as the
morphine dose is increased, a profound depression of total minute ventilation can result.
Morphine is metabolized by the liver, and its metabolites are normally excreted in the urine. The
elimination half-life of one of its main metabolites, morphine-6-glucuronide, is 1.5 to 4.0 hours, but
in patients with impaired renal function, drug effect can persist for more than 24 hours.
Fentanyl
Fentanyl is a synthetic opioid with high lipid solubility, which permits rapid penetration into the
central nervous system and equilibration between blood and brain drug levels. Fentanyl has a
more rapid onset of action and shorter duration of action than morphine and is 75 to 200 times
more potent. Because of its lipid solubility, it can accumulate in fat stores and, after repeated
doses, may have a significantly longer elimination half-life. When administered in the ICU by
continuous infusion, it offers little advantage over morphine. It is a useful drug for short-term,
painful ICU procedures, particularly when used in combination with a benzodiazepine. Systemic
hemodynamic effects resulting from vasodilatation are less significant with fentanyl than with
morphine, because less histamine is released. Fentanyl can be also delivered via a patch and
this route of administration may be helpful in stopping the infusion or intermittent intravenous
doses in patients with chronic pain or opioid dependency.
Meperidine
Meperidine is a synthetic opioid analgesic that is one-eighth as potent as morphine when
administered parenterally. It is metabolized in the liver to normeperidine, an active metabolite.
Meperidine has an elimination half-life of three to four hours. Normeperidine has an elimination
half-life of 15 to 30 hours. Often, toxic levels of normeperidine can be responsible for alterations
in a patient's mental status and can cause seizures. Therefore, meperidine should be used with
caution in patients with decreased hepatic or renal function and in patients who require repeated
doses. As a consequence, it is not widely used in ICU patients.
In summary, opioids can be titrated to a desired effect for analgesia. The most effective route of
administration in the ICU is continuous intravenous infusion. Repeated bolus injections of opioids
tend to result in peaks and troughs of oversedation and inadequate analgesia. Instead, it is more
efficient to supplement baseline continuous infusion with short-acting formulations for severe pain
associated with procedures in the ICU.
Non-steroidal anti-inflammatory drugs (NSAIDs)
Non-steroidal anti-inflammatory drugs (NSAIDs) are valuable in post-operative pain management
by decreasing the need for opioids, and thus probably decreasing the potential for opioid-related
complications such as post-operative ileus and respiratory depression. NSAIDs work by blocking
the cyclooxygenase enzymes (COX 1 & COX 2) thereby reducing prostaglandin production
throughout the body. As a consequence, ongoing inflammation, pain, and fever are reduced.
Since the prostaglandins that protect the stomach and support platelets and blood clotting also
25
ASCCA Residents Guide to the ICU
are reduced, NSAIDs can cause ulcers in the stomach and promote bleeding. Ketorolac is a very
potent NSAID and is used for moderately severe pain in combination with a narcotic. It can be
administered parenterally and, thus, is used often in the ICU setting. However, ketorolac causes
gastric ulcers more frequently than any other NSAID and is, therefore, not used for more than five
days. The Cox 2 inhibitors seemed promising in the ICU in both oral and intravenous form but
recent controversies with regard to side effects have retarded their use in the ICU.
Ketamine, clonidine, dexmedetomidine
The NMDA-receptor antagonist ketamine, and the alpha-2-receptor agonists clonidine and
dexmedetomidine have analgesic and sedative effects and can be used as adjuncts to opioids.
Using such therapy, the amount of opioids can be decreased and some of their side effects
minimized. Ketamine has been found to share or at least have synergetic effects with the opioid
receptors at the spinal cord level. It may be administered IV. Clonidine and dexmedetomidine
both produce a synergistic anti-nocioceptive effects. Clonidine can be given via PO, IV and
intrathecal/epidural/regional routes. It is unclear if topical administration (i.e. CatapresВ® patch)
has the same effects. Dexmedetomidine is given via IV, intrathecal/epidural or nasal routes.
EPIDURAL ANALGESIA
In recent years, the administration of local anesthetics and opioids in the epidural space has
become an increasingly popular method of post-operative analgesia. It has been shown to
facilitate the early return of function (ambulation, coughing, deep breathing, and enteral
alimentation) in patients after abdominal surgery. In the trauma population, patients with multiple
rib fractures and/or pulmonary contusion benefit the most from this type of therapy by
minimization of splinting allowing the patient to take larger breaths and to cough and by allowing
the institution of aggressive pulmonary toilet. A continuous infusion of bupivacaine and morphine
(in low doses), and placement of the epidural catheter close to the dermatomes of surgery, will
allow the use of the lowest doses of each medication, maximize the concentration of analgesics
at the site where the nociceptive fibers enter the spinal cord, and minimize the side-effects and
complication of the therapy. In addition, epidural local anesthetics probably have two beneficial
effects: they decrease the need for opioids (which reduces the gut-slowing opioid side-effects)
and may have direct stimulatory effects on the bowel by decreasing the sympathetic tone.
Although epidural analgesia has many benefits, it is not without complications. Thus, the decision
to place an epidural catheter must take into account the potential benefits of epidural analgesia
and the inherent risks. Some of the complications include but are not limited to: accidental dural
puncture, intravenous injection, hypotension, high spinal, and epidural abscess and hematoma
formation. The appearance of cerebrospinal fluid through either the epidural needle or catheter
signifies a dural puncture. If this occurs, the epidural may be attempted again at a different
interspace. The risk of postdural puncture headache is increased because of the large needle
size used. Large intravascular injections can be prevented by pre-injection aspiration of the
epidural catheter and the use of a 3 to 4 ml test dose of local anesthetic with epinephrine.
Sometimes, catheters migrate intravascularly hours after the initial placement in the epidural
space. Inadvertent intrathecal injection of an epidural dose of local anesthetic results in a high
spinal block. The use of a test dose before injection of the initial local anesthetic bolus,
incremental injections, and aspiration of the catheter before re-injection aid detection of an
intrathecally placed catheter and reduce the risk of a high or total spinal.
Although vasodilation and subsequent hypotension produced by the sympathetic blockade is
possible, it is dependent on the level of sympathectomy and the patient’s volume status.
Increasing the patient preload and administering the local anesthetics in small doses while
monitoring the hemodynamics should prevent large variations in the blood pressure.
Epidurals are generally avoided in patients who are grossly septic, have a localized infection at
the site of needle placement, or are therapeutically anticoagulated in order to prevent epidural
abscess and hematoma formation. These medications are not limited to anticoagulants such
unfractionated heparin, low molecular weight heparin or warfarin but anti-platelet drugs such as
clopidogrel as well. The American Society of Regional Anesthesia guidelines should be followed
in those cases if the benefits of epidural anesthesia will significantly improve the patient’s
outcome.
26
ASCCA Residents Guide to the ICU
In summary, the data showing a clear improvement in clinical outcome and/or decreased cost
with use of post-operative epidural analgesia are limited to a few categories of patients and
surgical procedures. However, the studies where a definite advantage has been shown are quite
helpful to the clinician because they suggest areas where post-operative epidural analgesia is
clearly indicated after the risk and benefits of this technique have been considered. Several
studies of patients randomized to receive either general anesthesia plus intravenous opioids
versus epidural anesthesia plus post-operative epidural analgesia have shown better outcomes
when epidural anesthesia/analgesia is used. For instance, for peripheral vascular surgery, the
use of an epidural was associated with improved graft survival. Cancer patients undergoing
major abdominal surgery who had epidurals showed decreased incidence of tachycardia,
myocardial ischemia, myocardial infarction, shorter time to tracheal extubation, earlier discharge
from ICU, earlier discharge from hospital, and lowered total cost than those who did not. Patients
undergoing partial colectomy and/or radical retropubic prostatectomy had decreased time to
recovery of bowel function and lowered hospital charges when epidurals were used for analgesia
than not.
This chapter is a revision of the original chapter authored by Reginald Neymour, M.D. and Thomas M. Fuhrman, M.D.
REFERENCES
1. Puntillo KA, White C, Morris AB,et al Patients' perceptions and responses to procedural pain: results
from Thunder Project II.Am J Crit Care. 2001;10:238-51.
2. Puntillo KA. Pain experiences of intensive care unit patients. Heart Lung. 1990;19:526-33
3. Schelling G, Stoll C, Haller M, et al Health-related quality of life and post-traumatic stress disorder in
survivors of the acute respiratory distress syndrome. Crit Care Med. 1998;26 :651-9.
4. Jacobi J, Fraser GL, Coursin DB,et al ; Task Force of the American College of Critical Care Medicine
(ACCM) of the Society of Critical Care Medicine (SCCM), American Society of Health-System
Pharmacists (ASHP), American College of Chest Physicians. Clinical practice guidelines for the
sustained use of sedatives and analgesics in the critically ill adult. Crit Care Med. 2002;30:119-41.
5. Giovannoni MP, Ghelardini C, Vergelli C, et al. Alpha-2-agonists as analgesic agents. Med Res Rev.
2009 Mar; 29(2):339-68.
QUESTIONS
7.1.
A.
B.
C.
D.
Epidural analgesia:
Is devoid of complications
Requires a careful assessment of the patient
Is not useful in trauma patients
Limits the patient ability to cough
A.
B.
C.
D.
Opioids produce all of the following effects EXCEPT:
Respiratory depression
Ileus
Amnesia
Pruritus
A.
B.
C.
D.
Ketamine:
Is an alpha-2 receptor antagonist
Has no role in the treatment of pain in the ICU
Depresses the respiratory drive
Can serve as an adjunct for pain management in the ICU
7.2.
7.3.
27
ASCCA Residents Guide to the ICU
7.4.
A.
B.
C.
D.
Characteristics of pain in the ICU include the following EXCEPT:
It can be iatrogenic in nature
It is easy to assess
May lead to PTSD in ICU survivors
It is one of the common causes of agitation
A.
B.
C.
D.
Deleterious effects of pain include the following EXCEPT:
Elevated oxygen consumption
Agitation
Urinary retention
Atelectasis
7.5.
28
ASCCA Residents Guide to the ICU
8. Neuromuscular Blockade
Ruben J. Azocar, M.D.
A 22-year-old male victim involved in a rollover accident, vomits and aspirates upon attempted
intubation in the field. In addition he has a right sided flail chest. Upon arrival to the ICU he is
severely hypoxemic and different modes of ventilation fail to improve his oxygenation. A decision
to prone the patient in a rotating bed is made and he is paralyzed with vecuronium to minimize
potential interference with the ventilator and or sudden movements while in the prone position.
He remains paralyzed for 7 days and as his oxygenation improves, the paralysis is stopped.
Steroids are started to minimize the fibrosis of late phase ARDS. However, 24 hours after
stopping the paralytic agent, his physical exam reveals generalized muscle weakness and
minimal adductor pollicis brevis muscle response to post-tetanic stimulation.
The decision to treat a patient in the intensive care unit (ICU) with neuromuscular blocking agents
(NMBAs) (for reasons other than the placement of an endotracheal tube) is a difficult one that is
guided more commonly by individual practitioner preference than by standards based on
evidence-based medicine. Furthermore, neuromuscular blockade can lead to deleterious effects
in patients. The following discussion is based in the SCCM Clinical Practice Guidelines for
sustained neuromuscular blockade in the adult critically ill patient. 1
Indications for NMBAs for an adult patient in an ICU frequently include management of difficult
mechanical ventilation, management of increased ICP, treatment of muscle spasms (tetanus),
and decreasing oxygen consumption when all other means have been tried without success. 1
Neuromuscular blocking agents can be divided by their chemical structure into either
benzylisoquinolines or steroidal nucleus group; by their duration of action into ultrashort, short,
intermediate or long; and by the type of block into depolarizing or nondepolarizing. (Table 8.1)
The choice of a neuromuscular blocking agent for sustained paralysis in the intensive care unit
must be guided by an understanding of the drug’s properties and by a cost-benefit analysis.2 The
practitioner should be familiar with the relevant pharmacologic features for each neuromuscular
blocking agent including but not limited to structure, ED95, usual bolus dose, infusion dose range,
onset time, duration and recovery times, major route of elimination, activity of major metabolites,
autonomic interactions, and other major side effects.
Ultra-Short
Depolarizing
Succinylcholine
NonDepolarizing
Rapacuronium*
Short
Intermediate
Long
-
-
-
Mivacurium*
Atracurium
cis-Atracurium
Rocuronium
Vecuronium
d-Tubocurarine
Doxacurium
Metocurine
Pancuronium
Pipecuronium
Table 8.1 Neuromuscular blocking agents by mechanism of action and duration. *no longer available.
Many drugs interact with the actions of neuromuscular blocking agents. Some drugs such as
aminoglycoside antibiotics and magnesium potentiate their actions and could be involved in the
pathophysiology of muscle weakness after the paralysis course. Others such as phenytoin may
antagonize their effects. (Table 8.2)
29
ASCCA Residents Guide to the ICU
Drug
Effect on Neuromuscular Blockade
Antibiotics
(aminoglycosides, vancomycin,
clindamycin, tetracycline, bacitracin)
Potentiation
Anticonvulsants
(phenytoin, carbamazepine)
Resistance
Antidysrhythmics
(lidocaine, calcium channel blockers,
quinidine, procainamide)
Potentiation
Antihypertensives
(trimethaphan, nitroglycerine)
Potentiation - pancuronium only for
nitroglycerine
Dantrolene
Potentiation
Furosemide
(dose related)
under 10 mcg/kg - Potentiation
1-4 mg/kg - Resistance
Ketamine
Potentiation
Local anesthetics
Potentiation
Magnesium Sulfate
Potentiation
Steroids
Potentiation
Table 8.2 Drugs that interact with neuromuscular blocking agents
In terms of drug selection, the majority of patients in an ICU who are prescribed an NMBA can be
managed effectively with pancuronium. For patients for whom vagolysis is contraindicated (e.g.,
those with cardiovascular disease or tachycardic), NMBAs other than pancuronium may be used.
(Vecuronium is recommended by the SCCM guidelines.) Because of their unique metabolism
(Hoffman’s metabolism) cis-atracurium or atracurium are recommended for patients with
significant hepatic or renal disease. 1
The need for NMB should be reassessed daily and administration should be stopped as early as
possible. Monitoring is also important. Monitoring the depth of neuromuscular blockade may allow
use of the lowest NMBA dose and may minimize adverse events. 1 Despite the lack of evidence
that monitoring prevents adverse effects and the lack of a standardized method of monitoring,
assessment of the depth of neuromuscular blockade in ICU patients is recommended. By using
the Train of Four (TOF) monitoring, the rate of infusion can be adjusted to achieve one or two
twitches. However, the TOF might be difficult to assess in a swollen and or diaphoretic patient.
Another option is to stop the paralytic on daily basis (“drug holiday”) to reassess the need for the
continuing the drug and assure rapid recovery from the drug effects.
The complications of the neuromuscular blockade in the Intensive Care Unit can be categorized
as short term (ventilator disconnect, accidental extubation and hyperkalemia with succinylcholine
use), midterm (edema, hypostasis, bedsores, venous thrombosis) and long term (muscle
weakness). Additionally, the possibility of awareness and its deleterious consequences is also
present. Awareness and muscle weakness will be discussed in some detail.
30
ASCCA Residents Guide to the ICU
NMBA are not sedatives therefore, before initiating neuromuscular blockade, patients should
receive appropriate sedative and analgesic drugs to provide adequate sedation and analgesia
and to avoid awareness and probably post-traumatic stress disorder (PTSD). The use of depth of
anesthesia monitors such as the Sedline В® (Hospira) or the BIS В® (Aspect Medical) to assure a
proper level of sedation while patients are paralyzed seems logical although there is not enough
evidence to support their use. 3
Although thought to be multifactorial, skeletal muscle weakness in ICU patients is closely related
to the use of NMBA.4,5 A confusing list of names and syndromes, including acute quadriplegic
myopathy syndrome (AQMS), floppy man syndrome, critical illness polyneuropathy (CIP), acute
myopathy of intensive care, rapidly evolving myopathy, acute myopathy with selective lysis of
myosin filaments, acute steroid myopathy, and prolonged neurogenic weakness have been
reported.1 However, the term critical illness myopathy (CIM) has become the more frequently
used name for this entity. Although the exact mechanisms of this problem are unknown, the
common factor seems to be damage to the neuromuscular membrane. The use of steroid based
NMBA (pancuronium, vecuronium) and/or the concomitant use of steroids have been clearly
associated in the development of muscle weakness. However benzylisoquinolinium based NMB
(cis-atracurium, atracurium) have also been reported to produce this problem. In addition, muscle
weakness has been associated with the persistent presence of the drug or its metabolites in
plasma. Alterations in clearance mechanisms such as hepatic and/or renal failure can contribute
to this problem. Unintentional overdose, drug interactions (Table 2), electrolyte imbalances
(hypermagnesemia, hypophosphatemia), acidosis, hypothermia and underlying muscle disorders
(polyneuropathy of critical illness, myasthenia gravis) are also involved.
In summary, NMB should be use as a last resource in the critically ill since the effects on the
neuromuscular membrane can lead to severe complications. If used, monitoring to minimize the
dosing and reassessment of the indications for NMB need to be done continuously. If prolonged
use is needed “drug holidays” should be instituted.
This chapter is a revision of the original chapter authored by Richard Prielipp, M.D.
READING LIST:
1. Murray MJ, Cowen J, DeBlock H, et al. Task Force of the American College of Critical Care Medicine
(ACCM) of the Society of Critical Care Medicine (SCCM), American Society of Health-System
Pharmacists, American College of Chest Physicians. Clinical practice guidelines for sustained
neuromuscular blockade in the adult critically ill patients. Crit Care Med. 2002;30:142-56.
2. Ortega R, Azocar R: Neuromuscular Blocking Agents in Surgical Intensive Care Medicine. Edited by
O’Donell J ,Nacul F. Kluwer Academic Publishers, Norwood MA 2001 pp163-180
3. Arbour R. Impact of bispectral index monitoring on sedation and outcomes in critically ill adults: a case
series. Crit Care Nurs Clin North Am. 2006;18:227-41
4. Murray MJ, Brull SJ, Bolton CF. Brief review: Nondepolarizing neuromuscular blocking drugs and
critical illness myopathy. Can J Anaesth. 2006;53:1148-56.
5. Friedrich O:Critical illness myopathy: what is happening? Curr Opin Clin Nutr Metab Care. 2006 ;9:403-
QUESTIONS:
8.1.
A.
B.
C.
D.
All the the following drugs potentiate the effects of NMBA except:
Quinidine
Phenytoin
Amino glycosides
Magnesium
31
ASCCA Residents Guide to the ICU
8.2.
A.
B.
C.
D.
Which of the following NMB drugs are NOT appropriately “paired,” based their duration of
action?
Vecuronium: intermediate
Mivacurium: short
Pipecuronium: short
Rocuronium: intermediate
A.
B.
C.
D.
Complications of use of nondepolarizing NMB drugs in the ICU may include all of the following
EXCEPT:
Awareness
Venous Thrombosis
Myopathy
Hyperkalemia
A.
B.
C.
D.
Potential causes of prolonged weakness in the ICU patient after NMB drug administration
include all the following EXCEPT:
Electrolyte disturbances
Drug or drug metabolites accumulation
Propofol interaction with the neuromuscular junction
Concomitant use of steroids
A.
B.
C.
D.
The following are true statements in relation to monitoring NMBA effects, EXCEPT:
The train of four is always easily obtainable and measurable
Monitoring is important in the prevention of myopathy
Drug holidays may be needed to assess effects and length of recovery
Adjustment of the dose to 2 twitches with the TOF is suggested
8.3.
8.4.
8.5.
32
ASCCA Residents Guide to the ICU
9. Acid-base Balance
Nathanael Slater, D.O., Khaldoun Faris, M.D.
A 68-year-old male with a history of hypertension and severe COPD is now post-operative day 5 from an
exploratory laparotomy for perforated diverticulitis. The surgery was complicated by significant blood loss
with subsequent fluid resuscitation, necessitating a prolonged intubation in the ICU. He is now tolerating
diuresis via furosemide infusion and weaning of the ventilator. He is currently awake and cooperative on
minimal pressure support ventilation with an FiO2 of 0.5. His morning labs reveal a chemistry panel as
follows: Na 138 mEq/L, K 4.0 mEq/L, Cl 104 mEq/L, HCO3- 34 mEq/L, BUN 27, Creat 1.2 mg/dl. His
ABG on rounds this morning has the following values: pH 7.41, PaCO2 54 mm Hg, PaO2 85 mm Hg,
HCO3- 33 mEq/L, BE +8 mEq/L.
Acid-Base disturbances are almost universal in critical ill patients. Clinicians must be prepared to
anticipate, diagnose and treat such conditions. A patient’s natural buffers and compensatory mechanisms
for mild acid-base perturbations are often overwhelmed by such insults as infection, trauma, hypovolemia,
and toxic ingestion. Furthermore, such insults are less tolerated by those with co-morbidities that directly
affect our natural compensatory mechanisms, such as chronic pulmonary disease, renal failure, or
malnutrition. The following outline will delineate the fundamental acid-base topics that need to be
understood by every critical care clinician. It is important to recognize that many patients present with
complex and mixed acid-base abnormalities. For example, the patient described above could be
presumed to have a severe metabolic alkalosis based on his chemistry panel and present treatment with
a furosemide infusion. However, based on his history of severe COPD and review of his blood gas
revealing a normal pH, it is clear that he has a mixed disorder, a metabolic alkalosis and concomitant
respiratory acidosis. Therefore, it may be ill-advised to treat the metabolic alkalosis by forcing the
excretion of bicarbonate in the setting of chronic pulmonary disease.
Normal acid-base balance
I. Henderson-Hasselbach Equation
A. Classic Equation: pH = pK + log [HCO3-] / [CO2-]
1. This equation expresses how pH changes in response to alterations in the concentration of
HCO3- and H+.
2. The pH is inversely proportional to [H+] and directly proportional to [HCO3-].
3. To use this equation clinically, multiply the pCO2 by 0.03 (solubility constant of CO2).
B. Modified Equation: [H+] x [HCO3-] = K x pCO2
II. Non-renal Buffering Mechanisms:
A. Hemoglobin: The buffering effect is via a histidine side group which has a pK very close to
physiologic pH, making it an excellent buffer.
B. Plasma Proteins: The most abundant protein in this group is albumin. The histidine side group
binds free H+, as it does with hemoglobin.
C. Phosphate (H3PO4): The pK of the diprotic form (H2PO4-) is 6.8, making it a good buffer at
physiologic pH. However, the buffering capacity is small compared to hemoglobin and albumin.
III. Bicarbonate buffer system: CO2 + H2O
H+ + HCO3-
A. Addition of H+ drives the equation to the left, producing CO2 that is removed by the lungs. If the
additional CO2 was not removed, the pH would fall dramatically.
B. For example, adding 5 mmol of HCl to the equation would drop the [HCO3-] by 5 mmol and
produce 5 mmol of CO2, producing a pH of 6.6.
1. pH = pK + log [HCO3-]/[CO2]
2. The physiologic levels are HCO3-: 24 mmol/L, CO2: 1.2 mmol/L, and pK: 6.1
3. The addition of the HCl will lead to: pH = 6.1 + log (19/6.2) = 6.6
4. If the CO2 is removed from the equation by ventilation, then the effect is ameliorated and
results in a pH compatible with life: pH = 6.1 + log (19/1.2) = 7.3
C. The effect of adding acid to this buffer system is blunted even further by the renal absorption of
HCO3- from the glomerular filtrate.
33
ASCCA Residents Guide to the ICU
IV. Renal Buffering Mechanism:
A. Bicarbonate reabsorption: Approximately 4320 mmol of HCO3- are filtered from the blood every
24 hours in to the lumen of Boman’s capsule. Roughly 85% of HCO3- is reabsorbed in the
proximal convoluted tubule, 15% in the collecting tubule.
1. H+ is secreted via a Na+ - H+ exchanger into the lumen of the proximal tubule. It reacts with
filtered HCO3- via carbonic anhydrase to produce H2O and CO2. CO2 passively diffuses
across the cell membrane from the tubular lumen, reacts with H2O and carbonic
anhydrase intracellularly, and produces HCO3-. The Na+ - HCO3- cotransporter moves
three bicarbonate ions across the cell membrane into the bloodstream along with one Na+
ion.
2. Excretion of H+ into the tubule via the Na+ - H+ exchanger is limited by the pH of the filtrate.
Excretion stops below a pH of 6.5-6.8
3. Reabsorption is similar in the distal tubule, however HCO3- is moved into the bloodstream in
exchange for Cl- instead of being cotransported with Na+. H+ is pumped into the lumen by
a proton pump, and can move against a steeper gradient (down to pH of 4.5).
B. Excretion of titratable acids: Phosphate, urate, creatinine, and beta-hydroxybutyrate are filtered
by the kidney into the lumen of Bowman’s capsule. H+ that is transported across the cell
membrane in the proximal and distal tubules is bound by these non-bicarbonate buffers.
Production of HCO3- intracellularly is the same as with bicarbonate reabsorption.
C. Formation and excretion of ammonium: Two molecules of ammonium (NH4+) are produced by
the metabolism of glutamine to alpha-ketoglutarate. Alpha-ketoglutarate is a divalent anion
and consumes two protons as it is metabolized to glucose or CO2 and H2O. NH4+ is excreted
via a Na+ - H+ exchanger into the lumen by substituting for H+.
V. Renal Response to Acid-base Imbalance
A. Metabolic acidosis: Acute compensation is accomplished primarily by increasing respiratory
removal of CO2. Additional buffering is accomplished by hemoglobin, plasma proteins, and
phosphate, and by carbonate from bones. The kidneys reabsorb 100% of filtered HCO3- and
generate additional HCO3- by excretion of titratable acids and formation of ammonium.
B. Respiratory acidosis: The kidneys respond to chronic elevation of pCO2 by increasing production
of ammonium. A response to an acute respiratory acidosis takes at least 72 hours to begin.
C. Metabolic alkalosis: The kidneys’ ability to adjust their resorptive threshold of HCO3- typically
allows for elimination of excess bicarbonate without accumulation. A condition that leads to
metabolic alkalosis must reset the threshold for reabsorption in addition to producing excess
amounts of HCO3-. Three inter-related mechanisms can lead to metabolic alkalosis in the
critically ill patient.
1. Hypokalemia causes intracellular K+ to move across the cell membrane into plasma. In order
to maintain electrical neutrality, an HCO3- ion diffuses out as well. The decrease in
intracellular HCO3- is seen by the cell as an acidosis, stimulating an increase in renal
resorption of bicarbonate.
2. Aldosterone secretion in response to a decrease in effective circulating volume increases Na+
reabsorption from the tubular lumen. This leads H+ to move into the lumen along an
electrical gradient, facilitating HCO3- reabsorption. The electrical gradient also pulls K+
into the lumen, maintaining hypokalemia.
3. Volume depletion combines the effects of the above processes. Aldosterone secretion is
stimulated, leading to HCO3- reabsorption and hypokalemia. Hypokalemia can be
exacerbated by diuretic use leading to further HCO3- reabsorption. Diuretics cause loss of
fluid, not loss of bicarbonate (unless using a carbonic anhydrase inhibitor). This loss of
fluid volume while maintaining the same amount of HCO3- leads to a “contraction”
alkalosis.
D. Respiratory alkalosis: Respiratory alkalosis is typically due to an acute physiology leading to
hyperventilation. Renal compensation in this situation is to decrease the amount of HCO3that is reabsorbed.
34
ASCCA Residents Guide to the ICU
VI. Role of Electrolytes
A. Renal management of potassium levels:
1. Approximately 756 mEq/day of K+ is filtered into Boman’s capsule. 65% is reabsorbed in the
proximal convoluted tubule and 25-30% in the thick ascending loop of Henle. The distal
convoluted tubule and collecting tubule can either absorb or secrete K+ depending on
serum [K+].
2. Principal cells in the distal and collecting tubules are responsible for secreting K+. A Na+ - K+
ATP-ase system pumps K+ into the cell in exchange for Na+. K+ then passively diffuses
into the tubular lumen. Secretion of K+ is stimulated by aldosterone that is produced in
response to hyperkalemia.
B. Sodium and chloride reabsorption:
1. Na+ and Cl- are actively reabsorbed throughout the length of the nephron with the exclusion
of the thin descending limb of the loop of Henle. The majority of reabsorption occurs in
the proximal convoluted tubule, followed by the thick ascending limb of the loop of Henle.
This is essential for establishing the “countercurrent” mechanism by which the kidney
produces concentrated urine.
2. The primary mechanism for regulating plasma osmolarity via [Na+] is secretion of ADH
(vasopressin) by the posterior pituitary. The distal convoluted tubule and collecting tubule
become permeable to H2O in the presence of ADH, producing concentrated urine.
3. Diabetes insipidus
a) Central diabetes insipidus is caused by inability to secrete ADH.
b) Nephrogenic diabetes insipidus occurs when the kidneys do not respond to ADH.
c) Both result in excretion of large volumes of dilute urine, rapidly leading to dehydration if
water deficits are not replaced.
Clinical Disorders
I.
Metabolic acidosis
A. Metabolic acidosis refers to any condition in which serum pH and HCO3- are abnormally low.
Severe metabolic acidosis can be defined as a serum HCO3- ≤ 8 mmol/L. Differential diagnosis of
metabolic acidosis is produced based on analysis of the anion gap.
B. Anion gap: The anion gap quantifies unmeasured anions in the blood. It is the mathematical
difference of serum Na+ minus the sum of HCO3- and Cl-. Addition of acid releases H+ that binds
one molecule of HCO3- and adds the conjugate base of that acid to the quantity of unmeasured
anions, increasing the anion gap. The anion gap can be misleading in critically ill patients due to
the presence of hypoalbuminemia.
1. Anion gap = [Na+] – ([HCO3-] + [Cl-])
2. Normal gap = 6-12
3. Low albumin correction: corrected AG = calculated AG + (2.5*(normal albumin - measured
albumin)
C. Elevated anion gap acidosis differential diagnosis (MUDPILES)
1. Methanol
2. Uremia (acute renal failure)
3. Diabetic/Alcoholic ketoacidosis
4. Paraldehyde
5. Ischemia (bowel, limb)
6. Lactic acidosis (sepsis, circulatory/respiratory failure)
7. Ethylene glycol
8. Salicylates
D. Normal anion gap acidosis differential diagnosis
1. GI losses (vomiting, diarrhea)
2. Acute renal failure (occasionally)
3. Renal tubular acidosis
a) Type I: defective distal H+ secretion
b) Type II: decreased proximal reabsorption of HCO3E. Base Deficit
1. The base deficit is calculated as part of the standard ABG panel. It represents the amount of
base that would be required to titrate one liter of whole blood to a pH of 7.4.
a) Normal: +2 to -2
35
ASCCA Residents Guide to the ICU
b) Mild elevation: -2 to -5
c) Moderate elevation: -6 to -14
d)Severe elevation: ≤ -15
2. In bleeding patients, the base deficit can indicate the degree of tissue acidosis due to
inadequate oxygen delivery and therefore may reflect overall volume status. Rapid correction
of the base deficit has been correlated with better outcomes in bleeding patients as opposed
to more gradual correction.
F. Consequences of severe acidosis
1. Cardiovascular
a) Decreased contractility
b) Arteriolar dilatation, venoconstriction, and centralization of blood volume
c) Increased pulmonary vascular resistance (worsens with hypoxemia)
d) Decrease in cardiac output, arterial blood pressure, hepatic and renal blood flow
e)Lower threshold for reentrant arrhythmias and ventricular fibrillation. Poor cardiovascular
response to catecholamines
2. Respiratory
a) Hyperventilation
b) Respiratory muscle fatigue and loss of strength
c) Dyspnea
3. Metabolic
a) Increase in metabolic demand
b) Insulin resistance
c) Inhibition of anaerobic glycolysis
d) Reduced ATP synthesis
e) Hyperkalemia
f) Increase in protein degradation
g) Lack of efficacy of sedatives, narcotics, and supportive medications
4. Cerebral
a) Inhibition of metabolism and cell-volume regulation
b)Delirium, obtundation, and coma
G. Management:
1. Treatment needs to be directed at the underlying mechanism of disease. For example, with
ethylene glycol ingestion, removing accumulated glycolic acid with dialysis, stopping the
metabolism of ethylene glycol with fomepizole, and aggressive fluid resuscitation are the
mainstay of therapy. Alkali therapy is reserved for pH below 7.2. The goal of alkali therapy is
to ameliorate the severe drop in [HCO3-] and thereby prevent or reverse the consequences of
acidosis listed above.
a) HCO3- deficit = Lean body mass (0.5)(24 – [HCO3-])
(1) 1 ampule has 50 mmol each of Na+ and HCO3(2)To replete give 1.5 ampules of NaHCO3 in 1/2NS or 3 ampules of NaHCO3 in D5W.
b) Consequences:
(1) Buffering of H+ releases CO2, which causes an additional acid load on the already
acidemic tissue (H+ + HCO3- в†’ CO2 + H2O).
(2) Excessive NaHCO3 may cause an “overshoot” alkalosis.
(3)NaHCO3 can stimulate 6-phosphofructokinase activity and the production of organic
acids which limits the utility of NaHCO3 therapy.
c) Alternative agents: Limited data available on efficacy
(1) Carbicarb
(2) THAM
H. Commonly encountered problems in the ICU:
1. Lactic acidosis
a) Lactate is produced by anaerobic metabolism and is cleared from the body by the liver
and, to a lesser extent, the kidneys. Oxygen is required for metabolism of lactate to
glucose or its breakdown to CO2 and H2O. Consequently, any clinical setting which
decreases delivery of oxygen to tissues increases the production of lactate and
decreases its consumption. Treatment is aimed at reversing the underlying cause.
b) Common causes:
(1) Acute hypoxemia
36
ASCCA Residents Guide to the ICU
(2) Shock (septic, cardiogenic, hemorrhagic)
(3) Impaired oxidative metabolism
(4) Impaired gluconeogenesis
(5)Thiamine deficiency
2. Diabetic ketoacidosis
a) Lack of insulin or insulin resistance due to infection or another stressor prevents glucose
from entering cells and storage and metabolism of glucose. Glucagon secretion is
increased and gluconeogenesis is stimulated in response to the lack of energy source for
cellular metabolism. Lipolysis releases fatty acids into the bloodstream which the liver
metabolizes into ketone bodies.
b) Diagnosis: Serum glucose of 400-800 mg/dL and concomitant metabolic acidosis
(1) [Na+] is lowered by 1.6 points per 100 mg/dL rise in serum glucose above normal; if
[Na+] is high, the patient is severely dehydrated.
(2) Patients will initially present with hyperkalemia, but will become hypokalemic with
treatment. K+ is shifted out of cells by acidosis and lost with osmotic diuresis.
Hypovolemia and vomiting can worsen hypokalemia.
(3) Serum HCO3- is lost due to buffering of ketone bodies.
c) Treatment: Aggressive fluid resuscitation and insulin therapy are the mainstay of
treatment.
(1) Hydrate with 0.9% saline initially, check electrolytes frequently, and add potassium to
fluid once serum levels begin to decrease.
(2) Begin insulin therapy only after fluids have been started. Initial rate of infusion should
be 5-10 units per hour, increasing as needed every hour to gradually drop blood
glucose. Serum glucose should not drop below 200 mg/dL within the first 24 hours of
therapy.
(3) Glucose should be added to IV fluid once serum glucose is less than 200 mg/dL.
(4) Insulin should never be stopped completely. It is required for metabolism of
circulating ketone bodies and proper glucose metabolism.
(5) Switch to subcutaneous dosing of insulin once the patient’s HCO3- levels have
returned to normal. Stop insulin infusion 2-3 hours later.
II. Metabolic Alkalosis
A. Metabolic alkalosis refers to abnormally elevated serum pH and HCO3-. Severe alkalemia is
defined as serum HCO3- ≥ 45 mmol/L
1. Types:
a) Volume responsive (also known as chloride-responsive): The more common of the two
types of alkalemia encountered in the ICU, and usually due to vomiting, gastric drainage,
diuretic use, or administration of NaHCO3-.
(1) Urinary chloride < 15 mmol/L or sodium < 10-15 mmol/L
(2) Bulimia/anorexia and surreptitious diuretic use; diuretic abuse will falsely elevate
urine Na and Cl
b)Volume resistant: Alkalosis in the absence of vomiting, gastric drainage, diuretic use, or
exogenous NaHCO3- should point to a volume/chloride resistant alkalosis.
(1) Urinary chloride > 35 mmol/L
(2) Bartter’s Syndrome, primary hyper aldosteronism and corticosteroid use
2. Consequences of severe alkalosis
a) Cardiovascular
(1) Arteriolar constriction
(2) Decrease in coronary blood flow
(3) Decrease in threshold for angina
(4) Lower threshold for refractory cardiac arrhythmias
b) Respiratory
(1) Hypoventilation
(2) Hypercapnia and hypoxemia
c) Metabolic
(1) Stimulation of anaerobic glycolysis and organic acid production
(2) Hypokalemia
(3) Decreased plasma ionized calcium concentration
37
ASCCA Residents Guide to the ICU
3.
(4) Hypomagnesemia
(5) Hypophosphatemia
d) Cerebral
(1) Decreased cerebral blood flow
(2) Tetany, seizure, lethargy, delirium, and stupor
Management
a) Volume responsive metabolic alkalosis:
(1) Stop gastric suctioning
(2) Treat nausea to prevent vomiting
(3) H2-blockers to decrease gastric acid secretion
(4) Decrease dose of, or discontinue diuretics
(a) In patients who continue to need diuresis, acetazolamide, Diamox, can be
given; either 250mg or 500mg q8 hours x3 doses then assess effect.
(b) Add spironolactone to prevent K+ loss and to minimize hypokalemic contribution
to alkalosis.
(5) Treat chloride deficit with saline solution
(6) Replete potassium deficit if present
(7) For refractory alkalosis consider
(a) Infusion HCL
(b) Dialysis with a chloride-rich solution
b) Volume resistant:
(1) Consider tapering corticosteroids
(2) Primary hyperaldosteronism
(a) Surgery if necessary
(b) Spironolactone (blocks effects of aldosterone)
(c) Potassium supplementation
(3) Bartter syndrome:
(a) Na+, K+ replacement
(b) Spironolactone
(c) ACE inhibitor
III. Respiratory acidosis:
A. Respiratory acidosis refers to abnormally low pH with an elevated PaCO2. Hypercapnia is due to
either inadequate ventilation or increased CO2 production. The acidosis can be either acute or
chronic.
B. Etiology:
1. Hypoventilation
a) Central depression
(1) Drugs such as opioids and benzodiazepines
(2) CNS disorder
b) Thoracic
(1) Neuromuscular disorders
(2) Obesity
(3) Airway obstruction
c) Iatrogenic
(1) Inadequate ventilator setting
(2) Permissive hypercapnia (acute lung injury and status asthmaticus)
2. Increased CO2 production
a) Hypermetabolic states
(1) Fever
(2) Sepsis
(3) Malignant hyperthermia
(4) Excess dextrose in TPN
C.Management
1. Treat the underlying disorder
2. Non-invasive ventilation, BiPAP
3. Intubation and mechanical ventilation
4. Tolerate hypercapnia in certain conditions
38
ASCCA Residents Guide to the ICU
Acute lung injury: Tidal volume is lowered in order to minimize baro/volutrauma and
further lung injury
b)Status asthmaticus: Respiratory rate is decreased to allow longer expiratory time.
IV. Respiratory Alkalosis
A. Respiratory alkalosis refers to abnormally decreased PaCO2, resulting from either excess
ventilation or decreased CO2 production. The alkalosis can either be acute or chronic.
B. Etiology
1. Hyperventilation
a) Central
(1) Drugs such as salicylates (combined metabolic acidosis and respiratory alkalosis)
and catecholamines
(2) CNS disorders
(3) Hypoxemia
(4) Sepsis
(5) Pregnancy
b) Iatrogenic, during mechanical ventilation
2. Decreased CO2 production
a) Hypothermia
b) Pharmacological paralysis
C. Management
1. Treat the underlying disorder
2. Adjust ventilatory setting by decreasing tidal volume, respiratory rate or both
3. In ventilated patients, the centrally mediated respiratory alkalosis does not respond to the
adjustment of the ventilatory setting. If the respiratory alkalosis is severe, sedatives may be
used.
a)
This chapter is an updated revision of the original chapter authored by Barry A. Shapiro, M.D.
READING LIST:
1. Androgue HJ, Madias NE. Management of Life-Threatening Acid-Base Disorders (Part I). N Engl J Med
1998;338:26-34
2. Androgue HJ, Madias NE. Management of Life-Threatening Acid-Base Disorders (Part II). N Engl J Med
1998;338:107-11
This two part article is an excellent review of common acid-base emergencies and their treatments.
3. Corey HE. Fundamental principles of acid-base physiology. Crit Care Med 2005; 9:184-92
The review describes new and more accurate approaches to the diagnosis of acid-base disturbances in the critically
ill patient. It includes calculation of strong-ion gap and strong-ion difference.
4. Kellum JA. Disorders of acid-base balance. Crit Care Med 2007; 35:2630-36
This review addresses the need for a different approach to acid-base disturbances in the critically ill patient. It
advocates for the use of strong-ion difference, pCO2, and total weak acid concentration as the basis for diagnosis.
5. Clive MC. Metabolic acidosis and metabolic alkalosis. In: Irwin RS, Rippe JM, eds. Manual of intensive care
medicine, 4th ed. Lippincott Williams and Wilkins, 2006; 341-46
This chapter provides a quick review of metabolic acidosis and alkalosis.
QUESTIONS:
9.1.
A.
B.
C.
D.
The most common acid-base abnormality associated with aspirin/salicylate toxicity is the following:
Metabolic Acidosis
Respiratory Acidosis
Metabolic Alkalosis
Respiratory Alkalosis
39
ASCCA Residents Guide to the ICU
9.2.
A.
B.
C.
D.
E.
According to Winters formula, the expected PaCO2 (plus or minus 2) observed in a patient with an acute
metabolic acidosis and HCO3- of 18 mmol/L is:
28 mmHg
30 mmHg
35 mmHg
40 mmHg
45 mmHg
A.
B.
C.
D.
E.
Acute alkalemia would have which of the following effects:
Increased cerebral blood flow
Decreased affinity of oxygen for hemoglobin
Reduces calcium binding to proteins
All of the above
None of the above
A.
B.
C.
D.
E.
Which of the following effects can be seen with Alkali (Bicarbonate) administration:
Hypernatremia
Hypercapnia
Volume overload
Cerebrospinal acidosis
All of the above
9.3.
9.4.
9.5.
A.
B.
C.
D.
Which of the following answers is true regarding the use of the “anion gap” to narrow the differential
diagnosis for a metabolic acidosis in the intensive care unit:
Hypoalbuminemia decreases the “normal” anion gap whereas hypophosphatemia increases the
“normal” anion gap.
Hypoalbuminemia and hypophosphatemia both decrease the “normal” anion gap range observed.
Hypoalbuminemia and hypophosphatemia both increase the “normal” anion gap range observed.
Albumin and phosphorous levels have no effect on the anion gap.
40
ASCCA Residents Guide to the ICU
10. Metabolic and Endocrine Abnormalities
Ruben J Azocar, M.D.
In the past 10 years the concept of endocrine dysfunction in the ICU has grown significantly. This case
presentation will highlight some of the newer developments in this area. Management of classical
endocrine related issues such as Diabetic Ketoacidosis, thyroid storm and others are well described in
textbooks and will not be discussed in this review.
70-year-old male with PMHx of CAD (s/p CABG in 99), CHF, HTN, CRI (Cr 1.8 mg% at baseline ), PVD
(s/p Bilateral LE bypasses), presented to outside hospital with mental status changes, DKA, hypotension,
elevated liver enzymes, gangrenous left foot. Initial management included intravenous fluids, insulin, and
empiric antibiotic therapy. A pulmonary artery catheter revealed a “septic physiology” picture.
Norepinephrine to keep MAP> 70 mm Hg was started. However the patient developed A-Fib with a rapid
ventricular response and his Troponin I was 11.7 ng/ml. Additionally he became oliguric and his creatinine
was now 4.1 mg%. He remained hypotensive . A decision to start Vasopressin at a rate of 0.04 units/min
was made started. Several hours later norepinephrine was discontinued and his urine output improved.
What were the data used to support this decision?
Landry, 1 compared the vasopressin levels in patient with septic shock vs. patient in cardiogenic shock. In
septic shock patients vasopressin levels were 3.1+/- 1 pg/ml. But in patients in cardiogenic shock the
levels were 22.7+/-2.1 pg/ml. To discard the possibility that this difference was secondary to an increased
catabolism, an infusion of vasopressin was started. The plasma concentration levels corresponded to the
predicted values discarding an accelerated breakdown of the vasopressin. Furthermore, the infusion of
0.04 units/min increased systemic blood pressure and the peripheral vascular resistance. In patients
receiving vasopressin, withdrawal resulted in hypotension. These data suggest the possibility of lower
levels of vasopressin in the patients with septic shock. Malay et found similar results.2
In a randomized double placebo control comparing vasopressin 0.04 u/min vs. placebo in patients with
septic shock, they found that SBP and SVR increased. They also noted that all vasopressin patients
survived the 24 hour trial and had other pressors discontinued. Traditionally vasopressin had been used
as a last resource for esophageal varices bleeding. Concerns regarding splanchnic circulation and
coronary circulation have been raised with the use of vasopressin despite the fact that the doses used for
esophageal varices were much higher. A small trial by Patel et al, tried to determine whether short-term
vasopressin infusion in patients with severe septic shock has a vasopressor sparing effect while
maintaining hemodynamic stability and adequate end-organ perfusion.3 The patients were randomized
to a double-blinded 4-h infusion of either norepinephrine (n = 11) or vasopressin (n = 13), and open-label
vasopressors were titrated to maintain blood pressure. To assess end-organ perfusion, urine output and
creatinine clearance were measured to assess the renal system, gastric mucosal carbon dioxide tension
was obtained to assess gut ischemia , and EKG with ST segment position were measured to discard
myocardial ischemia. The norepinephrine group went from a median pre-study norepinephrine infusion of
20.0 mcg/min to a blinded infusion of 17.0 mcg/min at 4h. Vasopressin group went from a median prestudy NE infusion of 25.0 mcg/min to 5.3 mcg/min at 4 h. Mean arterial pressure and cardiac index were
maintained in both groups. Urine output did not change in the norepinephrine group (25 to 15 ml/h) but
increased substantially in the vasopressin group (32.5 to 65 ml/h). The creatinine clearance did not
change in the NE group but increased by 75% in the vasopressin group. The gastric mucosal carbon
dioxide tension and EKG/ ST segments did not change significantly in either group. The authors conclude
that short-term vasopressin infusion spared conventional vasopressor use and improved some measures
of renal function in patients with severe septic shock.
The cause of this vasopressin deficiency is not well recognized but a possibility is the depletion of
vasopressin depletion in the neurohypophysis during septic shock.
Sharshar reported 3 patients with septic shock in whom the high signal intensity of the posterior lobe of
the pituitary gland on T1-weighted MRI images was absent suggesting that in septic shock,
41
ASCCA Residents Guide to the ICU
inappropriately low plasma levels of vasopressin are at least partly related to a depletion of vasopressin
stores in the neurohypophysis.4
Finally, the role of vasopressin in the therapy of septic shock remains ill-defined. The vasopressin in
septic shock trial (VASST) is a recently completed study to determine the effectiveness of Vasopressin
compared to Norepinephrine in reducing 28-day mortality11. No survival benefit to the use of vasopressin
was found compared to norepinephrine.
As the patient’s hemodynamics improved, he was taken to the operating room on hospital day 3 for
amputation. In the immediate post operative period persistent septic physiology, metabolic acidosis, and
hyperglycemia continued. Strict glucose control was instituted and antibiotics were started.
Glycemic control
Traditionally, hyperglycemia was considered as a “physiologic” response to stress due to stimulation of
neuroendocrine axis. Blood glucose levels in the 200 mg% range were accepted in the ICU most of the
time despite data suggesting deleterious effects.
A landmark study by van den Berghe revolutionized the perspective of glucose control in the ICU.5 In a
prospective, randomized, controlled study, 1548 patients were randomly assigned to either intensive
insulin therapy defined as maintenance of blood glucose at a level between 80 - 110 mg/dl or
conventional treatment defined as maintenance of glucose at a level between 180 -200 mg/dl. Intensive
insulin therapy reduced mortality from 8.0 % with conventional treatment to 4.6 % in the intense insulin
therapy group. The benefit of intensive insulin therapy was mostly attributable to its effect on mortality
among patients who remained in the ICU for more than five days (20.2% with conventional treatment vs.
10.6 % with intensive insulin therapy). In addition, the greatest reduction in mortality involved deaths due
to multiple-organ failure with a proven septic focus. Finally, intensive insulin therapy also reduced overall
in-hospital mortality by 34%, bloodstream infections by 46%, ARF requiring dialysis or hemofiltration by
41%, the median number of RBC transfusions by 50%, critical-illness polyneuropathy by 4%, decreased
ICU length of stay and increased ventilator free days. A subsequent study in a medical intensive care unit
by the same group found that intensive Insulin therapy significantly reduced morbidity but not mortality in
patients admitted to medical ICU (40% vs. 37.3%).6 However it is interesting to note that if the patients
were in the ICU longer than 3 days, the mortality was greater in the conventional group. Similar to the
prior study, less new kidney injury and accelerated weaning from ventilator and shorter ICU and hospital
stays were noted in the intensive therapy group.
Recently concerns about the incidence and impact of hypoglycemia in patients in tight glucose control
protocols have been raised. Hypoglycemia seems to have a negative impact in patient outcomes.
Protocols directed to avoid hyperglycemia while preventing hypoglycemia need to be instituted in the ICU.
POD #1: Clinically better, but persistent low BP and SVR. Also hypothermic. Abdominal CT negative.
TFTs: TSH 6.9 IU/ml, T3 69 ng/dl, T4 4.2 mcg/dl, rT3 2.29 ng/dl.
Sick Euthyroid syndrome
Any abnormal thyroid function test (TFT) in the setting of non-thyroidal illness (NTI) or systemic stress has
been denoted Sick Euthyroid Syndrome. Most recently the preferred term is “non-thyroidal illness
syndrome” (NTIS) as some patients may, in fact, be hypothyroid. This entity is more frequent in seriously
or critically ill patients and the abnormalities can be detected as soon as two hours after onset of stress.
Different clinical scenarios have been described in the syndrome 7. A low T3 level is the most commonly
encountered abnormality in non-thyroidal illness. In these cases, T3 levels fall rapidly within 30 minutes to
42
ASCCA Residents Guide to the ICU
24 hours of onset of illness, while rT3 levels increase. The finding of increased rT3 levels differentiates
this syndrome from true hypothyroidism, in which rT3, T3, and T4 levels would most likely all be low.
Thyrotropin (TSH) and total and free T4 levels are usually normal. Low T3 syndrome is thought to be due
to a decrease in T4 conversion to T3 by the hepatic deiodinase system. An exception is patients with
advanced AIDS, in whom baseline rT3 levels are already low.
Low T3 and low T4 may be encountered in patients who are moderately ill. This syndrome has been
described in up to 20% of patients treated in intensive care units. Free thyroid hormone levels are usually
normal but may be decreased in patients treated with dopamine hydrochloride or corticosteroids. TSH
levels may also be normal to low. The mechanism involved may be a deficiency in TBG, which leads to
low total thyroid hormone levels. Another possibility is the presence of a thyroid hormone-binding inhibitor,
which lowers total thyroid hormone levels. Another presentation is the low TSH, low T3, and low T4. This
presentation is the most severe non-thyroidal illness. Although most of these patients have TSH levels at
the low end of normal, TSH may be undetectable in some, even when third-generation assays are used.
The finding of low TSH and low total T4 and T3 levels suggests altered pituitary or hypothalamic
responsiveness to circulating thyroid hormone levels. During the recovery period, TSH levels return to
normal or may even rise transiently before returning to normal. Finally an elevated T4 might be
encountered in this condition - the total T4 level is elevated, TSH level is normal or elevated, and T3 level
is normal or high. It may be seen in primary biliary cirrhosis and acute and chronic active hepatitis, in
which TBG synthesis and release are increased. Drugs such as amiodarone hydrochloride, propranolol
and iodinated contrast agents also elevate T4 levels by inhibiting peripheral conversion of T4 to T3.
However, there is not data to support that thyroid replacement is of any benefit and that resolution of the
primary cause is the key of treatment.
The patient now developed worsening respiratory status and remains septic. A baseline cortisol level was
low and there was inadequate response to the stimulation test.
Under normal stress conditions, a normal adrenal response would increase the cortisol levels. However,
in the late 1990’s there was renewed interest in the role of relative adrenal insufficiency in the critically ill,
particularly the septic patient. It was noted that some septic patients requiring vasopressor support after
proper resuscitation would improve their hemodynamics with steroid given as bolus or as infusion. There
was also noted that most of these patient would either have a lower than expected baseline cortisone
level or an abnormal response to the stimulation test. Most of the studies were relatively small. Annane et
al published the landmark study on this topic.8 The goal of the study was to assess whether low doses of
corticosteroids improve 28-day survival in patients with septic shock and relative adrenal insufficiency.
This was a double blind placebo controlled, parallel-group trial performed in 19 French ICU’s. Three
hundred patients who fulfilled usual criteria for septic shock were enrolled after undergoing a short
corticotropin test. Patients were randomly assigned to receive either hydrocortisone (50-mg IV bolus q6
hours) and fludrocortisone (50 mcg tablet po daily) (n = 151) or matching placebos (n = 149) for 7 days.
After the corticotropin test they were also classified as responders or non-responders. They identified
229 non-responders to the corticotropin test, 115 received a placebo, and 114 receive the steroids. Of the
70 responders to the corticotropin test, 34 were assigned to the placebo group and 36 to the steroids
group.
In nonresponders, there were 73 deaths (63%) in the placebo group and 60 deaths (53%) in the
corticosteroid group (P =.02). Vasopressin therapy was withdrawn within 28 days in 46 patients (40%) in
the placebo group and in 65 patients (57%) in the corticosteroid group (P =.001). There was no
significant difference between groups in responders and adverse events rates were similar in the 2
groups. The authors concluded that a 7-day treatment with low doses of hydrocortisone and
fludrocortisone significantly reduced the risk of death in patients with septic shock and relative adrenal
insufficiency without increasing adverse events.
Recently the term critical illness related corticosteroid insufficiency (CIRCI) has been introduced to
characterize better this syndrome and avoid the confusing terminology of absolute vs. relative adrenal
43
ASCCA Residents Guide to the ICU
insufficiency and to imply that the problem might exist at multiple levels and not solely related to the
adrenal gland.
A point of controversy in reference with the etiology of the critically CIRCI is the use of etomidate (11ОІhydroxylase inhibitor thereby interfering with steroidal genesis). There is evidence that the incidence of
CIRCI in patients with septic shock is increased when the stimulation test is performed after the
administration of etomidate.9 Therefore when managing a hemodynamically unstable or potentially
unstable critically ill patients, the risk/benefits of using etomidate should be weighed and the ICU team
warned that CIRCI could manifest in this patient.
CONCLUSION:
Similar to other organ systems the adrenal system is affected during critical illness. Recent literature has
brought new insights in how to manage this key system.
This chapter is a revision of the original chapter authored by Thomas E. Shaughnessy, M.D
REFERENCES
1. Landry DW, Levin HR, Gallant EM, Ashton RC Jr, Seo S, D'Alessandro D, Oz MC, Oliver JA. Vasopressin
deficiency contributes to the vasodilation of septic shock. Circulation. 1997 ;95:1122-5
2. Malay MB, Ashton RC Jr, Landry DW, Townsend RN. Low-dose vasopressin in the treatment of vasodilatory septic
shock. J Trauma. 1999 ;47:699-703; discussion 703-5
3. Patel BM, Chittock DR, Russell JA, Walley KR. Beneficial effects of short-term vasopressin infusion during severe
septic shock. Anesthesiology. 2002;96:576-82
4. Sharshar T, Carlier R, Blanchard A, et al Depletion of neurohypophyseal content of vasopressin in septic shock.
Crit Care Med. 2002 Mar;30:497-500
5. van den Berghe G, Wouters P, Weekers F, et al Intensive insulin therapy in the critically ill patients. N Engl J Med.
2001 Nov 8;345(19):1359-67
6. Van den Berghe G, Wilmer A, Hermans G, et al. Intensive insulin therapy in the medical ICU.N Engl J Med. 2006
Feb 2;354(5):449-61
7. Camacho PM, Dwarkanathan AA: Sick euthyroid syndrome: What to do when thyroid function tests are
abnormal in critically ill patients. Postgraduate Medicine on line: http://www.postgradmed.com/issues/
1999/04_99/camacho.htm
8. Annane D, Sebille V, Charpentier C, et al: Effect of treatment with low doses of hydrocortisone and
fludrocortisone on mortality in patients with septic shock. JAMA. 2002 Aug 21;288(7):862-71
9. Mohammad Z, Afessa B, Finkielman JD. The incidence of relative adrenal insufficiency in patients with septic
shock after the administration of etomidate. Crit Care. 2006;10:R105
10.Marik PE, Pastores SM, Annane D, et al. Recommendations for the diagnosis and management of corticosteroid
insufficiency in critically ill adult patients: consensus statements from an international task force by the American
College of Critical Care Medicine. Crit Care Med. 2008 36:1937-49.
11.Russell JA, Walley KR, Singer J, et al. Vasopressin versus norepinephrine infusion in patients with septic shock. N
Engl J Med. 2008 358:877-87
QUESTIONS:
10.1.
A.
B.
C.
D.
During septic shock , the following is not indicated:
Tight glucose control
Consideration of vasopressin for refractory hypotension
Initiation of steroids early on the course despite hemodynamics
Early goal resuscitation
10.2.
A.
B.
C.
D.
In the medical ICU the following is true in relation to aggressive insulin therapy EXCEPT:
Improves mortality
Decreases ICU stay
Increases ventilator free days
Decrease hospital stay length
44
ASCCA Residents Guide to the ICU
10.3.
A.
B.
C.
D.
In a septic patient the use of vasopressin had been proven to:
Be superior than norepinephrine
Be safe for up to 3 weeks
Produce myocardial ischemia
Improve renal function
10.4.
A.
B.
C.
D.
Euthyroid sick syndrome is:
Only present in patients with preexisting thyroid disease
Always manifested by an elevation of T4
Never manifested by low T3
Related in most cases to a deficiency in the conversion from T4 to T3
10.5.
A.
B.
C.
D.
Critical illness related corticosteroid deficiency (CIRCI) is:
Rarely characterized by hypotension
Managed by treating all septic patients with steroids
Secondary to corticosteroid depletion at the adrenal gland level
Potentially caused by etomidate
45
ASCCA Residents Guide to the ICU
11. Fluid Management
Ronald W. Pauldine, M.D
A 31-year-old man falls 25 feet from a scaffold. He is unresponsive at the scene. Paramedics intubate
him and begin a normal saline infusion. While in the emergency department, he withdraws to noxious
stimuli with all four extremities but does not open his eyes. His pupils are equal, mid-size, and sluggish to
react to light. A peritoneal lavage is grossly bloody. His vital signs are blood pressure = 85/30 mm Hg,
heart rate = 130/minute, and respiratory rate = 25/minute. His hematocrit is 27%. En route to the
operating room for an exploratory laparotomy, a head CT scan reveals diffuse, small, punctate
intracerebral hemorrhages. The neurosurgeon would like to place an intracranial pressure monitor as
soon as his coagulation studies are known.
The goal of fluid management and hemodynamic monitoring is to ensure that oxygen delivery meets the
metabolic needs of the body. Many factors including critical illness, trauma and surgery affect the volume,
composition and distribution kinetics of fluids within the intracellular and extracellular compartments. In
the OR and in the ICU, the anesthesiologist is constantly monitoring and manipulating the various
components of oxygen delivery: cardiac output, hemoglobin concentration, and arterial oxygen saturation.
A basic understanding of the distribution and composition of fluids along with an appreciation of the
factors commonly causing shifts in these important variables is vital to caring for the critically ill.
I.
Composition of Body Fluids and Water Balance
A. Body Fluid Compartments
1. Total Body Water (TBW) = 0.6 X Lean Body Weight
a) TBW is the sum of: Intracellular Volume (ICV) = 0.4 X Lean Body Wt. plus
b) Extracellular Volume (ECV) = 0.2 X Lean Body Weight
c) Approximately 20% of the ECV is composed of Plasma Volume (PV)
d) The remaining 80% represents the Interstitial Fluid (IF)
B. Fate of Infused Fluids
1. The quantity of infused fluid initially remaining in the plasma volume can be estimated by
considering the distribution volume of the infused fluid:
PV increment = (volume infused X normal PV)/distribution volume
a) Example: For a given volume of D5W with a distribution volume approximating TBW,
roughly 7% of total volume infused will remain in the plasma volume. For a given volume
of lactated Ringers or normal saline with a distribution volume approximating the ECF
roughly 1/3 will remain.
b) This calculated increase is transient as it does not consider ongoing fluid excretion.
C. Adaptive Responses to Hypovolemia
1. Decreased GFR
2. Redistribution of Renal Blood Flow
3. ADH
4. Renin- Angiotensin-Aldosterone
5. Altered secretion of ANP
D. Factors Affecting Distribution of Fluid Between the ECV and ICV
1. Osmolality
a) Defined as number of somatically active particles per kg of solvent
b) Can be measured directly in lab
c) Can be estimated by:
Osmolality = ([Na+] X 2.0) + (Glucose/18) + (BUN/2.3)
2. Starling Equation
Q = kA[(Pcapillary - Pinterstitial) + Пѓ(ПЂcapillary - ПЂinterstitial)]
where:
Q = fluid filtration
k = capillary filtration coefficient
A = the area of the capillary membrane
P = hydrostatic pressure
46
ASCCA Residents Guide to the ICU
Пѓ = reflection coefficient
ПЂ = colloid osmotic pressure
II. Assessment of Fluid Requirements
Fluid requirements and potential advantages for fluids of varying compositions are determined by
the underlying clinical scenario. Frequent considerations include hypovolemia from any cause
including blood loss or dehydration, major burn injury, relative hypovolemia in situations such as
sepsis, and conditions resulting in increased intracranial pressure. Even for experienced
clinicians, clinical estimation of the intravascular volume can be difficult.
A. Formulas for maintenance, volume depletion, blood loss, “third space” fluid loss, trauma, and
burns
B. Recognize trends while making serial laboratory and clinical assessments
C. Role of invasive hemodynamic monitoring and echocardiography
D. Targets of resuscitation
1. Systemic Oxygen Delivery and Consumption
a) May be employed as a target for resuscitation, however data are unclear concerning
utility
b) Determinants of oxygen transport
(1) DO2 = Cardiac Index x CaO2 x 10 = SV x HR x [Hb x SaO2 x 1.34) + (0.003 x PaO2)]
(2) Regulation of oxygen transport
(3) Relationship between oxygen delivery, oxygen consumption, and oxygen extraction
(4) Flow-independent and flow-dependent oxygen consumption
(5) Supranormal oxygen delivery and impact on outcome
c) Clinical signs of adequacy of perfusion:
Assessment Methods
Noninvasive
Capillary refill
BUN
Laboratory
Skin color
BUN:serum creatinine
Invasive Monitoring
CVP, pulmonary arterial and wedge
pressures
Cardiac index
Pulse rate
Urine osmolality
Frank-Starling Curve
Blood pressure
Urine:plasma creatinine
Oxygen delivery
Pulse pressure
Fractional excretion of
sodium
Orthostatic blood pressure and Hematocrit
heart rate changes
Mental status
Acid-base balance
Oxygen consumption
Urine output
Right ventricular ejection fraction
Temperature
Serum lactate
Mixed venous oxygen consumption
Gastric tonometry
Transesophageal and transthoracic
echocardiography
E. Colloid vs. crystalloid
1. Ongoing debate - Large meta-analyses and RCTs demonstrate no advantage for one over
the other
F. Colloid
1. 5% Albumin
2. 25% Albumin
3. 6% hydroxyethyl starch
4. Dextran(in either normal saline or D5W)
G. Isotonic Crystalloid
1. Normal saline
2. Ringer’s lactate
3. Plasmalyte
H. Hypertonic Solutions
47
ASCCA Residents Guide to the ICU
1. Hypertonic (3%) saline
III. Electrolyte Replacement
A. Potassium
1. Internal Balance – acid-base, insulin, mineralocorticoids, catecholamines
2. External Balance – renal and gastrointestinal potassium excretion
3. Hypokalemia
a) Clinical presentation – Cushing’s syndrome, hyperaldosteronism, diabetic ketoacidosis,
diuretics, renal potassium wasting, hypomagnesemia
b) Approach to treatment
4. Hyperkalemia
a) Clinical presentation–mineralocorticoid deficiency, renal failure, cardiotoxicity
b) Approach to treatment:
(1) Membrane hyperexcitability – calcium
(2) Shift/removal of potassium – insulin, glucose, bicarbonate, catecholamines,
potassium-binding resins (kayexalate), dialysis
B. Sodium
1. Hyponatremia
a) Diagnostic Approach - Assessment of:
(1) Serum osmolality
(2) Total body sodium
(3) Subclassification of hypotonic hyponatremia based on volume status
b) Approach to treatment
(1) Based on etiology
(2) Rate of correction
c) Indications for hypertonic saline
(1) Severe symptomatic hyponatremia – stupor, coma, seizures
d) Central pontine myelinolysis
(1) Potential consequence of rapid correction
2. Hypernatremia
a) Diagnostic Approach - Assessment of:
(1) Extracellular volume
(2) Urine Osmolarity (may be useful in selected cases)
b) Approach to treatment
(1) Based on etiology
(2) Rate of correction
(3) Potential neurological complications
C. Calcium
1. Hypocalcemia
a) Clinical presentation
b) Causes–parathyroid hormone (PTH) deficiency, vitamin D deficiency, calcium chelation
(citrate), alcohol, hypomagnesemia, pancreatitis, hyperventilation, renal or hepatic
disease, ionized Ca depletion prevalence of 15% in ICU
c) Measurement
(1) Total
(2) Ionized
d) Approach to treatment: role of magnesium and phosphate, risk of metastatic calcification
(calcium-phosphate product)
2. Hypercalcemia
a) Clinical presentation
b) Causes - Excess PTH, malignancy, immobilization, granulomatous diseases
c) Approach to treatment–forced diuresis, pharmacological approach
D. Magnesium
1. Hypomagnesemia
a) Clinical presentation–prevalence in ICU 60%, primary loss through renal mechanism
(intrinsic and/or diuretic)
b) Effects on potassium, calcium, and phosphate homeostasis
2. Hypermagnesemia
a) Clinical presentation–iatrogenic (exogenous Mg+2 administration in pregnancy or renal
48
ASCCA Residents Guide to the ICU
disease)
E. Phosphate
1. Hypophosphatemia
a) Clinical presentation–ATP, neurologic, and hematologic presentations
b) Approach to treatment–role of calcium
2. Hyperphosphatemia – clinical presentation and approach to treatment
IV. Blood and Blood products
A. Indications for transfusion of various blood products
1. Whole blood
2. Packed red blood cells
a) Enhancement of oxygen carrying capacity
b) Perioperative ischemia
c) Acute vs. chronic anemia
d) Role of a trigger hemoglobin for transfusion
3. Washed red blood cells
4. Leukocyte-depleted red cells
5. Plasma–single donor, fresh-frozen
a) Reversal of warfarin therapy
b) Correction of coagulopathies
6. Cryoprecipitate
a) Fibrinogen deficiencies
b) von Willebrand’s disease
c) Consumptive coagulopathy
7. Platelet concentrate
a) Indications for prophylactic transfusion
b) Microvascular bleeding and correction of intraoperative thrombocytopenia
B. Differences between fresh and banked blood
C. Indications for Type and Screen vs. Type and Crossmatch orders
1. Process for complete crossmatch
2. Guidelines for emergency transfusion and use of uncrossmatched blood
D. Adverse Affects of Blood Transfusion
1. Effect on oxygen transport and the oxygen-hemoglobin dissociation curve
2. Hemolytic transfusion reactions: incidence, evaluation, and treatment
a) Allergic and febrile reactions
b) Microaggregates, acute lung injury, and use of blood filter
3. Transmission of Infectious Diseases
a) Viruses (HIV, CMV, Hepatitis)
b) Bacteria (Transfusion related sepsis)
c) Spirochetes
d) Parasites
e) Prions
4. Transfusion – Related Acute Lung Injury (TRALI)
5. Immunomodulation
6. Problems Associated with Massive Transfusion
a) Dilutional thrombocytopenia and dilutional coagulopathy
b) Hypothermia and use of blood warmers
c) Citrate intoxication
d) Metabolic complications–hypomagnesemia, hypocalcemia, acid-base disturbances
E. Alternatives to Transfusion
1. Minimize blood loss, role of controlled hypotension and DDADP
2. Tolerate lower hematocrit
3. Autologous transfusion
a) Preoperative donation and storage
b) Preoperative normovolemic hemodilution
c) Perioperative blood salvage
d) Blood Substitutes – remain experimental
This chapter is a revision of the original chapter authored by Karen J. Schwenzer, M.D
49
ASCCA Residents Guide to the ICU
READING LIST:
1. Adrogue HJ, Madias NE. Hyponatremia. N Engl J Med. 2000;342:1581-1589.
2. Adrogue HJ, Madias NE. Hypernatremia. N Engl J Med. 2000;342:1493-1499.
3. Bagshaw SM, Bellomo R. The influence of volume management on outcome. Curr Opin Crit Care. 2007;13:541-548.
4. Bhardwaj A, Ulatowski JA. Hypertonic saline solutions in brain injury. Curr Opin Crit Care. 2004;10:126-131.
5. Finfer S, Bellomo R, Boyce N, et al. A comparison of albumin and saline for fluid resuscitation in the intensive care
unit. N Engl J Med. 2004;350:2247-2256.
6. HГ©bert PC, Wells G, Blajchman MA, et al. A multicenter, randomized, controlled clinical trial of transfusion
requirements in critical care. transfusion requirements in critical care investigators, canadian critical care trials
group. N Engl J Med. 1999;340:409-417.
7. Perel P, Roberts I. Colloids versus crystalloids for fluid resuscitation in critically ill patients. Cochrane Database
Syst Rev. 2007;(4):CD000567.
8. Shander A, Goodnough LT. Update on transfusion medicine. Pharmacotherapy. 2007;27:57S-68S.
9. Shorr AF, Jackson WL, Kelly KM, Fu M, Kollef MH. Transfusion practice and blood stream infections in critically ill
patients. Chest. 2005;127:1722-1728.
10.Swanson K, Dwyre DM, Krochmal J, Raife TJ. Transfusion-related acute lung injury (TRALI): Current clinical and
pathophysiologic considerations. Lung. 2006;184:177-185.
11.Prough DS and Mathru M. Fluid Management in Critically Ill Patients In: Murray MJ, Coursin DB, Pearl RG,
Prough DS, editors. Critical care medicine: perioperative management, 2e. Philadelphia: Lippincott-Williams and
Wilkins; 2002.
QUESTIONS:
11.1.
A.
B.
C.
D.
All of the following would indicate hypovolemia EXCEPT:
BUN: Creatinine >20
Urine osmolality >350 mOsm/L
Urinary sodium <20 mEq/L
Fractional excretion of sodium (FENa) <1%
11.2.
A.
B.
C.
D.
All of the following are associated with hyperkalemia EXCEPT:
Alkalosis
ACE inhibitors
Rhabdomyolysis
Hypoaldosteronism
11.3.
A.
B.
C.
D.
E.
Euvolemic hyponatremia is most likely associated with:
Congestive heart failure
Cirrhosis
Aldosterone deficiency
SIADH
Diabetes insipidus
11.4.
A.
B.
C.
D.
The need for a massive transfusion contributes to overall mortality by all of the following EXCEPT:
Reflects that the patient is in shock
Causes microaggregate-induced acute lung injury
Blood is immunosuppressive
Bradykinin antagonism results in irreversible hypotension
11.5.
The available evidence for the use of albumin in critically ill patients suggests all of the following
EXCEPT:
Overall lack of superiority of albumin over crystalloid
Higher cost for albumin
Potential for harm with use of albumin in subgroups with head trauma
Clear clinical superiority of crystalloid over albumin
A.
B.
C.
D.
50
ASCCA Residents Guide to the ICU
12. Gastrointestinal Bleeding
Fahim Habib, M.D., Miguel A. Cobas, M.D., Juan Asensio, M.D.
A 58-year old male is admitted to the ICU following a motor vehicle collision in which he sustained
a traumatic brain injury, multiple rib fractures, pulmonary contusion and a pelvic fracture.
Following a period of prolonged mechanical ventilation, he develops a ventilator-associated
pneumonia and is started on broad-spectrum empiric antibiotics. He then progresses to develop
septic shock requiring pressors. His nasogastric tube begins to put out coffee-ground that rapidly
progresses to frank bright red blood. His proton pump inhibitors are changed from prophylactic to
therapeutic doses. The coagulopathy resulting from hepatic dysfunction due to sepsis is corrected
by administration of fresh frozen plasma. Endoscopy performed shows the presence of diffuse
superficial gastric ulcerations without a localized source of bleeding.
Gastrointestinal bleeding may either result in admission to the ICU or develop during the course
of the ICU stay. It results most commonly from upper gastrointestinal bleeding. Use of prophylaxis
in high-risk patients has helped reduce its occurrence, however patients requiring prolonged
mechanical ventilation, those with head injury or burns and patients on corticosteroids remain atrisk. The first step in management is to assess and ensure hemodynamic stability. Once
resuscitation is underway and the patient appears to be responding, a thorough history and
physical examination is performed to determine the likely source of the bleeding.
Esophagogastroduodenoscopy (EGD) is performed for suspected UGIB and may be diagnostic
and therapeutic. Colonoscopy is performed for most cases of LGIB. If both are negative,
evaluation for a small bowel source using enteroclysis/enterscopy or capsule endoscopy is
performed. Bleeding is best controlled by local measures. Additionally, hemostatic abnormalities
must be corrected and the systemic physiology restored. Surgical intervention is reserved for
refractory cases. Recombinant factor VIIa may be employed when all conventional measures fail.
The following review outlines the key aspects of gastrointestinal bleeding in critically ill patients.
I.
Epidemiology
A. Incidence: Decreasing: because- increased use of prophylaxis, aggressive resuscitation,
improved supportive care, preference of enteral feeds.
B. Causes
1. UGIB
a) Peptic ulcer disease
(1) Gastric ulcer
(2) Duodenal ulcer
b) Erosive esophagitis
c) Diffuse superficial gastritis
d) Variceal Bleeding
e) Mallory-Weiss tears
f) Vascular malformations
g) Carcinomas
h) Hemobilia
i) Aortoenteric fistula
2. LGIB
a) Diverticular disease
b) Vascular ectasia
c) Ischemic colitis
d) Inflammatory colitis
e) Polyps
f) Carcinoma
II. Risk Factors
A. Prolonged mechanical ventilation (>48 hrs)
51
ASCCA Residents Guide to the ICU
B.
C.
D.
E.
F.
G.
Coagulation disorders
Trauma
Burns
Patients on corticosteroids
Patients with multi-system failure, especially hepatic and renal
Use of vasoconstrictor agents: dopamine, epinephrine, norepinephrine, vasopressin etc.
III. Prophylaxis
A. Early provision of enteral nutrition
B. Restoration of systemic perfusion and oxygenation
C. Use of pharmacologic prophylaxis
1. Indicated in ICU patients:
a) Requiring prolonged (>48 hrs) mechanical ventilation, having coagulation
disorders, with renal failure, admitted after burns or trauma, and patients on
corticosteroids
2. Options for prophylaxis include:
a) Histamine blockers, proton pump inhibitors, sucralfate, and antacids
(1) Antacids: not preferred because:
(a) Need frequent administration
(b) Interact with many oral medications
(c) May cause diarrhea
(d) Bind phosphate
(e) Potential for magnesium toxicity
(f) Expensive
(2) Sucralfate: not preferred because:
(a) Reduces bioavailability of many oral medications
(b) May clog feeding tubes
(3) Histamine blockers: effective, may cause alteration in drug metabolism,
altered platelet function and confusion
(4) Proton pump inhibitors: highly effective, safe
3. Adverse effects of acid suppression
a) Increased incidence of nosocomial pneumonia
D. Avoidance of ulcerogenic medications: corticosteroids, slow release potassium,
nonsteroidal anti-inflammatory agents etc
IV.
Diagnosis
A. History
1. Hematemesis: indicates upper GI bleed
2. Melena: UGIB or LGIB from right colon
3. Hematochezia: brisk UGIB (10%) or LGIB (90%)
B. Gastric lavage: negative only if no blood returns in effluent that contains bile. In absence
of bile, bleeding from a duodenal source with a competent pylorus cannot be excluded.
C. Esophagogastroduodenoscopy (EGD)
D. Tagged red cell scans
E. Mesenteric angiogram
F. Enteroclysis/Enteroscopy/Capsule Endoscopy
G. Colonoscopy
H. Anoscopy
I. Digital rectal examination
V. Treatment
A. First step: assess hemodynamic stability
B. In unstable patients: Initiate resuscitation: assess/secure airway, ensure ventilation,
establish intravenous access, begin fluid resuscitation, obtain and send blood for
laboratory studies, arrange for the necessary blood products. Then begin obtaining a
history performing a physical examination and initiate studies to localize the source of
bleeding.
52
ASCCA Residents Guide to the ICU
IV access: preferred option- two large bore (14-gauge) peripheral catheters in the
antecubital fossa. Acceptable alternative- central venous introducer. Do not use: triple
lumen central venous catheter
2. Laboratory studies: sent at time IV access obtained. Includes: hematocrit,
electrolytes, creatinine, liver function tests, prothrombin time, and platelet count.
Samples are also sent for typing and cross-matching
3. Fluid resuscitation: initially use isotonic crystalloid. If need for resuscitation persists;
use specific blood components based on results of hematocrit, prothrombin time and
platelet count.
a) Goals of transfusion: Hematocrit ≥ 25%, platelet count > 50,000, near normal
prothrombin time.
C. In stable patients: obtain a history, perform a physical examination and initiate studies to
localize the source of bleeding.
D. Pharmacologic therapy: proton pump inhibitor infusion (esomeprazole, 80 mg bolus, then
8 mg/hr infusion)
E. Esophagogastroduodenoscopy
1. Benefits
a) May establish the diagnosis
b) May permit the control of bleeding
(1) Injection of epinephrine, injection of sclerosants, or thermal therapy (80%
successful)
(2) Banding of varices
c) May predict the risk of re-bleeding
2. Disadvantages
a) Need for sedation, may compromise ventilation
b) Need for a stomach empty of food and blood, may be achieved by administration
of 250 mg of IV erythromycin 30 min prior to the EGD
3. Re-bleeding
a) In 15-20%, usually occurs within 48 hours
b) Suggested by:
(1) Endoscopic findings: presence of bleeding varices, visible spurting vessel,
oozing vessel, ulcers containing a visible vessels, ulcers with overlying clot
(2) Location of the ulcer: high on the lesser curve (left gastric artery) and
posterior-inferior wall of the duodenum (gastroduodenal artery)
c) Treatment for re-bleeding: repeat endoscopy, surgery, angiographic occlusion
4. Failure to establish a diagnosis: bleeding that has stopped, bleeding from beyond the
reach of the endoscope
F. Balloon tamponade: for varices, utilizes the Sengstaken-Blakemore tube
G. Adjunctive pharmacotherapy for variceal bleeding:
1. Octreotide
2. Vasopressin
H. Surgical options: for refractory disease
1. Total gastrectomy for stress gastritis,
2. Porto-systemic shunts (most often trans-internal jugular, or TIPSS) for varices
I. Options for treatment of LGIB
1. Colonoscopy with laser or thermal coagulation
2. Angiographic infusion of vasoconstrictors or embolization
3. Surgical resection
J. Factor VIIa: for refractory bleeding unresponsive to conventional measures
VI. Outcome
A. Poor prognostic factors
1. Clinical
a) Large volume of hemorrhage
b) Shock at presentation
c) Presence of coagulopathy
d) Unidentified source of bleeding
e) Advanced age
f) Multiple organ failure: especially hepatic and renal
1.
53
ASCCA Residents Guide to the ICU
B. Mortality: 6-10%, has remained unchanged over the last several years
This chapter is a revision of the original chapter authored by Charles G. Durbin, M.D.
READING LIST
1. Conrad AS. Acute upper gastrointestinal bleeding in critically ill patients: causes and treatment
modalities. Crit Care Med. 2002 June;30(6 Suppl):S365-8.
This paper reviews the causes and treatment options for acute upper gastrointestinal bleeding in critically
ill patients.
2. Fennerty BM. Pathophysiology of the upper gastrointestinal tract in the critically ill patient: Rational of
the therapeutic benefits of acid suppression. Crit Care Med. 2002 June;30)6 Suppl):S351-5.
This paper reviews the evidence supporting the use of acid suppression in the critically ill patient.
3. Lin CC, Lee YC, Lee H, Ho WI, Chen TH, Wang HP. Bedside colonoscopy for critically ill patients with
acute lower gastrointestinal bleeding. Intensive care Med (2005) 31;743-46.
This paper evaluates the utility of bedside colonoscopy in the assessment of lower gastrointestinal
bleeding in the ICU.
4. Maury E, Tankovic J, Ebel A, Offenstadt G; Parisian Group of the Upper Gastrointestinal Bleeding Survey..
An observational study of upper gastrointestinal bleeding in intensive care units: is Helicobacter pylori
the culprit?. Crit Care Med. 2005 Jul;33(7):1513-8.
Demonstrates the role of Helicobacter pylori in patients developing gastrointestinal bleeding in the ICU.
QUESTIONS
12.1.
A.
B.
C.
D.
E.
Factors resulting in a reduced incidence of stress gastritis in the ICU setting include:
Greater awareness of the pathophysiology of the disease
Increased use of prophylactic agents
Increased use of enteral feeds
Improvements in resuscitation
All of the above
12.2.
A 22-year old healthy male sustains a pelvic fracture and multiple extremity injuries. Following
development of a pulmonary embolus, he is placed on therapeutic doses of heparin. Two days
later his nasogastric tube begins to put out large amounts of bright red blood. The patient
becomes progressively tachycardic and hypotensive. The first step in the management of this
patient is to:
Remove the NG tube
Administer a bolus dose of a proton pump inhibitor
Administer protamine sulfate to reverse the effect of heparin
Fluid resuscitation to reestablish hemodynamic stability
Perform an EGD
A.
B.
C.
D.
E.
12.3.
A.
B.
C.
D.
E.
Pharmacologic prophylaxis for prevention of stress gastritis is indicated in all of the following
EXCEPT:
Epidural hematoma in a patient receiving jejunal feeds
2nd degree burns with inhalation injury
Cirrhosis of the liver
Patients on steroids for adrenal insufficiency
Necrotizing fasciitis of the perineal area
54
ASCCA Residents Guide to the ICU
12.4.
A.
B.
C.
D.
E.
12.5.
A.
B.
C.
D.
E.
An 86-year old female is admitted to the ICU after presenting with hematochezia. An EGD is
negative. Colonoscopy is unsuccessful due to presence of large amount of blood and stool in
the colon. The patient remains hypotensive despite aggressive resuscitation. A mesenteric
angiogram was attempted but is unsuccessful due to severe atherosclerotic disease of the
mesenteric vessels. The next step in management of this patient is:
Subtotal colectomy
Administration of DDAVP
Initiation of a vasopressin infusion
Bolus 300 micrograms of octreotide followed by an infusion
Administration of recombinant factor VIIa
A 49-year old alcoholic is admitted to the ICU after developing an UGIB following severe
retching during a binge episode. After resuscitation an EGD is performed which shows linear
tears on the gastric side of the gastroesophageal junction. The most important aspect of
management is:
Expectant observation
Administration of antiemetics
Raising the gastric pH
EGD with thermal coagulation of the tear
Esophageal resection
55
ASCCA Residents Guide to the ICU
13. Thromboembolic Disease
Gustavo Angaramo, M.D.
A 28-year-old man is admitted to the trauma unit due to a thoracic spine injury with paraplegia (T10) after
a fall from a ladder. Three days after admission, he complains of shortness of breath. His CXR and ABG
are normal. A V/Q scan is obtained, and is read as “low probability” with matching sub segmental defects.
Eight days after admission, he develops sudden respiratory distress with hypotension. ABG on 50% face
mask reveals: pH = 7.50, PaCO2 = 30, PaO2 = 58. A spiral CT scan reveals a right inferior pulmonary
artery embolus.
Venous thromboembolism and its sequel pulmonary embolism are insidious and pervasive management
problems in the ICU and general hospital population. Autopsy studies indicate that the problem is under
diagnosed, and hospital surveys indicate that prophylaxis is not provided to many individuals at risk.
Additionally, screening studies for DVT lack sensitivity and specificity. The cornerstone of management
involves identification of high-risk groups and treatment with adequate prophylactic measures. In patients
with clinical history suspicious for PE, definitive diagnostic studies and therapy are warranted, given the
extremely high morbidity and mortality associated with PE.
Scope of the Problem
A. Incidence:
1. Underestimated, based on unselected autopsy studies–over 70% of those who die with PE
are not suspected of PE prior to death
2. Best recent estimate (early 90s): 23 per 100,000 leading to overall estimate of 260,000 cases
per year in U.S.
B. Morbidity and mortality:
1. Accounts for 10-15% of all acute care hospital deaths (may be higher in nursing home and
rehab hospital settings)
2. Case fatality rate = 12%
II. Risk Factors (in approximate descending order of overall incidence for diagnosed DVT; incidence of
DVT without prophylaxis noted, when known)
A. Age greater than 40–risk increases exponentially with age
B. Obesity
C. Prior history of DVT–8-fold increased risk
D. Malignancy–incidence approx. 15% in patients with 2 episodes idiopathic DVT
E. Immobilization > 5 days
F. High risk surgeries:
1. Lower extremity orthopedic surgery
a) Total hip replacement–50% incidence
b) Total knee replacement–even higher incidence
2. Open heart procedures
3. Gynecologic-oncology–19% incidence
4. Major urologic
5. Neurosurgery–24% incidence
6. Abdominal operations lasting >30 min–25% incidence
G. Major stroke–50% incidence
H. Congestive heart failure and MI–24% incidence
I. Trauma (without prophylaxis incidence of DVT):
1. Lower extremity ortho injuries–69%
2. Spinal injuries–38-62%
3. Major head injuries–54%
4. Major injuries of face, chest, or abdomen–50%
J. Primary or secondary brain tumors–DVTs in up to 28% of high grade tumors
K. High dose estrogen (oral contraceptives, not post-menopausal replacement)
L. Pregnancy
M. Hypercoagulable states–deficiencies of antithrombin III, protein C, or protein S, including recently
I.
56
ASCCA Residents Guide to the ICU
described activated protein C resistance present in up to 7% of the asymptomatic population
(occurs secondary to a mutant factor V that doesn’t bind protein C)
N. Multiple above risk factors increase risk exponentially
III. Pathophysiology
A. Virchow’s triad–venous stasis, hypercoagulable state, and endothelial injury
B. Involved vessels–usually lower extremity, although incidentally involves upper extremity when
associated with central venous catheterization.
IV. Prophylaxis
A. Pharmacologic agents
1. Heparins:
a) Formulations:
(1) Low-dose unfractionated heparin (LDH)
(a) Doses: 5000 IU SQ Q 8-12 hr
(b) Monitoring: not needed for anticoagulation
(2) Low molecular weight heparin (LMWH)
(a) Doses: 30 mg SQ Q 12 hr or 40 mg SQ QD (enoxaparin)
(b) Monitoring: not needed for anticoagulation
(3) Fondaparinux
(a) Pentasaccharide
(b) Does not bind to platelet factor 4 and will not cause heparin induced
thrombocytopenia
(c) Dose: 2.5 mg subcutaneously once a day
b) Complications
(1) Bleeding–5% risk major bleeding with continuous IV or SQ unfractionated
(2) Thrombocytopenia–occurs in 3% with unfractionated heparin(less with LMWH);
secondary to immune IgG-mediated response; may lead to arterial thrombosis.
(3) Resistance to heparin–antithrombin III deficiency.
(4) Osteoporosis–in 30% treated with long-term unfractionated heparin therapy
2. Oral anticoagulation (warfarin) – dose to INR 2-3
a) Complications:
(1) Bleeding
(2) Skin necrosis–in patients with protein C or S deficiency
B. Mechanical agents:
1. Early ambulation, leg elevation, physiotherapy
2. Graduated compression (elastic) stockings (ES)
3. Intermittent pneumatic compression (IPC)
4. Prophylactic IVC filter–in those for whom anticoagulation is contraindicated, or who
experience recurrences despite adequate anticoagulation
C. Current recommendations: See Table 13.1
Type of Risk
Category
Low risk
Moderate risk
Details of Group
•Minor surgery
•Mobile patients
•General, open gynecologic or urologic
surgery patients with low risk of bleeding
•Medical patients at bed rest or “sick”
•Hip or knee arthroplasty
•Hip fracture surgery
•Major trauma
•Spinal cord injury
Prophylaxis Recommended
Early ambulation
•LMWH
•LDH
•fondaparinux
LMWH, fondaparinux or oral vitamin K
High risk
antagonist (INR 2-3)
Moderate to high
VTE risk
High risk of bleeding
Mechanical thromboprophylaxis (IPC)
Table 13.1 Current Recommendations for VTE Prophylaxis. LDH = low-dose unfractionated heparin,
LMWH - low-molecular weight heparin, IPC = intermittent pneumatic compression, ES = graduated
compression (elastic) stockings
57
ASCCA Residents Guide to the ICU
V. Diagnosis of DVT/PE
A. Clinical findings (low sensitivity and specificity):
1. DVT–distal limb edema/pain, differential limb circumference, Homan’s sign, distended
collateral veins, increased temperature if infection present.
2. PE–dyspnea, chest pain, tachypnea (3 most common symptoms), tachycardia, hypoxemia,
hypocarbia, hemoptysis, infiltrate on CXR.
B. Diagnostic studies:
1. Impedance plethysmography
a) Good for thigh thrombosis (90%), but poorer sensitivity for calf thrombosis
2. Duplex doppler ultrasound
a) Reliability is tied to the technician and the interpreter
b) Can not distinguish between occlusion from external pressure vs. thrombosis
3. Contrast venography
a) Sensitive and very specific, compared with the preceding methods
4. Radionuclide lung scans (V/Q):
a) Significant inter-observer variability exists for distinguishing between intermediate and
low probability scans
b) Therefore, 14% of patients with low probability scan will have a PE on angiogram.
c) High clinical suspicion and high probability lung scan-PE in 96%
d) Low clinical suspicion and low probability lung scan-PE in 4%
5. Contrast enhanced helical CT scan
a) Sensitivity for central emboli is 95 %( overall 72%), specificity 95%
6. D-dimer: degradation products of fibrin, endogenous marker for fibrinolysis.
a) The value of the test is in ruling out venous thromboembolism.
7. Right heart ultrasound
8. Echocardiography/TEE
9. Pulmonary angiography–the gold standard
VI. Treatment
A. Local measures–elevation of extremity, warm compresses; to be used as adjunct to other
measures
B. Anticoagulation (should begin before diagnostic studies if PE is intermediate or high probability):
1. Adjusted-dose intravenous unfractionated heparin
a) The standard treatment
b) Adjust to keep PTT >1.5 - 2.0 times control
c) Follow with warfarin within 24 hours for at least 3 months
2. Subcutaneous unfractionated heparin–17,500 IU Q 12 hr
3. Low molecular weight heparin–fixed dose, SQ regimens proven as effective treatment for
DVT and PE
C. Thrombolysis
1. Considered when hemodynamic instability exists, or massive PE
2. Agents: tPA
D. Vena caval interruption (IVC filter)
1. For those patients who have absolute contraindications to anticoagulant therapy, who
develop serious bleeding problems with anticoagulation, or experience recurrent emboli
despite adequate anticoagulation.
E. Pulmonary embolectomy
1. For those with serious systemic symptoms from PE, and in whom thrombolysis is
contraindicated
2. Emergency surgical mortality ranges from 10 to 75%.
This chapter is a revision of the original chapter authored by Lucy A. Weston, Ph.D., M.D.
READING LIST:
1. Antithrombotic and Thrombolytic Therapy: American College of Chest Physicians Evidenced-Based Clinical
Practice Guidelines (8th Edition) Chest 2008;133: 67S-968S. Comprehensive review of thromboembolism in major
patient groups; includes set of usable recommendations.
58
ASCCA Residents Guide to the ICU
2. Rosenow EC. Venous and pulmonary thromboembolism: an algorithmic approach to diagnosis and management.
Mayo Clin Proc 1995;70:45-49.
3. Geerts WH, Jay RM, Code KI, Chen E, Szalai JP, Sail EA, et al. A comparison of low-dose heparin with lowmolecular-weight heparin as prophylaxis against venous thromboembolism after major trauma. N Engl J Med
1996;335:701-7. This study shows that LMWH, as compared to conventional unfractionated heparin, reduces the
risk of proximal DVT by 65% (vs. 12% reduction of risk).
4. Ralph DD. Pulmonary embolism: the implications of prospective investigation of pulmonary embolism diagnosis.
Rad Clin N Amer 1994;32(4):679-87. A synthesis of the prospective PIOPED study and others: this article
summarizes the true risk of angiographically determined PE according to clinical suspicion and results of V/Q
scanning.
5. Weitz JI. Low-molecular-weight heparins. N Engl J Med 1997;337(10):688-98. A great little review article on
LMWH: its pharmacology and uses.
6. Ginsberg JS. Management of venous thromboembolism (review). N Engl J Med 1996;335(24):1816-27. A nice little
review with good references.
7. The Columbus Investigators. Low-molecular-weight heparin in the treatment of patients with venous
thromboembolism. N Engl J Med 1997;337:657-62. This prospective, randomized study of 1021 patients show
similar outcomes for patients with symptomatic DVT treated with either low-molecular weight heparin or tradition
intravenous unfractionated heparin.
8. Anderson FA, Wheeler B. Venous thromboembolism: risk factors and prophylaxis. Clin Chest Med 1995;16(1):
235-251. This general review focuses on surgical aspects of DVT and includes a discussion of intermittent
pneumatic compression devices.
QUESTIONS:
13.1.
A.
B.
C.
D.
E.
13.2.
A.
B.
C.
D.
E.
13.3.
A.
B.
C.
D.
E.
A 66-year-old man is 5 days post repair of an abdominal AAA and develops dyspnea with pleuritic chest
pain. Vitals: T = 39C, BP - 110/60, P = 120. CXR and ECG are unrevealing. ABG on room air shows:
pH = 7.48, PaCO2 = 32, PaO2 = 72. What should be done next?
Intravenous heparin and no further diagnostic testing.
Intravenous heparin followed by either V/Q scanning or deep venous studies.
Subcutaneous heparin (5000 IU BID) followed by oral warfarin and observation.
Impedance plethysmography or duplex ultrasonography and, if no abnormalities detected, repeat every
2 to .4 days for 10 days.
Pulmonary arteriography.
A 68-year-old woman with acute dyspnea 10 days after hemiplegia from a non-hemorrhagic stroke has
a V/Q scan demonstrating a high probability for PE, with two segmental perfusion defects with normal
ventilation to these areas. She has a history of 90 pack-years of smoking, and has received heparin,
5000 IU SQ Q12 hrs. since her admission. Which one of these measures is most appropriate at this
point?
Discontinue heparin, as the dyspnea may be a complication of therapy.
Do a pulmonary arteriogram, because a V/Q scan is unreliable in heavy smokers.
Repeat the V/Q scan in several days.
Anticoagulate with IV heparin (PTT >1.5 normal), then start warfarin in 5 to 7 days.
Anticoagulate with IV heparin (PTT>1.5 normal), then start warfarin the same day.
Which one of the following statements is true about the initial clinical manifestations of acute PE?
Normal findings on deep venous studies including venography of both lower extremities exclude the
possibility of PE.
Hypoxemia (PaO2 <80) is present in almost all patients with PE.
More than half of the patients with features typical of acute PE do not have it.
Hormone replacement therapy in postmenopausal women is a risk factor for venous thromboembolism.
Patients with eventually fatal PE have characteristic manifestations that suggest its presence.
59
ASCCA Residents Guide to the ICU
14. Bleeding in the ICU
Ruben J. Azocar, M.D.
A 26-year-old male arrives from the OR after evacuation of a subdural hematoma, external fixation of his
left femur and exploratory laparotomy that revealed a grade 3 liver laceration and a shattered spleen. The
spleen was removed and the liver packed and he is brought to the ICU for further resuscitation. He
received 10 units of blood, 4 units of FFP and a pack of platelets in the OR in addition to 6 liters of
Lactated Ringers. On arrival his vital signs are HR 125/minute; BP 90/55 mm Hg; Temperature 35.5
degrees Celsius. He is “oozing” blood from the abdominal wound and puncture sites. Despite the
transfusion his HCT is 25.
Bleeding in the ICU can be sudden, dramatic, and immediately apparent or slow, insidious, and difficult to
diagnose. The bleeding may represent a life-threatening event via hypovolemia and loss of oxygencarrying capacity or by a mass effect, as with an expanding neck hematoma, space occupying intracranial
mass or pericardial tamponade.
Nature and Scope of the Problem
A. High priority–second only to airway and breathing
B. Presentations:
1. Dramatic rupture В± exsanguination
2. Slow dwindle in hematocrit–may be diagnostic challenge
3. Bleeding into a limited space–space occupation is the issue
C. Immediate concerns:
1. Secure the airway
2. Secure intravascular access
3. Maintain volume status
4. Secure blood products as appropriate
5. Investigate the bleed
a) Surgical bleeding
b) Medical bleeding
II. Specific Bleeding Situations in which Loss of Airway is a Concern
A. Upper airway bleeding
1. UPPP (uvulopalatopharyngoplasty) surgery
2. Tonsils and adenoids
3. Wegener’s granulomatosis
4. Massive epistaxis/facial trauma
B. Lower airway bleeding
1. Necrotizing pneumonia
2. Post-bronchoscopy
C. External compression of the trachea from a surgical site
1. Carotid endarterectomy
2. Thyroid/parathyroid surgery
3. Arterial injury during central line insertion
D. Massive hematemesis
1. Esophageal varices
2. Gastric/duodenal ulcer
III. Cases Where Blood Occupying a Space is a Danger Due to Mass Effect
A. Intracranial bleed
1. Subarachnoid hemorrhage
2. Subdural hematoma
3. Epidural hematoma
4. After stereotactic brain biopsy
5. During/after thrombolytic therapy
B. Pericardial tamponade
C. Hemothorax
I.
60
ASCCA Residents Guide to the ICU
IV. Vascular Access
A. Routes
1. Hands/antecubital/subclavian/IJ/EJ/Saphenous/femoral
2. Intraosseous needle in young children
B. Catheters
1. Resistance to flow related to (radius)4 and length
2. Short, large-bore catheters preferable
C. Fluids
1. Initially whatever is available
2. Preferably colloid - more intravascular expansion per ml
a) 5% Albumin–expensive, efficacious
b) Hetastarch – platelet aggregation difficulties at doses above 15 ml/kg
c) Order of preference of blood/blood products:
(1) Fully cross-matched
(2) Correct type, screened
(3) Type-specific
(4) O-negative blood
V. Etiologies
A. Surgical bleeding (AKA “Technical bleeding”)
1. Readily apparent
a) Disrupted suture lines–external losses
b) Drainage output increases (chest tubes, J-P drains, etc.)
c) Visible hematoma
B. Surgical bleeding, concealed
1. Subtle
a) Drainage tubes clotted, kinked
b) Time course of slow bleed may correspond to period of time over which ongoing “third
space” losses might be expected, thereby confounding the diagnosis
c) Clinically “silent” areas of bleeding (e.g., thigh, pelvis, retroperitoneal, etc.)
d) Initial hematocrit is of little help
2. Hematocrit trending down–extraordinary fluid requirement
3. Presentation:
a) Restlessness, anxiety
b) Tachycardia (if not masked by beta blockade, digitalis glycosides, or intrinsic heart
disease)
c) Rising diastolic BP (narrow pulse pressure)
d) Decreasing urine output with high specific gravity
4. Imaging–a compromise between the optimal study and the stability of the patient
a) Plain films
b) Ultrasound
c) CT/MRI scan
d) Tagged RBC scan
e) Angiography/surgical exploration
5. To operate?
a) Surgeon needs to be involved early and completely
b) Optimizing all factors while surgery is contemplated
(1) Airway/IV access/volume repletion
(2) Communicate with blood bank regarding needs
(3) Correcting all “medical” causes of bleeding
6. Communication with all team members
a) O.R./Nursing
b) Anesthesia team
c) ICU (especially if patient is in ER or ward on presentation)
d) Blood bank
e) Patient and family
VI. Investigate the Bleed
A. Medical bleeding (coagulopathy)
1. Inherited coagulopathy–usually have history of prolonged oozing/bleeding
61
ASCCA Residents Guide to the ICU
Hemophilia A: Factor VIII deficiency
Hemophilia B: Factor IX deficiency
Fibrinogen defects/deficiency
Von Willebrand’s disease
Treatment with infusion of the appropriate factor
(1) Hemophilia generally transfused with the appropriate factor concentrate to maintain
100% of factor activity in the perioperative or peri-bleed period
(2) Deficits in fibrinogen are corrected with FFP or cryoprecipitate
(3) Von Willebrand factor can be increased transiently by infusion of cryoprecipitate and/
or DDAVP
2. Acquired coagulopathy
a) Platelet dysfunction
(1) Hypothermia
(2) Drug-related (e.g., NSAIDs, clopidrogel)
(3) Post-CABG
(4) Uremia
b) Platelet deficiency
(1) Post-CABG/IABP
(2) Dilutional–Not as dramatic a decline as would be predicted from dilution of
endogenous whole blood with transfused PRBCs. Fixed formulae that suggest a
prophylaxis platelet transfusion after a given number of pRBC transfusions are not
appropriate.
(3) Drug-related (e.g., Heparin, quinine)
(4) ITP/TTP
(5) Sepsis/MODS/ARDS/DIC
(6) Marrow suppression/displacement
c) Factor deficiency
(1) Vitamin K/nutritional deficiency/antibiotics/biliary obstruction
(2) Hepatic dysfunction
(3) DIC
(4) Recently the use of activated factor VII have been advocated in situation where non
surgical bleeding arises.
(a) Factor VIIa is an initiator of thrombin generation. Factor VIIa acts primarily via
two pathways to activate Factor Xa. One pathway is at the site of tissue injury
complexed with Tissue Factor, and the other is on the surface of platelets,
independent of tissue factor.
(b) Recombinant Factor VIIa (FVIIa) is currently licensed for use in hemophiliacs
with antibodies to Factor VIII. At present, its use in trauma and hemorrhage is
on a 'compassionate' basis. Where the use of Factor VIIa is being considered,
hospitals should have a set of guidelines in place for the availability and use of
Factor VIIa.
(c) Some recommended guidelines include
i) Factor VIIa will not stop surgical hemorrhage.
ii) Factor VIIa should not be given instead of other blood product
administration. Adequate FFP, Cryoprecipitate and Platelets need to be
present for full effect.
iii) Factor VIIa should not be used too early, but neither should it be used only
after 'super-massive' transfusions of 40-60 units. Therapy at between 8
and 20 red blood cell infusions is probably appropriate. (Opinion only)
iv) The current recommended dose is 100 mcg/kg. This dose should be
repeated at 1-2 hourly intervals if required.
v) The Prothrombin time is used to monitor drug effect
vi) When the pH is below 7.2, consideration should be given to not using
FVIIa (futility) or increasing the dose of FVIIa
VII. Blood transfusions in the ICU
A. Transfusion risks: Risks of blood transfusions include viral transmission, bacterial infection, acute
transfusion reactions, transfusion related acute lung injury, immunosuppressive effects, and
inflammatory responses. The Serious Hazards of Transfusion (SHOT) Study analyzed
a)
b)
c)
d)
e)
62
ASCCA Residents Guide to the ICU
spontaneously reported deaths or major complications from blood transfusions in the UK and
Ireland during a 2-year period. In all, a total of 366 events were reported from 94 of 424
participating hospitals in the first year and from 112 of the 424 participating hospitals in the
second year, the majority of which could be attributed to incorrect blood/component transfusion
and preventable clerical/administrative errors, rather than immunologic or infectious
complications.
B. Are transfusions beneficial in the critically ill?: The Canadian Clinical Care Trials Group conducted
a series of trials known as Transfusion Requirements in Critical Care (TRICC), designed to
examine the impact of transfusion practice on mortality rates in the ICU. This was a landmark
randomized study in ICU patients, which sought to understand whether a restrictive transfusion
policy was different than a liberal transfusion policy with respect to mortality and severity of organ
dysfunction. Twenty-five ICUs participated and 838 patients who had a Hb concentration less
than 9 g/dL within 72 hours of ICU admissions were enrolled. Patients were randomized to
receive transfusions using two different strategies: Patients randomized to a restrictive group did
not receive a transfusion until their Hb dropped below 7 g/dL. Those in the liberal group received
transfusions when Hb dropped below 10 g/dL. Patients in the restrictive group had lower average
daily Hb levels, 8.5 g/dL, versus 10.7 g/dL in the liberal group. There was a 54% reduction in the
average number of transfusions administered to patients in the restrictive group (2.6 per patient)
versus those in the liberal group (5.6 per patient); 33% of those in the restrictive group received
no transfusions at all. The 30 day mortality was 23.3% in the liberal group vs. 18.7% in the
restrictive group and the in hospital mortality was 28.1% vs 22.2%.
VIII. Alternatives to transfusion:
A. Tolerance of anemia
B. Use of Epoetin alfa: EPO 3 (Corwin, et al. 2007).
1. Assessed efficacy of once a week dosing of erythropoietin in reducing RNC transfusion in
critically ill patients
2. Erythropoietin 40,000 units administered once a week
3. Did not reduce the number of blood transfusions or alter mortality (except in trauma patients).
Risk of thromboembolic complications was associated with its use.
4. Cannot be recommended.
C. Minimize blood draws
D. Reduce the “ease” of drawing blood – remove arterial and central venous catheters if not needed.
E. Use pedi tubes.
F. Point of care testing
This chapter is a revision of the original chapter authored by J. Eric Greensmith, M.D., Ph.D.
REFERENCES
1. HГ©bert PC, Wells G, Blajchman MA, et al. A multicenter, randomized, controlled clinical trial of transfusion
requirements in critical care. Transfusion Requirements in Critical Care Investigators, Canadian Critical Care Trials
Group. N Engl J Med. 1999 Feb 11;340:409-17
2. Williamson LM, Lowe S, Love EM, et al Serious hazards of transfusion (SHOT) initiative: analysis of the first two
annual reports. BMJ. 1999 Jul 3;319(7201):16-9.
3. Chin-Yee I, Arya N, d'Almeida MS. The red cell storage lesion and its implication for transfusion. Transfus Sci. 1997
Sep;18(3):447-58.
4. www.trauma.org/resus/FactorVIIa.htlm
5. Corwin HL, Gettinger A, Pearl RG et al ; EPO Critical Care Trials Group. Efficacy of recombinant human
erythropoietin in critically ill patients: a randomized controlled trial. JAMA. 2002 Dec 11;288(22):2827-35.
6. Rizoli SB, Nascimento B Jr, Osman F, et al:Recombinant activated coagulation factor VII and bleeding trauma
patients. J Trauma. 2006 Dec;61(6):1419-25
63
ASCCA Residents Guide to the ICU
QUESTIONS:
14.1.
A.
B.
C.
D.
A 67-year-old man several hours after a transurethral prostatectomy develops massive bleeding. The
most important treatment is:
100 gm factor VII
10 units of platelets
Volume and blood replacement
FFP and cryoprecipitate
14.2.
A.
B.
C.
D.
The most common inherited bleeding disorder is:
Hemophilia-A
Antithrombin III deficiency
ОІ-Thalassemia
Von WillebrandКјs disease
14.3.
A.
B.
C.
D.
Activated factor VII is indicated in cases of:
Heparin overdose
Protamine excess
Hemophilia
Abnormal platelet function
14.4.
A.
B.
C.
D.
Strategies to minimize blood transfusion in the ICU include the following EXCEPT:
Autologous blood donations
Epoetin alfa once a week
Minimize blood draws
Acceptance of lower hemoglobin levels
14.5.
A.
B.
C.
D.
Risks of blood transfusion include the following EXCEPT:
Transfusion related acute lung injury
Immunosuppressive effects
Hypercoagulability
Fever
64
ASCCA Residents Guide to the ICU
15. Specialized Nutrition Support in Critically Ill
Patients
Gustavo Angaramo, M.D.
A 27-year-old male involved in an MVA is transferred to the ICU with a distended abdomen, a right chest
tube, a right femur fracture, and left open tibia and fibula fractures. He is intubated and receiving
mechanical ventilation. Chest X-ray shows a right pulmonary contusion. A CT scan of the abdomen
reveals a grade 4 liver laceration with significant intraperitoneal blood. After CT scan (complicated by
hypotension), the patient goes to the OR and undergoes an exploratory laparotomy.
Other injuries discovered in surgery include a crush injury to the cecum and a devascularized section of
the mid small bowel of approximately three feet in length. After a small bowel resection and anastomosis,
an ileostomy and a colonic mucous fistula are constructed. A needle catheter jejunostomy is placed, and
the patient is transferred back to the ICU.
Upon arrival to the ICU, the patient requires dopamine (15 mcg/kg/min) and epinephrine (0.06 to mcg/kg/
min) infusions to maintain hemodynamic parameters. The patient remains hemodynamically unstable
over the next several days and requires multiple blood transfusions.
The metabolic response to multiple trauma, burns, or sepsis is often characterized by a rise in the resting
energy expenditure, erosion of lean body mass, and a negative nitrogen balance. This stress response is
mediated by several cytokines, including IL-1, IL-6, and TNF-О±. The actions of these cytokines may result
in elevations of serum cortisol, glucagon, epinephrine, and counter-regulatory hormones associated with
the mobilization of energy stores and protein breakdown. Although cytokines exert many beneficial
effects such as T-lymphocyte recruitment, tissue repair, and mobilization of energy substrates, excess
production may result in an exaggerated inflammatory response and depletion of protein and energy
stores. As a consequence of altered substrate and energy requirements manifested in these patients,
specialized nutrition support becomes an essential component in the management of critically ill and
injured patients.
Metabolic Considerations
A. Increased stress hormones
1. Glucagon
2. Steroids
3. Catecholamines
B. Increasing resting energy expenditure
C. Increased protein catabolism
D. Role of endogenous cytokines
II. Identification of Patients at Risk for Malnutrition
A. Involuntary weight loss ≥10% within 6 months
B. Inadequate nutrition intake for > 7 days
III. Selection of Route for Delivery of Specialized Nutrition Support
A. Advantages of enteral nutrition over parenteral nutrition:
1. Ease and safety of administration
2. More economical
3. More physiologic:
a) Maintenance of GI integrity
b) Bacterial translocation prevention
c) Decreased septic complications
B. Contraindication to use of enteral nutrition:
1. Nonfunctioning gut.
IV. General Practice Guidelines for Parenteral Nutrition
A. Candidates for parenteral nutrition are patients who cannot, should not, or will not eat to maintain
their nutrient stores
I.
65
ASCCA Residents Guide to the ICU
B. Peripheral parenteral nutrition should not be used for greater than 2 weeks
C. Central parenteral nutrition is necessary when patients cannot ingest or absorb oral nutrients for
> 2 weeks
V. Parenteral Nutrition Solutions
A. Nonprotein energy
1. Dextrose–available from D5W to D70W
a) Usual requirements: do not exceed 5-7 mg/kg/min (equivalent to 25-30 kcal/kg/day)
b) Should not exceed 5 mg/kg/min in the critical care setting
2. Lipid–available in concentrations from 10% to 30%
a) Should provide 1-4% of total calories as lipid to prevent essential fatty acid deficiency
b) Should limit to < 30% of total calories provided and administer over 24 hrs. to avoid
immunosuppressive effects
3. Calculating caloric requirements:
a) Basal Energy Expenditure (BEE) is calculated with the Harris Benedict Equation as
follows:
(1) Females = 655 + (9.6 x Wt) + (1.7 x Ht) - (4.7 x A)*
(2) Males = 66 + (13.7 x Wt) + (5 x Ht) – (6.8 x A)*
*Wt (kg), Ht (cm), A (yrs)
b) The BEE can be multiplied by the following stress factors to estimate patients’ total
caloric requirements:
(1) Maintenance or minimal stress:
BEE x 1.2-1.3
(2) Moderate stress:
BEE x 1.5
(3) Severe stress (thermal injury):
BEE x 1.5-2
c) Total caloric requirements can also be estimated based upon patients’ stress levels and
their weight (kg):
(1) Maintenance or minimal stress:
25-30 kcal/kg/day
(2) Moderate stress:
30-35 kcal/kg/day
(3) Severe stress (thermal injury):
35-40 kcal/kg/day
B. Protein
1. Dosing
a) Maintenance or minimum stress:
1.0-1.5 g/kg/day
b) Moderate stress or repletion:
1.5-1.75 g/kg/day
c) Severe stress (thermal injury)
1.75-2 g/kg/day
C. Micronutrients
1. Suggested electrolyte requirements per 1000 kcal:
• Na+
40-50 mEq
• Mg+2 8-12 mEq
• K+
40 mEq
• Ca+2
2-5 mEq
• C140 mEq
• PO4-3 15-25 mM
2. Vitamins–most institutions use a 12-vitamin preparation which contains all known vitamins
EXCEPT Vitamin K (MVI-12В®, MVI 9+3В®)
3. Trace element requirements (MTE-4В®, MTE-5В®):
• Zinc
5 mg/day
• Copper
0.5-1.5 mg/day
• Manganese
0.15-0.8 mg/day
• Chromium
10-15 mcg/day
• Selenium
40-60 mcg/day
VI. General Practice Guidelines for Enteral Nutrition
A. Candidates for enteral nutrition are patients at risk for malnutrition in whom oral feedings are
inadequate to maintain their nutritional status.
B. Enteral access should be obtained in critically ill patients whenever possible; either at the time of
surgery with direct enteral access or with nasoenteric feeding tubes.
C. Parenteral feedings should be administered ONLY when enteral access cannot be obtained or
when feeding into the GI tract is contraindicated.
VII. Enteral Nutrition Administration Routes and Techniques
A. Route
66
ASCCA Residents Guide to the ICU
Oral
Nasally-place tubes
a) пЈїпЈїNasogastric (NG), nasoduodenal (ND), nasojejunal (NJ), orogastric (OG). Small
bore feeding tubes should be inserted without the aid of capnography or colorimetric CO2
detection or electromagnetic guidance to reduce the risk of intrapulmonary placement.
3. Tube enterostomy
a) Open gastrostomy vs Percutaneous endoscopic gastrostomy (PEG)
b) Jejunostomy
(1) Percutaneous endoscopic jejunostomy (PEJ)
(2) Permanent jejunostomy
VIII.Categories of Enteral Nutrition Products
A. Polymeric, nutritionally-complete enteral products
B. Polymeric, nutritionally-complete concentrated enteral products
C. Polymeric, nutritionally-complete oral supplements
D. Chemically-defined, nutritionally-complete enteral products
E. Fiber-containing, nutritionally-complete enteral products
F. Peptide-based, nutritionally-complete enteral products
G. Disease specific enteral products (several are nutritionally incomplete)
1. Renal failure (low protein, low electrolytes)
2. Hepatic failure (increased branched-chain AAs, decreased aromatic AAs)
3. Pulmonary failure (increased fat content, decreased carbohydrate content)
4. Diabetes (increased fat content, decreased carbohydrate content, added fiber)
5. Immunocompromised states (enhanced with arginine, omega-3 fatty acids, nucleotides, betacarotene)
6. Stress states (enhanced branched-chain AAs or increased protein or both)
IX. Monitoring Guidelines or Parenteral and Enteral Nutrition
A. Verify placement of central venous catheter or feeding tube prior to starting feeding
B. Order strict In/Outs during feeding
C. Initiate a glycemic protocol.
D. For enteral nutrition, elevate patient’s head of bed 300 at all times
E. For enteral nutrition, check gastric residuals Q 6 hr and hold feeds for >200 mL
F. If parenteral nutrition is interrupted for any reason, start D5W or D10W at the same rate
G. Order appropriate labs (minimum of Chem 7, Chem 15, prealbumin weekly)
1.
2.
This chapter is a revision of the original chapter authored by Gordon Sacks, Pharm.D.
READING LIST:
1. A.S.P.E.N. Board of Directors. Guidelines for the use of parenteral and enteral nutrition in adult and pediatric
patients. J Parent Enter Nutrit JPEN 1993;17(supplement):1SA-52SA. A through collection of practical advice on
providing nutritional support to adult and pediatric patients with numerous disease states.
2. McMahon MM, Farnell MB, Murray MJ. Nutritional support of critically ill patients. Mayo Clin Proc
1993;68:911-20. A short, concise, easy-to-read review of nutrition support in ICU patients.
3. Cerra F, et al. Applied nutrition in ICU patients: A consensus statement of the American College of Chest
Physicians. Chest 1997;111:769-778. Addresses nutritional considerations in ICU patients in addition to
highlighting new developments in nutritional support of these patients (e.g., glutamine and arginine).
4. The Veterans Affairs Total Parenteral Nutrition Cooperative Study Group. Perioperative total parenteral nutrition
in surgical patients. N Engl J Med 1991;325-525-532. Landmark study to demonstrate that at least 7-15 days of
preoperative parenteral nutrition is beneficial in decreasing noninfectious complications (anastomotic leaks, wound
dehiscence) in severely malnourished patients; in this study infectious complications were actually increased in
borderline or mildly malnourished patients.
5. American Gastroenterological Association. Medical position statement: Guidelines for the use of enteral nutrition.
Gastroenterology 1995;108:1280-1301. The most current and comprehensive review of enteral nutrition.
6. Klein S, Kinney J, Jeejeebhoy K, et al. Nutrition support in clinical practice: review of published data and
recommendations for future research directions. J Parent Enter Nutrit JPEN 1997;21:133-156. A critical review of
67
ASCCA Residents Guide to the ICU
the medical literature summarizing important points on nutrition assessment and nutrition support in
gastrointestinal, wasting, critically ill, and perioperative realms.
7. Talpers SS, Romberger DJ, Bunce SB, et al. Nutritionally associated increased carbon dioxide production: Excess
total calories vs. High proportion of carbohydrate calories. Chest 1992;102:551-555. An important study in which
the results suggested that lowering total calories, not just replacing carbohydrate calories with fat, would lower the
RQ in patients receiving parenteral nutrition while requiring mechanical ventilation.
8. Kudsk KA, Croce MA, Fabian TC, et al. Enteral vs. parenteral feeding: effects on septic morbidity following blunt
and penetrating trauma. Ann Surg 1992;215:503-513. Landmark study to demonstrate the beneficial effects of
enteral over parenteral nutrition in trauma patients.
9. Mirtallo JM, Schneider PJ, Mavko K, et al. A comparison of essential and general amino acid infusions in the
nutritional support of patients with compromised renal function. J Parent Enter Nutrit JPEN 1982;6:109-113. One of
the few well-designed studies to demonstrate that general amino acids (essential plus nonessential) are clinically
efficacious and more cost-effective than essential amino acids in patients with renal insufficiency.
10.Nompleggi DJ. The basic principles of nutritional support in the intensive care unit. Manual of Intensive Care
Medicine 2006; 86: 463-465. Fully updated chapter containing precise information regarding nutrition in critically ill
patients for bedside consultation.
QUESTIONS:
15.1.
A.
B.
C.
D.
Which of the following mineral deficiencies is most closely associated with a reduction in
cellular respiration resulting from a decrease in oxygen transport and oxygen uptake by
tissues?
Calcium
Chromium
Phosphorus
Magnesium
15.2.
A.
B.
C.
D.
Which of the following are considered to be essential fatty acids in the human?
Acetate, propionate, butyrate
Leucine, isoleucine, valine
Oleic, palmitoleic, linoleic
Linoleic,О±-linolenic,arachidonic
15.3.
When compared to patients receiving parenteral nutrition, early enteral feeding after major
trauma is associated with:
Greater caloric intake
Decreased mortality
Decreased days of antibiotics
Fewer postoperative infections
A.
B.
C.
D.
15.4.
A.
B.
C.
D.
15.5.
A.
B.
C.
D.
A 60-year-old WF (40 kg) has Stage III ovarian CA and experiences enteritis secondary to
extensive radiation therapy. Her GYN team desires bowel rest for 2 weeks and parenteral
nutrition to help restore her nutritional status. Which parenteral nutrition regimen is the most
appropriate endpoint for her?
D25W, A4.25%, IVFE 20% (125 mL) at 100 mL/hr
D35W, A6%, IVFE 20% (125 mL) at 42 mL/hr
D15W, A5%, IVFE 20% (125 mL) at 50 mL/hr
D20W, A4%, IVFE 20% (250 mL) at 63 mL/hr
Which of the following surgical procedures is most often associated with malabsorption
requiring parenteral nutrition?
Esophagogastrectomy
Pyloroplasty
Subtotal gastrectomy
Vagotomy
68
ASCCA Residents Guide to the ICU
15.6.
A.
B.
C.
D.
15.7.
A.
B.
C.
D.
What is the minimum period that severely malnourished patients can receive preoperative
parenteral nutrition and still achieve a significant reduction in postoperative morbidity and
mortality?
1-2 days
3-5 days
7-15 days
21-28 days
Initiation of parenteral nutrition should be considered in adult patients with nonfunctional
gastrointestinal tracts and:
3% weight loss of usual body weight over 1 month
Inadequate oral nutrient intake for 3 days
5% weight loss of usual body weight over 1 month
Inadequate oral nutrient intake for 5 days
69
ASCCA Residents Guide to the ICU
16. Routine Monitoring in the ICU
Daniel R. Brown, M.D., Ph.D.
A 65-year-old male has just undergone a carotid endarterectomy under general anesthesia. His arterial
line was inadvertently pulled while he was transferred to the ICU bed. He is somnolent but easily
arousable. Past medical history is significant only for peripheral vascular disease.
Monitoring in the ICU may exist for diverse reasons: observation for detection of changes in physiologic
status, intensive titration of therapy, and/or detection of breaches of safety (e.g., ventilator disconnection),
among others. By definition, patients are admitted into the ICU because they require a higher level of
care. Monitoring is a significant part of that care, whether it involves monitoring by human caregivers
and/or medical machines. The patient’s condition and the likelihood that his condition will rapidly change
should determine the level of monitoring. One task of the ICU physician is to continually evaluate the
need of a patient for a particular level of monitoring. Ideally, monitors should be used to confirm or further
define problems that have been discovered by careful physical examination and clinical assessment.
I.
Noninvasive monitoring in the ICU
A. Blood pressure
1. Use of noninvasive BP vs. invasive BP determined as in O.R.
2. Similar concerns as to cuff size and site of measurement
B. ECG
1. 5-lead and ST-segment analysis in patients at risk for ischemia
2. Arrhythmias may be first sign of electrolyte imbalance
3. Rhythm and rate are monitored as part of vital signs
C. Pulse oximetry (SpO2)
1. A monitor of oxygenation
a) Use of supplemental oxygen negates ability to use SpO2 as monitor of ventilation
b) Not a monitor of perfusion unless plethysmographic waveform is utilized; only a crude
measure if used
c) More artifacts in ICU (vs. O.R.) with patient movement
D. Temperature
1. Site of measurement important, must be noted with each measurement
2. Fever is associated with significant increase in mortality
E. Peripheral nerve stimulator
1. Mandatory use when patient under neuromuscular blockade
2. Response to train of four (TOF) to be recorded with regular vital signs
F. Capnography
1. Can be used in non-intubated patients as a monitor of ventilation
2. Difficulty in long-term use with humidification of ventilator circuit
3. End-tidal CO2: arterial CO2 gradient changes can be due to ventilation or perfusion changes;
end-tidal CO2 should be noted with each blood gas
G. Non-invasive cardiac output (CO)/cardiac function
1. Echocardiography: transthoracic (TTE) or transesophageal (TEE)
• Accurate TTE may not be possible in edematous patients; can be used for detection of
ischemia, assessment of preload, and ejection fraction. TEE better for assessment of
right ventricle, pulmonary circulation and aortic dissection.
2. Doppler noninvasive/thoracic impedance
• Newer monitors; can correlate well with invasive CO monitor. Can be used for trending
and rapid evaluation of changes in therapy.
3. Respiratory CO2-derived cardiac output measurements in mechanically ventilated patients via
proprietary technology
H. Radiology/Ultrasound
1. Consult with radiologist to make sure the test is the right one for the presumed diagnosis
2. Many tests may need proper preparation of the patient (NPO, bowel prep, contrast, etc.).
3. Prepare patients with poor renal function (avoid hypovolemia, consider bicarbonate and/or N-
70
ASCCA Residents Guide to the ICU
I.
acetylcysteine) or allergy to contrast (consider histamine blockers and/or steroids).
4. Bedside ultrasound can be used for serial evaluation and follow-up exams
Gastric tonometry
1. Measures changes in gastric mucosal CO2 by allowing equilibration of CO2 partial pressures
between a fluid-filled balloon and the mucosal layers
2. Used as a monitor of tissue hypoxia because the increase in CO2 is a reflection of decreased
perfusion or anaerobic metabolism
II. Invasive monitoring
A. Arterial lines
1. Used with unstable patients or those receiving vasoactive infusions
2. Insertion either percutaneous or by cut-down
a) Cut-down associated with more morbidity
3. Sites: radial, femoral, dorsalis pedis, axillary, posterior tibial, temporalis
a) Check for collateral circulation (controversial)
b) Axillary site with higher risk of infection
B. Central lines (see section 17: Rational Use of the PA Catheter)
1. Sites: Internal/external jugular, subclavian, femoral
2. Cardiac output measurements via non-pulmonary artery catheter devices; may include
additional measurements such as lung water
3. Associated morbidity and mortality (including infection, bleeding, arrhythmias, emboli, arterial
puncture/catheterization, pneumothorax (subclavian and internal jugular), thrombosis,
catheter misdirection, or perforation of insertion vessel, more central vein or even the heart).
C. ICP monitoring
1. Useful in closed head injury patients at risk for intracranial hypertension and herniation
2. Ventriculostomy vs. bolt: differences in infection rates, ease of placement, therapeutic
possibilities.
This chapter is a revision of the original chapter authored by Clayton C. Cowan, M.D. and Thomas H. Fuhrman, M.D.
READING LIST:
1. Marino P. The ICU Book. Philadelphia: Lea & Febiger; 1991, pp. 89-90. An excellent overview of critical care
concepts. Very readable and concise.
2. Zimmerman ZL, editor. Critical Care Refresher Course. Society of Critical Care Medicine; 1997, p. 171. Useful as a
review text for exam study. Less detailed than the usual textbook.
3. Mark JB. Getting the most from a CVP catheter. ASA Refresher Course Lectures, 1997. A great review of CVP
waveform interpretation.
4. Howell MD, Curley FJ, and Smyrnios NA: Routine monitoring in critically ill patients. In: Irwin RS and Rippe JM
(eds) Irwin and Rippe’s Intensive Care Medicine 6th Edition. Wolters Kluwer/Lippincot Williams & Williams
(Philadelphia) 2008.
5. Zimmerman ZL, editor. Critical Care Refresher Course. Society of Critical Care Medicine; 1997, pp. 112-113.
QUESTIONS:
16.1.
A.
B.
C.
D.
E.
All of the following statements concerning noninvasive BP monitoring are true EXCEPT:
In elderly, hypertensive subjects, the diastolic pressure may exceed the actual pressure by 10 mmHg.
Noninvasive BP measurements are significantly more inaccurate than arterial line pressures in low-flow
states.
Noninvasive BP measurements overestimate the actual systolic pressure of patients with cardiac
failure.
An indication for the use of invasive arterial pressure monitoring is the need to begin vasoactive drug
therapy.
In obese patients, if the noninvasive BP cuff is too small, the BP will read higher than the actual
pressure.
71
ASCCA Residents Guide to the ICU
16.2.Indications for inserting or maintaining a PA catheter include all of the following EXCEPT:
A. High-risk surgery planned in a patient with poor left ventricular function
B. Shock which is refractory to multiple therapeutic modalities
C. Pulmonary edema of uncertain etiology
D. Oliguria in a patient with normal left ventricular function
E. Titration of pressors and/or inotropes
16.3.
A.
B.
C.
D.
E.
16.4.
A.
B.
C.
D.
E.
16.5.
A.
B.
C.
D.
E.
All of the following statements concerning the CVP tracing are true EXCEPT:
“A” waves result from atrial contraction.
“C” waves are a result of tricuspid valve closure and ventricular contraction.
When a patient has a junctional rhythm, prominent “A” waves (cannon “A” waves) may result.
The waves of a CVP tracing tend to be sharper and more distinct with fast heart rates than with normal
heart rates.
Cardiac tamponade changes the normal, biphasic CVP tracing into a monophasic tracing, obliterating
the “Y” descent.
True statements concerning the internal jugular approach of central venous cannulation include all of
the following EXCEPT:
In patients with tracheostomies, the internal jugular approach has a higher risk of infection than the
subclavian approach.
With an internal jugular approach, there is no risk of pneumothorax.
It is safe to attempt an internal jugular approach on the same side as a failed subclavian approach
without first obtaining a chest X-ray.
Accidental puncture of the carotid artery occurs in 2-10% of internal jugular vein cannulation attempts.
The presence of an internal jugular catheter usually results in greater patient discomfort than a
subclavian catheter.
All of the following are true concerning intracranial pressure monitoring EXCEPT:
The ventriculostomy is a more accurate and reliable method for measuring ICP than the subarachnoid
hollow screw (bolt).
An advantage of the ventriculostomy over the bolt is that it allows for the drainage of CSF with
increases in ICP.
Ventriculostomies carry an 8%-11% risk of infection within 5 days of placement.
Since a ventriculostomy is directly attached to a water column manometer, it requires no electrical
equipment.
The subarachnoid hollow screw or bolt carries a higher risk of infection and bleeding complications than
ventriculostomy.
72
ASCCA Residents Guide to the ICU
17. Rational Use of the Pulmonary Artery Catheter
(PAC)
Fanni Nhuch, M.D., Ricardo Martinez-Ruiz, M.D., Miguel A. Cobas, M.D.
A 73-year-old man presents to the hospital emergency medicine department with a chief complaint of
progressive shortness of breath. The patient has a history of a dilated cardiomyopathy thought to be
ischemic in nature with an ejection fraction of less than 20%. Additionally, he has a history of lifethreatening cardiac arrhythmias and has an automatic implantable cardiodefibrillator. Over the week prior
to admission, the patient has complained of a cough and other cold symptoms with progressive
respiratory failure. Chest x-ray on admission shows a prominent left lung infiltrate involving the entire
lung and some infiltrate on the right. The patient is admitted to the medical/surgical intensive care unit.
The following day the patient develops progressive respiratory failure with tachypnea and is intubated and
started on mechanical ventilation. Because the etiology of his respiratory failure is unclear (CHF vs.
pneumonia), a pulmonary artery catheter (PAC) is inserted. His hemodynamic profile revealed a low
blood pressure (BP), low filling pressures with a low cardiac output (CO). Modest amounts of fluids were
given which resulted in an increase in CO with just a slight increase in filling pressures. The calculated
systemic vascular resistance (SVR) was low after volume expansion, suggesting a picture of low tone
state from sepsis. After the patient responded to appropriate antibiotics he was able to be weaned from
the ventilator. The patient spent some extra days at the facility and was discharged home in his previous
state of health.
Although condemned in some recent literature, the PAC has enjoyed a long and robust history of use for
clarification of cardiovascular status and manipulation of hemodynamic parameters. With the preceding
case, the data clarified the diagnosis of pneumonia, as the patient had adequate cardiac function without
evidence of congestive heart failure and fluid overload. Management of this patient based only on clinical
skills could certainly have missed the proper diagnosis and delayed the appropriate treatment. Proper
use of the PAC, however, requires a sophisticated understanding of proper data collection and
interpretation. When used for the proper indications and interpreted appropriately, data derived from a
PAC is indispensable in augmenting information about a critically ill patient’s clinical situation.
I.
Indications for use of the PAC: The PAC is used to obtain a hemodynamic picture of the
cardiovascular system. This means we are trying to establish the relationship between the filling
pressures of the cavities and their function i.e. generate a Starling curve for the ventricle. This is done
by obtaining a pulmonary capillary wedge pressure (PCWP) and CO, two of the distinctive
parameters that this catheter can provide. It is also used to assess oxygen delivery by measuring
mixed venous oxygen saturation (SvO2).
A. Monitoring filling pressures: PCWP
1. When the balloon is inflated the catheter tip moves from a central position to wedge in a more
peripheral vessel. Flow then stops and a static fluid column is created.
2. The pulmonary venous system is a low-resistance circuit; the pressure at this junction
normally closely approximates mean left atrial pressure. Mean left atrial pressure
approximates left ventricular end-diastolic pressure unless an atrial myxoma, mitral disease
or severe left ventricular failure exists.
3. By itself PCWP is an inadequate measure of the distending force exerted upon the ventricular
myocardium and an unreliable estimator of preload or fiber stretch actually achieved. Two
other important variables must be taken into account: the distensibility of the ventricular
myocardium (myocardial compliance) and the pressure surrounding the heart and great
vessels (transmural pressure).
4. Ventricular Compliance: the failure of PCWP to correlate with the left ventricular diastolic
volume is at least partially accounted for by the variations in left ventricular compliance,
secondary to events such as myocardial ischemia, ventricular hypertrophy, etc.
5. Transmural Pressure= PCWP - juxtacardiac pressure (stretching force exerted upon the left
ventricle). Under most circumstances juxtacardiac pressure closely parallels mean
73
ASCCA Residents Guide to the ICU
B.
C.
D.
E.
II.
III.
intrapleural pressure, and, at the end-expiration is when pressure inside the chest cavity is
closest to atmospheric (that’s why we should always measure PCWP at the end of
expiration).
Thus to accurately interpret the PCWP, one must always consider the extramural pressure:
pleural pressure, PEEP, auto-PEEP, increased intra-abdominal pressure (abdominal
compartment syndrome), cardiac tamponade, etc.
Monitoring cardiac output
1. After injection of a known quantity of cold solution (usually 10 ml or normal saline), blood
temperature is measured continuously by a rapidly responding thermistor positioned
downstream in the lumen of central pulmonary artery.
2. The change in temperature over time (area under the curve) is inversely proportional to the
blood flow acting to restore the thermal baseline. A normal curve peaks early and exhibits a
smooth contour. For variations to be minimized, each recorded value should be the average
of three or more determinations.
3. Except during significant hypothermia or very high cardiac output states, there will be no
difference if the solution injected is at 0 degree Celsius or ambient temperature, if the
computer is adjusted to accept relevant input.
4. The port at which the cold solution will be injected does not affect the recorded value as long
as it is upstream from the tricuspid valve. Also a slow and interrupted injection of the solution
may give rise to erroneous values.
5. Errors can also result from operator negligence: Cardiac output will not be accurate if the
injectate temperature constant is entered incorrectly, if the injected volume differs from the
expected amount, or if the injectate probe is not immersed in the environment of its reference
temperature. An intracardiac shunt or significant disease of the tricuspid or pulmonic valve
may also interfere with accurate output determination.
Monitoring oxygen transport:
1. The PAC catheter allows sampling of pulmonary artery blood for determination of SvO2.
2. The tip of the catheter should be located proximally in the pulmonary artery, and blood should
be withdrawn slowly so that admixture with oxygenated post capillary blood does not occur.
3. SvO2 is a good parameter of the relationship between oxygen delivery and consumption.
4. Changes in SvO2 vary directly from changes in cardiac output, hemoglobin concentration,
and SaO2 and inversely with oxygen consumption.
5. SvO2 normal value is 75%.
Provide cardiac pacing capability as well as patient monitoring:
1. Pacing PAC
2. Paceport catheter with special lumen for pacing wire
Provide special right ventricular function monitor:
1. Right ventricular ejection fraction
2. Right ventricular end diastolic volume
Insertion procedure
A. Intravascular route
1. Right or left internal jugular
2. Right or left subclavian
3. Right or left femoral
• “If the blood can float to the heart, then you can float the Swan to the heart”
B. Technique of insertion
1. Access vein with side arm introducer inserted by Seldinger technique
2. 8-French catheter will pass through 9-French introducer
3. Balloon-tipped catheter floated into position with flow of blood through right ventricle into
pulmonary artery (Remember: advance with the balloon inflated, pull back with the balloon
deflated!)
4. Position of catheter during insertion determined by identifying right atrial(20 cm), right
ventricle(35 cm), pulmonary artery(45 cm) and pulmonary capillary wedge(50 cm) traces on
bedside monitor.
5. Alternatively, fluoroscopy can guide the passage of the catheter
Choice of catheters determined by monitoring and/or treatment needs of the patient
74
ASCCA Residents Guide to the ICU
A. Standard thermodilution catheter provides cardiac output, pulmonary artery pressures, and
pulmonary capillary wedge pressure
B. Continuous cardiac output catheter provides the same information but a heated wire in the
catheter determines flow or cardiac output data regularly and automatically
C. Fiberoptic catheter measures mixed venous oxygen saturation by reflectance spectrophotometry
and can be combined with standard or continuous cardiac output catheter
D. Pacing catheters permit cardiac pacing by insertion of a wire through a pace port or by the use of
pacing electrodes on the surface of the catheter
E. Right ventricular ejection fraction catheter also determines end diastolic volume and right
ventricular ejection fraction
IV.
Complications of a PAC
A. Insertion complications:
1. Vascular injury
2. Bleeding
3. Cardiac arrhythmias
4. Pneumothorax
5. Air embolism
B. Monitoring complications:
1. Infection
2. Pulmonary infarction
3. Pulmonary artery rupture
4. Right bundle-branch block (or complete heart block if existing left bundle-branch block)
5. Venous thrombosis
6. Mural thrombus
7. Valvular or endocardial vegetations
V.
Value of the PAC:
A. The right diagnosis leads to the right treatment.
B. It is only a diagnostic tool; its benefits depend on what you do with the numbers you get. Recent
criticism of the catheter claims increased mortality rate with its use. Possible causes for these
findings are:
1. Poorly matched patient populations
C. Poor understanding and improper utilization of data obtained by the catheter may explain the lack
of benefit.
D. Careful interpretation of data helps clarify important questions in complicated patients.
E. It is important to make a hemodynamic diagnosis and then treat appropriately i.e. if the patient
has a low tone state with good or high CO and low BP, the therapy should be to increase tone by
giving a vasopressor. If the numbers give you a low CO with low filling pressures, the presumptive
diagnosis is hypovolemia and fluids should be administered.
F. Always interpret the data taking into account the transmural pressure gradient and compliance of
the ventricle.
G. A filling pressure in isolation tells you very little about the cardiovascular system status. Always
associate a filling pressure with a function parameter (CO, CI, SV). You are generating the
Starling curve for that ventricle!
This chapter is a revision of the original chapter authored by John W. Hoyt, M.D.
READING LIST:
1. Connors AF Jr, Dawson NV, Shaw PK. Hemodynamic status in critically ill patients with and without acute heart
disease. Chest 1990;98:1200-1206.
This paper might be characterized as a second generation justification for use of the PA catheter. Now the issue
was not demonstrating the adequacy or inadequacy of clinical skills when compared to the PA catheter but instead
75
ASCCA Residents Guide to the ICU
the impact of hemodynamic information on clinical decisions. Dr. Connors found that physicians changed
treatment 50% of the time when they had hemodynamic information obtained from a PA catheter.
2. Connors AF Jr, McCaffree DR, Gray BA. Evaluation of right heart catheterization in the critically ill patient without
acute myocardial infarction. N Engl J Med 1983;308:263-267.
Appearing in the New England Journal, this paper had a substantial effect on the use of PA catheters. The authors
demonstrated that clinical skills were not adequate to predict cardiac output and pulmonary capillary wedge
pressure. They demonstrated that only by invasive monitoring could one obtain accurate information for better
medical decision making.
3. Connors AF, Speroff T, Dawson NV, et al. The effectiveness of right heart catheterization in the initial care of
critically ill patients. JAMA 1996;276(11):889-897.
This paper is the latest in attacks on the value of a PAC. The authors compare a population of patients monitored
with a PA catheter to a matched population of unmonitored patients. The results of the paper show a higher
mortality rate in the monitored population. Accompanying editorial suggested the moratorium on the use of a PA
catheter if clear evidence cannot be obtained to demonstrate the value of the PA catheter.
4. Eisenburg PR, Jaffe AS, Schuster DP. Clinical evaluation compared to PACization in the hemodynamic assessment
of critically ill patients. Crit Care Med 1984;12(7):549-553.
This manuscript is a repeat of the Connors paper in the New England Journal from a year before. The authors
substantiated Connors’ findings that clinical skills are inadequate for accurate information about cardiac function
and good medical decision-making.
5. Fein AM. Is PACization necessary for the diagnosis of pulmonary edema? Am Rev Respir Dis 1984;129:1006-1009.
Pulmonary edema may have a cardiac or noncardiac diagnosis. This paper demonstrates the difficulty of
accurately categorizing pulmonary edema by clinical skills alone. Once again, this manuscript has been commonly
quoted as a justification for invasive hemodynamic monitoring with a PA catheter.
6. Forrester JS, Diamond G, McHugh TJ, et al. Filling pressures in right and left sides of the heart in acute myocardial
infarction: A reappraisal of central venous pressure monitoring. N Engl J Med 1971;285:190.
This historical paper was a significant clinical force in increasing the use of PA catheters in the ICU. The paper
demonstrated that patients with cardiac disease and a normal CVP might have low or normal or high pulmonary
capillary wedge pressures. Thus the clinical value of the PA catheter was substantiated and the need was
demonstrated for more invasive monitoring of intravascular volume for better clinical decision-making about fluid
administration.
7. Gore JM, Goldberg RJ, Spodick DH, et al. A community-wide assessment of the use of Pulmonary Artery Catheters
in patients with acute myocardial infarction. Chest 1987;92;721-727.
This was the original clinical attack on the value of the PA catheter. A retrospective study of cardiac patients in
community hospital CCUs demonstrated a higher mortality rate when a PA catheter was used. There was no
attempt to match monitored and unmonitored patients except for diagnosis. An accompanying editorial suggested
that many lives might be saved if clinicians stopped using the PAC.
8. Iberti TJ, Fischer EP, Leibowitz AB, et al. A multicenter study of physician’s knowledge of the Pulmonary Artery
Catheter. JAMA 1990;264:2928-2932.
One of the key issues in the most recent Connors paper demonstrating an increased mortality rate with the use of a
PA catheter is user education. In this paper Iberti demonstrated a surprisingly poor knowledge level among
clinicians commonly using the PA catheter. This paper has been recently repeated by Trotter once again
demonstrating a knowledge defect. It is now being speculated that increased mortality rate with the use of the PA
catheter might be due to user error and poor knowledge.
9. Swan HJC, Ganz W, Forrester J, et al. Catheterization of the heart in man with the use of flow directed balloontipped catheter. N Engl J Med 1970;283-447.
This is a historical paper of great significance to critical care. It was in this manuscript that the use of the balloontipped and flow-directed PAC was first introduced.
10.Tuman KJ, McCarthy RJ, Speiss BD, et al. Effect of PACization on outcome in patients undergoing coronary artery
surgery. Anesthesiology 1989;70:199-206.
This paper is a more recent and better designed prospective study of the value of the PA catheter. Unfortunately,
Tuman was unable to demonstrate a difference in cardiac surgery patients. His population size may be small, and
in the sickest patients who were unmonitored there could be some information that eventual monitoring made a
difference in reversing their cardiac problems. The paper is certainly not convincing in its support of
hemodynamic monitoring.
11.Wheeler AP, Bernard GR, Thompson BT, Shoenfeld D, et al. Pulmonary-Artery versus Central Venous Catheter to
Guide Treatment of Acute Lung Injury. The New England Journal of Medicine. Boston: May 2006. Vol. 354, Iss 21;
pg 2213-2225.
This study is a randomized trial comparing hemodynamic management guided by PAC with hemodynamic
76
ASCCA Residents Guide to the ICU
management guided by central venous catheter. The primary outcome was mortality during the first 60 days before
discharge home. The conclusion shows that PAC-guided therapy did not improve survival or organ function, but
was associated with more complications than CVC guided therapy.
12.The ESCAPE Investigators and ESCAPE Study Coordinators: Evaluation Study of Congestive Heart Failure and
PACization Effectiveness (The ESCAPE Trial). JAMA, October 5,2005. Vol 294, No 13: 1625-1633.
Randomized Controlled Trial to determine wheter PAC use is safe and improves clinical outcomes in patients
hospitalized with severe symptomatic and recurrent heart failure. No affect in overall mortality and
hospitalization.
13.Marini JJ. Obtaining Meaningful Data From the Swan-Ganz Catheter. Respiratory Care. July 1985, Vol 30, No 7:
572-584.
Classic review article about hemodynamics and PA catheter. It discusses the issue of the transmural pressure
gradients, the effect of compliance in the PCWP and its relation to volume assessment.
QUESTIONS:
17.1.
A.
B.
C.
D.
Appropriate indications for the PAC include all of the following EXCEPT:
Monitor intravascular volume in the hypovolemic patient
Monitor cardiac output in the patient with an acute myocardial infarction
Measure left ventricular ejection fraction in a patient with cardiomyopathy
Determine oxygen transport and extraction in the septic patient
17.2.
A.
B.
C.
D.
Which of the following answers best justifies the use of a fiber-optic PAC:
There is a reliable correlation between SvO2 and cardiac output
There is a reliable correlation between SvO2 and the global balance of oxygen supply and demand
There is reliable correlation between SvO2 and oxygen consumption
There is reliable correlation between SvO2 and arterial oxygenation
17.3.
Which of the following patient populations are most likely to have a rupture of the pulmonary artery
during hemodynamic monitoring with a PAC?
Patients with pulmonary artery hypertension
Patients with cardiogenic shock
Patients with septic shock
Patients with a low left ventricular end diastolic volume
A.
B.
C.
D.
17.4.
A.
B.
C.
D.
17.5.
A.
B.
C.
D.
In an average size adult male of approximately 70 kg one might expect to reach the right ventricle
during insertion of the PAC through the right internal jugular vein at which of the following distances:
20-25 cm
30-35 cm
40-45 cm
50-55 cm
On which condition the mixed venous oxygen saturation correlates with the level of perturbation in
systemic oxygen delivery?
Septic shock
Anaphylactic shock
Spinal shock
Cardiogenic shock
77
ASCCA Residents Guide to the ICU
18. Echocardiography in the Intensive Care Unit
Frank Rosemeier, M.D., Miguel A. Cobas, M.D.
A known hypertensive, 68-year-old obese, female accident victim with a femur fracture and blunt chest
and abdominal trauma becomes acutely hypoxemic and hemodynamically unstable within the first hour of
admission to the intensive care unit. She was endotracheally intubated and sedated for respiratory
distress in the ER with otherwise stable vital signs and transferred to the intensive care unit after initial
management according to ATLS protocol. On examination, she is agitated, HR 110, BP 75/55, SpO2 92%
on FiO2 1.0. Breath sounds are heard bilaterally, JVP is not visualized, heart sound appear distant,
abdominal examination is negative for acute peritoneal signs and distention, but limited due to body
habitus. Despite manual bag ventilation with 100% oxygen and a fluid challenge via a large bore
intravenous catheter her cardiopulmonary status continues to deteriorate.
Unexplained hemodynamic instability is an archetypical indication for the use of ultrasonography as a
diagnostic tool in the critically ill patient. Both, transthoracic (TTE) and transesophageal (TEE)
echocardiography are useful as rapid, noninvasive bedside techniques to diagnose and manage
refractory hypotension in a time sensitive manner. Especially after standard management including
volume resuscitation and administration of inotrope and vasopressor infusions fail to stabilize the patient,
additional information obtained via real-time images or Doppler flow patterns may lead to an elusive
diagnosis. Frequently, resuscitation efforts are redirected based on the information obtained from
echocardiography. Some observational studies indicate that in about half of critically ill patients, TEE
changes management strategies despite the presence of a pulmonary artery catheter. Like most other
monitors and diagnostic procedures however, echocardiography should be regarded as an adjunct, which
requires expertise to integrate the images with other data relevant to the overall assessment of the
patient.
The preceding case presentation demonstrates the diagnostic dilemma of unexplained and delayed
shock in a multitrauma patient. The scenario is compatible with hypovolemia secondary to concealed
hemorrhagic shock from a delayed ruptured spleen, but is also consistent with a variety of other scenario
such as ventricular failure from acute myocardial infarction, acute valvular dysfunction, cardiac
tamponade, fat embolism, and traumatic aortic dissection. Echocardiography may circumvent the timeconsuming process of placing invasive catheters, which require additional monitoring equipment,
accurate interpretation of hemodynamic parameters and have been to be shown to be of variable benefit.
The following outline aims to provide a basic understanding of the use of echocardiography in the
critically ill patient. However, the content will not substitute for formal echocardiographic education and
training.
Indications:
National guidelines base their recommendations according to the level of scientific evidence.
Class I: conditions for which there is evidence and/or general agreement that a given procedure
or treatment is useful or effective.
Class IIa: the evidence is in favor of the treatment or procedure, but opinion may diverge.
Class IIb: evidence is less convincing
Class III: evidence may potentially be harmful.
The list of Class I indications below can be extended to include preexisting cardiovascular and pulmonary
conditions in the critical ill patient such as acute coronary syndromes, valvular dysfunction, and
pulmonary disease.
A. Class I indications for echocardiography in the ICU
1. Critically ill patient
a) Hemodynamic instability
(1) Hypovolemia
(2) Vasodilatation
(3) Ventricular failure
(a) Left ventricular failure
I.
78
ASCCA Residents Guide to the ICU
2.
(b) Right ventricular failure
(c) Biventricular failure
(d) Sepsis related cardiomyopathy
(e) Left ventricular diastolic failure
(4) Pulmonary embolus
(5) Acute valvular dysfunction
(6) Cardiac tamponade
b) Suspected aortic dissection (TEE)
c) Unexplained hypoxemia
d) Sources of emboli (TEE if considering left atrium)
Critically injured patient
a) Serious blunt or penetrating chest trauma for suspected pericardial effusion or
tamponade or aortic dissection
b) Mechanically ventilated multiple-trauma or chest trauma patients
c) Suspected preexisting valvular or myocardial disease in the trauma patient
d) Hemodynamic instability in the multitrauma patient without chest trauma, but with crush
or deceleration injury mechanism with potential cardiac or aortic injury
e) Radiographic widening of the mediastinum with suspected thoracic aortic injury
f) Needle injury from central venous catheter or pericardiocentesis with or without signs of
tamponade.
II. Indications for primary transesophageal echocardiography
TTE devices facilitate an urgent, focused, qualitative assessment in many patients. Frequently in critically
ill patients an adequate TTE assessment cannot be achieved and progression to a TEE examination is
indicated. Although TEE is more invasive and requires considerable expertise, it may provide superior
imaging and overcomes some of the limitation of TTE.
A. Inferior image quality of TTE in:
1. Mechanical ventilation with high PEEP
2. Severe obesity, emphysema
3. Pneumothorax, pneumopericardium or subcutaneous emphysema
4. Presence of chest and mediastinal tubes
5. Surgical incisions and dressings
B. Superior image quality of TEE in:
1. Thoracic aortic dissection
2. Assessment of endocarditis
3. Intracardiac thrombus
4. Visualization of thoracic aorta, left atrial appendage, prosthetic valves
5. Postcardiotomy patients to rule out mediastinal tamponade
III.
Contraindications to insertion of a TEE probe
A. Absolute contraindications
1. Esophageal pathologies
a) Stricture
b) Mass or tumor
c) Diverticulum
d) Mallory Weiss tear
e) Dysphagia or odynophagia not evaluated previously
2. Cervical spine instability
B. Relative contraindications
1. Esophageal varices
2. Recent esophageal or gastric surgery
3. Oropharyngeal carcinoma
4. Upper gastrointestinal bleeding
5. Severe cervical arthritis
6. Atlantoaxial disease
IV. Preparation of the patient
Informed consent should be obtained from the patient, if conscious, or the medical power of attorney. The
79
ASCCA Residents Guide to the ICU
risk and benefit and conduct of the procedure should be explained in simple, understandable language.
A. Incidence of complication for intraoperative TEE
1. Mortality (0.01 to 0.0.3%)
2. Morbidity (0.2%)
a) Odynophagia (0.1%)
b) Dental injury (0.03%)
c) Endotracheal tube malposition (0.03%)
d) Upper gastrointestinal hemorrhage (0.03%)
e) Esophageal perforation (0.01%)
The patient is ideally positioned in the left lateral decubitus position for either TTE or TEE. TTE is well
tolerated, but requires a cooperative patient. For a TEE examination the patient should be NPO for six
hours, and tube feeds should be held with the exception of emergencies. Adequate sedation and
appropriate hemodynamic and cardiovascular monitoring according to American Society of
Anesthesiologists monitoring standards is essential for the TEE examination for both the non-intubated
and intubated patient. Administration of sedatives may cause hemodynamic instability and the presence
of a qualified care provider and availability of vasopressor agents are recommended.
V. Image acquisition
A comprehensive TEE evaluation encompasses the acquisition and interpretation of 2D images and
Doppler data of 20 cross sectional views, in contrast to 8 image plans in a focused, basic assessment.
One of the most useful cross sections is the transgastric short axis view at the level of the midpapillary
muscles (see Figure 18.1). This TEE image is obtained by forwarding the TEE probe into the stomach
and directing the ultrasound beam across the heart. The image obtained of the left ventricle resembles a
donut with the right ventricle hugging the left in a crescent shape. This image plane allows for the
evaluation of the left and right ventricular myocardium, papillary muscles, and pericardium. Pathological
processes such as ventricular dysfunction and the presence of pericardial fluid can be identified.
However, an examination, which is limited and focused needs to be accurate as erroneous interpretation
of limited data may lead to a wrong diagnosis and possibly harmful management strategies.
VI. The hemodynamically unstable patient
Unexplained hypotension or hypotension refractory to appropriate fluid resuscitation and or vasopressor
therapy is a common problem in the emergency room and intensive care unit alike. Table 1 presents
some chief echocardiographic features of common etiologies for hypotension. For an initial, rapid
examination the transgastric midpapillary view should be developed. Simple visualization in real-time by
an experienced echocardiographer, who can rapidly estimate systolic and diastolic areas, myocardial wall
thickening and endocardial wall excursion, allows for fast assessment of volume status and global
ventricular function,. Wall motion can be described in terms of normokinesia, hypokinesia, akinesia, and
dyskinesia, the latter being characterized by the ventricular wall moving outwards instead of contracting
towards the middle in systole.
Table 18.1. TEE findings for common etiologies of refractory hypotension in the ICU population
Etiology
Hypovolemia
Left ventricular outflow tract
obstruction
Failing left ventricle
TEE findings
Decreased end-diastolic area
Normal or decreased end-systolic area
Increased fractional area change
Normal myocardial thickening
Normal to increased endocardial excursion
“Kissing” papillary muscles
Small, hyperdynamic left ventricle
Mosaic pattern of flow in left ventricular cavity on Doppler
Systolic anterior motion of the mitral valve
Increased end-diastolic area
Increased end-systolic area
Decreased fractional area change
80
ASCCA Residents Guide to the ICU
Failing right ventricle
Vasodilatation
Cardiac tamponade
Aortic dissection
Pulmonary embolus
“D” shape left ventricle in transgastric view
Intraventricular septum bulges towards the left ventricle
Increased right ventricular size > 1/3 of left
Normal to low end-diastolic area
Decreased end-systolic area
Increased fractional area change
Increased myocardial thickening
Increased endocardial excursion
Pericardial effusion
Diastolic collapse of right-sided chambers or left atrium
Dilated, on-collapsible inferior vena cava
Intimal flap
Aorta with two lumen
Aortic regurgitation
Pericardial effusion
Right ventricular dysfunction
Dilated right-sided chambers
Flow through a patent foramen ovale
Echogenic density
VII. Other use of ultrasound in the ICU setting
A. Central line placement
B. Assessment of pleural effusions
C. Assessment of intra-abdominal fluid collections
D. Urinary bladder scan
E. Focused assessment of the trauma patient
F. Assistance in placement of intra-aortic balloon counter pulsation
G. Evaluation of flow in ventricular assist devices
H. Weaning from ventricular assist devices or extracorporeal membranous oxygenator
VIII. Conclusion
The use of echocardiography is growing in the ICU setting and may rapidly assist the clinician in obtaining
additional information in the unstable critically ill patients. Often, management strategies are redirected or
optimized based on the echo information, even in the presence of invasive monitoring. Prospective,
randomized trials on the impact of echocardiography may be difficult to conduct. As guidelines for the
conduct of ICU echocardiography are developed, the importance of operator expertise, as with any
technology, need be emphasized as inappropriate use and inaccurate interpretation may result in
inappropriate interventions.
READING LIST:
1. Beaulieu Y, Gorcsan J. Bedside Ultrasonography. In: Fink MP, Abraham E, Vincent J-L, Kochanek P, eds. Textbook of
Critical Care. 5th ed. Philadelphia: Elsevier Saunders; 2005:1757 - 83.
Excellent review on the application of bedside ultrasonography in the critical ill patient.
2. Benjamin E, Griffin K, Leibowitz AB, et al. Goal-Directed Transesophageal Echocardiography Performed by
Intensivists to Assess Left Ventricular Function: Comparison with Pulmonary Artery Catheterization. J
Cardiothorac Vasc Anesth 1998;12(1):10-5.
Intensivists can be trained rapidly and safely in limited-scope, goal-directed TEE to evaluate LV function.
3. Cheitlin MD, Armstrong WF, Aurigemma GP, et al. ACC/AHA/ASE 2003 Guideline Update for the Clinical
Application of Echocardiography: Summary Article: A Report of the American College of Cardiology/American
Heart Association Task Force on Practice Guidelines (ACC/AHA/ASE Committee to Update the 1997 Guidelines
for the Clinical Application of Echocardiography). Circulation 2003;108(9):1146-62.
Document includes recommendations for the use of echocardiography in both specific cardiovascular disorders
and the evaluation of patients with frequently observed cardiovascular symptoms and signs, common presenting
complaints, or findings of dyspnea, chest discomfort, and cardiac murmur.
81
ASCCA Residents Guide to the ICU
4. Chirillo F, Totis O, Cavarzerani A, et al. Usefulness of Transthoracic and Transoesophageal Echocardiography in
Recognition and Management of Cardiovascular Injuries after Blunt Chest Trauma. Heart 1996;75(3):301-6.
TEE is more sensitive, specific and accurate in diagnosing cardiac aortic in blunt chest trauma than TTE.
5. Colreavy FB, Donovan K, Lee KY, Weekes J. Transesophageal Echocardiography in Critically Ill Patients. Crit Care
Med 2002;30(5):989-96.
A retrospective chart review, in which the utilization of a TEE led to a significant change in management in 32% of
255 hypotensive patients.
6. Donovan KD, Colreavy F. Echocardiography in Intensive Care: The Basics. Crit Care Resusc 1999;1(3):291-310.
A review of echocardiography in critically ill patients with special reference to the advantages and disadvantages
of the transthoracic and transoesophageal approaches.
7. Kallmeyer IJ, Collard CD, Fox JA, Body SC, Shernan SK. The Safety of Intraoperative Transesophageal
Echocardiography: A Case Series of 7200 Cardiac Surgical Patients. Anesth Analg 2001;92(5):1126-30.
TEE is a relatively safe diagnostic monitor with an overall morbidity (0.2%) and mortality (0%) rates in a
retrospective case series of 7200 adult, anesthetized cardiac surgical patients.
8. Reeves ST, Payne KJ, Ramsay J, Shanewise J, Insler S, Stewart WJ. TEE in the Critical Care Setting. In: Savage RM,
Aronson S, eds. Comprehensive Textbook of Intraoperative Transesophageal Echocardiography. 1st ed.
Philadelphia: Lippincott Williams & Williams; 2005:303 - 24.
Standard textbook on perioperative TEE. This chapter focuses on ultrasound application in the critical care
medicine setting.
9. Seward JB, Douglas PS, Erbel R, et al. Hand-Carried Cardiac Ultrasound (HCU) Device: Recommendations
Regarding New Technology. A Report from the Echocardiography Task Force on New Technology of the
Nomenclature and Standards Committee of the American Society of Echocardiography. J Am Soc Echocardiogr
2002;15(4):369-73.
The American Society of Echocardiography believes HCU will provide a rapid assessment of cardiovascular
anatomy, function, and physiology, but cautions that appropriate user-specific training are essential to ensure the
most accurate acquisition, interpretation, and use of the data.
10.Vignon P, Frank MB, Lesage J, et al. Hand-Held Echocardiography with Doppler Capability for the Assessment of
Critically-Ill Patients: Is It Reliable? Intensive Care Med 2004;30(4):718-23.
HCU had a lower diagnostic accuracy compared with conventional TTE.
11.Wake PJ, Ali M, Carroll J, Siu SC, Cheng DC. Clinical and Echocardiographic Diagnoses Disagree in Patients with
Unexplained Hemodynamic Instability after Cardiac Surgery. Can J Anaesth 2001;48(8):778-83.
Retrospective review of unstable patients after cardiac surgery noting that the lack of TEE data is associated with
inappropriate management strategies in over 50% of patients.
QUESTIONS:
18.1.
A.
B.
C.
D.
E.
Class I indication for echocardiography in the critical care setting include:
Monitoring function of assist devices
Hemodynamic instability in an intubated patient with invasive monitoring lines
Facilitation of pulmonary artery catheter positioning
Spontaneously ventilating, hemodynamically stable chest trauma patient
Monitoring placement of intra-aortic balloon counterpulsation devices
18.2.
A.
B.
C.
D.
E.
Absolute contraindication to transesophageal echocardiography includes:
Rheumatic arthritis
Know esophageal varices
Bleeding diathesis
Esophageal stricture
Recent gastric surgery
18.3.
A known hypertensive, 68-year-old obese, female accident victim with a femur fracture, blunt chest, and
abdominal trauma becomes acutely hypoxemic and hemodynamically unstable. Initial management
step(s) in order of importance include primarily.
Establishment of an airway, breathing, circulation, and possible drug administration
Intravenous fluid challenge
Emergent sedation, intubation, and ventilation
Administration of a vasopressor
Urgent bedside echocardiography
A.
B.
C.
D.
E.
82
ASCCA Residents Guide to the ICU
18.4.
A.
B.
C.
D.
E.
18.5.
A.
B.
C.
D.
E.
18.6.
A.
B.
C.
D.
E.
The above patient was stabilized and intubated in the emergency room and transferred to the ICU
where refractory hypoxemia developed. Echocardiographic features of traumatic dissection of the aorta
consist of all of the following, EXCEPT:
Intimal flap in the lumen of the aorta
Pericardial effusion
Aortic insufficiency
D-shape left ventricle
Two lumens in the ascending aorta
The above patient underwent emergent repair of a thoracic dissection with aortic valve replacement
complicated by postoperative hemorrhage. Within 4 hours postop she gets progressively hypotensive
requiring frequent adjustment of vasopressor therapy and repeated fluid challenges. Echocardiographic
features consistent with pericardial tamponade include all of the following, EXCEPT:
Loculated fluid in the posterior portion of the mediastinum
Diastolic collapse of the right ventricle
Decreased left ventricular end-diastolic area
Small, collapsible inferior vena cava
Right ventricular dysfunction
On postoperative day 3 the above patient develops atrial fibrillation with ST segment changes.
Echocardiographic features of ventricular dysfunction may include:
Increased myocardial thickening
Decreased end-diastolic area
Increased right ventricular size
Increased endocardial excursions
Kissing papillary muscles
83
ASCCA Residents Guide to the ICU
19. Airway Management in the Intensive Care Unit
Todd W. Sarge, M.D.
A 47-year-old female patient who was an unrestrained driver in an MVA is admitted to the ICU following
surgical repair of a liver laceration and stabilization of a femur fracture. She also exhibits significant
pulmonary infiltrates on CXR, possibly secondary to aspiration. Her past medical history includes asthma,
DM, and chronic renal insufficiency. On arrival to the ICU, an endotracheal tube (ETT) cuff leak is noted
and breath sounds are diminished in all lung fields. The ETT balloon is found to be incompetent and
reintubation is required. The patient remains in a hard cervical collar for spine precautions but does not
appear to have facial trauma. After administering midazolam 3 mg and vecuronium 6 mg, the hard
cervical collar is removed while a physician manually holds the patient’s head with in-line stabilization. An
ETT exchanger (Cook catheter) is placed inside of the ETT and the ETT is then withdrawn and removed.
Laryngoscopy is performed to help visualize and assist the ETT exchange. The new ETT is placed over
the Cook catheter and advanced with a grade II view by direct laryngoscopy. The vocal cords and the
larynx appear edematous but the ETT passes without difficulty and the Cook airway exchange catheter is
then removed. The patient’s oxygen saturation is now 82%. The placement of the ETT is confirmed by
end tidal CO2 and bilateral breath sounds. After a recruitment maneuver, her oxygen saturation returns
to 94%.
Airway management in the ICU involves all of the difficulties that are encountered in the operating room.
However, unlike the operating room, the problems are rarely elective, the patients are much less able to
tolerate even the smallest degree of hypoxia or hypoventilation, and the physical environment of the ICU
often does not allow for easy access to the patients or the airway as in the operating room. Patients often
have cardiovascular instability, agitation / delirium, and injuries to the head and neck, all of which may
increase the difficulty of intubation. In addition, the equipment available for securing an airway in the ICU
is often less sophisticated and not as accessible as the operating room. In sum, airway management in
the ICU requires a great deal of skill and an appreciation for the unique physiology and pathophysiology
of the critically ill patient.
I.
Basic patient conditions requiring airway support
A. Obstruction
1. Relaxed tongue, redundant tissue (sleep apnea patients), foreign body
2. Head and neck trauma, tumors, burns, inhalation injury, infections
B. Respiratory failure (hypoxemic or hypercarbic) requiring a need for positive pressure ventilation
secondary either to pathology or to anesthetic medication effect.
C. Alterations in mental status with an inability to protect the airway
D. Inability to control secretions
E. Cardiovascular instability
II. Evaluation of the airway
A. General exam
1. Purpose: To predict the relative ease or difficulty of tracheal intubation.
2. Mallampati classification: with the patient sitting upright, the neck extended, and the tongue
protruding, evaluate the size of the tongue in relation to the oral cavity. Higher grades are
thought to predict a more difficult intubation
a) I: The uvula, faucial pillars, and the soft palate are visualized
b) II: The uvula and soft palate are visualized
c) III: The base of the uvula and the soft palate are visualized
d) IV: The tongue and hard palate are visualized only
3. Oral opening: less than 4 cm may increase difficulty of intubation
4. Thyromental distance: less than 6 cm may increase difficulty of intubation
5. Range of motion of neck: limited mobility secondary to cervical collar, concern of cervical
spine injury and chronic neck pathology (arthritis) make intubation more difficult.
6. Dentition: note loose teeth that may be displaced during laryngoscopy.
7. Head and neck exam: distorted anatomy secondary to trauma, abscesses, prior surgery and
84
ASCCA Residents Guide to the ICU
other pathologies can make intubation more difficult
NPO status–considerations:
a) Trauma patient
b) Patient on gastric tube feeds
c) Morbid obesity, DM, pregnancy
B. Common ICU problems which heighten difficulty or make mask ventilation or intubation difficult:
1. Uncooperative patients: may make awake intubation difficult
2. Edematous pharyngeal tissues; fluid shifts, long surgical procedures
3. Facial fractures, basilar skull fractures
4. Cervical spine precautions
5. Airway hemorrhage
6. Unstable cardiovascular status
7. Increased or potentially increased intracranial pressure
8.
III. Intubation in the ICU
A. Preparation of the Patient
1. Emergency vs. urgent vs. semi-elective intubation
2. Evaluation of airway as above
3. Evaluation of the patient
a) Cardiovascular and respiratory status
b) Medications and allergies.
c) Mental status
B. Pharmacology of intubation in the ICU
1. Is any agent required? Often ICU patients may require little or no medication
2. If medications are chosen, the intensivist must know the pharmacologic profile of the drug, its
side effects and how to counter them.
3. Amnestic/Induction agents: pentothal, propofol, midazolam, etomidate, ketamine: these drugs
have cardiovascular, neurologic, respiratory and endocrinologic effects which should be
known prior to using.
4. Analgesics
a) Have sedating properties and may decrease need for induction agent
b) Mitigate pain of laryngoscopy
c) Morphine, fentanyl, remifentanyl
5. Muscle relaxants
a) Do not assume every patient needs to be paralyzed
b) Succinylcholine, rocuronium–short acting, rapid onset.
c) Can use less rapid onset agents with longer duration if patient can be ventilated via mask
while awaiting adequate relaxation, provided there is an empty stomach.
d) The disadvantage of longer acting muscle relaxants is that if the trachea is not able to be
intubated immediately, mask ventilation on the paralyzed patient needs to be adequate
for a relatively long period of time. Mask ventilation may become inadequate over this
period.
6. Miscellaneous agents
a) Local anesthetics–reduce response to intubation or facilitate awake intubation.
b) Vasoconstrictors for nasal intubation
C. Techniques of intubation in the ICU
1. General considerations
a) Monitor patient for changes in condition that may require changes in plans
b) Always have a backup plan in mind (see Reference 1)
c) Check all equipment first, never assume everything is ready
d) Anticipate the need for special equipment and if time allows gather this equipment. This
may include special blades such as the McCoy laryngoscope (flexible tip), Eschmann
catheters (bougies), intubating LMAs, indirect laryngoscopes (e.g. Bullard, GlideScope,
Airtraq) and a fiberoptic-bronchoscope.
e) Oxygenate the patient first and during the procedure. May not be possible in patients with
cardiorespiratory disease. Use of noninvasive pressure ventilation may increase
oxygenation.
f) Patients who require urgent/emergency intubation may have high sympathetic tone which
85
ASCCA Residents Guide to the ICU
2.
3.
4.
5.
can be rapidly reduced when oxygenation and ventilation are supported; long-acting
agents to curb sympathetic response may precipitate significant hypotension after
intubation. In addition, patients may have hypovolemia which can be unmasked by
positive pressure ventilation; long-acting agents could worsen that response.
Awake vs. Asleep intubation:
a) An awake intubation is generally undertaken in patients where there is a significant
possibility of being unable to secure an airway after induction with amnestic and paralytic
agents. Examples are patients with significant facial and neck pathology and those with
poor mouth opening and high Mallampati score.
b) To perform an awake intubation, the nerves of the airway- the glossopharyngeal, the
superior laryngeal +/- the recurrent laryngeal nerve should be blocked by local
anesthetics either via inhalation or direct nerve block
c) With adequate local anesthesia, an awake intubation can be performed in several ways –
blindly via the nares, with direct laryngoscopy (DL) or more commonly with a flexible
fiberoptic bronchoscope.
d) Disadvantages: An awake intubation requires a cooperative patient, perhaps obtained by
sedation and a patient that is stable (non-hypoxemic) from a respiratory standpoint
because of the time required to perform the procedure.
e) An intubation under sedation or anesthesia is generally opted for in a patient that the
practitioner feels confident of being able to secure an airway quickly after induction.
Ideally, the patient should have an empty stomach in order to prevent aspiration while
mask ventilation is being used. If not, and the patient is deemed to be a difficult intubation
by exam or history, then an awake intubation should be considered even if mask
ventilation is possible. The steps involved include: preoxygenation, induction of
anesthesia (with or without paralytic), attempt at mask ventilation (unless using the rapid
sequence technique), and intubation. Securing the airway can be can be achieved in
several ways: either by DL or via other devices, such as an intubating Laryngeal Mask
Airway (LMA), an Eschmann tube or a flexible fiberoptic bronchoscope.
f) With DL, the views obtained is graded as follows:
(1) Grade I: The entire glottis and vocal cords are seen.
(2) Grade II: Only the posterior aspect of the glottis is seen (arytenoid cartilage)
(3) Grade III: Only the epiglottis is seen
(4) Grade IV: The epiglottis is not visualized
Rapid sequence intubation
a) A variant of intubation performed in patients with a full stomach where induction agent
and a rapid acting paralytic are given in sequence and mask ventilation is not performed
during the onset of muscle relaxation in an effort to prevent aspiration
b) Cricoid pressure (Sellick maneuver) is performed to occlude the esophagus.
c) A backup plan is imperative should intubation be difficult
d) A surgical airway may be required, surgeon may not be available
Management of the difficult airway
a) A difficult airway may be anticipated or unanticipated before induction
b) If anticipated, an awake intubation should be performed if possible
c) If discovered after induction then the following steps should be taken:
(1) Call for the assistance of people knowledgeable in emergency airway management
(2) Attempt bag-mask ventilation with appropriate devices (i.e. oral airway)
(3) If unsuccessful with bag-mask ventilation, attempt ventilation either with an LMA,
Trans-tracheal jet ventilation, or by surgical airway
(4) If attempts unsuccessful, try to awaken the patient (although in the ICU setting this
may not be realistic as they are usually being intubated for emergency purposes)
Oral vs. nasal intubation:
a) Oral intubation:
(1) Often technically easier and faster
(2) Can interfere with patient’s ability to communicate and hinders oral care
(3) Usually permits a larger endotracheal tube to be used
b) Nasal intubation:
(1) Often better tolerated by patients–can mouth words for communication
(2) Usually requires a smaller tube–smaller size and angulations may make suctioning
86
ASCCA Residents Guide to the ICU
more difficult
(3) Can be done blindly or aided by laryngoscope or fiberoptic scope; the fiberoptic
technique is usually easier nasally
(4) May lead to an increased incidence of sinusitis
IV. Complications of Endotracheal Intubation
A. Complications during intubation
1. Trauma: dental injury; perforation or dislocation of structures of pharynx, larynx and trachea
2. Acute Respiratory complications: unrecognized esophageal intubation, laryngospasm with
pulmonary edema, bronchospasm
3. Cardiovascular problems: PVCs, VT, bradycardia, hypotension, hypertension
4. Neurologic problems: increased intracranial pressure, spinal cord injury, passage of tube into
cranial vault (nasal intubation in patient with basilar skull fracture
B. Complications with the ETT in place
1. Blockage, kinking or movement to mainstem bronchus
2. Trauma to airway structures
3. Sinusitis
C. Complications after extubation
1. Aspiration and laryngospasm
2. Ulceration of airway structures from mouth to trachea
3. Vocal cord paralysis
4. Laryngeal edema or trauma with stridor
5. Tracheal stenosis
V. Controversies in Airway Management in the ICU
A. Long-term intubation
1. Convert to tracheostomy?
a) Some centers go to a surgical airway “early” (≤4 days) in patients felt to require
prolonged intubation
b) Percutaneous or surgical tracheostomy
(1) Percutaneous can be done at bedside in the ICU; some centers will do regular
tracheostomy at bedside. If going to OR, coordinate with other required procedures.
c) Tracheostomy decreases dead space in ventilator circuit, also easier suctioning
d) Less oropharyngeal/nasopharyngeal damage with tracheostomy
e) Less vocal cord trauma with tracheostomy
f) Tracheostomy is another procedure; less is better
g) Tracheostomy procedure has complications that may be significant
(1) Pneumothorax, tracheoinnominate artery fistula, infection, etc.
(2) Estimates of up to 1% significant morbidity/mortality due to tracheostomy (from older
studies that included many emergency tracheostomies, by nature more likely to have
complications)
VI. Extubation in the ICU
A. Criteria for extubation is more complicated than in the OR
1. The patient, of course, must no longer require mechanical ventilation
2. Can the patient’s oxygenation need be met without an airway?
a) FiO2 should be less than 0.5
3. Is the condition that required intubation fixed or at least controlled?
a) Secretions controlled–can the patient cough adequately?
b) Mental status improved–evaluate patient responsiveness
c) Can the patient protect his/her airway?
d) Airway edema evaluation
(1) Direct laryngoscopy may not be tolerated; fiberoptic upper airway exam may suffice
(2) Is there a leak around the tube with the cuff deflated)?
e) Are the patient’s respiratory mechanics as judged by a spontaneous breathing trial
adequate for extubation?
f) Is the patient going to require reintubation for another procedure in the very near future?
Controversial.
87
ASCCA Residents Guide to the ICU
This chapter is a revision of the original chapter authored by Thomas M. Fuhrman, M.D.
READING LIST:
1. Levitan R, Ochroch EA. Airway management and direct laryngoscopy. A review and update. Crit Care Clin. 2000
Jul;16(3):373-88, v. Review.
2. Practice Guidelines for Management of the Difficult Airway: An Updated Report by the American Society of
Anesthesiologists Task Force on Management of the Difficult Airway. Anesthesiology. 98(5):1269-1277, May 2003.
3. Crosby ET. Airway management in adults after cervical spine trauma. Anesthesiology. 2006 Jun;104(6):1293-318.
Review.
4. Bridges to establish an emergency airway and alternate intubating techniques. Crit Care Clin. 2000 Jul;16(3):429-44,
vi. Review.
5. Mort TC. Preoxygenation in critically ill patients requiring emergency tracheal intubation. Crit Care Med. 2005; 33:
2672-5.
Preoxygenation does not reliably increase PaO2 prior to intubation.
6. Baillard C, Fosse JP, Sebbane M, et al. Noninvasive ventilation improves preoxygenation before intubation of
hypoxic patients. Am J Respir Crit Care Med. 2006; 174:171-7.
Noninvasive ventilation provided better oxygenation than standard bag-valve-mask in critically ill patients
undergoing intubation.
QUESTIONS:
19.1.
A.
B.
C.
D.
In a patient with suspected cervical spine injury:
Nasal intubation is the route of choice.
The neck can be flexed and extended as needed during an awake intubation.
In-line stabilization is mandatory for all intubation attempts.
A cross-table lateral film can be used to “clear” the cervical spine.
19.2.
A.
B.
C.
D.
Absolute indications for endotracheal intubation include:
Positive pressure ventilation
Obtundation
Hypercarbia
None of the above
19.3.
Which of the following statements regarding cricoid pressure for rapid sequence intubation
are true:
Despite adequate cricoid pressure, aspiration may still occur, particularly in patients with full
stomachs or active vomiting.
Cricoid pressure should be released immediately after the endotracheal tube is inserted by
the person performing the intubation.
When applied correctly, backwards pressure on the cricoid cartilage will always prevent
aspiration during intubation.
None of the above.
A.
B.
C.
D.
19.4.
A.
B.
C.
D.
In the case presented above, imagine that after removing the incompetent ETT, the Cook
catheter becomes dislodged out of the trachea and initial DL reveals a grade IV view with a
blind attempt at intubation being unsuccessful. After calling for skilled assistance, what is the
next most appropriate course of action?
Reattempt intubation by DL.
Place the Cook catheter back in the oropharynx and attempt passage into the trachea.
Attempt Bag-Mask ventilation.
Attempt a fiberoptic intubation.
88
ASCCA Residents Guide to the ICU
20. Chronic Airway Management
Ruben J. Azocar, M.D.
A 52-year-old male sustains a C4 cervical fracture and complete spinal cord injury as an unrestrained
passenger in a motor vehicle accident. He is admitted to the ICU on full ventilatory support and develops
ARDS during his hospitalization. After being intubated with an oral endotracheal tube for 15 days, it
appears that the patient will continue to require ventilatory support for at least another 1-2 weeks.
Potential questions relevant to the care of this patient are as follows: What are the concerns about
prolonged oral endotracheal intubation? How is a tracheostomy performed? How is a tracheostomy tube
maintained? What types of tracheostomy tubes can be placed? How can we allow the patient to
vocalize?
The management of the chronic artificial airway encompasses elements of acute airway management, as
well as elements unique to the chronically instrumented airway. The indications for chronic artificial
airway placement are usually similar to those of acute artificial airway management; however, the time
course for resolution of these indications is either prolonged or permanent. Anesthesiologists should
understand the indications for chronic airway management, the options available, and clinical
management techniques. This outline provides a framework for approaching the patient who requires a
chronic artificial airway.
I.
Indications for establishment of an artificial airway
A. Ventilatory support
B. Suboptimal bronchial hygiene
C. Upper airway obstruction
D. Airway protection
E. Facilitate weaning from Mechanical Ventilation
Recent data has looked into outcomes in patient that receive tracheotomies. It seems that in terms of inhospital mortality rates patient who receive tracheostomies have better outcomes. Also the benefit of
early versus late has been a matter of controversy. Data for the trauma literature suggests that early
tracheostomy (performed within 7 days of admission) is associated with shorter duration of mechanical
ventilation and ICU length of stay (LOS) but with no effects on outcomes. Similar data was gathered from
medical intensive care units: shorter LOS and lower hospital cost in patient with early
tracheotomies’ (within 10 days of admission). Other data reveals that tracheotomies after 21 days of
intubation were associated with a higher rate of failure to wean, longer ICU stays and higher ICU
mortality. Therefore it seems early tracheostomy in carefully selected patients in whom long term
ventilation is predicted might be beneficial in terms of outcomes, LOS and cost.
II.
Advantages of tracheostomy
A. Avoidance of laryngeal injury
1. Reversible–ulceration, erythema, inflammation
2. Long-term–laryngeal granuloma, vocal cord adhesions and stricture
B. Improved patient comfort
C. More efficient bronchial hygiene (i.e., suctioning)
D. Communication is facilitated
E. More effective swallowing when compared to endotracheal intubation
F. Early mobilization
III.
Performance of the tracheostomy (open)
A. Incision is made below the cricoid membrane (2nd-3rd tracheal rings)
B. Thyroid isthmus is retracted or divided
C. Endotracheal tube cuff is deflated and ETT is retracted to subglottic position allowing
tracheostomy tube placement
D. Presence of carbon dioxide and bilateral breath sounds confirmed
E. Endotracheal tube removed after tracheostomy is secured
89
ASCCA Residents Guide to the ICU
IV.
Performance of the tracheostomy (percutaneous)
A. A fiberoptic scope is introduce in the ETT and the tube is retracted to subglottic position
1. The tracheal is access with an introducer needle
2. A wire is then advanced into the trachea
3. Using a Seldinger technique, the trachea is dilated with successive dilators or by a single
dilator
4. Once the tracheostomy is established, the tracheotomy tube is placed and the wire is
removed. The fiberoptic scope confirms tracheal placement
5. Presence of carbon dioxide and bilateral breath sounds confirmed
6. Endotracheal tube removed after tracheostomy is secured
7. Percutaneous tracheostomy is safe and cost effective
V.
Tracheostomy tube components
A. Single cannula
B. Double cannula
1. Outer cannula
2. Inner cannula–facilitates cleaning of the artificial airway without removal of the entire artificial
airway
C. Neck flange–facilitates securing artificial airway and prevents migration of artificial airway
D. Obturator–used to insert cannula
E. Cuff
F. Pilot balloon
VI.
Tracheostomy tube designs
A. Cuffed
1. Inflatable
2. Self-inflating foam cuff
B. Cuffless
C. Fenestrated
D. Communication tracheostomy
VII. Tracheostomy tube materials
A. Metal
B. Polyvinylchloride
C. Silicone
D. Polyethylene
E. Teflon
F. Nylon
VIII. Management of the tracheostomy tube
A. Cuff inflation and management (low pressure, high volume)
1. Minimal occlusive volume-minimal amount of air in cuff to produce no leak at the end of the
inspiratory phase of a positive pressure ventilation
2. Minimal leak volume-cuff is inflated in 1mL increments until a slight leak is heard at the end of
the inspiratory phase of a positive pressure ventilation
3. Manometer can be used to check cuff pressure and maintain it less than 20 mmHg to prevent
trachea mucosal hypoperfusion
B. Stoma care
1. Cleansing of stoma
2. Tracheostomy dressing to prevent bronchial secretions from contributing to wound infection
C. Initial airway change-this is performed after a secure tract has formed (3-7 days). Unintentional
decannulation prior to this time may result in loss of the airway and inadequate ventilation. A
recent paper advocates the rationality and safety of this practice.
D. Artificial airway advancement
1. Vocalization
2. Downsizing
3. Decannulation
90
ASCCA Residents Guide to the ICU
IX.
Complications of tracheostomy
A. Acute
1. Tube malposition
2. Pneumothorax
3. Pneumomediastinum
4. Subcutaneous emphysema
5. Stomal hemorrhage
B. Chronic
1. Pulmonary infections
a) Colonization versus pneumonia versus tracheobronchitis
2. Tracheal stenosis
a) Cuff site
b) Stoma site
3. Tracheoinnominate artery fistula
4. Tracheomalacia
5. Tracheoesophageal fistula
6. Swallowing dysfunction
7. Stoma infections
X.
Tracheostomy buttons–prevent stoma closure
A. Kistner button
B. Olympic button
C. Montgomery button
XI.
Vocalization
A. “Air leak” through vocal cords is necessary
1. Ventilator manipulation:
a) Adequate tidal volume with built-in leak
b) Adequate inspiratory time is necessary as vocalization occurs during inspiration (consider
lower ventilatory support rate with larger tidal volume)
B. Capping of tracheostomy tube (cuffless or fenestrated)
C. Electrolarynx
D. One-way speaking valves–always ensure exhalation is possible through glottis
E. Communication tracheostomy
This chapter is a revision of the original chapter authored by Michael L. Ault, M.D. and William T. Peruzzi, M.D.
REFERENCES
1. Combes A, Luyt CE, Nieszkowska A et al: Is tracheostomy associated with better outcomes for patients requiring
long-term mechanical ventilation Crit Care Med 2007; 35:802-7
2. Kollef MH, Ahrens TS, Shannon W: Clinical predictors and outcomes for patients requiring tracheostomy in the
intensive care unit. Crit Care Med 1999;27:1714-20
3. Arabi Y, Haddad S, Shirawi N, et al Early tracheostomy in intensive care trauma patients improves resource
utilization: a cohort study and literature review. Crit Care 2005;8:R347-52
4. Brook AD, Sherman G, Malen J, et al:Early versus late tracheostomy in patients who require prolong mechanical
ventilation. AmJ Crit Care 200;9: 352-9
5. Hsu CL, Chen KY, Chang CH et al: Timing of tracheostomy as a determinant of weaning success in critically ill
patients: a retrospective study. Crit Care 2005; 9:R46-52
6. Higgins KM, Punthakee X. Meta-analysis comparison of open versus percutaneous tracheostomy. Laryngoscope.
2007 Mar; 117(3):447-54.
7. Tabaee A, Lando T, Rickert S, et al:Practice patterns, safety and rational for tracheostomy tube changes: a survey of
otolaryngology training programs. Laryngoscope 2007;117:573-6
91
ASCCA Residents Guide to the ICU
QUESTIONS:
20.1.
A.
B.
C.
D.
Which of the following is NOT an indication for tracheostomy?
PaCO2 of 80 mmHg and pH of 7.2 in a patient with myasthenia gravis who has a history of unsuccessful
attempts at endotracheal intubation via direct laryngoscopy; the patient refuses awake intubation
Inability to activate swallow reflex in response to appropriate oropharyngeal stimuli in a patient who has
sustained a large subarachnoid hemorrhage and has been intubated for 14 days
Stridor in a patient with supraglottitis
PaO2 of 62 mmHg in a patient with Pneumocystis carinii pneumonia on 4 liters O2 by nasal cannula
20.2.
A.
B.
C.
D.
The following is true about percutaneous tracheostomy except:
Requires the patient to be transported to the OR
It generates higher costs than open tracheotomies as the equipment is very expensive
It has a higher complication rate than the open procedure
Fiberoptic guidance is recommended
20.3.
A.
B.
The purpose of a tracheal “button” is to:
Discourage suctioning of the patient
Maintain the tracheostomy stoma in the event that a tracheostomy is again required in a patient from
whom it has been removed
Restore normal swallowing function
Allow more rapid healing of the tracheostomy stoma
C.
D.
20.4.
A.
B.
C.
D.
Chronic complications of tracheostomies include the following except:
Tracheoinnominate artery fistula
Tracheomalacia
Acute Pneumothorax
Tracheoesophageal fistula
20.5.
A.
B.
C.
D.
The following is true about tracheostomy tube exchange:
Should be performed 48 hours after the initial tracheostomy
If timed correctly, it is devoid of risks
Loss of the airway is unlikely
It is a potential risky procedure and its indication should be well rationalized
92
ASCCA Residents Guide to the ICU
21. Management of Mechanical Ventilation
Edward A. Bittner, M.D., Ph.D.
A 66-year-old gentleman with a history of CAD and bronchospastic emphysema is admitted to the ICU
after an AAA repair. Overnight he remains on full vent support (AC = 10, VT = 800, FiO2 = 40%, PEEP =
5). His fluid requirements have diminished and he is hemodynamically stable. How would you wean from
here? IMV? PSV? T-piece?
Airway control and ventilatory support are commonly encountered ICU methodologies. At present, there
are numerous techniques available for the initial control and subsequent support of the respiratory
system. A thorough understanding of these techniques leads to individualized treatment strategies and
attenuation of complications.
I.
Airway
A. Indications for tracheal intubation
1. Hypoxemic respiratory failure
a) Intubation and positive pressure ventilation allow delivery of a high fractional inspired
oxygen concentration(FiO2) and reduces shunt by maintaining alveoli open.
2. Hypercapnic respiratory failure
a) Positive pressure ventilation reduces the work of breathing and thus prevents respiratory
muscle fatigue or speeds recovery when fatigue is present.
3. Airway protection
a) Intubation secures the airway in order to reduce the risk of aspiration.
4. Impending hemodynamic collapse
5. Facilitation of suctioning/pulmonary toilet
B. Endotracheal tubes
1. Oral
Oral intubation is the most familiar technique for intubation. It is usually performed with direct
laryngoscopy although fiberoptic laryngoscopy may also be used. Adequate mandible and
neck mobility are required to allow direct visualization.
2. Nasal
Nasotracheal intubation may be performed as a blind procedure guided by breath sounds or
under direct vision with laryngoscopy or fiberoptic bronchoscopy. Nasotracheal intubation can
be performed when the oral route is difficult or impossible. Disadvantages of nasotracheal
intubation include the potential for nasal hemorrhage during placement, increased airway
resistance and difficulty with suction catheter passage due to kinking of the tube in the
nasopharynx and increased risk of sinusitis.
3. Double lumen
a) Double lumen tubes are composed of two tubes: one is inserted into a main stem
bronchus and the other ends in the trachea. Both tubes have cuffs that ensure
independent ventilation. Proper positioning of the double lumen tube is essential and
placement is usually confirmed with bronchoscopic guidance. Minimal patient movement
may cause dislodgment so sedation and paralysis are usually necessary.
b) Placement of a Double lumen tube is indicated for lung protection (for significant
hemoptysis or unilateral infection) , bronchoalveolar lavage, independent lung ventilation
or surgical exposure. Complications of lung isolation techniques include collapse of an
obstructed segment of the lung, airway trauma, hypoxia and hypoventilation during
placement and as a result of malposition.
4. Bronchial blockers
Bronchial blockers are used in situations in which it is not possible to place an endobronchial
tube, such as in pediatric patients, in those with difficult airway anatomy or where satisfactory
lung isolation cannot be achieved by other means. Univent tubes are large caliber
endotracheal tubes encompassing a small channel for a bronchial blocker.
C. Surgical airway
1. Cricothyrotomy
93
ASCCA Residents Guide to the ICU
The primary indications for cricothyrotomy are establishment of an emergent airway when
endotracheal intubation is unsuccessful and when oropharyngeal airway obstruction is
caused by trauma or a foreign body. Cricothyrotomy can be accomplished by either needle
technique or surgical incision. Cricothyrotomy takes less time to perform than tracheostomy
and is associated with less bleeding.
2. Tracheostomy
The primary indication for tracheostomy is to maintain a secure airway in patients who require
long term ventilation. There is considerable controversy as to the optimal timing for
performing a tracheostomy. Advantages of a tracheostomy compared with an endotracheal
tube include lower airway resistance, greater patient comfort, increased easy of suctioning,
allowing patients to swallow secretions and nutrition, and providing the ability to
communicate. Risks associated with tracheostomy placement include hemorrhage, infection,
tracheostomy dislodgment, and tracheal stenosis.
D. Endotracheal tube management
After insertion, the endotracheal tube is subject to dynamic changes due to patient pathology as
well as to medical treatment. Complications can occur if the tube is not well managed: inspissated
secretions can block the tube inhibiting gas exchange; the endotracheal tube can slip into the
mainstem bronchus with neck flexion or out of the trachea with neck extension; if the amount of
air in the pilot balloon is too little an air leak and reduced tidal volumes will result. Conversely if
the pilot balloon is overinflated tracheal ischemia can result. Ideally cuff pressures should be less
than 25 mmHg to minimize ischemic complications.
II. Ventilators
A. Positive pressure ventilation applies positive pressure to the airway during inspiration. Positive
pressure ventilation is used almost exclusively in the ICU.
B. Negative pressure ventilators create intermittent negative pressure around the thorax and
abdomen. The “iron lung” which was popular during the polio outbreaks of the 1940s-50s is the
prototypical example. Negative pressure ventilation is almost never used in the ICU.
III. Modes of Ventilation
A. The mode of mechanical ventilation describes the control (pressure, volume, flow, time) and
phase variables (trigger, limit, cycle) that define how ventilation is provided. The control variable
remains constant throughout inspiration, regardless of impedance. Most common types are
volume control and pressure control. The phase variables include the trigger, limit and cycle
variables. The trigger variable is adjusted to sense patient effort for initiation of the inspiratory
phase. The limit-ventilation variable rises no higher than a given preset value or increases to a
preset value before inspiration ends. Cycle is the variable which terminates inspiration, which is
commonly volume, time or flow. Common modes of mechanical ventilation are described below:
B. Assist-control
With assist-control ventilation, a minimal respiratory rate and tidal volume (or pressure) is set.
The ventilator senses an inspiratory effort by the patient and responds by delivering a preset tidal
volume. Every inspiratory effort which satisfies the ventilator’s trigger initiates delivery of the tidal
volume. A control mode backup rate is set to prevent hypoventilation.
C. Intermittent mandatory ventilation/SIMV
Intermittent mandatory ventilation (IMV) is a ventilatory mode in which mandatory breaths are
delivered at a set rate and volume. Synchronized intermittent mandatory ventilation (SIMV) is IMV
in which the breaths are delivered in synchrony with the patient’s own inspiratory effort. Between
mandatory breaths, the patient is allowed to breathe spontaneously without support or supported
with levels of pressure support or CPAP. If a patient does not make spontaneous breathing efforts
the ventilator delivers breaths to achieve the set rate.
D. Pressure support
Pressure support ventilation is a pressure-preset, flow-cycled ventilator mode used to support
spontaneous respiratory efforts. With each inspiratory effort the patient triggers the ventilator,
which maintains the preset pressure level in the circuit throughout inspiration. Depending on the
ventilator, the inspiratory cycle ends when the flow rate decreases to a predetermined level (less
than 25% peak flow rate, flow rate is < 5L/minute or pressure change).
E. Pressure control
With pressure control ventilation, the pressure applied to the airway is constant throughout the
94
ASCCA Residents Guide to the ICU
F.
G.
H.
I.
J.
K.
L.
inspiratory time. The inspiratory flow during pressure control ventilation is decelerating and is
determined by the pressure control setting, airway resistance and respiratory system compliance.
Pressure control can be used with either assist-control or SIMV modes of ventilation.
Inverse ratio ventilation
Inverse ratio ventilation (IRV) increases the mean airway pressure by prolonging the inspiratory to
expiratory (I:E) ratio. The prolonged inflation time can help prevent alveolar collapse resulting in
improved oxygenation. Heavy sedation or neuromuscular blockade is usually required for patients
to tolerate IRV.
Airway pressure release ventilation
Airway pressure release ventilation (APRV) cycles between a high continuous positive airway
pressure (hCPAP) and a low continuous positive airway pressure(lCPAP). Transition from hCPAP
to lCPAP allows deflation and transition from lCPAP to hCPAP allows inflation. APRV improves
oxygenation by maximizing alveolar recruitment and maintaining ventilation-perfusion matching.
Noninvasive Positive Pressure Ventilation
1. Noninvasive positive pressure ventilation (NPPV) is the delivery of mechanically assisted or
generated breaths without an endotracheal or tracheostomy tube. Ventilation in NPPV is
delivered via nasal or facial mask or helmet. Advantages of NPPV include: avoiding the risks
related to intubation, sedation and less development of nosocomial pneumonia.
Disadvantages of NPPV include: it does not protect against aspiration, less airway pressure
is tolerated and there is no access to the airway for suctioning. NPPV is most beneficial for
patients with acute COPD exacerbations and patients with cardiogenic pulmonary edema.
Other indications include weaning (postextubation), obesity-hypoventilation syndrome, acute
respiratory failure postsurgery and in patients deemed not to be intubated (DNI).
2. Continuous positive airway pressure (CPAP) works by generating a continuous airflow that
maintains a continuous positive pressure to the respiratory system during inspiration and
expiration thus preventing airway and alveolar collapse. By maintaining end expiratory lung
volume, CPAP may improve alveolar ventilation and oxygenation by preventing airway
obstruction and reversing atelectasis.
3. Bi-level positive airway pressure (BiPAP) therapy is independently set inspiratory positive
airway pressure (IPAP) and expiratory positive airway pressure (EPAP). Tidal volume results
from the difference between the IPAP and the EPAP. BiPAP therapy is effective at reducing
PaCO2.
PEEP: Positive end expiratory pressure (PEEP) may increase oxygenation in lung diseases
characterized by lung collapse.
1. Extrinsic PEEP (PEEP set on the ventilator) maintains alveolar recruitment, increases
functional residual capacity, decreases pulmonary shunt and may improve lung compliance.
The application of PEEP may have disadvantages: it increases intrathoracic pressure which
may decrease venous return and compromise cardiac output. In addition, since PEEP has
the greatest effect on the compliant regions of the lungs, overdistention can occur resulting in
increased deadspace and high levels of PEEP may contribute to ventilator induced lung injury
(VILI).
2. Intrinsic or auto-PEEP results from pressure developing from gas trapping (dynamic
hyperinflation) due to insufficient expiratory time and/or increased expiratory airflow
resistance. Increasing the expiratory time and reducing airway resistance are methods to
reduce auto-PEEP.
Permissive hypercapnia is an approach to limit ventilator associated acute lung injury through
deliberate tolerance of alveolar hypoventilation and acceptance of hypercapnia. Contraindications
include increased intracranial pressure, right ventricular failure and ongoing acidemia.
Recruitment maneuvers refer to the application of elevated pressures and volumes for variable
duration, magnitude and frequency in an effort to recruit atalectatic lung.
Complications
1. Oxygen toxicity: High concentrations of oxygen for long periods may cause lung damage.
Oxygen toxicity primarily affects normal lung units since these areas receive most of the
ventilation. The FiO2 should be reduced when possible provided that arterial oxygenation is
adequate. An FiO2 of less than 0.5 is generally considered safe.
2. Ventilator induced lung injury (VILI) occurs when the lung is directly damaged by the action of
mechanical ventilation. “Barotrauma” refers to injury resulting from alveolar overdistention
and rupture when high inspiratory pressures are applied. Pneumothorax,
95
ASCCA Residents Guide to the ICU
pneumomediastinum, pneumoperitoneum are all associated with alveolar rupture from
overdistention. “Volutrauma” refers to lung injury resulting from excessive volume, rather than
excessive pressure. The cyclic opening and closure of alveoli during inspiration and
expiration may also be injurious to the lungs causing “atalectrauma”. Finally, ventilation of the
lungs in a manner that promotes alveolar overdistention and derecruitment increases the
release of inflammatory mediators resulting in “biotrauma”.
IV. Monitoring
A. Oxygenation
1. Pulse oximetry provides noninvasive measurement of hemoglobin oxygen saturation using an
oximeter based on the principle of differential light absorption characteristics of different types
of hemoglobin. Pulse oximetry uses two wavelengths of light, red (660 nm) and infrared
(900-940 nm) which are passed through a pulsating vascular bed to a photodetector to
discriminate between oxyhemoglobin and deoxyhemoglobin. Pulse oximeters use empiric
calibration curves developed from studies of healthy volunteers. The accuracy of pulse
oximetry is В± 4% to 5% at saturations greater than 80% and less at lower saturations. Causes
of inaccurate pulse oximeter readings include: dyshemoglobinemias, dyes and pigments, low
perfusion, motion, abnormal pulses, anemia and external light sources.
2. Arterial Blood Gas (ABG)
a) Intermittent: In addition to measurement of the systemic acid–base status and CO2
levels, the arterial blood gas provides measurement of arterial oxygenation. The arterial
partial pressure of oxygen (PaO2) is a measurement of the amount of oxygen that is
dissolved in the blood. It is important to remember that the amount of oxygen dissolved in
the blood, the PaO2, makes up only a small proportion of the total arterial oxygen content.
The oxygen content of arterial blood (CaO2) consists of two components the oxygen
bound to hemoglobin and the oxygen dissolved in blood. The CaO2 can be quantified by
the equation:
CaO2 = [1.34 * Hgb * SaO2] + [PaO2 * 0.003]
where 1.34 ml O2 per gm Hgb is a constant, Hgb is hemoglobin in gm/dL,
SaO2 is the arterial oxygen saturation, PaO2 is in mm Hg, 0.003 ml O2 per mmHg per dL is a constant
b) Continuous arterial blood gas (ABG) monitors use sensors that detect variations in light
and fluorescence to allow the continuous display of ABG results and permit the
observation of trends over time. The O2 sensor uses fluorescent die and the H+ and CO2
sensors use fluorescent acids; fluorescence is inversely proportional to the amount of
substance being measured. Limitations associated with these monitors include less
accurate readings at higher PaO2 and PaCO2 levels, motion artifact, inaccuracies
resulting from clot formation at the catheter tip and the sensor adjacent to vessel wall.
Continuous monitors are not routinely used in the ICU.
3. Transcutaneous oxygen tension: Transcutaneous oxygen monitoring uses the polarographic
principle to measure PO2 at the skin surface. The device warms the skin to ensure adequate
perfusion to produce a transcutaneous oxygen tension (TcO2) approximating the PaO2. The
electrode site is changed every 4-6 hours to prevent skin burns. The technique is particularly
useful in neonates and infants, in whom arterial sampling may be difficult. However, adults
have thicker skin with decreased capillary density and TcO2 measurements do not correlate
well with PaO2. In addition, compromised hemodynamics cause underestimation of the PaO2.
Given the technical and physiologic limitations of transcutaneous monitoring it is rarely used
in the adult ICU.
4. The shunt fraction is the proportion of cardiac output that does not participate in gas
exchange. The normal shunt fraction is approximately 3% and is due to the bronchial arterial
circulation. The degree of shunt can be quantified by the shunt equation:
Qs/Qt = (CcO2 – CaCO2) / (CcO2 – CvO2)
where Qs/Qt is shunt fraction, CcO2 is end-capillary O2 content,
CaO2 is arterial O2 content and CvO2 is mixed venous oxygen content
5. Global oxygen delivery (DO2) is calculated as the product of the arterial oxygen content of the
blood (CaO2) and the cardiac output (CO):
DO2 = CaO2 * CO * 10
where CO is cardiac output and CaO2 is arterial content of blood.
6. Oxygen consumption (VO2) is calculated by the Fick equation:
VO2 = CO * (CaO2 – CvO2) * 10
where CaO2 and CvO2 are the arterial and mixed venous oxygen contents of blood respectively.
96
ASCCA Residents Guide to the ICU
B. Ventilation
1. Capnography/capnometry: The capnometer is a device that quantitatively measures the
amount of exhaled carbon dioxide while a capnograph provides the display and ability to
track changes in end tidal carbon dioxide. Quantitative capnometers measure CO2 using the
principles of infrared spectroscopy, Raman spectroscopy or mass spectroscopy.
Nonquantitative capnometers indicate CO2 by color change of an indicator material.
Capnometry in the ICU setting is used to confirm endotracheal tube placement and is
sometimes used to non-invasively estimate the arterial partial pressure of carbon dioxide.
However capnometry may be inaccurate in the presence of significant intrinsic lung disease.
2. ABG: The arterial partial pressure of CO2 (PaCO2) reflects the balance between carbon
dioxide production and alveolar ventilation. PaCO2 varies directly with carbon dioxide
production (VCO2) and inversely with alveolar ventilation (VA) as described by the equation:
PaCO2 = VCO2 / VA
Minute ventilation affects the PaCO2 only to the extent that it affects the alveolar ventilation.
3. CO2 production is a function of oxygen consumption and CO2 that is liberated in the buffering
of H+ ions. Increased CO2 production is usually accompanied by an increase in minute
ventilation which eliminates the excess CO2 and maintains a constant arterial CO2. However,
in patients with impaired ability to eliminate CO2 (e.g. by neuromuscular weakness or lung
disease), an increase in CO2 production can result in an increase in PaCO2. Overfeeding is a
recognized cause of hypercapnia in patients with severe lung disease and respiratory failure.
Overfeeding with carbohydrates is especially problematic since metabolism of carbohydrates
generates more CO2 than lipids or proteins.
4. Dead space refers to ventilation that does not participate in gas exchange. The dead space
ratio (VD/VT) is calculated from the Bohr equation which measures the ratio of dead space to
total ventilation:
VD/VT = (PaCO2 – PECO2) / PaCO2
where PECO2 is the CO2 concentration in mixed expired gas.
The normal dead space ratio is 0.3 to 0.4.
5. Transcutaneous carbon dioxide monitoring (TcCO2) has been used in the neonatal ICU but
with limited acceptance in adults. TcCO2 has been shown to correlate with PaCO2, but results
vary at higher CO2 levels.
C. Mechanics
1. Airway pressures
a) Peak
Peak airway pressure (Ppeak) is the pressure reached at end inspiration during positive
pressure volume controlled ventilation. Ppeak is the sum of the pressure required to
overcome airway resistance and the pressure required to overcome the elastic properties
of the lung and chest wall.
b) Plateau
The plateau pressure (Pplat) reflects the pressure required to overcome the elastic recoil
of the lung and chest wall. The plateau pressure is an estimate of peak alveolar pressure
which is an indicator of alveolar distention. Measurement of the plateau pressure requires
the absence of patient effort and is obtained with a short inspiratory hold.
c) Mean
Average pressure within the airway during one complete respiratory cycle; directly related
to the inspiratory time, respiratory rate, peak inspiratory pressure and PEEP.
2. Compliance is the change in volume (usually tidal volume) divided by the change in pressure
required to produce that volume. There are two types of compliance –static and dynamic.
a) Static compliance is measured when airflow is absent. It is calculated as:
Cstat= tidal volume/(Pplat – PEEP)
When airflow is absent, airway resistance is not a determining factor. Thus static
compliance reflects the distensibility of the lung and chest wall only.
b) Dynamic compliance is measured when airflow is present (at end inflation). Since airflow
is present, airway resistance contributes to the measurement of dynamic compliance. It is
calculated as:
Cdyn= tidal volume/(Ppeak – PEEP)
Comparing static and dynamic compliance can help in identifying the cause(s) for
difficulty in ventilating or weaning from mechanical ventilation. In conditions where airway
resistance is increased (e.g. bronchospasm) dynamic compliance will decrease while
97
ASCCA Residents Guide to the ICU
static compliance while remain unaffected.
3. Resistance (R) is the ratio of driving pressure to flow and is calculated as:
R= (Ppeak –Pplat)/F
where Ppeak is the peak airway pressure, Pplat is the plateau pressure and F is the peak
inspiratory flow rate.
D. Miscellaneous
1. Esophageal pressure changes reflect changes in pleural pressure (the absolute esophageal
pressure does not reflect absolute pleural pressure). Esophageal pressure is measured with
a thin walled balloon containing a small volume of air which is placed in the lower esophagus.
Changes in esophageal pressure can be used to assess respiratory effort, work of breathing
and auto-PEEP during spontaneous breathing and patient triggered modes of ventilation. It
can also be used to assess chest wall compliance during full ventilatory support.
2. Transdiaphragmatic pressure specifically measures diaphragm strength. Balloon tipped
catheters placed in the mid esophagus and stomach are used to measure esophageal (Pes)
and gastric (Pga) pressures, respectively, and the difference in pressures (Pga –Pes) provides
the transdiaphragmatic pressure. Due to its invasiveness measurement of
transdiaphragmatic pressure is not routinely used in clinical practice.
3. Work of breathing: To achieve normal ventilation, work is performed to overcome the elastic
and frictional resistances of the lung and chest wall. Work of breathing can be calculated by
plotting transpulmonary pressure against tidal volume and measuring the subtended area.
Transpulmonary pressure is calculated from the difference between pleural pressure
(estimated by esophageal pressure) and the airway pressure.
V. Cardiopulmonary interactions
A. Interventricular dependence refers to the interaction between the right and left ventricles in which
enlargement of the right ventricle pushes the interventricular septum to the left and compromises
left ventricular size, filling and cardiac output.
B. Decreased venous return results from the increased intrathoracic pressure caused by positive
pressure ventilation. Intravascular volume replacement may counteract the negative
hemodynamic effects of positive pressure ventilation.
C. Reduced afterload: Lung expansion increases extramural pressure which helps pump blood out
of the thorax and thereby reduces LV afterload. In conditions where cardiac function is mainly
determined by changes in afterload rather than preload (e.g. hypervolemic patient with systolic
heart failure), positive pressure ventilation may be associated with improved cardiac output.
D. Shunt effects: In patients with an intracardiac shunt, positive pressure ventilation and PEEP may
increase right to left shunt and result in hypoxemia. The mechanism of increased shunting is
likely the result of Valsalva-like activity (e.g. breathing against the ventilator) or an increase in
pulmonary vascular resistance secondary to PEEP.
VI. Weaning
A. Assessment of Ventilator Discontinuation Potential
A few simple criteria should be evaluated before the patient undergoes an assessment for
ventilator discontinuation:
1. Resolution of underlying disease: Before mechanical ventilation can be safely withdrawn
there must be some evidence that the disease that lead to respiratory failure is improving.
2. Mental status: Patients should be awake, alert, able to manage secretions and protect their
airway. Sedation level should be adjusted so that the patient is able to cooperate with the
weaning process.
3. Hemodynamic stability: The patient should be hemodynamically stable without evidence of
myocardial ischemia and clinically significant hypotension requiring vasopressors.
4. Adequate gas exchange: The adequacy of gas exchange must be assessed before
discontinuation of mechanical ventilation. Gas exchange is adequate when arterial
oxygenation (PaO2 exceeds 60 mmHg with FiO2 of 0.5 or less and PEEP of 5 cm H2O or
less), and when there is adequate ventilation (PaCO2 such that pH ≥ 7.25).
5. Adequate respiratory muscle strength: For weaning to be successful the load placed on the
respiratory muscles must not exceed the muscles’ capacity to manage the load. Common
causes of high load are high airway resistance, low lung compliance and high minute
ventilation. Reduced muscle strength may result from disease, disuse, malnutrition, hypoxia
98
ASCCA Residents Guide to the ICU
or electrolyte imbalance. The maximal inspiratory pressure (MIP, described below) is an
indicator of overall respiratory muscle strength.
B. Weaning Parameters
Weaning parameters are objective measures used as predictors of weaning success. Most of
these parameters reflect only a single component of the respiratory system and thus are limited
predictors of weaning outcome.
1. RSBI: The rapid shallow breathing index (RSBI) is calculated by dividing the respiratory rate
by the tidal volume (in liters). A low RSBI (<105) has been used to predict successful
liberation from mechanical ventilation.
2. MIP: The maximal inspiratory pressure (MIP) also called the negative inspiratory force (NIF)
is a measure of respiratory muscle strength. It measures the maximal inspiratory pressure
that is generated during inspiration against an occluded airway. A NIF more negative than –30
cm H2O has been used to predict successful ventilator liberation.
3. P0.1: The airway pressure generated 100 ms after initiating an inspiratory effort against an
occluded airway (P0.1) is believed to reflect central respiratory drive. A high respiratory drive
(P0.1 of –4 to –10 cm H2O) has been used to predict successful ventilator liberation.
C. Ventilatory modes and weaning: The method of weaning varies with ventilatory mode as
described below:
1. PSV: With pressure support ventilation the level of inspiratory pressure assistance is
gradually decreased until the patient is able to breath without assistance (usually a pressure
support of < 10 cm H20).
2. SIMV: With SIMV the mandatory rate is gradually decreased until the patient is able to breath
without assistance (usually when the patient receives < 4 mandatory breaths/min). Several
studies suggest that SIMV weaning is inferior to other approaches and may prolong the
duration of mechanical ventilation.
3. SBT: A spontaneous breathing trial (SBT) is more predictive of a patient’s readiness to
breathe without ventilatory support than is any other weaning parameter reported to date. An
SBT can be performed on the ventilator with minimal pressure support and PEEP settings or
off of the ventilator with a T-piece or tracheostomy collar. Most patients who tolerate an SBT
of 30 to 120 minutes can be successfully discontinued from mechanical ventilation.
VII. Miscellaneous
A. Hyperbaric oxygen
Hyperbaric oxygen (HBO) treatment involves intermittently breathing 100% oxygen at > 1
atmosphere of pressure. Mechanisms of action for HBO therapy stem from 2 types of effects:
hyperoxygenation of perfused tissues and reduction of gas bubble volume. HBO therapy has
been used in the treatment of a heterogeneous group of disorders:
1. Air embolism: The use of HBO is based on Boyles Law since the volume of air (nitrogen)
bubbles is inversely related to the pressure exerted on them. Nitrogen bubbles are further
reduced by the replacement of nitrogen with oxygen which is rapidly metabolized. Air
embolism can result from surgical procedures when air is entrained through a disrupted
vascular wall.
2. Decompression sickness: Divers breathing compressed air who return to the surface too
rapidly are at risk for decompression sickness (“bends”). Bubble formation in tissues and
blood occurs as the partial pressure of inert gas (mostly nitrogen) exceeds that of ambient air.
HBO is the primary treatment for decompression sickness through its effects on decreasing
bubble size and relief of hypoxia.
3. Carbon monoxide poisoning: HBO reduces the half life of carboxyhemoglobin (CO) which
may prevent the late neurocognitive deficits associated with severe CO poisoning.
4. Soft tissue infections: HBO has been used as an adjunct therapy for severe life- or limbthreatening necrotizing infections such as clostridial myonecrosis, necrotizing fasciitis and
Fournier’s gangrene.
B. Split-lung
Patients with severe unilateral lung disease may require different ventilation strategies applied to
each lung. Examples where split lung ventilation may be used include unilateral pulmonary
contusion, bronchopleural fistula or following single lung transplant. Split lung ventilation may be
achieved thru the use of a double lumen tube.
C. Heliox is gas mixture of helium and oxygen that is used in conditions of high airflow resistance.
99
ASCCA Residents Guide to the ICU
Helium is less dense than air and flows more readily through regions of reduced cross sectional
area where flow is turbulent. Heliox may be useful in conditions of high airflow resistance such as
acute exacerbations of asthma or COPD, and extrathoracic or tracheal obstruction. Heliox is
generally well tolerated but use may be limited in critically ill patients due to the high
concentration of helium required which limits FiO2 delivery. Most studies use helium:oxygen
mixtures of 80:20 or 70:30 to achieve benefit.
D. High Frequency ventilation
High frequency ventilation achieves gas exchange by combining very high respiratory rates with
very low tidal volumes (smaller than anatomic dead space). Potential advantages include a lower
risk of barotrauma due to smaller tidal volumes and improved gas exchange due to more uniform
distribution of ventilation, enhanced diffusion and greater alveolar recruitment.
1. High frequency jet ventilation (HFJV) delivers pulses of gas at high velocity and rapid
frequency into the trachea through a small catheter. The high flow jet pulse produces a jet
mixing effect that creates an area of negative pressure and entrains additional gas.
Exhalation is a passive process. HFJV had been used in patients with ARDS with the goal of
reducing airway pressures and ventilator induced lung injury.
2. High frequency oscillatory ventilation (HFOV) delivers low tidal volumes using an oscillatory
pump. During HFOV both inspiration and expiration are active processes. The amplitude of
the oscillation is adjusted to obtain adequate “chest wiggle”. HFOV oscillates around a
constant mean airway pressure which allows maintenance of alveolar recruitment while
avoiding high peak airway pressure. HFOV has been used primarily in children and neonates
where its use is associated with improved oxygenation and decreased barotrauma.
This chapter is a revision of the original chapter authored by Todd Dorman, M.D.
READING LIST:
1. Bigatello LM ed. Critical Care Handbook of the Massachusetts General Hospital, 4th Ed. Philadelphia:Lippincott
Williams & Wilkins, 2006.
2. Fink MP, Abraham E, Vincent JL and Kochanek eds. Textbook of Critical Care,5th Ed. Philadelphia: Elsevier
Saunders, 2005.
3. Hess D, MacIntyre NR, Mishoe SC et al eds.. Respiratory Care: Principles and Practice. W.B. Saunders, 2002.
4. Marino PL. The ICU Book, 3rd Ed. Philadelphia: Lippincott Williams & Wilkins, 2007.
5. McLean B and Zimmerman JL eds. Fundamental Critical Care Support, 4th Ed. Society of Critical Care Medicine.
Mount Prospect Il, 2007.
QUESTIONS:
21.1.
A.
B.
C.
D.
E.
Endotracheal cuff pressure should not exceed:
10 mmHg
15 mmHg
18 mmHg
25 mmHg
40 mmHg
21.2.
A.
B.
C.
D.
Static compliance will be independently affected by:
Bronchospasm
Pulmonary edema
Kinked endotracheal tube
Mucous plugging
100
ASCCA Residents Guide to the ICU
21.3.
A.
B.
C.
D.
E.
A patient is paralyzed and mechanically ventilated at a set rate and tidal volume. A sudden increase in
PaCO2 can represent all of the following EXCEPT:
Pulmonary embolism
Overfeeding syndrome
Sepsis
Increased shunt
Increased dead space
21.4.
A.
B.
C.
D.
Goals of adding PEEP to routine ventilatory settings include all of the following EXCEPT:
Improved lung compliance
Improved oxygen transport
Decreased work of breathing
Reduced dead space
21.5.
A.
B.
C.
D.
Which of the following is most likely to occur from the initiation of positive pressure ventilation?
Decreased preload and decreased afterload
Decreased preload and increased afterload
Increased preload and increased afterload
Increased preload and decreased afterload
21.6.
A.
B.
C.
D.
Helium can be beneficial to patients with airway obstruction because:
The viscosity of helium is less than oxygen
The density of helium is less than oxygen
Helium is non-toxic
Reduces the FiO2 administered
21.7.
Which weaning mode/method is inferior to other approaches and may prolong the duration of
mechanical ventilation?
Pressure support ventilation (PSV)
Synchronized intermittent mandatory ventilation (SIMV)
Spontaneous breathing trial (SBT)
Proportional assist ventilation (PAV)
A.
B.
C.
D.
101
ASCCA Residents Guide to the ICU
22. Side-Effects of Mechanical Ventilation
Jean Kwo, M.D. and Luca M. Bigatello, M.D.
A 30-year-old man with the diagnosis of ARDS is transferred to your hospital. He arrives with a blood
pressure of 80/60, a heart rate of 140, and an SpO2 of 93%. Ventilator settings are: tidal volume: 1 liter,
rate: 10, PEEP: 10 cm H2O, FiO2: 60%, with a mean airway pressure of 52 cm H2O. On exam, he has
diminished breath sounds on the right side.
Patients with acute respiratory failure constitute a significant portion of admissions to a critical care unit,
and constantly challenge the skills of the intensivist to maintain patient homeostasis. Typically,
oxygenation and ventilation are primary concerns, as well as maintaining concomitant cardiovascular
integrity. Numerous ventilatory strategies have evolved to meet these demands, each of which has
attendant side- effects. This section outlines and characterizes some of the side- effects of mechanical
ventilation that should be understood by physicians in the ICU.
I.
Side-Effects of Positive Pressure Ventilation
A. Ventilator- induced lung injury (VILI)
1. Large inspiratory volumes and high airway pressures can injure the lung. While this effect
may not be clinically relevant in subjects with healthy lungs (e.g., during general anesthesia),
it has been shown to worsen lung injury and even worsen the outcome of patients with
severe acute respiratory failure.
2. The pathogenesis of VILI is multifactorial, including mechanical (�stretch’) and biological
(inflammation and injury) damage. In addition, inflammatory mediators from the damaged
lung may reach distant organs and contribute to multiorgan system dysfunction syndrome
(MODS).
3. VILI results in damage (edema, fibrosis) of the affected areas of the lung, and occasionally
progresses to more acute manifestations related to the rupture of overdistended alveoli.
4. Clinical manifestations:
a) In the majority of patients, VILI has no acute manifestations. It progressively aggravates
the pathologic and clinical course of the underlying injury.
b) Occasionally, VILI may result in macroscopic damage to the lung that includes interstitial
emphysema, subpleural air cysts, pneumomediastinum, pneumopericardium,
pneumoperitoneum, subcutaneous emphysema, and pneumothorax.
5. Although the exact threshold of pressure and volume that is dangerous to the lung is not
known, data suggest that VILI can occur at inspiratory pressures above 25- 30 cm H2O. Tidal
volumes above 6 mL/kg may result in an increased mortality rate.
6. It is important to understand that airway pressure is just an approximation of the alveolar
pressure. In patients with a low chest wall compliance (e.g., from morbid obesity, high intraabdominal pressure), airway pressure overestimates the alveolar pressure. Airway pressure
underestimates alveolar pressure in patients who are making spontaneous breathing efforts
(e.g., during pressure support ventilation).
B. Cardiovascular Effects of Positive Pressure Ventilation
1. Effects on right ventricular (RV) performance:
a) Improved RV performance:
(1) Decreased pulmonary vascular resistance results from improved oxygenation and
ventilation.
b) Worsened RV performance:
(1) Decreased venous return: positive intrathoracic pressure decreases the gradient for
systemic venous return.
(2) Increased pulmonary vascular resistance can result from lung overdistention В±
hypoxemia/hypercarbia.
2. Effects on left ventricular (LV) performance:
a) Improved LV performance
(1) Decreased LV transmural pressure required to generate the same systemic blood
pressure, effectively reducing afterload
102
ASCCA Residents Guide to the ICU
b) Worsened LV performance:
(1) Decreased LV filling as a result of decreased systemic venous return and/or
ventricular interdependence in the presence of RV dilatation.
3. Hemodynamic compromise is correlated with increased mean airway pressures (reflecting
mean alveolar pressures). Mean airway pressure is influenced by:
a) The level of intrinsic or applied PEEP
b) Tidal volume and minute ventilation
c) Inspiratory time: prolonged inspiratory times (e.g., inverse I:E ratios) prevent alveolar
emptying and promote auto-PEEP
d) The likelihood of hemodynamic compromise is increased by hypovolemia
e) Compliant lungs or a stiff chest wall may increase the effect of airway pressures on
hemodynamics
C. Oxygen Toxicity
1. High inspired O2 concentrations can injure the respiratory mucosa, increase permeability of
the alveolar-capillary membrane, and impair mucociliary transport. Over prolonged periods of
time, they can lead to pulmonary fibrosis.
2. In the setting of critical illness, the variables of O2 concentration and duration of exposure that
produce significant lung damage have not been established. These factors probably interact
with severity of disease and individual susceptibility.
3. An F1O2 of less than 50% is generally considered safe.
II. Side-Effects of Airway Intubation
A. Route of intubation:oral vs. nasal
1. Oral intubation is less traumatic than nasal and allows the use of larger diameter tubes which
facilitate suctioning and minimize airway resistance.
2. Nasal intubation has fallen out of favor because of the possible damage at the time of
insertion of the tube (nose bleeding), the need for small tubes, and the high incidence of
sinusitis.
B. Increased risk of nosocomial pneumonia
1. Both tracheobronchitis and pneumonia are frequent complications of mechanical ventilation
delivered through an endotracheal tube.
2. Nosocomial pneumonia is common in critically ill patients and is associated with a high
mortality.
3. Endotracheal tubes bypass the natural upper airway defense mechanisms and impair
mucociliary transport and secretion clearance. Bacterial overgrowth along the lumen of the
tube (biofilm) has been observed within 24 hours of tracheal intubation.
4. Oropharyngeal and gastric colonization by nosocomial organisms (multi-resistant Gram
positive cocci, enteric and non-enteric Gram-negative bacilli) is believed to be the primary
pathogenic mechanism of nosocomial respiratory infection.
5. Non- invasive mask ventilation decreases the incidence of pneumonia and has been
associated with higher survival rates in selected patient populations, particularly in patients
with COPD exacerbation and cardiogenic pulmonary edema.
C. Tube misplacement and dislocation
1. Usually results from inadequate external fixation coupled with excessive neck movement.
Neck flexion and extension can move the tube up to 5 cm.
2. Mainstem intubation–can cause overdistention of the ventilated lung, and hypoventilation and
atelectasis of the nonventilated lung.
3. Upward migration of the tube to the level of the vocal cords or above results in low tidal
volumes, the ability to phonate, and escape of air through the mouth and nose.
4. Inadvertent extubation is a potentially lethal complication and is associated with an increase
in subsequent laryngotracheal complications.
5. Confirmation of the adequacy of tube placement by a chest radiograph may reduce the
incidence of tube misplacement.
D. Airway injury
1. Intubation can result in injury to the hypopharynx, larynx, and trachea.
2. Glottic edema and minor vocal cord lesions occur frequently, but they resolve rapidly in most
patients.
3. Tracheal stenosis is a major, though rare, complication. Important determinants include:
103
ASCCA Residents Guide to the ICU
Cuff pressure (most important factor) above capillary perfusion pressure (approximately
25 cm H2O) may lead to ischemic ulceration and mucosal damage.
b) Less specific factors include: duration of intubation, tube diameter relative to laryngeal
and tracheal diameter, tube movement, hypotension, tube and cuff design
Tracheostomy complications: tracheal erosion, tracheoinnominate fistula, tracheoesophageal
fistula, stomal granulations, and tracheal stenosis
a)
4.
III. Effects of Sedation and Paralysis
A. Sedation can cause vasodilation, hypotension and decreased cardiac output
B. Paralytic agents can lead to the retention of secretions, atelectasis, and muscle wasting.
Neuromuscular blocking agents have been associated with prolonged neuromuscular damage
that persists long after the cessation of drug administration
IV. Other Side-Effects
A. Renal Dysfunction
1. Reduced circulating blood volume related to increased intrathoracic pressures and decreased
cardiac output may lead to altered perfusion of the renal parenchyma, redistribution of
intrarenal blood flow, release of antidiuretic hormone, and/or inhibition of atrial natriuretic
peptide. These physiologic effects can contribute to:
a) Disorders of sodium and water excretion and acute renal failure.
b) Decreased free water clearance and generalized fluid retention.
B. Gastrointestinal Dysfunction
1. Gastrointestinal complications associated with acute respiratory failure and mechanical
ventilation include gastric distention, ileus, diarrhea, and gastrointestinal bleeding.
2. The use of PEEP has also been associated with hyperbilirubinemia and mild elevations of
liver enzymes related to altered hepatic perfusion, and impeded venous and biliary drainage.
C. Increased Intracranial Pressure
1. Increased intrathoracic pressure can increase jugular venous pressure and impede
intracranial venous drainage resulting in increased intracranial pressure. Furthermore,
decreased cardiac output during mechanical ventilation can produce hypotension and, thus,
reduce cerebral perfusion pressure.
D. Psychological Disturbances
1. Sleep deprivation, pain, fear, inability to communicate, immobility and use of drugs (e.g.,
benzodiazepines) with dissociative properties during mechanical ventilation may produce
long-lasting psychological disturbances.
This chapter is a revision of the original chapter authored by Jean Kwo, M.D. and James G. Cain, M.D.
READING LIST:
1. The Acute Respiratory Distress Syndrome Network. Ventilation with lower tidal volumes as compared with
traditional tidal volumes for acute lung injury and the acute respiratory distress syndrome. N Engl J Med
2000;342:1301–1308.
2. Garpestad E, Brennan J, Hill NS. Noninvasive Ventilation for Critical Care. Chest 2007;132:711-720.
3. Kuiper JW, Groeneveld ABJ, Slutsky AS, Plotz FB. Mechanical ventilation and acute renal failure. Crit Care Med.
2005;33:1408-1415.
4. Keith RL, Pierson DJ. Complications of mechanical ventilation: a bedside approach. Clin Chest Med
1996;17:439-451.
5. McGuire G, Crossley D, Richards J, Wong D. Effects of varying levels of positive end-expiratory pressure on
intracranial pressure and cerebral perfusion pressure. Crit Care Med 1997;25:1059-1062. Discussion of issues in
mechanically ventilated neurosurgical patients.
6. Porzecanski I, Bowton DL. Diagnosis and Treatment of Ventilator-Associated Pneumonia. Chest. 2006;130:597-604.
7. Slutsky, AS. Lung Injury Caused by Mechanical Ventilation. Chest. 1999;116:9S-15S.
104
ASCCA Residents Guide to the ICU
QUESTIONS: (K-type)
A:
B:
C:
D:
E:
1,2,3 are correct
1,3 are correct
2,4 are correct
4 is correct
1,2,3,4 are correct
22.1.
1.
2.
3.
4.
Ventilator-associated Lung Injury:
Is due to large tidal volumes and high airway pressures
Results in macroscopic lung damage in most patients
Injury may be due to mechanical and biological factors
Can cause lung damage even in healthy lungs
22.2.
1.
2.
3.
4.
Mean airway pressure:
Is decreased by increased inspiratory time
Does not affect oxygenation
Is not correlated with hemodynamic changes
Is determined by tidal volume, minute ventilation, and level of PEEP
22.3.
1.
2.
3.
4.
Decreased cardiac output may be a result of:
Increased PEEP
Increased right ventricular afterload
Decreased venous return
Increased intrathoracic pressures
22.4.
1.
2.
3.
4.
Intrinsic PEEP:
Decreases venous return
May cause airway pressure to underestimate alveolar pressure
May increase difficulty triggering ventilator
May be an unrecognized etiology of hemodynamic collapse
22.5.
1.
2.
3.
4.
Nosocomial pneumonia:
Is due to colonization of oropharyngeal and gastric mucosa by nosocomial organisms
Is a common but benign complication of endotracheal intubation
Biofilm has been observed within 24 hours of endotracheal intubation
Is easily diagnosed
22.6.
1.
2.
3.
4.
Noninvasive ventilation:
May be preferred in patients with acute COPD exacerbation
Requires heavy sedation for patient compliance
May cause gastric distention
Does not place patient at risk for aspiration
22.7.
1.
2.
3.
4.
Tracheal injury:
Is diagnosed early by evidence of tracheal dilation on chest X-ray
May be avoided by maintaining cuff pressure less than 25 cm H2O
Only occurs with prolonged intubation
May occur when cuff pressure exceeds capillary perfusion pressure
22.8.
1.
2.
3.
4.
Mechanical ventilation affects urine output by:
Inhibiting atrial natriuretic peptide release
Decreasing splanchnic blood flow
Increasing antidiuretic hormone release
Directly compressing renal parenchyma by depressing the diaphragm
105
ASCCA Residents Guide to the ICU
23. Lung Protection Strategies
Paul G. Jodka, M.D.
A 28-year-old woman is admitted to the ICU with multiple injuries, including pulmonary contusions,
following a motor vehicle collision. ABG shows PaO2 = 40 mmHg, PaCO2 = 55 mmHg, and pH = 7.28
with an FiO2 = 100%. A pulmonary artery (PA) catheter measures a PA pressure of 55/28 and a PA
wedge pressure of 13.
This case is typical of a patient admitted to the ICU requiring mechanical ventilation to treat severe lung
injury in addition to other concerns. Over the last several years, well-executed multicenter studies have
shed light on aspects of treatment of acute lung injury (ALI) and the Acute Respiratory Distress syndrome
(ARDS) that should help clinicians standardize key aspects of the treatment of such patients It is
presently well-accepted that it is of crucial importance to pay close attention to the ventilator strategy and
fluid balance in patients with ALI/ARDS in order to minimize the likelihood of worsening not only lung
function, but remote organ function as well.
Etiology of injury: After the initial injurious event [which could result from direct alveolar injury or from
remote processes (e.g. systemic inflammation)], imprudent ventilation can be further harmful to patients.
Regional alveolar overdistention during mechanical ventilation through the delivery of “normal” tidal
volumes can induce or exacerbate lung injury in this setting. This occurs because the lung injury in ALI/
ARDS is generally heterogeneous, and gas will preferentially flow to the most compliant (non-injured)
regions. Thus, delivery of “normal” tidal volumes may result in areas of healthy lung becoming
overstretched, while diseased segments effectively do not participate in gas-exchange. Alveolar collapse
at end expiration with cyclic reopening may additionally exacerbate lung injury by shear force generation
and cytokine release.
Excessive alveolar or transpulmonary pressure may produce pulmonary barotrauma. End-inspiratory
occlusion pressure (i.e., plateau pressure), measured when there is no airflow, is believed to represent
mean end-inspiratory alveolar pressure better than peak inspiratory pressure (PIP), which includes the
contribution of inspiratory flow and airway resistance. Oxygen toxicity is a potential concern, though its
clinical importance in the injured human lung is uncertain.
Strategies for Minimizing Lung Injury
Ventilation should be performed with tidal volumes ranging within the mid-portion of a static pressurevolume curve of the respiratory system. PEEP may be adjusted to keep functional residual capacity
(FRC) above the lower inflection point (prevents damage due to repetitive shearing open of collapsed
alveoli), and the tidal volume can be reduced to avoid exceeding the upper inflection point (prevents
damage due to alveolar overdistention). Since the generation of pressure-volume curves is not universally
practical, we suggest the following initial approach to the mechanical ventilation of the patient with ALI/
ARDS:
Initial Ventilator Settings:
Tidal volume: Starting with a Vt of 8 ml/kg predicted/ideal body weight (IBW) the Vt is incrementally
lowered to a target volume of 6 ml/kg IBW (IBW in hg: male = 50 + 0.91(centimeters of height – 152.4);
female = 45.5 + 0.91(centimeters of height – 152.4)). This adjustment of Vt to predicted/ideal body weight
effectively normalizes Vt to lung size (which most closely correlates with gender/height).
Respiratory rate: an initial rate should be set between 18 – 22 breaths per minute. Adjustments to the
respiratory rate can be made as needed based on blood gas analysis, with rates not to exceed 35 breaths
per minute.
106
ASCCA Residents Guide to the ICU
PEEP/FiO2: The application of PEEP raises mean airway pressure, increases the proportion of aerated
lung, and is thought to prevent derecruitment of small airways and alveoli, resulting in an improvement of
gas-exchange in addition to lowering risk of lung injury (from repetitive shear/collapse). Conceptually,
optimizing PEEP is a balance between recruiting the “recruitable” alveoli of diseased lung without
overdistending aerated uninjured alveoli. Excessive FiO2 might itself be injurious, and it appears prudent
to adjust PEEP/FiO2 in tandem as delineated by the ARDSnet trials (detailed in reference 2).
Assessment of ventilatory strategy:
Airway pressures: the most relevant variable to consider during lung protective ventilation is the
transpulmonary pressure (plateau pressure minus pleural pressure). Normal lung is maximally distended
at transpulmonary pressures of 30 – 35 cm of H2O and pressures exceeding those will result in
overdistention. Since pleural pressure assessments are not universally available, the plateau pressure
itself is the key variable to assess in response to the delivery of a given tidal volume. If the plateau
pressure exceeds 30 cm H2O at the target tidal volume of 6 ml/kg IBW, the volumes should be lowered in
one cm H2O increments to a minimum of 4 ml/kg IBW for a target Vt less than 30 cm H2O. The respiratory
rate can be increased concomitantly, with a limit at 35 breaths per minute.
Blood gas analysis: Limitation of tidal volumes (and resultant net alveolar ventilation) can result in
hypercarbia, which can be tolerated if the resultant respiratory acidosis is not extreme (permissive
hypercarbia). A general goal of PaO2 of 55 mmHg or greater and SpO2 of 88% or greater is acceptable.
Salvage Therapies:
High-Frequency Ventilation (HFV): HFV refers to ventilatory support using higher than normal breathing
rates, often at rates of several hundred “breaths” per minute. At these rates, Vt is often less than anatomic
dead space, and peak airway pressures are reduced. Attractive features of HFV include that reduced
airway pressure swings may reduce barotrauma, and that altered flow delivery characteristics may
improve ventilation-perfusion matching. Dedicated ventilators are used for this form of ventilation (either
high-frequency jet ventilators or oscillators). Most experience with this form of ventilatory support has
been gained in the neonatal and pediatric populations, with more limited experience in adults (trials using
HFV in adults with ARDS are ongoing; trend towards decreased mortality using HFV vs. conventional
ventilation in 2002, though standard ventilation group used tidal volumes larger than what is now
considered desirable). When to initiate HFV is uncertain; early application seems physiologically
advisable, but firm data is lacking at this point. Proponents of HFV suggest consideration of this mode of
ventilation for patients failing conventional lung-protective ventilation (e.g. persistent plateau pressures
greater than 30 cm H2O despite minimal Vt, with continued high FiO2 requirements).
Pressure-Control/Inverse-ratio Ventilation: Limitation of airway pressures is an attractive feature of
pressure-limited ventilatory modes. However, pressure-limitation is only part of the challenge with ALI/
ARDS, as excessive lung stretch (volutrauma) is a major concern in this condition. Changes in respiratory
system compliance are reflected in changes in Vt in this mode, making careful titration of Vt difficult.
Other interventions to improve oxygenation in ALI/ARDS include the use of inhaled vasodilators (Nitric
oxide, Prostacyclin) and partial-liquid ventilation (perflubron). No benefit regarding patient survival has
been demonstrated with these modalities. Prone positioning and the use of Surfactant are being presently
studied (prior trials have been negative). Extracorporeal Membrane Oxygenation is technically feasible at
dedicated centers, though optimism regarding this intervention is at present guarded.
Circulatory Management:
Reduction of extravascular lung water decreases ventilator and ICU days. A fluid conservative (versus a
fluid liberal) approach improves organ failure outcome in ARDS. CVP monitoring was equally effective
compared to use of a Swan-Ganz catheter (with fewer complications) to guide a fluid conservative
strategy in treating ARDS.
This chapter is a revision of the original chapter authored by Kevin Dennehy, M.D. and Brian J. Poore, M.D.
107
ASCCA Residents Guide to the ICU
REFERENCES:
1. Ware LB, Matthay MA: The acute respiratory distress syndrome. N Engl J Med 2000;342:1334-1349.
2. The Acute Respiratory Distress syndrome Network: Ventilation with lower tidal volumes as compared with
traditional tidal volumes for acute lung injury and the acute respiratory distress syndrome. N Engl J Med
2000;342:1301-1308.
3. Malhotra A: Low-tidal volume ventilation in the acute respiratory distress syndrome. 2007;357:1113-1120.
4. Girard TD, Bernard GR: Mechanical ventilation in ARDS: A state-of-the art review. CHEST 2007;131:921-929.
5. The National Heart, Lung, and Blood Institute Acute Respiratory Distress Syndrome (ARDS) Clinical Trials
Network: Pulmonary-artery versus central venous catheter to guide treatment of acute lung injury. N Engl J Med
2006;354:2213-2224.
6. The National Heart, Lung, and Blood Institute Acute Respiratory Distress Syndrome (ARDS) Clinical Trials
Network: Comparison of two fluid-management strategies in acute lung injury. N Engl J Med 2006;354:2564-2575.
QUESTIONS:
23.1.
A.
B.
Acute Lung Injury and ARDS are characterized by alveolar damage that is completely homogeneous in
severity throughout the lungs.
True
False
23.2.
A.
B.
C.
D.
Diagnostic criteria for ALI and ARDS include:
CXR alone is sufficient
Combination of CXR and PAO2/FiO2 ratio
Combination of CXR, PAO2/FiO2 ratio, and absence of hydrostatic pulmonary edema.
“I know it when I see it……..”
23.3.
A.
B.
C.
D.
Initial Ventilator settings for patients with ALI/ARDS should consider:
Height of the patient
Gender of the patient
Arterial blood gas analysis
All of above
23.4.
A.
B.
C.
D.
E.
Critical components of the management of patients with ALI/ARDS include:
Tidal volume of 6 ml/kg IBW
Achievement of plateau pressures less than 30 cm H2O
Negative fluid balance in hemodynamically stable patients
None of the above
All of the above
108
ASCCA Residents Guide to the ICU
24. Weaning From Mechanical Ventilation
Stephen O. Heard, M.D
A 75-year-old man with severe emphysema undergoes bilateral lung volume reduction surgery under
general anesthesia. The post-operative course is complicated by air leaks, pneumonia, and atrial
fibrillation requiring reintubation, mechanical ventilatory support, antibiotic therapy, and heart rate control
with intravenous diltiazem. One week following reintubation, the pneumonia is resolving but the patient
remains in atrial fibrillation with a rate of 80/min. During the day, the patient is weaned using pressure
support ventilation starting at 25 cm H2O and progressively decreasing to 15 cm H2O. At night the
patient, in addition to the pressure support, is rested on SIMV of 8 breaths/min. Despite this progress, the
patient develops severe dyspnea with a respiratory rate of 35/min and a heart rate of 140/min when the
PS is decreased to less than 15 cm H2O. Prepare a plan to wean the patient from mechanical ventilation.
Weaning is the abrupt or gradual discontinuation of mechanical ventilatory support. Although, for the
majority of patients, weaning is uncomplicated and spontaneous ventilation easily achieved, there are
patients with severe acute respiratory distress syndrome, chronic obstructive pulmonary disease, and
neuromuscular disorders in which the weaning process can account of up to 40% of the time that the
patient requires mechanical ventilation. The main modes of weaning are T-piece and continuous positive
airway pressure (CPAP), synchronous intermittent mandatory ventilation (SIMV), pressure support
ventilation (PSV), and combinations of the above. Although extubation (the removal of the endotracheal
tube, tracheostomy, or artificial airway from the airway) usually follows weaning, it is a different process,
and criteria related to the structure and function of the upper airway play an important role in the decision
to extubate:
I.
Weaning Classification
A. Immediate–approximately 75-80% of intubated intensive care patients
B. Acute–within 72 hours, approximately 10-15%
C. Chronic–greater than 72 hours, approximately 5-10%.
D. Terminal–following decisions to withhold/withdraw non-beneficial care
II. Weaning Readiness
A. Resolution of the pathological process–improvement in the underlying process causing
respiratory failure (pneumonia ARDS, cardiogenic pulmonary edema)
B. Reversal of the pharmacological process–reversal of neuromuscular blockers, decreased
concentrations of opioids and sedatives
C. Treatment of patient’s comorbidities:
1. Stable hemodynamics on minimal inotrope/vasopressor support
2. Fluid balance problems resolved
3. Acid/base balance restored to the patient’s pre-morbid status
4. Electrolytes corrected, especially potassium, calcium, and phosphate important for muscle
function
5. Patient’s mental status adequate to allow cooperation with weaning – daily cessation of
sedation to allow the patient to awaken and participate in the weaning trial.
6. Adequate peripheral nerve function and muscle strength
III. Weaning Criteria
A. Single index predictors–gas exchange and perfusion, ventilatory performance, and lung
mechanics
B. Multiple indices predictors (see below) – rapid shallow breathing, CROP weaning index
1. Positive predictive value–patient will wean if the indices predict success
2. Negative predictive value–patient will not wean if the indices indicate failure
3. If a predictor has a low negative predictive value compared to its positive predictive value,
weaning may inadvertently be delayed while waiting for the predictor to improve.
109
ASCCA Residents Guide to the ICU
IV. Single Index Predictors
A. Gas exchange and perfusion:
1. PaO2 > 60 mmHg and F1O2 < 0.4
2. P(A-a)O2 < 350 mmHg on F1O2 = 1.0
3. PaO2/PAO2 > 238
4. PaO2/PAO2 > 0.47
5. Shunt fraction < 20%
6. pH > 7.30
7. Vd/Vt <0.6
8. PaCO2 increase post-disconnect < 8 mmHg
B. Ventilatory Performance:
1. Vital capacity >10 mL/kg
2. Tidal volume >5 mL/kg
3. Respiratory rate < 25/min
4. Minute ventilation <10 L/min
5. Maximum minute ventilation >2 times resting minute ventilation
C. Lung Mechanics and Work of Breathing:
1. Maximum inspiratory force <-30 cm H2O
2. Occlusion pressure P0.1 < 6 cm H2O
3. Dynamic compliance > 25 mL/cm H2O
4. Oxygen cost of breathing < 15%
5. Work of breathing–poor predictor of weaning
V. Multiple Indices Predictors
A. Rapid swallow breathing (RSB):
1. RSB = respiratory frequency/tidal volume (liters) = f/Vt
2. Tachypnea and decrease in tidal volume indicates failure
3. RSB < 105 breaths/liter predicts success
4. Positive predictive value = 0.78
5. Negative predictive value = 0.95
6. Independent of patient effort
B. CROP Index:
1. Acronym for an index combining compliance, respiratory rate, oxygenation, and airway
pressure
2. >13 breaths per minute predicts weaning outcome
3. Positive predictive value = 0.71
4. Negative predictive value = 0.72
C. Weaning Index:
1. Pressure time and gas exchange index
2. <4 breaths per minute
3. Positive predictive value = 0.95
4. Negative predictive value = 0.96
VI. Weaning techniques
A. Abrupt discontinuation of ventilatory support:
1. Useful in immediate and acute weaning
2. Inexpensive
3. Simple
B. T-piece trials and continuous positive airway pressure (CPAP):
1. CPAP:
a) A mode of ventilation
b) Constant level of positive pressure during spontaneous ventilation
c) Decreases extrinsic and intrinsic respiratory work load of the patient
d) Systems–demand flow, continuous flow, and mixed systems
2. Useful in immediate, acute, and chronic weaning
3. Observe patient for two hours
a) If patient tolerates–extubate
b) If patient does not tolerate then:
110
ASCCA Residents Guide to the ICU
C.
D.
E.
F.
(1) Daily trials, gradually increase duration of T-piece trials
(2) Rest on control, assist control, or IMV
(3) If tolerates 24 hour period–extubate
4. Combine with continuous positive airway pressure
Synchronous Intermittent Mandatory Ventilation (SIMV):
1. Ventilator breath–delivered at set rate and volume or pressure
2. Patient’s inspiratory effort–mandatory breath delivered in synchrony
3. If no inspiratory effort–mandatory machine breath, determined by preset interval
4. Triggered by patient pressure or flow
5. SIMV and Weaning:
a) Initial total support is provided with SIMV
b) Decrease machine breaths (usually by 2s) and allow increase in spontaneous minute
ventilation
c) If SIMV = 2 with no respiratory distress and adequate gas exchange–extubate
Pressure-support ventilation (PSV):
1. Pressure targeted ventilation is a spontaneous ventilatory mode
2. Each breath is patient initiated, and minute ventilation determined by patient’s respiratory
drive
3. Decreases work of breathing, improving spontaneous respiration
4. No minimum minute ventilation; therefore, risk of hypoventilation
5. Modern ICU ventilators have backup minute ventilation if apnea occurs, but ventilators differ;
therefore, need to know each ventilator
6. PSV and Weaning;
a) Set pressure level of support, aim to achieve Vt = 8 mL/kg, RR <25/min
b) Decrease by 2-4 cm H2O, according to patient’s response
c) Attempt decrease at least every 12 hours
d) When PSV of 8-10 cm H2O reached, consider extubation
Combined modes
Utilizing protocol-driven techniques
VII. Weaning Trial Studies
A. PSV Weaning Better than SIMV and T-piece (Reference 5):
1. Entry criteria–patient assessed ready to wean, but has failed 2 hr trial
2. 25% of all mechanically ventilated patients entered study
3. After 21 days–significantly larger number of patients weaned by PSV
4. IMV was without PSV and T-piece was without CPAP
B. T-piece better than PSV and SIMV (Reference 6):
1. Similar entry criteria to trial quoted in reference 5
2. Once daily and multiple intermittent T-piece trials vs. PSV vs. SIMV
3. At 14 days:
a) Successful extubation rate higher with T-piece
b) Median duration of weaning shorter with T-piece
4. Higher re-intubation rate in T-piece group
C. Two hour T-piece screening better than usual respiratory care (Reference 7):
1. Prospective, randomized patients once daily respiratory assessment and T-piece trial vs.
usual care by managing physicians
2. Those with T-piece trials–shorter weaning time and duration of mechanical ventilation
3. Fewer total complications, decreased rates of reintubation, and decreased costs
VIII.Failure to Wean Checklist
A. Weaning to exhaustion–rest
B. Excessive work of breathing, auto PEEP, technical problems
C. Nutritional status–malnutrition vs. overfeeding
D. Electrolyte Balance–hypomagnesemia and hypophosphatemia
E. Acid/Base Status–metabolic alkalosis, acidosis
F. Cardiovascular status–myocardial ischemia, left ventricular failure
G. Neuromuscular disorders–acquired neuropathy, myopathy
H. Infection
111
ASCCA Residents Guide to the ICU
I.
Organ dysfunction/failure
This chapter is a revision of the original chapter authored by Barry A. Harrison, M.D. and Curtis F. Buck, CRNA, RRT
REFERENCES:
1. Tobin MJ. Principles and practice of mechanical ventilation. 2nd edition. New York: McGraw-Hill;2006.
Compressive textbook on the fundamentals of mechanical ventilation.
2. Smyrnios NA, Connolly A, Wilson MM, et al. Effects of a multifaceted, multidisciplinary, hospital-wide quality
improvement program on weaning from mechanical ventilation. Crit Care Med 2002; 30: 1224-30. The program
described in this article resulted in large reductions in the duration of mechanical ventilation, length of stay and
hospital costs despite an increase in severity of illness
3. Girard TD, Kress JP, Fuchs BD, et al. Efficacy and safety of a paired sedation and ventilator weaning protocol for
mechanical ventilated patients in intensive care (Awakening and Breathing Controlled trial): a randomized
controlled trial. Lancet 2008; 371: 126-34. The authors found that a daily spontaneous awakening trial coupled with
a spontaneous breathing trial increased the number of days breathing without assistance, reduced intensive care
unit and hospital length of stay and was associated with a lower risk of death over one year.
4. Mechanical Ventilator Manuals. When in doubt, consult the directions. Although similar ventilation modes are
available on most intensive care unit ventilators, the actual workings of the modes may be different. Thus, it is
important to know about the ventilator that you are using to wean the patient.
5. Brochard L, Ruass A, Benito S, et al. Comparison of three methods of gradual withdrawal from ventilatory support
during weaning from mechanical ventilation. Am J Respir Crit Care Med 1994;150(4):896-903.
6. Esteban A, Frutos F, Tobin MJ, et al. A comparison of four methods of weaning patients from mechanical
ventilation. N Engl J Med 1995;332(6):345-350.
7. Ely E, Baker A, Dunagan D, et al. Effect on the duration of mechanical ventilation of identifying patients capable of
breathing spontaneously. N Engl J Med 1996;335:1864-69.
QUESTIONS:
24.1.
A.
B.
C.
D.
E.
24.2.
A.
B.
C.
D.
E.
A 75-year-old man who is intubated and ventilated has resolving ARDS secondary to acute pancreatitis.
His gas exchange indices are a PaO2 of 75 mmHg on F1O2 of 0.45, a Vd/Vt of 0.50, and a shunt
fraction of 35%. A T-piece trial is administered and after two hours the patient=s minute ventilation is 9
liters/min, ABG analysis show PaO2 unchanged with a PaCO2 of 45 mmHg. The next step is to:
Extend the T-piece trial for another two hours and re-evaluate.
Place the patient on mechanical ventilation at previous settings, repeat T-piece trial in another 24 hours.
Leave the patient on the T-piece until the patient=s respiratory rate, and heart rate increases and the
tidal volume increases and the tidal volume decreases.
Extubate the patient and place on a close face mask oxygen at 100%.
Extubate the patient, and titrate the inspired oxygen to maintain an oxygen saturation oxygen of 92%.
Pressure control and inverse ratio ventilation is successful in managing a 50-year-old man with ARDS
secondary to aspiration pneumonia and a tricyclic overdose The patientКјs ventilator is switched to
pressure support ventilation at 20 cm H2O without SIMV, and CPAP is set at 10 cm H2O . In the early
hours of the morning, 10 mg of morphine sulfate is administered intravenously. The patientКјs respiratory
rate decreases to two breaths per minute. The following will occur:
If the patient is on a Siemens Servo C ventilator, the PaCO2 will increase because there is no apnea
backup; however, the low minute ventilator alarm will sound.
If the patient is on a Puritan Bennett 7200, a preset minute ventilation will take over if the apnea time
limit is exceeded.
If the patient is on a Hamilton Veolar ventilator, a present minute ventilation will take over only if apnea
develops and the pressure support is administered in the minimum mandatory minute ventilation mode.
All of the above.
None of the above.
112
ASCCA Residents Guide to the ICU
24.3.
A.
B.
C.
D.
E.
24.4.
A.
B.
C.
D.
E.
24.5.
A.
B.
C.
D.
24.6.
A.
B.
C.
D.
E.
A 65-year-old female undergoes emergency surgery for a ruptured abdominal aortic aneurysm which
requires an intraoperative 10 unit blood transfusion. Two days later the patient is hemodynamically
stable and her CXR is clear. The surgeon states the patient is ready to be weaned and asks you which
weaning technique will minimize the time to wean:
Weaning in the pressure support mode
Weaning in the SIMV mode
Weaning with T-piece trials
Weaning with abrupt disconnect and extubation
Weaning utilizing any technique, but with a protocol driven technique
A 65-year-old 70 kg man with a pre-existing left bundle branch block and COPD without carbon dioxide
retention is recovering from severe ARDS that required the use of neuromuscular relaxants. He is now
on total parenteral nutrition receiving 1500 kcals/24 hours and can lift his head off the pillow for 7
seconds. He is being weaned using intermittent T-piece trials. These trials are terminated after 15
minutes due to severe dyspnea; however, the heart rate remained unchanged. The next step is:
Measure the resting energy expenditure and calculate the respiratory quotient.
Measure the pulmonary capillary wedge pressure and the cardiac output during the weaning episode.
Measure function of the peripheral nerves and muscle function using electroneurography and
electromyogram.
Measure the airway pressure after an end expiratory hold (auto-PEEP).
Measure the base excess.
A 20-year-old 50 kg woman with an exacerbation of myasthenia gravis requires intubation and
ventilation. Following corticosteroid treatment and plasmapheresis the patient is undergoing weaning
from mechanical ventilation. Which of the following should be done?
Vital capacity 500 mL, PiMax > -60 cm H2O, PeMax 60 cm H2O – extubate
Vital capacity 200 mL, PiMax -10 cm H2O, PeMax 15 cm H2O – extubate
Vital capacity 400 mL, PiMax -25 cm H2O, PeMax 25 cm H2O with no respiratory distress after 24
hours on a T-piece–leave intubated
Vital capacity 200 mL, PiMax -10 cm H2O - check ABGs, if within normal limits–extubate
A 70-year-old man with severe chronic obstructive pulmonary disease with baseline ABG on room air of
PaO2 = 55 mmHg, PCO2 = 58 mmHg, pH = 7.42, has an infective exacerbation requiring intubation and
mechanical ventilation. All of the following are helpful with respect to weaning EXCEPT:
Treatment with aerosol bronchodilators.
Measurement of auto-PEEP.
The use of SIMV mode of ventilation to decrease the duration of weaning.
PS and T-piece weaning may be just as effective weaning tools.
A daily period of rest on increased ventilatory support to prevent respiratory muscle fatigue.
113
ASCCA Residents Guide to the ICU
25. Support of the Failing Circulation Shock
Michael Wall, M.D., F.C.C.M.
Shock is a syndrome of impaired tissue oxygenation and perfusion where oxygen demands exceed
oxygen supply. Shock can result from a decrease in oxygen delivery, decreased tissue perfusion or
impaired utilization of oxygen. This chapter will outline the important considerations for the diagnosis and
management of shock.
I.
Clinical Presentation
A. History
B. Physical exam, signs, symptoms
1. Neurologic
a) Level of consciousness
b) Paralyzed ?
2. CV
a) Heart rate
b) Pulse pressure
c) Blood pressure
d) Valvular disease ?
e) CHF?
3. Pulmonary
a) Oxygen saturation
b) Work of breathing
4. Renal
a) Oliguira/anuria
b) Acute renal failure
5. GI
a) Exam
b) Shock liver
6. Skin/extremities
a) Temperature
b) Pulses
c) Edema
C. Laboratory
1. Arterial blood gas
2. Liver function, bilirubin
3. Lactate
4. ScvO2/SvO2
5. CBC
6. Coagulation tests
7. Electrolytes, BUN, Cr, Ca++, Mg++
D. Imaging: role of:
1. CXR
2. Abdominal films
3. Chest/abdominal CT
4. Echocardiogram
5. CT angiogram (pulmonary)
II.
Types of Shock
A. Understand initial physical exam, differential diagnosis and hemodynamic profiles of
1. Hypovolemic shock
2. Distributive shock
3. Cardiogenic shock
4. Obstructive shock
III.
Physiologic Principles
A. Oxygen delivery
114
ASCCA Residents Guide to the ICU
1. DO2 = CaO2 x CO
a) CaO2 = (1.38 x Hb x SaO2) + (0.003 x PaO2)
B. Perfusion
1. Determinants of blood pressure
a) Ohm’s law (V=ri)
b) MAP = CO x SVR
2. Determinants of cardiac output
a) CO = HR (rhythm) x SV
b) Stroke volume determined by
(1) Preload (Starling’s law)
(2) Afterload
(3) Contractility
3. Perfusion pressure
a) Cerebral perfusion pressure
b) Coronary perfusion pressure
c) Renal perfusion pressure
d) Autoregulation
C. Oxygen supply and demand
1. Determinants of ScvO2 and SvO2
a) SaO2
b) VO2
c) CO
d) Hb
2. ScvO2 vs SvO2
3. Causes of increased and decreased ScvO2/SvO2
4. Pitfalls of ScvO2/SvO2 monitoring
IV. Initial Management
A. Restore perfusion rapidly while looking for and treating initial cause
B. Support airway and breathing
1. Role of intubation, mechanical ventilation and noninvasive ventilation
2. Work of breathing (WOB)
3. Goals
a) Decrease WOB
b) SaO2 > 95%
C. Restore perfusion
1. Goals
a) MAP ≥ 65 mmHg
b) Adequate cardiac output
(1) Rate, rhythm, preload, afterload, contractility
c) Adequate Hb
d) ScvO2 > 70% or SvO2 > 60%
e) Normal lactate, pH, base deficit
f) Urine output > 0.5 cc/kg/hr
D. Identify cause
V.
Specific Management
A. Hypovolemic shock
1. ATLS algorithm
2. Crystalloid
3. Colloid
4. Blood and blood products
B. Distributive shock
1. Early goal directed therapy
2. Steroid use
3. Surviving sepsis guidelines
4. Antibiotic use
5. Diagnosis and treatment of:
115
ASCCA Residents Guide to the ICU
a) Anaphylactic shock
b) Adrenal crisis, thyroid storm
c) Neurogenic shock
d) Toxic/drugs
C. Cardiogenic shock
1. Diagnosis and treatment of:
a) Acute myocardial dysfunction
(1) Ischemia/infarction
(2) Cardiomyopathies
(3) Structural defects (valve dysfunction, septal defects, etc.)
(4) Acute right heart failure
D. Obstructive shock
1. Diagnosis and treatment of:
a) Tamponade
(1) Post cardiac surgical tamponade vs traditional tamponade
b) Restrictive myocarditis and pericarditis
c) Obstructive cardiomyopathies
d) Pneumothorax
VI.
Monitoring
A. Indications, contraindications, placement, maintenance and complications of:
1. Arterial lines
2. Central lines
3. Pulmonary arterial catheters
B. Role of echocardiography in the diagnosis of shock
1. TEE vs TTE
C. Understand the following monitoring devices and laboratory tests:
1. Noninvasive blood pressure
2. ECG
3. Noninvasive CO monitors
a) Li++ dilution
b) Esophageal doppler
c) Pulse contour analysis
4. Pulse pressure variation
5. Tonometry
6. Tissue oxygen tension
7. Lactate
8. Base deficit
9. Near infrared spectroscopy
VII. Pharmacologic Therapy in Shock
A. Antiobiotics
B. Steroids
C. Vasodilators
1. Nitroglycerin
2. Nitroprusside
3. Nesiritide
4. Nicardipine
5. Nitric Oxide
6. Epoprostenol (Flolan)
D. Vasoconstrictors
1. Phenylephrine
2. Norepinephrine
3. Vasopressin
E. Inotropes
1. Epinephrine
2. Dopamine
F. Inodilators
116
ASCCA Residents Guide to the ICU
1. Dobutamine
2. Milrinone/inamrinone
G. Miscellaneous
1. Digoxin
VIII. Mechanical Support Options
A. Indications, contraindications, complications for:
1. Intraaorta balloon pump
2. Right/left ventricular assist devices
3. Extracorporeal membrane oxygenators (ECMO)
This chapter is a revision of the original chapter authored by Robin J. Hamill-Ruth, M.D.
REFERENCES
1. American College of Surgeons Committee on Trauma. Advanced Trauma Life Support for Doctors (ATLS). 7th Ed.
Chicago, IL: American College of Surgeons; 2004
2. American Thoracic Society. Evidence-based colloid use in the critically ill: American Thoracic Society consensus
statement. Am J Respir Crit Care Med 2004;170:1247
3. Annane D, Sebille V, Charpentier C, et al. Effect of treatment with low doses of hydrocortisone and
fludrocortisones on mortality in patients with septic shock. JAMA 2002;288:862
4. Beca J, Willcox T, Hall RMO: Mechanical Cardiac Support. Sidebotham et al (eds.). Cardiothoracic Critical Care.
Butterworth Heineman Elsevier, Philadelphia. 2007; p.342-365
5. Hollenberg SM: Vasopressor Support in Septic Shock [Review]. Chest. 132(5):1678-1687
6. Marx G, Reinhart K. Venous Oximetry. Curr Opin Crit Care 2006;12:263
7. McLean B, Zimmerman JL (eds). Diagnosis and Management of Shock. Fundamental Critical Care Support, 4th
Edition, 2007; p.7-1 and 11-20
8. Rivers E, Nguyen B, Havstad S, et al. Early goal-directed therapy in the treatment of severe sepsis and septic shock.
N Engl J Med 2001;345:1368
9. Sprung CL, Annane D, et al: Hydrocortisone therapy for patients with septic shock. N Engl J Med 2008;358:111-124
WEB-SITES:
10.American College of Cardiology. http://www.acc.org. Guidelines for management of acute coronary syndromes
and related cardiovascular topics
11.American Heart Association. http://www.americanheart.org. Guidelines for management of acute coronary
syndromes and related cardiovascular topics
12.Eastern Association for the Surgery of Trauma. http://www.east.org/tpg.html. Best site for evidence-based
trauma care guidelines
13.Surviving Sepsis Campaign. www.survivingsepsis.org. This Web site contains guidelines for the management of
severe sepsis and septic shock, information on the Surviving Sepsis Campaign, and sepsis-related information and
resources for healthcare professionals, patients, and the general public
14.Trauma.org. http://www.trauma.org. Image bank and links
QUESTIONS
25.1.
A.
B.
C.
D.
E.
A PA catheter is placed in a critically ill trauma patient in the supine position with good wave
tracing and PA occlusion at 52 cm. The patient is then placed head up because of a high
intracranial pressure. The pulmonary artery tracing is noted to be damped several minutes
later. What is the most likely cause for the damped wave form?
PA catheter tip has moved to West lung zone 1 from zone 3
Tricuspid regurgitation
Congestive heart failure
Transducer zeroed at the hip
Increased ICP
117
ASCCA Residents Guide to the ICU
25.2.
A.
B.
C.
D.
E.
25.3.
A.
B.
C.
D.
E.
25.4.
A.
B.
C.
D.
E.
You are called to evaluate a patient just admitted from the emergency room after a “head on”
MVC. A CVP catheter was placed in the ER. The patient currently has a CVP of 24.
Auscultation of the chest reveals bilateral breath sounds without rales. What is the most likely
diagnosis?
Chordae tendinae rupture
Tension pneumothorax
Anterior myocardial infarction
Myocardial contusion
Aortic transection
A 70kg 65 year-old intubated female is admitted to the ICU with hypotension, altered mental
status, hypoxemia, and anuria. Vitals: BP = 60/30, HR = 145, Temp = 35.5В° C. No history is
available. ECG = tachycardia with lateral ST depression. A PA catheter placed by the ER
resident reveals CVP = 1 mmHg, PAOP = 7 mmHg, CI = 1.8 L/min/m2 with SVRI = 3200
dyne/sec/cm-5. Arterial blood gas results show pH = 7.15, PaCO2 = 40, PaO2 = 89, HCO3 =
14.
Rapid infusion of 1000 cc normal saline
Start a dobutamine infusion
Start a milrinone infusion
Increase the mechanical ventilation to normalize the pH
Start inhaled nitric oxide
A 45 year-old diabetic make is admitted to the ICU with hypotension, nausea, diaphoresis,
severe dyspnea and hypoxemia of 4 hours duration. His PMH is significant for 70 pack-year
history of cigarette abuse. After receiving 2 liters of crystalloid, his BP = 82/30, HR = 130, RR
= 36, arterial O2 saturation = 88% on 100% face tent. ECG shows sinus tachycardia with a
left bundle branch block. CVP = 18, PAOP = 24, CI = 1.3 L/min. Which of the following
therapies is most appropriate?
Metoprolol 5g IV
1 liter NS bolus
Start nitroglycerine infusion
Start phenylephirine infusion
Start dobutamine infusion
25.5.
A.
B.
C.
D.
E.
Which of the following should decrease the mixed venous O2 saturation?
Fluid resuscitated, early sepsis
End-stage liver disease
Malignant neuroleptic syndrome
Dobutamine
Neuromuscular blockade
25.6.
A.
B.
C.
D.
E.
The greatest increase in oxygen delivery will be accomplished by:
Increasing the PaO2 from 79 mmHg to 250 mmHg
Adding dobutamine to increase to cardiac output from 4.5 to 6.0 L/min
Transfusing to increase hemoglobin from 6.5 to 11.0 gm/dL
Increasing the PAOP from 18 to 22 mmHg
Increasing the patients core temp from 33.5 to 35.2 degrees
118
ASCCA Residents Guide to the ICU
26. Diagnosis and Treatment of Dysrhythmias
Michael Wall, M.D., F.C.C.M.
Dysrhythmias are extremely common in critically ill patients. An understanding of the pathophysiology and
treatment of common dysrhythmias is essential for the ICU clinician.
I.
BLS and ACLS Algorithms
II. Bradycardia and Sinus Node Dysfunction
A. Pathophysiology, diagnosis and treatment of:
1. Bradycardia
a) Non-pathologic
(1) Sinus bradycardia
(2) Nocturnal bradycardia
(3) Sinus arrhythmia
(4) Wandering pacemaker
b) Pathologic conditions
(1) Multiple etiologies, for example; increased ICP, sepsis, following MI, and heart
transplantation, drugs, etc.
2. Sinus node dysfunction
a) Persistent sinus bradycardia
b) Sinus pause or arrest
c) Sinus atrial exit block
d) Bradycardia/tachycardia syndrome
III. Vagally Medicated Bradycardias
A. Pathophysiology, diagnosis and treatment of:
1. Sinus arrest
2. Bradycardia
3. Heart block
IV. Conduction Disturbances
A. Pathophysiology, diagnosis and treatment of:
1. Left bundle branch block (LBBB)
2. Right bundle branch block (RBBB)
3. Bifascicular block
4. First-degree AV block
a) Normal QRS
b) Wide QRS
5. Second-degree AV block
a) Type I (Wenckebach)
b) Type II
c) Third-degree AV block
(1) Acquired
(2) Congenital
V. Premature Beats
A. Pathophysiology, diagnosis and treatment of:
1. Atrial
2. Junctional
3. Ventricular
VI. Supraventricular Tachycardia
A. Pathophysiology, diagnosis and treatment of:
1. Paroxysmal Supraventricular Tachycardia
a) AV nodal reentry tachycardia (AVNRT)
b) AV reentry tachycardia (AVRT)
119
ASCCA Residents Guide to the ICU
2.
3.
4.
5.
6.
7.
8.
9.
c) Intraatrial reentry
d) Automatic atrial tachycardia
e) Sinus nodal reentry tachycardia (SNRT)
Wolff-Parkinson-White and variants
Nonparoxysmal AV junctional tachycardia
Paroxysmal atrial tachycardia with block
Automatic AV junctional tachycardia
a) AKA junctional ectopic tachycardia
Multifocal atrial tachycardia
Sinus tachycardia
Atrial flutter
Atrial fibrillation
a) Prophylaxis for cardiac surgery
VII. Ventricular Arrythmias
A. Pathophysiology, diagnosis and treatment of:
1. Premature ventricular contractions
2. Nonsustained ventricular tachycardia
3. Sustained monomorphic ventricular tachycardia
4. Sustained polymorphic VT
5. Ventricular fibrillation
6. Idiopathic ventricular tachycardias
a) RV outflow tract tachycardia
b) Fascicular ventricular tachycardia
(1) AKA Verapamil - sensitive ventricular tachycardia, Belhassen’s ventricular
tachycardia
c) Arrhythmogenic right ventricular dysplasia/cardiomyopathy (AVRD)
d) Congenital long QT syndrome
e) Acquired QT prolongation
f) Brugada syndrome
g) Catecholaminergic polymorphic VT
h) Short coupled torsade de pointes
i) Short QT syndrome
VIII. Radiofrequency Ablation
A. Atrial fibrillation
B. Atrial flutter
C. AVNRT
D. Accessory pathways (WPW)
E. Ectopic atrial tachycardia
F. AVRD
IX. Pacemakers and Implantable Cardiac Defibrillators (ICD)
A. Pacemakers
1. 5 letter code
2. Temporary
a) Transvenous
b) Transcutaneous
c) Transesophageal
d) Epicardial
3. Permanent
4. Indications
5. Contraindications
6. Electromagnetic interference
7. Effect of magnet application
B. ICD
1. Indications
2. Electromagnetic interference
120
ASCCA Residents Guide to the ICU
3. Effect of magnet application
C. Biventricular pacing
1. Indications
This chapter is a revision of the original chapter authored by Stuart M. Lowson, M.B., B.S.
REFERENCES
1. 2005 American Heart Association Guidelines for Cardiopulmonary Resuscitation and Emergency Cardiovascular
Care. Electrical Therapies: Automated external defibrillators, defibrillation, cardioversion, and pacing. Circulation
2005; 112(suppl IV):IV-35-46
2. Aasbo JD, Lawrence AT, Krishnan K, et al: Amiodarone prophylaxis reduces major cardiovascular morbidity and
length of stay after cardiac surgery: A meta-analysis. Ann Intern Med 2005; 143:327-336
3. American Heart Association. Advanced Cardiovascular Life Support: Provider Manual. Dallas, TX: American
Heart Association; 2006
4. Connolly SJ, Hallstrom AP, Cappato R, et al: Meta-analysis of the implantable cardioverter defibrillator secondary
prevention trials. AVID, CASH and CIDS studies. Antiarrhythmics vs Implantable Defibrillator study. Cardiac
Arrest Study Hamburg. Canadian Implantable Defibrillator Study. Eur Heart J 2000; 21:2071-2078
5. Fuster V, Ryden LE, Cannom DS, et al: ACC/AHA/ESC 2006 guidelines for the management of patients with atrial
fibrillation: Full text: A report of the American College of Cardiology/American Heart Association Task Force on
Practice Guidelines and the European Society of Cardiology Committee for Practice Guidelines (writing committee
to revise the 2001 guidelines for the management of patients with atrial fibrillation) developed in collaboration
with the European Heart Rhythm Association and the Heart Rhythm Society. Europace 2006; 8:651-745
6. Gessman LJ, Trohman R: Cardiac Arrhythmias. Parrillo JE, Dellinger RP (eds.) Critical Care Cardiovascular
Disease, 3rd Edition. Butterworth Heineman Elsevier, Philadelphia. 2008; p.647-675
7. Mitchell LB, Exner DV, Wyse DG, et al: Prophylactic Oral Amiodarone for the Prevention of Arrhythmias that Begin
Early After Revascularization, Valve Replacement, or Repair: PAPABEAR: A randomized controlled trial. JAMA
2005; 294:3093-3100
8. Smith W, Hood M: Arrhythmias. Sidebotham D, et al (eds): Cardiothoracic Critical Care. Butterworth Heineman
Elsevier, Philadelphia. 2007; p316-341
9. The American Heart Association in Collaboration with the International Liaison Committee on Resuscitation
(ILCOR): 2005 American Heart Association Guidelines for Cardiopulmonary Resuscitation and Emergency
Cardiovascular Care. Circulation 2005; 112:IV58-IV66
WEB-SITES:
10.American College of Cardiology. http://www.acc.org. Guidelines for management of acute coronary syndromes
and related cardiovascular topics
11.American Heart Association. http:www.americanheart.org. Guidelines for management of acute coronary
syndromes and related cardiovascular topics
Phone Numbers
Boston Scientific: CRM Division (formerly Guidant)
Medtronic
St. Jude
1-800-cardiac
1-800-medtron
1-800-paceicd
QUESTIONS
26.1.
A.
B.
C.
D.
E.
A 30 year-old female with a history of Wolff-Parkinson-White Syndrome goes into atrial
fibrillation with a wide QRS in the PACU. Her HR is 150, BP 92/63. SaO2 100%.
Spontaneously breathing 2L nc. Which of the following is the most appropriate initial therapy?
Verapamil 5mg IV
Metoprolol 5mg IV
Digoxin 0.5 mg IV
Cardioversion
Amiodarone 150mg IV
121
ASCCA Residents Guide to the ICU
26.2.
A.
B.
C.
D.
E.
A VOO pacemaker
Paces the ventricle
Paces both the atria and ventricle
Senses the ventricle
Senses both the atria and the ventricle
Is inhibited by both the atria and ventricule
26.3.
A 75 year-old male acutely goes into atrial fibrillation 2 days following CABG. His ejection
fraction was 25% on postoperative TEE. His HR is 150, BP 88/62, SaO2 90% on 2L nc. He is
confused. The most appropriate initial therapy is:
Verapamil 5mg IV
Metoprolol 5 mg IV
Digoxin 0.5 mg IV
Cardioversion
Amiodarone 150 mg IV
A.
B.
C.
D.
E.
26.4.
A.
B.
C.
D.
E.
26.5.
A.
B.
C.
D.
E.
26.6.
A.
B.
C.
D.
26.7.
A.
B.
C.
D.
E.
A 50 year-old male with COPD and intubated for severe pneumonia has 3 blocked P waves in
a row immediately following endotracheal suctioning. This spontaneously resolves. What is
the most appropriate therapy?
Tilt table testing
Dopamine infusion
Transvenous pacing
Permanent pacemaker placement
Observation
A 70 year-old male patient is in the ICU for alcohol withdrawal and is being sedated with
haloperidol. He develops hemodynamically stable torsade de pointes. Which of the following
is the most appropriate initial therapy?
Magnesium
Procainamide
Ibutilide
Sotalol
Cardioversion
A 70 year-old patient is intubated and undergoing CPR with chest compressions for VF. What
is the most appropriate way to ventilate this patient?
2 breaths following every 30 compressions
4 breaths following every 30 compressions
20 breaths per minute continuously
8-10 breaths per minute continuously
Which of the following pacemaker modes would have the least interference from
electrocautery?
VOO
DDD
AAI
VVI
DVI
122
ASCCA Residents Guide to the ICU
27. Diagnosis and Treatment of Myocardial Ischemia
Ruben J. Azocar, M.D.
A 66-year-old male presents for elective shoulder surgery. Nine months before he suffered an acute
myocardial infarction in his left anterior descending coronary artery and underwent emergent cardiac
catheterization and placement of 3 drug eluting stents. He stopped his clopidogrel and aspirin 8 days prior
to his current surgery as instructed. A recent dobutamine echocardiogram demonstrated no evidence of
ischemia and he has excellent exercise tolerance.
Coronary artery disease (CAD) remains one of the leading causes of death in the industrial world and it is
the most common cause of death in the US. Clinically, a continuum ranges from a temporary imbalance
between coronary oxygen consumption and delivery representing myocardial ischemia (that may not
result in myocardial damage) to myocardial infarction and death. As anesthesiologist and intensivists we
might encounter patients arriving to the emergency department with myocardial ischemia/myocardial
infarction but also with patients developing such changes in the perioperative period. As there are some
differences between the two clinical presentations, notes describing pertinent issues in regards to the
perioperative ischemia/infarction are included in the chapter.
Classification
There are many classifications based on the extent of the infarction. In a transmural MI the full thickness
of myocardium, from endocardium to epicardium, is involved. In a nontransmural MI the damage to the
myocardium is limited to the endocardium. The endocardial /subendocardial zones are poorly perfused
compared to rest of heart making them more vulnerable to ischemia. Clinically the use of the terms Q
Wave MI (indicating the presence of Q waves on ECG) or non Q Wave MI are used as descriptors. A
differentiation between the presence of ST segment elevation on ECG (STEMI) or no ST segment
elevation on ECG (NSTEMI) has been also used. Although it does not indicate the degree of myocardial
involvement, STEMI patients have higher early morbidity and mortality. However this classification does
not predict long term sequelae. Finally, emerging data suggest that ischemia/infarction occurring in the
perioperative period (PMI) may differ from non-perioperative injury.
I.
Factors that determine myocardial oxygen delivery/supply
A. Heart rate
1. When the HR increases, it is the diastolic time which decreases. Significant coronary blood
flow occurs during diastole, particularl;y to the left heart
B. Coronary perfusion pressure
1. The aortic diastolic BP is the primary determinant of coronary perfusion pressure
2. Ventricular end diastolic pressure (EDP) (the higher the EDP the larger the compromise to
the subendocardial circulation)
C. Arterial oxygen content
1. Primary determinants:
a) Arterial oxygen saturation
b) Hemoglobin concentration
D. Coronary vessel diameter
1. Thrombus superimposed on ulcerated or unstable atherosclerotic plaque; most common
cause of MI
2. High Grade (>75%) stenosis with coronary vasospasm
II. Factors that determine myocardial oxygen demand/consumption
A. Basal requirements
1. Increased metabolic demands such as physical exertion, severe HTN or severe aortic
stenosis elevates the demand
B. Heart rate
1. Increases in HR increase oxygen demand
C. Wall tension
1. Increases in wall tension increase oxygen demand
123
ASCCA Residents Guide to the ICU
D. Contractility
1. Increases in contractility increase oxygen demand
III. Pathophysiology of Myocardial ischemia/infarction
A. Occlusion of Coronary Circulation
1. Disruption of Vascular Endothelium with Unstable Plaque that Stimulates Intracoronary
Thrombus.
2. 20-40 min to irreversible cell damage.
3. Most common cause on the patient coming to ED
4. Plaque forms over many years to decades
a) Fibromuscular Cap & Underlying Lipid-Rich Core
b) Shoulder Region – Cap meets Vessel Wall
c) Platelet-Mediated Thrombus formation on any disruption of vessel wall
B. “Demand ischemia”
1. Imbalance between myocardial oxygen consumption and delivery. Appears to be the most
likely cause in the early Perioperative period
IV. Extent of Myocardial Damage
A. Level of Occlusion in Coronary Artery
1. More proximal thrombus, more myocardium at risk
B. Length of Time
C. Collateral Circulation
V. PMI Mechanisms
A. Risk of PMI peaks within first 3 postoperative days (Days 0/1 highest)
1. Mobilization of fluids, resulting in an increase on preload
2. Pronounced thrombotic risk by activation of coagulation cascade during surgery
3. Increases in HR/BP associated with catecholamines/ postoperative pain worsens myocardial
supply/demand mismatch
4. Recent data suggest ischemia starts at end of surgery and emergence from anesthesia.
5. It might possible that most “early” PMI are related to demand ischemia and as time passes to
plaque rupture and thrombosis similar to the general population
VI. Risk Factors for Arteriosclerosis/MI
A. Hyperlipidemia
1. Major Component of Plaque
2. High LDL levels associated with higher MI rate
B. Diabetes Mellitus
C. Hypertension
D. Smoking
E. Family History
F. Male Gender
G. Age
VII. Risk factors for PMI
The ACA/AHA guidelines provide a blueprint for risk stratification, determining need for further
interventions and management of patients with coronary stents. Basically establishes a relationship
between patient risk factors and exercise tolerance and surgery risk factors to determine the need for
further interventions prior to surgery and anesthesia.
A. Patient factors:
1. Active cardiac conditions characterized by
a) Unstable coronary syndromes, characterized by unstable or severe angina, or recent MI
(less than one month)
b) Decompensated heart failure
c) Significant arrhythmias
d) Severe valvular disease
B. Clinical risk factors include
1. History of heart disease
124
ASCCA Residents Guide to the ICU
2.
3.
4.
5.
6.
History of compensated or prior heart failure,
History of cerebrovascular disease,
Diabetes mellitus
Renal insufficiency
Note that other risk factors such age (>70), abnormal ECG , uncontrolled HTN are not listed
as risk factors anymore as there is no evidence that they increase risk of PMI independently.
C. Surgery factors
1. Vascular surgery (High risk)
2. Intermediate risk :Intraperitoneal, intrathoracic, CEA, head and neck, orthopedic and prostate
surgery
3. Low risk Endoscopic, ambulatory, cataract ,breast and superficial surgery
D. Recent Stent Placement
1. Must know type of stent (bare metal vs drug eluting, length of time since insertion and status
of anticoagulation
2. Follow recent ASA practice alert
VIII.Signs & Symptoms
A. Chest pain described as a pressure sensation, fullness, or squeezing in the midportion of the
thorax
B. Radiation of chest pain into the jaw/teeth, shoulder, arm, and/or back
C. Dyspnea or shortness of breath
D. Epigastric discomfort with or without nausea and vomiting
E. Diaphoresis or sweating
F. Syncope or near-syncope without other cause
G. mpairment of cognitive function without other cause
H. More common in early morning or around time of physical activity
I. Some MI’s are without symptoms (Higher with DM)
J. PMI might be asymptomatic (silent)
IX. Diagnosis
A. Electrocardiography
1. ST segment elevation
2. Nonspecific ST and/or T wave changes
B. Blood Tests
1. Release of specific enzymes and cell wall proteins of myocardial cells that become ischemic
or infarted
2. CK-MB, troponin
3. Change of serum levels indicate amount of heart muscle affected
C. Echocardiography
1. Identifies heart region/ coronary anatomy that is affected by MI
2. May be old or new wall motion changes
3. Usefulness for diagnosis is limited
D. Concerns regarding diagnosis of PMI
1. Pain may be masked by analgesics
2. Telemetry misses significant ST changes
3. 12 lead ECG only diagnostic 50% time
4. V2-V4 with majority of 12 lead ECG changes
5. Usually NSTEMI in nature
6. Serum biomarkers
a) CK-MB, less sensitive/specific in PMI
b) Troponin T & I, markers of choice; Can be elevated in CHF, PE, Sepsis.
c) Diagnosis for acute MI requires change from baseline.
X. Therapy
A. Goals:
1. Restoration of Coronary Blood Flow
2. Limit myocardial damage
B. Antiplatelet Agents
125
ASCCA Residents Guide to the ICU
1. Aspirin (ASA)
a) Immediate therapy 160-325 mg upon signs or symptoms of MI; interferes with platelet
adhesion and cohesion at disruption site
b) Greatly reduces AMI Mortality
c) Long term use once MI diagnosed
d) Clopidogrel (Plavix) & Ticlopidine (Ticlid): Not superior to ASA; Used in ASA allergy
2. Oxygen
a) Maximize oxygen carrying capacity of RBCs
b) No published studies that demonstrates improved morbidity/mortality with supplemental
oxygen
3. Nitrates
a) Nitrates are given in the setting of MI with: CHF, persistent ischemia, long term Use,
HTN and Large Anterior Wall MI
b) Reduce preload and afterload
c) Decreases myocardial oxygen requirements
d) Effective in early stages (48hrs) of MI but no long term mortality advantages
4. Beta Blockers
a) Recommended in MI early and often!
b) Reduces Mortality in Acute MI setting
(1) 28% reduction mortality if used in first week post MI
(2) Indefinite Use
c) Decrease oxygen demand by lowering heart rate and force of
contraction and
increases supply by increasing diastolic time
d) Antiarrhythogenic, Decreases VF threshold
e) Antagonizes adrenergic effects of catecholamines
5. Unfractionated Heparin
a) Inhibits additional formation and propagation of thrombus
6. Glycoprotein 2b/3a Antagonists
a) Inhibit Platelet Aggregation by antagonizing receptors on platelets (glycoprotein 2b/3a)
bind fibrinogen
b) Use during PCI reduces mortality, reinfarction, and need for further revascularization
procedures
c) Abciximab, Eptifibatide, & Tirofiban
7. ACE Inhibitors
a) Recommended in MI patients within first 24 hours
b) Reduces Afterload via Vasodilatation
c) Continued if MI patients have:
(1) CHF
(2) LVEF < 40%
(3) HTN
(4) DM
d) Relative contraindication if hypotensive or with declining renal function
8. Lipid Management
a) New evidence that all MI patients should be on statin therapy regardless of HDL/LDL
levels(Schwartz et al. Am J Cardiol 2005)
b) Statins can reduce circulating markers of inflammation within days post MI
c) Improve coronary endothelium function
d) Reversal of prothrombotic states
e) Reduces atherosclerotic plaque volume
f) In a review of 6 randomized, controlled trials high intensity statin therapy (atorvastatin
80mg) reduced early recurrent ischemic events compared to moderate therapy (40mg) or
placebo
9. Fibrinolytics
a) Indicated for MI and ST elevation > 0.1mV in 2 contiguous leads or new bundle block
b) Restore coronary blood flow in 50-80% of cases
c) Best when door to needle time is less than 30 minutes
d) Runs risk of significant bleeding in PMI patients
10. Percutaneous Coronary Intervention (PCI)
126
ASCCA Residents Guide to the ICU
a)
b)
c)
d)
e)
f)
“Door to balloon “ goal of less than 90 minutes
Restores coronary flow 90-95% of MI patients
Better than fibrinolysis in short term mortality, bleeding rates, and reinfarction rates
Better if pt in cardiogenic shock
Stents reduce need for subsequent target-vessel revascularization
Postoperative MI in Noncardiac Surgery - PCI preferable to Fibrinolysis in immediate
postoperative phase due to lower risk major bleeding
g) Berger et al. with 48 postop pts for PCI
(1) 65% survival rate much better than those untreated
(2) No significant surgical site bleeding in cath lab
(3) Pts with sudden onset ST elevation from acute thrombotic occlusion did best with
immediate intervention (PCI vs CABG) compared to no intervention in postop pt.
11. Surgical Revascularization (CABG)
a) Urgent if failed PCI and unstable pt
b) Must have anatomy amenable for grafting
c) If Mechanical Complications of MI present
(1) Ventricular Septal Defect
(2) Free Wall Rupture
(3) Acute Mitral Regurgitation
d) Emergency procedure riskier than elective
(1) 3-7 days post MI similar risks to elective
e) Elective CABG improves survival in pts with:
(1) Left Main Coronary Disease
(2) 3 Vessel Disease
(3) 2 Vessel Disease not amenable to PCI
12. Current controversy exists over use of PCI vs CABG. See Reference 8 for more details.
This chapter is a revision of the original chapter authored by David T. Porembka, D.O.
Bibliography
1. “Acute Myocardial Infarction” by Christopher T. Bajzer, MD Cleveland Clinic Disease Management Project. May,
2002.
2. Fleisher L,. Beckman J, Brown K,:ACC/AHA 2007 Guidelines on Perioperative Cardiovascular Evaluation and
Care for Noncardiac Surgery: Executive Summary: A Report of the American College of Cardiology/American
Heart Association Task Force on Practice Guidelines Anesth. Analg. 2008 106: 685-712
3. A Report by the American Society of Anesthesiologists Committee on Standards and Practice Parameters. Practice
Alert for the Perioperative Management of Patients with Coronary Artery Stents: Anesthesiology, January 2009 ;
110 : 22-23
4. Adesanya, AO et al. Management of Perioperative Myocardial Infarction in Noncardiac Surgical Patients. Chest
2006;130:2.
5. Schwartz, GG et al. The Case for Intensive Statin Therapy After Acute Coronary Syndromes. Amer J Cardiol
2005;96 [suppl]:45F-53F.
6. Kopecky, SL Effect of Beta Blockers, particularly Carvedilol, on Reducing the Risk of Events After Acute
Myocardial Infarction. Amer J of Card 2006; 98:1115-1119.
7. Berger et al. An Immediate Invasive Strategy for the Treatment of AMI early after Noncardiac Surgery. Am J
Cardiol 2001; 87:1100-1102.
8. Serrruys PW, Morice MC, Kappetein AP, et al. Percutaneous Coronary Intervention versus Coronary Artery Bypass
Grafting for Severe Coronary Artery Disease. New Engl J Med 2009: 360(10):961-972.
Questions
27.1.
A.
B.
C.
D.
Perioperative myocardial infarction can be characterize by the following EXCEPT:
It is usually silent
It is usually NSTEMI
It is often transmural
Enzymes values might be difficult to interpret
127
ASCCA Residents Guide to the ICU
27.2.
A.
B.
C.
D.
27.3.
A.
B.
C.
D.
27.4.
A.
B.
C.
D.
27.5.
A.
B.
C.
D.
27.6.
1.
2.
3.
4.
Provision of ACE inhibitors is recommended after acute MI in the following situations
EXCEPT
To decrease afterload
In patients developing renal failure
Patients with low LVEF
Patient with HTN
In relation to пЃў-Blockers the following statements are true, except:
They should be stopped after the acute phase of ischemia is resolved
Decrease incidence of VF
Decrease myocardial oxygen consumption
Improve oxygen delivery
The goals of managing the patient with coronary ischemia include the following EXCEPT:
Restore coronary blood flow
Minimize extension of the infarct
Decrease oxygen delivery
Decrease oxygen consumption
In managing patient with coronary stents the following statement are true EXCEPT:
It is important to know the type of stent
The length of time since stent placement is an important piece of information
Once the patient has a stent his risk a new coronary event is close to zero
Management of anticoagulation is of paramount importance to avoid complications
Patient Risk factors for PMI include
Uncontrolled hypertension
Age>65
Presence of LBBB on EKG
Renal failure
128
ASCCA Residents Guide to the ICU
28. Valvular Heart Disease
Frank Rosemeier, M.D. and Miguel A. Cobas, M.D.
A 64-year-old female with acute coronary syndrome develops pulmonary edema in the intensive care unit
and becomes acutely dyspneic and hypoxemic. A new soft pansystolic murmur is audible at the apex.
Despite emergent intubation, ventilation and inotropic support, her cardiorespiratory status continues to
deteriorate. A bedside transthoracic echocardiogram reveals severe mitral regurgitation and a left
ventricular ejection fraction of 40%. Hemodynamic stabilization is achieved with an intraaortic balloon
pump (IABP). Following coronary angiography, she undergoes emergent coronary bypass surgery with
mitral valve repair.
Valvular heart disease in the critically ill patient typically presents either as acute valvular dysfunction with
acute heart failure or as chronic, but decompensated valvular disease secondary to increased metabolic
demands. Valvular regurgitant lesions are far more common than stenosis in the acute setting. However,
patients with previously asymptomatic or compensated stenotic lesions may acutely deteriorate triggered
by entities such as infection, sepsis, acute hemorrhage, trauma, anemia, or pregnancy.
The following outline aims to provide a basic framework of the causes, diagnostic approaches and
therapeutic interventions of emergent valvular disorder frequently encountered in the critically ill patient. It
will first provide a general overview and then discuss selected valvular dysfunction specifically.
I.
Causes of Acute Valvular Dysfunction in the Critical Ill Patient
A. Mitral regurgitation
1. Acute myocardial infarction
a) Papillary muscle rupture
b) Regional wall motion abnormality
c) Left ventricular dilation and systolic dysfunction
2. Myxomatous disease with flail leaflet (mitral valve prolapse)
3. Spontaneous chordal rupture
4. Endocarditis
B. Mitral Stenosis
1. Rheumatic heart disease, especially in younger women
2. Exacerbation by pregnancy, trauma, shock
C. Aortic Regurgitation
1. Endocarditis
2. Aortic dissection
3. Spontaneous rupture of a congenital fenestration
D. Aortic Stenosis
1. Calcification or bicuspid aortic valve in the elderly
2. Exacerbation by trauma, shock, infection, anemia
E. Tricuspid Regurgitation
1. Endocarditis
2. Penetrating chest trauma
3. Blunt chest wall trauma
F. Prosthetic Valves
1. Endocarditis
2. Valve thrombosis
3. Paravalvular dehiscence
4. Leaflet tear
II. Diagnosis of Acute Valvular Dysfunction
A. Physical Examination
1. Variable accuracy
2. High index of suspicion: Consider valve dysfunction in all patients with pulmonary edema
129
ASCCA Residents Guide to the ICU
and/or cardiogenic shock
B. Transthoracic echocardiography
1. Accurate diagnosis of disease
2. Quantification of severity of stenosis or regurgitation
3. Estimation of ejection fraction (EF)
4. Estimation of pulmonary artery pressures
C. Transesophageal Echocardiography
1. Sensitive for detection of valvular vegetations and paravalvular leaks
2. Detection of paravalvular abscess
3. Essential for prosthetic mitral valve dysfunction
4. Useful for prosthetic aortic valve dysfunction
D. Right-Sided Heart Catheterization
1. Not reliable for diagnosis of valve disease
2. May be helpful for optimizing loading conditions
E. Chest computer tomography
1. Sensitive and specific for diagnosis of aortic dissection
F. Coronary Angiography
1. Identifies coronary artery disease
III. Therapeutic Interventions
A. Accurate diagnosis with echocardiography
1. Differentiate acute valve dysfunction from acute decompensation in chronic valve disease
B. Treat the underlying disease process associated with decompensation (i.e. endocarditis, acute
myocardial infarction, and anemia)
C. Optimize pre- and afterload conditions using diuretics, vasodilators, and other agents guided by
hemodynamic monitoring
D. Consult the cardiac surgery team as soon as the diagnosis is made
E. Use intra-aortic balloon pump for acute mitral regurgitation
F. Consider surgical or percutaneous intervention for acute valve dysfunction
IV. Mitral Regurgitation (MR)
A. Etiology
1. Distortion or disease of any one component of the mitral valve apparatus (mitral annulus,
leaflets, chordae, papillary muscles, and shape and function of the left ventricular walls)
2. Increased volume load on the left ventricle results in left ventricular (LV) dilation and
eventually LV failure
3. In decompensated heart failure, the geometry of the mitral valve apparatus is further
compromised by worsening MR and LV dilation
B. Clinical presentation
1. Chronic MR is typically well tolerated, even in the presence of systemic infection, pregnancy
or acute hemorrhage
2. Acute MR presents as pulmonary edema
3. Chordal rupture in patients with mitral valve prolapse
4. Superimposed symptoms and signs of endocarditis
5. Acute ischemic papillary muscle dysfunction or subacute papillary muscle rupture several
days after myocardial infarction
C. Diagnosis
1. High level of clinical suspicion
2. Holosystolic murmur at the apex radiating to the axilla
a) Caveat: Loudness of murmur may not correlate with severity. Noisy environment and
transmitted sounds from pulmonary edema makes auscultation challenging
3. Echocardiography
a) Determines etiology by 2D echo and severity by area and width of the regurgitant jet on
color flow Doppler
4. Blood cultures to exclude endocarditis
5. Prominent v wave on pulmonary artery catheter occlusion pressure tracing (wedge), but may
be absent
D. Management
130
ASCCA Residents Guide to the ICU
1.
2.
3.
4.
For chronic, but decompensated MR
a) Treat the underlying cause such as fluid overload, anemia, infection
b) Optimize preload and afterload with diuretics, and vasodilators
For acute MR
a) Supportive with careful management of pre- and afterload conditions
b) Afterload reduction with IABP (which also improves diastolic flow)
Coronary revascularization for:
a) Regional wall motion abnormalities causing mitral valve apparatus dysfunction
Surgical considerations:
a) Mitral valve annuloplasty for annular dilation and regurgitation
b) Mitral valve repair for leaflet prolapse
c) Replacement, reimplantation, elongation or shortening of the chordae
d) Valve resection and replacement for endocarditis with heart failure to avoid further
structural damage and paravalvular abscess formation
e) Partial or complete papillary muscle rupture
V. Aortic Regurgitation (AR or AI)
A. Etiology
1. Congenital bicuspid valve
2. Rheumatic valve disease
3. Aortic root dilation
a) Hypertension
b) Cystic medial necrosis
c) Marfan’s syndrome
d) Bicuspid aortic valve
B. Clinical Presentation
1. Pulmonary edema from acute volume overload of the left ventricle
2. Cardiogenic shock from decreased forward flow and decreased coronary perfusion
C. Diagnosis
1. High index of suspicion (endocarditis, family history of aortic root disease, features consistent
with aortic dissection)
a) To-and-fro murmur, narrow pulse as signs of low cardiac output in contrast to bounding
pulse in chronic aortic regurgitation
b) Classical signs of de Musset, Traube, Quincke and Corrigan’s pulse unreliable in acute
decompensation with low cardiac output
2. Echocardiography
a) Transesophageal echocardiography for suspected thoracic aortic dissection
b) Thickened valve leaflets, flail leaflets, prolapse, vegetation, aortic root dilation
c) Regurgitant jet across the aortic valve on color flow Doppler
D. Management
1. Surgical emergency
2. Stabilize and support cardiovascular and respiratory status
a) Ventilatory support and invasive hemodynamic monitoring
b) Use diuretics, inotropic agents, and vasodilators to optimize pre- and afterload conditions
and improve contractility with emphasis of acute afterload reduction and faster heart rate
to allow forward flow and reduce regurgitant diastolic time
c) IABP contraindicated as balloon inflation during diastole will increase the regurgitant
volume
VI. Mitral Stenosis (MS)
A. Etiology
1. Nearly always due to rheumatic disease
2. Calcific mitral stenosis of the elderly is rare
3. 80% of cases are women
4. Large left-sided atrial myxoma may mimic mitral stenosis
5. Rare: Congenital, malignant carcinoid, lupus, amyloidosis
B. Clinical Presentation
1. Slowly progressive disease with insidious decline of effort tolerance level
131
ASCCA Residents Guide to the ICU
Acute decompensation with increased systemic hemodynamic demands in asymptomatic or
compensated patients with severe stenosis
3. Heart failure in pregnant women is most common emergency presentation
4. Pulmonary edema triggered or exacerbated by atrial fibrillation
C. Diagnosis
1. Diastolic rumble and opening snap difficult to hear in ICU environment
2. ECG with left atrial enlargement, atrial fibrillation
3. Echocardiography
a) Thickened valve, enlarged left atrium, stenotic flow on color Doppler
D. Treatment
1. Control fever, maintain normal hemoglobin level
2. Rate control for atrial fibrillation
3. Consider beta-blockers to prolong diastolic filling time
4. Hemodynamic monitoring and ventilatory support in decompensated CHF
5. Percutaneous transseptal balloon mitral valvotomy as emergency intervention:
a) Contraindication: Left atrial thrombus, coexisting moderate to severe mitral regurgitation,
or heavily calcified and deformed mitral valve
6. Urgent mitral valve replacement if valvotomy fails or contraindicated)
2.
VII. Aortic Stenosis (AS)
A. Etiology
1. Calcification of a normal or congenital bicuspid valve
2. Rheumatic aortic stenosis less common, but usually associated with mitral valve involvement
3. Restenosis after prior commissurotomy in childhood
B. Clinical Presentation
1. Chronic, slowly progressive disease
2. Syncope, pulmonary edema, angina or heart failure during pregnancy
3. Heart failure in the setting of pneumonia, anemia, or other conditions with increased
metabolic demands
C. Diagnosis
1. Classical physical findings of pulsus parvus et tardus with ejection systolic murmur unreliable
as decompensated stenosis may coexist with other valvular disorders. Loading conditions
may reduce or change the systolic murmur
2. ECG: LV hypertrophy, T-wave inversion and ST-segment depression
3. Echocardiography
a) Calcified, immobile leaflets
b) Stenotic flow across the aortic valve on color flow Doppler
c) Assessment of pressure gradient with Doppler
D. Management
1. Conservative approach; restore normal loading conditions
2. Urgent aortic valve replacement for severe AS and acute CHF
3. Balloon valvotomy or afterload reduction with nitroprusside controversial
VIII. Right-Sided Valve Disease
A. Pulmonic valve disease is nearly always congenital
B. Tricuspid valve stenosis relates to rheumatic mitral valve disease and is rare
C. Tricuspid regurgitation can present acutely due to endocarditis or to blunt or penetrating chest
wall trauma
1. Blunt chest wall trauma is often accompanied by cardiac injury
a) May result in pericardial effusion and tamponade
b) May result in valve rupture, with associated valve regurgitation
IX. Prosthetic Heart Valves
A. Mechanical Valves
1. Valve thrombosis
a) Caused by inadequate anticoagulation
b) May result in:
(1) Functional valve stenosis if movement of the leaflet(s) is restricted
132
ASCCA Residents Guide to the ICU
(2) Valve regurgitation if clot prevents full closure of the valve
Perivalvular leak
a) Early presentation often related to suture dehiscence
b) Associated with hemolytic anemia
c) Rule out endocarditis if leak occurs past the early stage of valve replacement
3. Clinical presentation similar to native valve stenosis or regurgitation
a) Echocardiography to assess valve dysfunction
4. Treatment
a) Controversial
b) For small thrombus consider conservative therapy with full dose intravenous
anticoagulation for several days
c) Valve replacement for severe hemodynamic compromise
d) Operative mortality from 17% to 40%
e) When surgical risk unacceptably high, consider systemic thrombolytic therapy, but:
(1) 20 % mortality, 16 % systemic embolism, still need emergency surgery in 20%
B. Tissue Valves
1. Slow degeneration of leaflets with calcification over 10 to 15 years
2. Acute decompensation in chronic prosthetic valve dysfunction by superimposed
hemodynamic stress
3. Rule out endocarditis in acute regurgitation
4. Diagnosis with echocardiography
5. Treatment similar to native valve with attempts to stabilize and optimize patients prior to valve
replacement surgery
2.
This chapter is a revision of the original chapter authored by David T. Porembka, D.O.
READING LIST:
1. Moore RA, Martin DE. Anesthetic Management For The Treatment Of Valvular Heart Disease. In: Hensley FA,
Martin DE, Gravlee GP, eds. Practical Approach to Cardiac Anesthesia. Third ed. Philadephia: Lippencott Williams
& Wilkins; 2002:302 - 35.
An easy to read manual about the practical aspect of hemodynamic management for valvular abnormalities
including prosthetic valves.
2. Bonow RO, Carabello BA, Kanu C, et al. ACC/AHA 2006 Guidelines For The Management Of Patients With
Valvular Heart Disease: A Report Of The American College Of Cardiology/American Heart Association Task Force
On Practice Guidelines. Circulation 2006;114(5):e84-231.
Practice guideline describing a range of generally accepted approaches for the diagnosis and management of
valvular disorders.
3. Boon NA, Bloomfield P. The Medical Management Of Valvar Heart Disease. Heart 2002;87(4):395-400.
Review of the medical treatment of the four major left heart valve lesions (aortic stenosis, mitral stenosis, aortic
regurgitation, and mitral regurgitation)
4. Carabello BA, Crawford FA, Jr. Valvular Heart Disease. N Engl J Med 1997;337(1):32-41.
A succinct, readable review of valvular heart disease
5. Mohan SB, Stouffer GA. Timing of surgery in aortic stenosis. Curr Treat Options Cardiovasc Med 2006;8(6):421-7.
Describes percutaneous balloon valvuloplasty for patients with aortic stenosis, who are not surgical candidates.
6. Otto CM. Emergent Valvular Disorders. In: Fink MP, Abraham E, Vincent J-L, Kochanek P, eds. Textbook of Critical
Care. 5th ed. Philadelphia: Elsevier Saunders; 2005:861 - 70.
A synopsis of emergency presentation and management of critical valvular disorders including a description of key
echocardiographic features of the major valvular lesions.
133
ASCCA Residents Guide to the ICU
QUESTIONS:
28.1.
A.
B.
C.
D.
E.
28.2.
A.
B.
C.
D.
E.
28.3.
A.
B.
C.
D.
E.
28.4.
A.
B.
C.
D.
E.
An 82-year-old man presents with an acute abdomen for exploration. He has a loud late
peaking systolic murmur at the apex radiating to his neck bilaterally. The most important
consideration is:
Decreasing afterload
Avoid volume overload
Prophylactic antibiotics
Placement of Invasive Monitoring Pre-Operatively
Prevent tachycardia
A patient with known severe mitral stenosis decompensates with rapid ventricular response
72 hours after the onset of new atrial fibrillation. When should electrical cardioversion be
attempted?
A transthoracic echocardiographic exam reveals no left atrial thrombus
There is adequate anticoagulation
Transesophageal echocardiography reveals no left atrial appendage thrombus
The cross-sectional dimensions are >4.5 cm2
There is normal left ventricular function
A patient is suspected of having an incompetent prosthetic mitral valve. Which diagnostic tool
will best assist in assessing any pathology?
Computed tomography
Transesophageal echocardiography
Transthoracic echocardiography
Angiogram
Pulmonary artery catheter
In a patient who has hemodynamic compromise, intraaortic balloon pump (IABP) may be
useful in the following conditions EXCEPT:
Acute myocardial ischemia
Ventricular septal defect
Acute aortic regurgitation
Acute mitral regurgitation
Congestive heart failure
134
ASCCA Residents Guide to the ICU
29. The Systemic Inflammatory Response Syndrome
(SIRS), Sepsis and Multiple Organ Dysfunction
Syndrome (MODS)
Sean M. Quinn, M.D., Miguel A. Cobas, M.D.
A 44-year-old male is admitted to the SICU after multiple traumatic injuries following a motor vehicle
crash. He sustained a subclavian vein injury and a lower extremity fracture, which were both repaired
emergently in the operating room. He required massive fluid and blood administration, but arrives to the
ICU intubated and hemodynamically stable on no vasoactive support. Over the first 24 hours of his ICU
course, the patient continues to improve and is on minimal mechanical ventilatory support. However, on
postoperative day 2, he required increasing ventilatory support for worsening hypoxia. The following two
days, he remains hypotensive, which is no longer responding to fluid boluses and requires vasopressor
support. A pulmonary artery catheter is inserted to guide therapeutic management, and shows increased
cardiac output and decreased systemic vascular resistance. Despite no obvious source of infection,
cultures are drawn and empiric antibiotics are started.
The sepsis syndromes include a number of specifically defined processes, which range from the Systemic
Inflammatory Response Syndrome and Sepsis to Septic Shock and Multiple Organ Dysfunction
Syndrome (MODS). Patients present with SIRS due to a wide variety of severe clinical insults and are
diagnosed by a number of objective clinical data. Sepsis results when this is coupled with a documented
infection, and severe sepsis further includes hypoperfusion and organ dysfunction. The final stage in this
continuum is MODS, in which multiple organ systems fail in an acutely ill patient which require significant
intervention to maintain homeostasis. As the incidence of sepsis continues to increase, it remains the
number one cause of death in the noncardiac intensive care unit. In order to reduce mortality and
morbidity of this disease process, early diagnosis and treatment is crucial. In order to facilitate this, it is
essential that the anesthesiologist be familiar with the basic pathophysiology and treatment modalities
associated with these syndromes, both in and out of the ICU.
I.
Definitions of the Sepsis Syndromes
A. The Society of Critical Care Medicine and the American College of Chest Physicians Consensus
Conference provide specific definitions for the classification of systemic inflammation, sepsis, and
organ failure.
B. Systemic Inflammatory Response Syndrome (manifested by two or more conditions)
1. Temperature >38ВєC or <36ВєC
2. Heart rate > 90 beats/min
3. Respiratory rate > 20 breaths/minute or PaCO2 <32 mmHg
4. WBC > 12,000/ВµL or < 4,000/ВµL, or > 10% immature (band) forms
C. Sepsis в€’ systemic inflammatory response to a documented infection.
D. Severe sepsis в€’ sepsis associated with organ dysfunction, hypoperfusion, or hypotension.
E. Septic Shock в€’ subset of sepsis with sepsis induced hypotension despite adequate fluid
resuscitation with the presence of perfusion abnormalities (i.e., lactic acidosis, oliguria, or altered
mental status).
F. Multiple organ dysfunction syndrome в€’ presence of organ dysfunction in multiple systems such
that homeostasis can not be achieved without intervention. MODS may result from sepsis,
trauma, massive acute blood loss, severe burns, or hemorrhagic pancreatitis.
1. Respiratory Dysfunction
a) Development of acute respiratory failure marks onset of acute lung injury (ALI) or acute
respiratory distress syndrome (ARDS)
b) Persistent hypoxia despite increasing FiO2 requires noninvasive (CPAP) or mechanical
ventilatory support
c) Inadequate ventilation due to reduced compliance and effective lung volume requires a
substantial elevation in work of breathing and minute ventilation to normalize PaCO2
2. Cardiovascular Dysfunction
135
ASCCA Residents Guide to the ICU
Elevation of cardiac output but variable stroke volume and diastolic volume (i.e., high
PCWP or LVEDV required to maintain cardiac output/stroke volume)
b) Severe endocapillary leak requiring high levels of fluid administration to maintain
intravascular volume, resulting in severe anasarca
c) Refractory, malignant arrhythmias (i.e., frequent PVCs, R-on-T pattern, SVT)
Renal/Electrolyte Dysfunction
a) Progressive rise in serum creatinine and decreased creatinine clearance (acute tubular
necrosis)
b) Progressive acidosis, often with anion gap, indicating the presence of unmeasured,
unexcreted acid byproducts of metabolism
c) Elevation of serum levels of intracellular electrolytes K+, Mg+2, PO4-3 reflecting loss of
body cell mass and renal insufficiency
d) Progressive elevations of urea nitrogen reflecting protein catabolism and impaired
excretory function
Hepatic Dysfunction
a) Hyperbilirubinemia and hyperbilirubinuria
b) Decreased synthetic capacity, often reflected in elevated prothrombin time
c) Failure to clear lactate (high lactate/pyruvate ratio)
d) Intrahepatic cholestasis
Neurological Dysfunction
a) Encephalopathy
b) Peripheral neuropathy
Hematologic Dysfunction
a) Anemia
b) Thrombocytopenia
c) Variable abnormalities in count and function of white cells
d) Consumptive coagulopathy (DIC)
Immune Dysfunction
a) Development of anergy
b) Development of opportunistic infections
Metabolic Dysfunction
a) Elevated resting energy expenditure
b) Elevated oxygen consumption
c) Elevated carbon dioxide production
d) Mobilization of muscle mass
e) Hyperglycemia
f) Fat intolerance
Endocrine Dysfunction
a) Elevations of serum levels of cortisol, glucagon, and epinephrine, but evidence of
diminished activity
b) Insulin resistance
c) Sick euthyroid syndrome
d) Relative adrenal insufficiency (“Baldwin’s syndrome”)
a)
3.
4.
5.
6.
7.
8.
9.
II. Epidemiology and Clinical Patterns
A. Most common cause of death in noncardiac intensive care unit patients and the eleventh cause of
death overall.
1. Annual incidence of 750,000 new cases of severe sepsis in the U.S.
2. Accounts for approximately 15-25% of all patients requiring critical care.
3. Mortality ranges from 15% in sepsis to 40-60% in septic shock.
B. Two basic clinical patterns:
1. Primary Respiratory Pattern with MODS as a terminal event
a) Development of cardiopulmonary dysfunction (ARDS) without involvement of other organ
systems two-to-five days following initial insult
b) Triggers for this pattern of MODS most often involve direct lung injury or infection (i.e.,
pneumonia, aspiration, isolated pulmonary trauma, respiratory failure in COPD)
c) Continued cardiopulmonary dysfunction for about three weeks
d) After three weeks, either
136
ASCCA Residents Guide to the ICU
(1) Resolution of pulmonary dysfunction, or
(2) Rapid development of dysfunction in other organ systems, most often kidney and
liver
e) Process continues for about one more week, with patients either recovering (60%) or
expiring (40%)
2. MODS as a Primary Disease Entity
a) Development of cardiopulmonary dysfunction (ARDS) without involvement of other organ
systems two-to-five days following initial insult
b) Development of progressive dysfunction, most often kidney and liver, within 3-to-5 days
c) Continued dysfunction for about three weeks
d) Resolution of process (30%) or dysfunction in other systems progressing to death (70%)
C. Most common terminal causes of death
1. Cardiovascular collapse
2. Refractory acidosis/electrolyte imbalance precipitating malignant arrhythmias
3. Progressive hypercarbia and hypoxemia
4. Refractory GI bleed
D. Risk Factors and Precipitating Events:
1. Historically, initial reports of MODS involved trauma victims resuscitated from severe insults,
patients surviving rupture of abdominal aortic aneurysms with massive blood loss, and
sepsis.
2. Any severe metabolic insult can precipitate MODS, including not only major trauma and
hemorrhagic shock, but also bacteremia, burns, pancreatitis, acute myocardial infarction, or
severe neurological insults (stroke, subarachnoid hemorrhage, intracerebral bleeding).
3. Patient characteristics play a major role in morbidity and mortality. Important factors include:
a) Patient age
b) Presence of preexisting systemic disease processes
c) Patient gender
d) Patient genotype
e) Patient lipid status
III. Pathogenic Hypotheses
A number of hypotheses have been advanced to explain the development of MODS. There are
considerable areas of overlap and common links. There are also significant problems with most.
Elements of each are probably important, as MODS is a multifactorial disease.
A. The Cytokine Hypothesis
1. Cytokines are mediators produced and secreted primarily by inflammatory and endothelial
cells. They are bound to cell surface receptors and activate intracellular events, most often
transcription. Cytokines are primarily autocrine or paracrine mediators, affecting cells near
the site of secretion. Occasionally, they may be released into the circulation and act in an
endocrine fashion.
2. The cytokines tumor necrosis factor (TNF)-О±, interleukins (IL)-1 and IL-6 are pro-inflammatory
mediators that have been implicated in MODS. IL-10 and О±-interferon are anti-inflammatory
mediators which are believed to be important in MODS
3. It is hypothesized that the pro-inflammatory mediators are overproduced and the antiinflammatory mediators are underproduced. This results in prolonged and uncontrollable
vasodilation and damage to endogenous tissue by primed macrophages and neutrophils.
4. TNF-О± likely is the most proximal mediator, activating IL-1 and IL-6 in sequence.
Experimental data derived from administration of TNF and endotoxin to volunteers and
animals support this theory. However, data from peritonitis models contradict this view and
large clinical trials using antibodies to TNF and IL-1 have not improved outcome.
B. Microcirculatory Hypothesis
1. A failure of cells or organs to receive an adequate supply of oxygen or some other important
nutrient
2. Low flow may be secondary to hypotension, shock, or hypovolemia, or reflect vasoactive
mediators and vascular congestion caused by microemboli.
3. Reperfusion is an important component of injury. Free radicals may mediate the injury.
4. The endothelium may be directly affected and participate actively in continuation of the
inflammatory cell-mediated damage process.
137
ASCCA Residents Guide to the ICU
C. Gut Hypothesis
1. The occult translocation of bacteria or endotoxin from the gut into the circulation mediates
MODS.
2. Multiple insults are required for experimental bacterial translocation to occur.
3. Exposure to intestinal flora can activate cytokine pathways.
D. Two-Hit Hypothesis
1. A second insult, following an initial “hit” may be required to precipitate MODS. The first hit
“primes” the patient to develop MODS.
2. Priming could also activate white cells or platelets, disrupt the GI mucosal barrier, and
facilitate translocation/induce free radical formation in the gut.
E. Connectionist Hypothesis
1. Biological systems can be viewed as oscillators, with continual variation around a mean level
of function.
2. Variability reflects a constant inflow of continuously changing external stimuli
3. Loss of these stimuli or loss of the ability to respond to them is pathological.
4. In MODS, changes in variability reflect an inability of cells or organ systems to communicate
with each other.
5. This hypothesis supplements and complements the above mentioned hypotheses.
F. Transcriptional Hypothesis
1. The expression of a specific group of genes leads to a characteristic cellular phenotype, e.g.,
a hepatocyte expresses genes for gluconeogenic enzymes, coagulation factors, bile
transporters, etc., while a pneumocyte expresses surfactant proteins and an enterocyte
expresses polysaccharidases.
2. Loss of the ability to express these cell-specific proteins leads to cellular de-differentiation
and an impairment of function.
3. In animal models of MODS, a loss of organ-specific transcription has been documented in
hepatocytes and pneumocytes.
IV. Pathophysiology of Organ Dysfunction
A. Pulmonary
1. Clinical scenario similar to ARDS, with bilateral pulmonary infiltrates secondary to increased
capillary membrane permeability, impaired gas exchange, and absence of left ventricular
dysfunction.
2. ARDS with PaO2/FiO2 ≤ 200; ALI with PaO2/FiO2 ≤ 300
3. Acute injury may be superimposed on preexisting COPD or lung disease.
4. Barotrauma or volutrauma from conventional mechanical ventilation strategies may result in
additional lung injury.
B. Cardiovascular
1. Hyperdynamic hemodynamic profile seen after volume replacement leading to tachycardia,
increased cardiac output, and decreased systemic vascular resistance.
2. Myocardial depression reflected by decreased stoke volume, decreased ejection fraction,
decreased left ventricular compliance, and high filling pressures.
3. Cardiovascular depression likely mediated by endotoxin.
4. Vasodilation can be refractory to vasopressors due to a defect in endothelial and vascular
smooth muscle responses to catecholamines, perhaps by the result of overproduction of nitric
oxide.
5. Right ventricular dysfunction secondary to elevated pulmonary artery pressures.
C. Renal
1. Initial pathologic changes are typical to acute tubular injury including basement membrane
disruption, patchy necrosis, interstitial edema, and formation of tubular casts.
2. The clinical hallmark is a reduction in creatinine clearance.
3. The pathologic results include loss of concentrating ability, impaired excretion of potassium
and acid, and anion gap acidosis.
D. Hepatic
1. Hypovolemia from splanchnic venous pooling and increased capillary membrane permeability
contribute to reduced cardiac output and hepatic blood flow.
2. Microvascular changes include decreased sinusoidal blood flow, fibrotic acinar distortion,
microthrombi, and endothelial cell swelling.
138
ASCCA Residents Guide to the ICU
Inhibition of drug metabolism and cytokine biotransformation by inhibition of cytochrome
P-450 pathway.
4. Nonspecific indicators of liver dysfunction are increased serum bilirubin, prolongation of the
prothrombin time, and elevated transaminases.
E. Gastrointestinal
1. Abnormal gastric motility includes gastric atony and generalized bowel adynamic ileus (most
common after bowel surgery).
2. Intestinal ischemia and bowel wall edema from vascular leak may contribute to decreased
motility.
3. Bleeding secondary to stress-related mucosal disease is a potential problem, especially in
patients with prolonged mucosal ischemia.
4. Rapid cannibalization of mucosa leads to barrier and absorption failure.
F. Metabolic
1. IL-1 mediates hypoalbuminemia as protein synthesis shifts from albumin to acute phase
reactant proteins, and stimulates protein catabolism.
2. Increased endogenous catecholamines, glucocorticoids, and glucagon contribute to impaired
glucose use in sepsis.
3. Failure to clear exogenous long chain fatty acids results in ectopic fat deposition
4. Pre-renal azotemia may result from amino acid deamination in Krebs cycle.
3.
V. Support and Therapy
A. Initial Resuscitation
1. Rapid fluid resuscitation to obtain CVP of 8 to 12 mmHg.
2. Vasoactive drugs to maintain MAP of 65-70 mmHg.
3. Inotropic support to maintain CO if depressed myocardium, or SvO2 > 70%.
4. Goal is to maintain urine output > 0.5 ml/kg/hr.
B. Source Control
1. Appropriate cultures should be drawn prior to starting antibiotics.
2. Broad spectrum antibiotics should be started depending in the common organisms growing in
each particular unit.
3. Antibiotics should be streamlined to the bacteria found and stopped after 72 hours if no
organism is found.
4. Control local source of infection if possible (abscess drainage, surgical debridement, removal
of vascular device).
5. Common source of infection are pulmonary, urinary tract, vascular access device,
intraabdominal abscess, sinusitis.
6. Uncommon sources include acalculous cholecystitis, vaginitis/endometritis and Clostridium
difficile colitis among others.
7. In 50% of patients, no source of infection can be found.
C. Hemodynamic Optimization
1. Aggressive fluid challenge with crystalloid or colloid in patients with perfusion deficits.
2. Vasopressors should be used in patients who remain hypotensive after volume resuscitation
and target CVP reached.
3. Norepinephrine is a first-line drug (supports central perfusion to heart, brain, visceral organs).
4. Inotropic support (e.g., dobutamine) should be used in patients with low cardiac output to
obtain a cardiac output 1.5 times normal.
D. Steroids
1. Consider intravenous steroids (hydrocortisone) for 7 days in patients who continue to require
vasopressor support despite adequate fluid replacement.
2. ACTH stimulation test may be performed on patients with low vasopressor requirements, and
steroids withheld if adequate response noted.
E. Recombinant Active Protein C
1. Recommended in patients with septic shock requiring vasopressors or in patients with three
or more sepsis-related organ systems dysfunction.
2. Contraindicated in patients with high risk of bleeding.
3. Patients with APACHE scores of > 24 should receive therapy unles contraindicated.
F. Blood Product Administration
1. Red blood cell transfusion should occur when hemoglobin decreases to < 7.0 g/dL (in
139
ASCCA Residents Guide to the ICU
patients with no other comorbidities or ongoing bleeding).
Erythropoietin is recommended for treatment of anemia only when a patient has another
accepted reason for use (i.e., kidney disease).
3. FFP and platelets should only be administered for ongoing clinical bleeding.
G. Mechanical Ventilation
1. High tidal volumes and high plateau pressures should be avoided in all ALI/ARDS patients.
2. Tidal volumes should be near 6 mg/kg with plateau pressures < 30 cm H20.
3. Permissive hypercapnia can be tolerated to minimize plateau pressures.
4. Spontaneous breathing trials help to facilitate more rapid extubation.
H. Metabolic Therapy
1. Glucose should be maintained under 150 mg/dL.
2. Bicarbonate therapy is not routinely recommended for improving hemodynamics or reducing
vasopressor requirements unless pH < 7.15.
3. Initiate enteral nutrition as soon as patient is stabilized. Parenteral nutrition should be
avoided, but is still better than prolonged fasting.
I. Prophylactic Therapy
1. H2 blockers or proton pump inhibitors should be used in all patients.
2. Pharmacological (low-molecular weight or unfractionated heparin) or mechanical deep vein
thromboprophylaxis is recommended in all patients unless contraindicated.
2.
VI. Outcomes
A. Mortality from severe sepsis is directly proportional to number of organ system dysfunctions.
1. Several organ dysfunction scores (i.e., Multiple Organ Dysfunction Score) have been
developed to objectively quantify degree of organ failure and describe morbidity or mortality
risk.
2. APACHE II score is the most widely utilized prediction model of mortality.
B. Mortality can be minimized in treating sepsis by a high index of suspicion and early goal-directed
interventions.
1. Immediate resuscitation and hemodynamic stabilization of the patient.
2. Rapid clearance of microorganisms from blood and source infection control.
3. Implement lung protective ventilator parameters to minimize lung injury.
4. Early appropriate metabolic and enteral nutritional support.
5. Administration of Activated Protein C in patients with high mortality risk.
C. Protocols integrating the Surviving Sepsis Campaign recommendations may help to ensure early
goal-directed therapy, although care needs to be individualized to specific patient populations and
organ dysfunctions.
This chapter is a revision of the original chapter authored by Clifford Deutschman, M.S., M.D.,
Zoltan G. Hevesi, M.D. and Thomas M. Fuhrman, M.D.
READING LIST:
1. Abraham E, Laterre PF, Garg R et al. Drotrecogin alfa (activated) for adults with severe sepsis and a low risk of
death. New Engl J Med. 2005; 353:1332-1341.
Randomized controlled trial demonstrating that drotrecogin alfa (activated) should not be used on patients with
severe sepsis with single-organ failure or an APACHE II score of less than 25 due to an increased incidence of
serious bleeding complications.
2. Angus DC, Linde-Zwirble WT, Lidicker J et al. Epidemiology of severe sepsis in the United States: analysis of
incidence, outcome, and associated costs of care. Crit Care Med 2001; 29:1303-1310.
Observational cohort study detailing the incidence and outcomes of severe sepsis, including the morality
associated with increasing severity of organ dysfunction.
3. Bone RC, Balk RA, Cerra FB, et al. American College of Chest Physicians / Society of Critical Care Medicine
Consensus Conference Committee: Definitions for sepsis and organ failure and guidelines for the use of innovative
therapies in sepsis. Chest. 1992; 101:1644-55.
Consensus guidelines for standardized definitions and criteria for classification and diagnosis of sepsis and organ
failure.
140
ASCCA Residents Guide to the ICU
4. Bernard GR, Vincent JL, Laterre PF, et al. Recombinant human protein C worldwide evaluation in severe sepsis
(PROWESS) study group. Efficacy and safety of recombinant human activated protein C for severe sepsis. N Engl J
Med. 2001; 344:699–709.
A multicenter randomized controlled trial of patients with severe sepsis which demonstrates a significant survival
benefit with the use of activated drotrecogin alfa.
5. Dellinger RP, Carlet JM, Masur H, et al. Surviving sepsis campaign guidelines for management of severe sepsis and
septic shock. Crit Care Med. 2008; 36(1):296-327.
Systematic review of evidence-based recommendations of the acute management of sepsis and septic shock in an
attempt to improve outcomes for the critically ill patient.
6. Hotchkiss RS, Karl IE. The pathophysiology and treatment of sepsis. New Engl J Med. 2003; 348:138-150.
Review article which explores the cellular mechanisms involved in sepsis and new concepts and modalities in the
treatment of sepsis.
7. Rivers E, Nguyen B, Havstad S, et al. Early goal-directed therapy in the treatment of severe sepsis and septic shock.
New Engl J Med. 2001; 345:1368-1377.
Randomized trial which demonstrates that early intervention and goal-directed therapy in the initial management
of sepsis results in less severe organ dysfunction.
QUESTIONS:
29.1.
A.
B.
C.
D.
E.
29.2.
A.
B.
C.
D.
E.
29.3.
A.
B.
C.
D.
E.
29.4.
A.
B.
C.
D.
E.
An 18 y.o. male is in the ICU after sustaining a stab wound to the groin with vascular
compromise that was emergently repaired 4 hours ago. He is mechanically ventilated and his
RR is 32/min on pressure support ventilation of 10 cm H20, HR 135, BP 150/100, temp 37.3
C, WBC 13000/μL. This patient’s condition is most likely due to:
Septic shock
Thyroid Storm
Malignant Hyperthermia
Opioid Overdose
Systemic Inflammatory Response Syndrome
A patient in the ICU has the following hemodynamic parameters: C.O: 4.5 L, PCWP: 19
mmHg. Mean Arterial Blood Pressure remains in the 50’s. The pt received over 9L of fluids
over the last 24 hours. The most appropriate next step in management would be:
Order a transthoracic Echocardiogram
Obtain a CAT scan of the abdomen
Start furosemide drip
Start vasopressor therapy
Continue administration of fluids
You’re called to evaluate a patient on the floor with high fevers, chills, WBC 23000/μL and
hypotension. All the next steps would be appropriate EXCEPT:
Obtain cultures from likely sources
Consider transfer patient to the ICU
Wait to start antibiotic therapy until cultures results are reported
Become familiar with your local antibiogram
Assure adequacy of intravenous lines for fluid administration
One of your septic patients in the ICU starts to become oliguric. Which of the following
findings is consistent with sepsis-related Acute Tubular Necrosis:
Urine Osmolality > 500 mOsm
Fractional Excretion of Sodium <1%
Urine/Plasma Creatinine >30
Urine Sodium concentration > 20 mEq/L
Concentrated Urine
141
ASCCA Residents Guide to the ICU
29.5.
A.
B.
C.
D.
E.
Septic Shock is characterized by the following signs and symptoms EXCEPT:
Cortisol Deficiency
Hypotension despite adequate fluid replacement
Increased Lactate
Evidence of Infection
Tachycardia
142
ASCCA Residents Guide to the ICU
30. Nosocomial Infections
Todd W. Sarge, M.D.
A 60-year-old woman remains intubated and ventilated in an intensive care unit six days following rupture
of a cerebral aneurysm. She is now post-operative day 5 from clipping of the aneurysm and is being
watched closely for vasospasm. Early this morning, she is noted to have a fever to 102.3 degrees
Fahrenheit. Her laboratory studies are notable for a white blood cell count of 16 thousand, with left shift
(~89 % neutrophils). The respiratory therapist and nurse report increased need to suction her
endotracheal tube for thick, tan secretions.
Infections are a common adversary to clinicians caring for patients in the intensive care unit. Critically ill
patients, even when admitted for reasons other than infection as in the case above, are at high risk of
developing nosocomial infections during their recovery. Nosocomial infections are endemic, given that
patients who are critically ill have excess catecholamine production and often lack adequate nutrition,
both leading to compromised immune function. In addition, critically ill patients are further threatened with
infection by the placement of invasive therapies and monitors. For example, the patient described above
is likely to have the following invasive devices: 1) an intra-ventricular drain; 2) endotracheal tube; 3)
nasogastric tube; 4) central venous catheter; 5) peripheral arterial catheter; 6) foley catheter. All of these
devices have set the stage for an opportunistic infection by creating a defect in the body’s first line of
defense, i.e. the skin.
I.
Nosocomial Infection Epidemiology
A. Several studies have reported nosocomial infection rates of 28% - 45%
B. Nosocomial infections have reportedly increase associated in-hospital mortality by 1.9 fold
C. Most common infections are as follows:
1. Pneumonia – ~30 – 40%
a) 83% associated with mechanical ventilation, i.e. VAP
2. Urinary tract infection UTI - ~23%
a) 97% associated with urinary catheters
3. Blood Stream Infection BSI - ~14%
a) 87% assoc. with central venous catheters, i.e. CRBSI
D. Risk Factors
1. Length of stay in the ICU
2. Invasive medical devices – CVL, Urinary Catheters, Mechanical Ventilation
3. Stress ulcer prophylaxis increases VAP risk
II. Signs and symptoms
A. Fever
1. Definition – criteria varies, but usually > 101.4 degrees Fahrenheit
2. Non-specific - occurring in over half of all ICU patients
3. Insensitive – elderly and severely immunocompromised may not generate fever
B. Leukocytosis or leukopenia - Differential should be performed looking for left shift / bandemia
C. Localizing signs – pain, swelling, erythema, wound discharge
III. Ventilator Associated Pneumonia (VAP)
A. Definition – any pneumonia not present on admission that develops in a patient mechanically
ventilated greater than 48 hours.
B. Epidemiology
1. Frequency – the most common nosocomial infection in intubated patients (90%)
2. Incidence – occurs in 10 – 65% of all patients mechanically ventilated > 48 hours
3. Mortality – 24 – 50% (May be as high as 75% for resistant organisms)
C. Pathogens
1. Early Onset – H. Flu; MSSA; Klebsiella; S. Pneumoniae
2. Late Onset (higher resistance) – Pseudomonas aeruginosa; Acinetobacter; MRSA
143
ASCCA Residents Guide to the ICU
D. Diagnosis
1. At least one of the following:
a) Fever (> 38 degrees C) with no other recognized cause
b) Leukopenia (< 4,000 WBC/mm3) or leukocytosis (> 12,000 WBC/mm3)
2. AND At least 2 of the following:
a) New onset of purulent sputum or change in character of secretions
b) New onset or worsening cough, dyspnea, or tachypnea
c) Rales or bronchial breath sounds
d) Worsening oxygenation (PaO2 / FiO2 < 240; increasing oxygen requirement/PEEP)
3. Radiographic evidence
a) New, progressive or persistent infiltrate
b) Consolidation, opacity, or cavitation
4. Microbiology with quantitative cultures results remains controversial for diagnosis
a) Blind tracheal aspirate vs. protected brush specimen vs. directed bronchoalveolar lavage
b) Generally accepted criteria is quantitive culture yielding > 105 colony forming units
E. Prevention
1. Handwashing
2. Oral care / gut decontamination
3. Head of Bed elevation > 30 degrees (If clinical contraindicated, use reverse Trendelenberg)
4. Patient turning and rotational therapy
5. GI/DVT prophylaxis
6. Ventilator circuit care – minimize disconnects, in-line suctioning, humidification
7. Aggressive ventilator weaning / extubation protocols
F. Treatment
1. Empiric broad spectrum antibiotics
2. Initial antibiotic choice guided by pathogens common to institution
3. Should be initiated early while cultures are pending
4. De-escalate antibiotics when culture results are known
IV. Urinary Tract Infections
A. Colonization occurs at a rate of 5 – 10% per day
B. Prevention
1. Sterile insertion
2. Early removal in ambulatory patients
3. Maintain closed drainage system
C. Diagnosis
1. Generally accepted quantitative culture of > 105 bacteria / mL from freshly collected urine is
definitive for infection
2. Qualitative culture is difficult to interpret given colonization in ICU patients
D. Common Pathogens
1. E. coli the most common > 30% of UTI’s
2. Other common species: Enterococcus, Pseudomonas, Klebsiella, Proteus
3. Candida – difficult to establish as infection except in the setting of sepsis and fungemia
E. Treatment
1. Appropriate antibiotic therapy should be established in critically ill patients
2. Unobstructed urinary flow should be established if possible
3. Catheters should be removed if possible
V. Catheter Related Blood Stream Infections (CRBSI)
A. Epidemiology
1. Incidence – may affect up to 20% of critical ill patients.
2. May account for up to 70% of all hospital-acquired bacteremias
3. Case fatality rate remains between 10 – 20%
4. Increased length of stay and costs ($25,000 - $30,000 per episode)
B. Mechanisms
1. Bacterial migration from skin to blood via catheter
2. Colonization occurs from circulating microorganisms at distant site
3. Infusion of contaminated intravenous fluid or drug
144
ASCCA Residents Guide to the ICU
C. Risk Factors
1. Diabetes Mellitus
2. Immunosuppressive therapy/drugs
3. Immune deficiency
4. Skin diseases at the insertion site
5. Sepsis from a distant source
6. Operator inexperience
7. Surgical cut down at site
8. Prolonged catheterization at one site
9. Emergency placement with non-sterile conditions
10. Insertion of large/multi-lumen catheters
11. Insertion site – Subclavian < Internal Jugular < Femoral
12. Infusion of parenteral nutrition or lipid containing compounds
D. Prevention
1. Education programs
2. Use of maximum barrier precautions when inserting catheters
3. Avoiding the femoral insertion site
4. Adequate skin antisepsis - chlorhexidine currently preferred
5. Removal of catheters when no longer needed is paramount
6. Minimizing blood draws through catheters
7. Minimizing number of times a port is accessed
8. Daily evaluation and monitoring of site for phlebitis
9. Routine rewiring is NOT preventative and may increase risk
10. Careful dressing changes with sterile techniques and gloves
11. Adjunct measures - antibiotic impregnanted catheters, antiseptic ointment, BioPatchВ®
E. Diagnosis
1. Redness, pain, swelling and erythema at the site are suggestive of infection but relatively
insensitive
2. Differential time to positivity: positive blood culture drawn from the catheter and consistent
with positive blood culture drawn at distant site is considered a CRBSI if the time difference to
positive cultures is greater than two hours.
3. Distal catheter tip culture with > 15 colonies consistent with a patient’s peripheral blood
cultures (by antibiogram) is also compelling evidence for a CRBSI
F. Common Pathogens
1. S. aureus, S. epidermidis and enterococcus cause most catheter related infections
G. Treatment
1. Removal and culture of the catheter should be the first step
2. Empiric antibiotics should be directed at resistant organisms and then de-escalated
3. Persistent bacteremia requires a work-up for endocarditis with echocardiography
VI. Prevention and Control Measures
A. Infection control precautions
1. CDC and Hospital Infection Control Practices Advisory Committee guidelines
2. Standard precautions for all patients
3. Additional, more stringent, transmission-based precautions for those with colonization or
infection with easily transmissible or resistant infections
B. Hospital design
1. Single patient rooms for ICU and those with highly transmissible or resistant infections.
2. Isolation rooms (with negative pressure) for patients with airborne transmissible disease.
3. Hand hygiene stations and sinks.
C. Hospital surveillance programs
1. Tracking of endemic outbreaks and antimicrobial resistance
2. Monitoring hand hygiene and personnel compliance with control precautions
D. Quality Improvement Initiatives
1. Standardized central venous line insertion with regard to equipment, supervision, site and
dressing care.
2. Bundles for care of mechanically ventilated patient to include hand hygiene, aggressive
weaning, oral decontamination and suctioning, head of bed elevation, etc.
145
ASCCA Residents Guide to the ICU
READING LIST:
1. Vincent JL, Bihari DJ, Suter PM, et al: The prevalence of nosocomial infection in intensive care units in Europe.
Results of the European Prevalence of Infection in Intensive Care (EPIC) study. EPIC International Advisory
Committee. JAMA 274(8):639—644, 1995.
2. National Nosocomial Infections Surveillance (NNIS) system report, data summary from January 1992 through June
2004, issued October 2004. Am J Infect Control 32(8):470—485, 2004.
3. O’Grady NP, Alexander M, Dellinger EP, et al: Guidelines for the prevention of intravascular catheter-related
infections. Clin Infect Dis 35:1281, 2002.
4. Warren DK, Cosgrove SE, Diekema DJ. A multicenter intervention to prevent catheter-associated bloodstream
infections. Infect Control Hosp Epidemiol 27(7): 662-669, 2006.
5. American Thoracic Society/Infectious Diseases Society of America. Guidelines for the management of adults with
hospital-acquired, ventilator-associated, and health care–associated pneumonia. Am J Respir Crit Care Med
171:388–416, 2005.
QUESTIONS:
30.1.
A.
B.
C.
D.
E.
Which of the following are risk factors for acquiring a nosocomial infection in the ICU:
Length of stay in the ICU.
Stress ulcer prophylaxis
Central venous catheter
Mechanical ventilation
All of the above
30.2.
Arterial catheters appear to have infectious complication rates much lower than observed for
venous catheters.
A. True
B. False
30.3.
A.
B.
C.
D.
E.
30.4.
A.
B.
C.
D.
E.
Good hand hygiene with alcohol-based hand rubs has been shown to reduce the incidence of
most nosocomial pathogens except:
Fungi
Influenza virus
Clostridium Difficile
Human Immunodeficiency virus
Gram-negative bacteria
Universal precautions for placement of an arterial catheter in a septic patient with gramnegative bacteria include which of the following:
Fluid-resistant face mask
Gloves
Goggles or equivalent eye protection
Fluid-resistant gown
All of the above
30.5. Routine replacement of central venous catheters is not recommended.
A. True
B. False
146
ASCCA Residents Guide to the ICU
31. Infections and Antibiotic Therapy in the
Intensive Care Unit
Ronald W. Pauldine, M.D.
A 24-year-old woman is involved in a high speed motor vehicle accident, sustaining a severe
closed head injury, flail chest, pulmonary contusion, pelvic fracture, and ruptured spleen
(removed at laparotomy). On postoperative day 5, she becomes febrile, and sputum obtained by
blind tracheal aspiration through an orotracheal tube reveals many PMNs and 2+ Pseudomonas
aeruginosa. Her chest x-ray shows an unchanged pulmonary contusion and her white blood cell
count is 14,000.
Infection can be defined as the isolation of a predominant pathogen from a site in the body that is
normally sterile. Importantly, infection contributes to more than half of all deaths in intensive care
unit patients. However, most infected patients do not go on to develop sepsis. The relationship
between infection, inflammation, and sepsis is complex. Host factors are extremely important in
defining an individual patient’s response to infection. Most infections observed in ICU patients
will fall into one of 3 categories. Patients will be admitted for primary infections, develop
nosocomial infections or infections will develop in immune-compromised patients. Infection is
often suspected in the setting of fever and leukocytosis. Localizing signs and symptoms such as
erythema, swelling, drainage, pain or cough may be present. However, the diagnosis of true
infection may be difficult due to non-specific physical findings, lack of characteristic findings in
some patient populations, overlap of findings with a number of non-infectious processes and
confounding culture results often seen with colonization. Accurate diagnosis is a cornerstone in
ensuring proper antimicrobial therapy. Indiscriminate antibiotic usage is a significant problem in
promoting selection of antibiotic resistant organisms which are then potentially transmitted to
other patients in the ICU.
I.
General considerations for ICU infections
A. High morbidity and mortality
B. Most common cause of death in critically ill patients, particularly when involved in
initiation and maintenance of multisystem organ failure
C. Endemic nosocomial organisms will colonize most patients within 72 hours of
hospitalization and are very important when choosing empiric therapy – be familiar with
your ICU antibiogram
D. Frequently caused by multiple drug resistant organisms
E. Can be difficult to diagnose, as critically ill patients may or may not be able to mount the
expected host response
F. Blood cultures should be obtained before initiation of empiric therapy
G. Empiric therapy should be initiated with broad spectrum agents that have activity against
commonly encountered organisms in the ICU then tailored to specific agents based on
the culture results
H. Source control (mechanical removal of the source of infection if appropriate) is a
governing principle of treatment. e.g. abscesses drained, infected hardware removed,
decompression of biliary obstruction, etc
II. Risk Factors
A. Severity of illness
B. Airway instrumentation and mechanical ventilation
C. Bladder catheterization
D. Intravascular catheters
E. Multiple, broad spectrum, long-term antibiotic usage
F. Chronic, pre-existing illness
G. Medications: steroids, other immunosuppressants, chemotherapeutic agents
147
ASCCA Residents Guide to the ICU
H. Endemic resistant organisms
I. Many of the risk factors are probably just indicators of severity of illness and not
independent of each other.
III. Commonly encountered patterns of infection in the ICU (GPC = Gram positive cocci, GNR =
Gram negative rods)
A. Pneumonia
1. Clinical setting and Organisms
a) Community-acquired
(1) First positive bacterial culture within 48 hrs of admission
(2) Common pathogens – Strep pneumoniae, H. flu, M. catarrhalis, M.
pneumoniae, Legionella, Chlamydia, MRSA, Influenza
b) Healthcare-associated
(1) First positive bacterial culture within 48 hrs of admission AND transferred
from another hospital OR receiving outpatient therapy for hemodialysis,
wound care or intravenous treatment OR prior hospitalization ≥3 days within
90 days of positive culture OR immunocompromised
c) Hospital-acquired
(1) First positive bacterial culture 48 hrs after admission
d) Ventilator-associated (VAP)
(1) Mechanically ventilated patients with first positive bacterial culture ≥ 48 hrs of
tracheal intubation
(2) Causative organisms for healthcare associated, hospital acquired, and VAP
include – MRSA, Pseudomonas aeruginosa, Klebsiella pneumoniae,
Acinetobacter spp. Stenotrophomonas spp. Legionella pneumophila
2. Diagnosis
a) Microbiologically confirmed
Abnormal CXR (new or progressive infiltrate), high clinical suspicion, clinical
pulmonary infection score (CPIS) ≥6 , recovery of a likely pulmonary pathogen
via uncontaminated specimen in high concentration on semi-quantitative culture
(≥ 103 CFU if protected brush or ≥ 104 CFU if bronchoalveolar lavage), or positive
serology. CPIS system considers 6 elements graded from 0-2pts each including
tracheal secretions, radiographic infiltrates, fever, leukocytosis, PaO2/FiO2 ratio,
and microbiology
b) Probable
Abnormal CXR, high clinical suspicion (CPIS≥6), no microbiological or
serological confirmation (pathogens recovered below diagnostic threshold)
c) Possible
Abnormal CXR of uncertain cause, low to moderate clinical suspicion but positive
microbiological or serological criteria
3. Treatment–antibiotics and pulmonary toilet
B. Bloodstream
1. Clinical setting and Diagnosis
a) Primary
(1) Unknown origin
(a) Blood culture positive for a recognized pathogen OR
(b) 2 or more blood cultures positive for common skin contaminant AND
(c) Cultured organism is not related to an infection at another site
(2) Catheter-related (CRBSI)
(a) suspected if local signs (erythema, swelling, drainage) present at
insertion site
(b) positive culture of removed catheter tip or hub (≥ 15 CFU)
(c) positive blood cultures for same organism drawn simultaneously from
suspected catheter and peripheral blood and no other apparent source
other than the catheter. More likely if catheter blood cultures turn
positive 2 or more hours before peripheral sample or if ≥5:1 quantitative
ratio of catheter to peripheral blood cultures.
148
ASCCA Residents Guide to the ICU
C.
D.
E.
F.
b) Secondary
(1) related to an established infection at another site
(a) Blood culture positive for a recognized pathogen related to an infection
with the same organism at another site
(b) Organisms – GPCs, fungi, GNRs
(2) Treatment
(a) Antibiotics
(b) Removal of catheter if suspected as source
Infective Endocarditis
1. Diagnostic Criteria
a) Modified Duke
2. Organisms – most frequently S. aureus, but B-hemolytic strep, enterococci. GNRs
and candida also seen
3. Treatment – Antibiotics
Intra-abdominal
1. Clinical Setting and Classification
a) Primary peritonitis – (spontaneous bacterial peritonitis) infection of peritoneal
fluid in the absence of gastrointestinal perforation, abscess or other localized GI
tract infection
b) Secondary peritonitis – Infection of the peritoneal space following perforation,
abscess, ischemic necrosis or penetrating injury of the abdomen
c) Tertiary peritonitis – persistent intra-abdominal inflammation related to
nosocomial pathogens after an episode of secondary peritonitis
2. Organisms – GNR, anaerobes, enterococci, occasionally fungi
3. Diagnosis – history of intraabdominal operation or trauma, physical examination, CT
scanning or ultrasound
4. Treatment–drainage (operative or percutaneous), antibiotics
Urinary Tract
1. Clinical Setting
a) Catheter-associated Urinary Tract Infection
(1) most common nosocomial infection (up to 50% of patients with indwelling
catheter ≥ 5 days develop bacteriuria or candiduria)
(2) silent catheter-associated bacteriuria represents a significant reservoir for
resistant organisms
(3) rarely symptomatic and rarely a source of severe sepsis
(4) urinalysis can help discern colonization from infection, but this can be difficult
(5) ≥ 103 CFU on culture (note this is different than routine outpatient UTI)
b) Urosepsis
(1) Usually community acquired in association with pyelonephritis
(2) My be associated with obstruction or instrumentation of the urinary tract
2. Organisms – enteric GNRs, enterococcus, pseudomonas, candida
3. Diagnosis - culture threshold for “infection” is unclear – but > 105 cfu/mL is associated
with infection
4. Treatment – removal of catheter if possible, catheter exchange for bacteriuria,
antibiotics for infection, decompression for obstructive etiology
Skin and Soft Tissue
1. Clinical Setting
a) Surgical Site Infections
b) Cellulitis
c) Necrotizing Fasciitis
d) Abscesses
2. Organisms – Most are polymicrobial, GPCs, anaerobes, GNRs
3. Diagnosis
a) Clinical exam
b) Imaging (soft tissue stranding, fluid collections, gas)
c) Gram Stain and Culture of infected material
4. Treatment
a) Drainage and debridement if indicated
149
ASCCA Residents Guide to the ICU
b) Antibiotics
G. Central Nervous system Infections
1. Clinical Setting
a) Meningitis
b) Encephalitis
c) Brain Abscess
2. Organisms
a) Meningitis - Pneumococcus, Neisseria meningitides, Haemophilus influenzae,
Listeria, Cryptococcus
b) Encephalitis – Herpes Simplex Virus, Varicella Zoster Virus
3. Diagnosis
a) Cerebrospinal Fluid Analysis
b) Gram Stain
c) Antigen and antibody testing
4. Treatment – Antibiotics, add dexamethasone for adult patients with pneumococcal
meningitis
H. Septic thrombophlebitis
1. Organisms–GPC, GNR
2. Diagnosis–history, physical examination, culture
3. Treatment–remove catheter, antibiotics, anti-coagulation if in deep vein, occasionally
excision or lysis
I. Sinusitis
1. Organisms–GNR, GPC
2. Diagnosis–physical examination, CT scan, aspiration
3. Treatment–remove or relocate nasoenteric or nasotracheal tubes, decongestants,
antibiotics, rarely drainage
IV. Considerations for the prevention of antibiotic resistance
A. Infection Control Practices
1. Hand washing – Use of alcohol based hand sanitizer enhances compliance but some
organisms (e.g. C. difficile) are resistant and mandate use of soap and water when
providing care for affected patients
2. Standard Precautions
3. Full barrier techniques for inserting vascular devices
4. Ventilator “Bundles” for prevention of VAP
B. Rational Use of Antibiotics
1. Effective Therapy
2. Appropriate Duration
3. De-escalation when appropriate
4. Antibiotic Utilization Programs
C. Mechanisms of Antibiotic Resistance
1. Genetic Mutation
2. Selection
3. Genetic Exchange
a) transformation
b) conjugation
c) transduction
4. Examples
a) Production of inactivating enzymes
(1) ОІ-lactamase and extended-spectrum ОІ-lactamase
b) Efflux pumps
c) Altered cell wall synthesis
d) Altered binding proteins
e) Altered channel proteins
V. Special Considerations:
Emerging organisms and changing patterns of disease provide considerable challenges in
diagnosis, management and prevention of several infectious disease processes. Cases may
150
ASCCA Residents Guide to the ICU
be sporadic but have the potential to occur as endemic or epidemic outbreaks. The ability to
provide sufficient critical care resources figures prominently in such events. Mass casualty
scenarios and acts of bioterrorism also have the potential to overwhelm healthcare
infrastructure and consume scarce resources.
A. Emerging and Evolving Infections
1. Viral Diseases
a) Influenza
(1) Seasonal
(2) Influenza A (Avian)
b) West Nile Virus
c) HIV
2. Bacterial Diseases
a) Methicillin Resistant Staphylococcus Aureus (MRSA)
b) Vancomycin Resistant Enterococcus (VRE)
c) Multi-drug Resistant GNRs
(1) Acinetobacter
(2) E. coli
(3) Klebsiella
(4) Enterobacter
d) Clostridium Difficile Colitis
(1) Increasing frequency and severity of infection
(2) Increasing rates of treatment failure
(3) Diagnosis based on screening for presence of antigen (sensitive but not
specific) then confirming presence of toxin by tissue culture or ELISA.
(4) Treatment – Stop offending antibiotics if able. Oral vancomycin or
metronidazole. Metronidazole can be used IV for patients unable to take po.
IV vancomycin is not effective.
3. Fungal Diseases
a) Increasing Fluconazole resistance
(1) Non-albicans species of Candida
b) Molds
B. Bioterrorism
1. Preparedness
a) Familiarity with Disaster/Mass Casualty Plan is important
b) Local Incident Command System
c) National Disaster Medical System
2. Familiarity with possible presentation, precautions and treatment for likely organisms
a) Effective weaponization is difficult but feasible
b) High priority agents are characterized by easy dissemination, person to person
transmission, lethality and impact on public health system. They include:
(1) Smallpox
(2) Anthrax
(3) Plague
(4) Botulism
(5) Tularemia
(6) Hemorrhagic Fever Viruses
VI. Considerations in Dosing Antibiotics in Critical Illness
A. Pharmacokinetics
1. The volume of distribution for many agents is altered in disease states such as sepsis
and major burn injury where Vd can be greatly increased and underdosing may occur
2. Pharmacokinetic parameters may change frequently in critically ill patients.
Alterations in clearance for example, may create increased likelihood of toxicity
3. When possible, serum levels should be followed and dosing optimized
B. Pharmacodynamics
1. Aminoglycoside antibiotics exhibit concentration dependent killing where effective
killing of a susceptible organism is related to the maximal concentration of agent
achieved.
151
ASCCA Residents Guide to the ICU
2.
3.
4.
Beta-lactams and vancomycin exhibit time dependent killing where effective killing of
susceptible organisms is related to the time the organism is exposed to
concentrations of agent above the minimum inhibitory concentration (MIC).
Aminoglycosides exhibit a significant post-antibiotic effect where bacterial growth
remains suppressed well after serum levels fall below the MIC.
These factors influence decisions in dosing that maximize efficacy while limiting
toxicity
VII. Antibiotics commonly used in the intensive care unit
A. Aztreonam
1. Spectrum–GNR (generally somewhat less broad than aminoglycosides)
2. Toxicity–minimal cross-reactivity in penicillin allergic patients
B. Cephalosporins
1. Spectrum:
a) First generation (cephalexin, cefazolin)
(1) GPC, some GNR
b) Second generation (cefoxitin, cefuroxime)
(1) GPC, many GNR, also:
(a) Cefoxitin–anaerobes including B. frag.
(b) Cefuroxime–H. influenzae
c) Third generation (cefotaxime, ceftriaxone, ceftazidime)
(1) Most gram negative bacilli, also:
(a) Ceftazidime– some P. aeruginosa
d) Fourth generation (cefepime)
(1) GNR including P. aeruginosa, GPC
2. Toxicity–10% cross-reactivity in penicillin allergic patients
C. Quinolones (ciprofloxacin, levofloxacin, moxifloxacin)
1. Spectrum–GNR, GPC (some Streptococcus spp. resistant)
2. Toxicity–rare, impairs cartilage growth
3. Moxifloxacin undergoes hepatic metabolism and biliary excretion – does not
concentrate in urine and is not effective for UTI
D. Clindamycin
1. Spectrum–anaerobes, GPC
2. Toxicity–C. difficile colitis
E. Colisitin
1. Old antibiotic (polymyxin) finding resurgent utility in some multi-drug resistant
infections (usually acinetobacter or pseudomonas).
2. Spectrum – GNRs including P. aeruginosa, A. baumanii, E coli, some enterobacter
spp, H. flu, B. pertussis, Legionella, Salmonella, Shigella, and Stenotrophomonas.
However, Burkhoderia cepacia, Serratia, M. catarrhalis, Proteus, Providentia and
Morganella are resistant.
3. Toxicity – Renal failure and neurotoxic effects including peripheral neuropathy
F. Daptomycin (cyclic lipopeptide)
1. Spectrum - GPCs
2. Toxicity – myopathy (monitor CPK), peripheral neuropathy possible
3. May be preferred for MRSA endocarditis
G. Macrolides (erythromycin, clarithromycin, azithromycin)
1. Spectrum–GPC, H. influenzae, Mycoplasma spp., Chlamydia spp., also:
Clarithromycin–Hl. pylori
2. Toxicity–nausea, vomiting, diarrhea
H. Aminoglycosides (gentamicin, tobramycin, amikacin)
1. Spectrum–GNR, synergy against some GPC
2. Toxicity–nephrotoxicity, ototoxicity
I. Imipenem, meropenem
1. Spectrum–GPC, GNR including P. aeruginosa, anaerobes
2. Toxicity–Seizures (imipenem), minimal cross-reactivity in penicillin allergic patients
J. Linezolid
1. Spectrum – GPC
152
ASCCA Residents Guide to the ICU
K.
L.
M.
N.
O.
P.
Q.
R.
S.
2. Toxicity – Thrombocytopenia
3. May be preferred for MRSA pneumonia
Metronidazole
1. Spectrum – anaerobes, E. histolytica (amebic abscess), Trichomonas spp., C. difficile
2. Toxicity–nausea and vomiting (esp. when taken with alcohol)
Penicillins
1. Spectrum:
a) Penicillin G/V–Some GPC, Clostridia spp., syphilis, mouth anaerobes
b) Nafcillin, dicloxacillin – GPC (not MRSA or Enterococcus spp.)
c) Ampicillin, amoxicillin–GPC (inc. some Enterococcus spp.) some GNR
d) Piperacillin, ticarcillin, mezlocillin – GPC (inc. some Enterococcus spp.), GNR, P.
aeruginosa, anaerobes
e) Combinations with β-lactamase inhibitors–similar to parent drugs, but with βlactamase resistance, increased anaerobic activity e.g. Piperacillin/Tazobactam
2. Toxicity–allergy, mild (rash) to severe (anaphylaxis)
Quinupristin/Dalfopristin
1. Spectrum – GPC, no activity against E. faecalis
2. Toxicity – Arthralgia/Myalgia, hyperbilirubinemia, phlebitis
Tigecycline
1. Spectrum - GPCs and GNRs many pseudomonas isolates are resistant
2. Toxicity - GI
Trimethoprim/sulfamethoxazole (TMP/SMX)
1. Spectrum–some GPC, some GNR, Chlamydia spp., Pneumocystis carinii, Legionella
spp., S. maltophilia
2. Toxicity–rash, toxic epidermal necrolysis (Stevens-Johnson syndrome)
Vancomycin
1. Spectrum–GPC (including MRSA), Clostridia spp. (including C. difficile}
2. Toxicity–face and neck rash (red man syndrome), ototoxic, nephrotoxic
Amphotericin B/Ampotericin Lipid complex/Liposomal Amphotericin
1. Spectrum–fungi
2. Toxicity–nephrotoxicity, hypokalemia, hypersensitivity
3. Toxicity decreased with lipid formulations
Echinocandins – Caspofungin, Micafungin
1. Spectrum – Candida spp.
2. Toxicity – well tolerated, dosage adjustment in hepatic failure, not concentrated in
urine
Azoles – Fluconazole, voriconazole
1. Spectrum–C. albicans, Cryptococcus neoformans, increasing resistance among nonalbicans Candida spp.
2. Toxicity–Visual disturbances, rash, fever and elevated liver enzymes can occur with
voriconazole.
3. Voriconazole preferred for Aspergillus spp.
This chapter is a revision of the original chapter authored by Robert G. Sawyer, M.D.
READING LIST:
1. American Thoracic Society, Infectious Diseases Society of America. Guidelines for the management of
adults with hospital-acquired, ventilator-associated, and healthcare-associated pneumonia. Am J Respir
Crit Care Med. 2005 Feb 15;171(4):388-416.
2. Bagshaw SM, Laupland KB. Epidemiology of intensive care unit-acquired urinary tract infections. Curr
Opin Infect Dis. 2006 Feb;19(1):67-71.
3. Bartlett JG. Narrative review: The new epidemic of clostridium difficile-associated enteric disease. Ann
Intern Med. 2006 Nov 21;145(10):758-64.
4. Bashore TM, Cabell C, Fowler V,Jr. Update on infective endocarditis. Curr Probl Cardiol. 2006 Apr;31(4):
274-352.
153
ASCCA Residents Guide to the ICU
5. Calandra T, Cohen J, International Sepsis Forum Definition of Infection in the ICU Consensus Conference.
The international sepsis forum consensus conference on definitions of infection in the intensive care unit.
Crit Care Med. 2005 Jul;33(7):1538-48.
6. Chastre J, Fagon JY. Ventilator-associated pneumonia. Am J Respir Crit Care Med. 2002 Apr 1;165(7):
867-903.
7. Karwa M, Currie B, Kvetan V. Bioterrorism: Preparing for the impossible or the improbable. Crit Care
Med. 2005 Jan;33(1 Suppl):S75-95.
8. Kollef MH. Bench-to-bedside review: Antimicrobial utilization strategies aimed at preventing the
emergence of bacterial resistance in the intensive care unit. Crit Care. 2005 Oct 5;9(5):459-64.
9. Morris A, Masur H, Huang L. Current issues in critical care of the human immunodeficiency virusinfected patient. Crit Care Med. 2006 Jan;34(1):42-9.
10. O'Grady NP, Alexander M, Dellinger EP, Gerberding JL, Heard SO, Maki DG, et al. Guidelines for the
prevention of intravascular catheter-related infections. centers for disease control and prevention. MMWR
Recomm Rep. 2002 Aug 9;51(RR-10):1-29.
11. O'Grady NP, Barie PS, Bartlett JG, Bleck T, Carroll K, Kalil AC, et al. Guidelines for evaluation of new
fever in critically ill adult patients: 2008 update from the american college of critical care medicine and the
infectious diseases society of america. Crit Care Med. 2008 Apr;36(4):1330-49.
12. Paterson DL, Bonomo RA. Extended-spectrum beta-lactamases: A clinical update. Clin Microbiol Rev.
2005 Oct;18(4):657-86.
13. Piddock LJ. Clinically relevant chromosomally encoded multidrug resistance efflux pumps in bacteria.
Clin Microbiol Rev. 2006 Apr;19(2):382-402.
14. Roberts JA, Lipman J. Antibacterial dosing in intensive care: Pharmacokinetics, degree of disease and
pharmacodynamics of sepsis. Clin Pharmacokinet. 2006;45(8):755-73.
15. Tenover FC. Mechanisms of antimicrobial resistance in bacteria. Am J Infect Control. 2006 Jun;34(5 Suppl
1):S3,10; discussion S64-73.
QUESTIONS:
31.1.
A.
B.
C.
D.
Which is not an accepted risk factor for infections in the intensive care unit?
Gender
Airway intubation
Steroid use
Underlying chronic disease
31.2.
How long does it take most patients to become colonized with nosocomial flora in the
intensive care unit?
6 hours
72 hours
1 week
2 weeks
A.
B.
C.
D.
31.3.
A.
B.
C.
D.
Which are the two most common organisms causing nosocomial pneumonia?
E. coli and B. fragilis
C. albicans and E. aerogenes
S. aureus and P. aeruginosa
S. pneumoniae and H. influenzae
31.4.
A.
B.
C.
D.
What minimal level of bacteriuria is indicative of infection?
10 colony forming units (CFU)/ ml
100 CFU/ ml
10,000 CFU/ ml
100,000 CFU / ml
154
ASCCA Residents Guide to the ICU
31.5.
A.
B.
C.
D.
What is the first line treatment of an infected intravascular catheter?
Local line care
Removal of catheter
Antibiotics
Antiseptic through the catheter
31.6.
A.
B.
C.
D.
Which antibacterial does not have significant anti-Pseudomonas activity?
Tobramycin
Imipenem
Cefepime
Tigecycline
31.7.
What measures have been demonstrated to decrease the incidence of catheterrelated blood stream infections (CRBSI)?
Use of prophylactic antibiotics prior to insertion of the catheter
Routine use of full barrier precautions during vascular access procedures
Use of any dressing after placement
Routine rewiring of central venous catheters every 5 days
A.
B.
C.
D.
31.8.
A.
B.
C.
D.
What is the antifungal of choice in a septic patient with Aspergillosis?
Itraconazole
Fluconazole
Voriconazole
Flucytosine
31.9.
A.
B.
C.
D.
What is the most common serious side-effect of imipenem therapy?
Seizures
Neutropenia
Renal failure
Thrombocytopenia
155
ASCCA Residents Guide to the ICU
32. Antibiotic Prophylaxis
Ronald W. Pauldine, M.D.
A 56-year-old man admitted to the Medical Intensive Care Unit for an exacerbation of COPD develops a
rigid abdomen and has a plain film which demonstrates free intraperitoneal air. A perforated duodenal
ulcer is suspected. He is taken to the operating room where the diagnosis is confirmed and he is treated
with an omental patch.
Prophylactic antibiotic administration is designed to decrease postoperative morbidity, shorten
hospitalization, and reduce costs associated with infection, particularly surgical site infection. Surgical
site infection is defined as infection occurring in a surgical incision. These infections should not be
confused with infections occurring in traumatic wounds. Surgical site infections complicates up to 3% of
all surgical procedures and can be as high as 20% in the setting of emergency intra-abdominal surgery.
Antibiotic administration should result in effective drug levels during the procedure and for a short time
afterwards. Patients receiving pre-operative antibiotics generally do not require additional coverage for
endocarditis prophylaxis.
I.
Antibiotic prophylaxis: General considerations
A. Unlike treatment, refers to therapy designed to prevent an infection before it occurs
B. Intended to prevent wound infections, though also may be useful in preventing deep infections,
such as intraabdominal abscesses
C. Serum antibiotic peak should be at the time of incision; thus, antibiotics should be given close to
the time of induction (not “on call”)
1. Antibiotics should be administered within 60 minutes of skin incision
2. Infusion of vancomycin or fluoroquinolone antibiotics should begin 60-120 minutes before
skin incision (infusion should be complete within 1 hour of skin incision)
D. Antibiotic given (if any) based on wound class (see below) and location
1. Should be directed against the most likely culprit organisms
2. Should be given for less than 24 hours unless obvious signs of infection (dirty wounds)
E. Redosing is based on blood loss and effective antibiotic half life
1. For procedures over 4 hours in duration or procedures involving major blood loss redosing
should occur every 1-2 half-lives of the chosen antibiotic in patients with normal renal
function.
II. Wound classification and average infection rate for all pooled procedures (in parentheses) Data from
National Nosocomial Infections Surveillance System (NNIS).
A. Clean (2.1%): elective, no violation of colonized or inflamed sites, good sterile technique,
procedure restricted to integumentary and musculoskeletal soft tissues
B. Clean-contaminated (3.3%): procedures where a hollow viscus has been opened under controlled
circumstances (such as elective resection of the colon)
C. Contaminated (6.4%): procedures where bacteria have been in contact with a
1. normally sterile body cavity, but for an insufficient period of time for
2. infection to become established (such as penetrating abdominal trauma or
3. traumatic wounds <4 hours old
D. Dirty (7.1%): procedure performed to address established infection
1. Traumatic wounds >4 hours old or with foreign body, devitalized tissue, or fecal contamination
2. Perforated viscus
3. Pus encountered
E. Risk of infection increases in relation to number of risk factors present
1. Wound Classification
2. American Society of Anesthesiologist (ASA) Physical Class 3 or higher
3. Prolonged operative time (>75th percentile for a given procedure)
4. Type of Procedure
5. Laparoscopic approaches decrease risk
6. Host factors likely to contribute but not included in NNIS risk index
156
ASCCA Residents Guide to the ICU
a)
b)
c)
d)
e)
f)
Diabetes Mellitus
Obesity
Malnutrition
Dyslipidemia
Tobacco use
Steroid use
III. Common bacteria by operative site* (GPC = Gram positive cocci, GNR = Gram negative rods):
Site
Organisms
Head and Neck
GPC, enteric GNR, anaerobes
Esophagus, stomach, duodenum
Biliary tree
Sterile (unless obstructed or acid-suppressed), GPC, enteric
GNR
GNR, enterococci, clostridia
Small bowel
Sterile (unless obstructed), GPC, enteric GNR
Large bowel
GNR, anaerobes, enterococci
Genitourinary system
GNR, enterococci
Gynecologic and Obstetric
Enteric GNR, anaerobes, Gp B Strep, enterococci
Thoracic (Non-cardiac)
GPC, enteric GNR
Skin (includes coverage for Neuro,
Orthopedic, and Vascular Surgery)
GPC
*Note: patients who have been hospitalized for more than 48 hours have a high rate of colonization at
all sites with nosocomial organisms, primarily GNRs.
IV.
Commonly used antibiotics, general spectrum, and intraoperative dosing intervals:
Antibiotic
Spectrum
Dosing interval
Cefazolin
Skin flora
Q 2-5 hrs
Cefoxitin
Skin and colonic flora (inc. anaerobes)
Q 2-3 hrs
Ampicillin/Sulbactam
Skin flora, Enterococcus spp., anaerobes
Q 2-4 hrs
Ciprofloxacin
GNR, Some Staphylococcus spp.
Q 4-8 hrs
Gentamicin
GI, GU flora (GNR)
Q 6-8 hrs
Clindamycin
Skin flora, anaerobes
Q 3-6 hrs
Metronidazole
Anaerobes
Q 6-8 hrs
Vancomycin
Skin Flora (all GPC)
Q 6-12 hrs
V.
General Recommendations for Prophylaxis
A. Cardiac Surgery
1. Careful attention to redosing in procedures lasting > 400 min
B. GI Surgery
1. Esophageal procedures with obstruction
2. Gastroduodenal procedures in setting of decreased motility or gastric acidity (obstruction,
hemorrhage, malignancy, morbid obesity, H2-blocker or PPI therapy)
3. Biliary procedures with risk factors including age >70, acute cholecystitis, obstruction or
known common duct stones
4. Colorectal surgery
5. Perforated appendicitis (Treatment)
C. GU Surgery
157
ASCCA Residents Guide to the ICU
Usually not needed if urine is sterile
Beneficial for TURP, transrectal biopsy, and procedures involving placement of prosthetic
material
Gynecology and Obstetrics
1. Hysterectomy
2. Cesarean section (after cord clamp or before skin incision)
Head and Neck Surgery
1. Procedures involving incision of oral or pharyngeal mucosa
Neurosurgery
1. Craniotomy
2. Spine
a) Fusion or placement of foreign material
Orthopedic
1. Total Joint Replacement
2. Operative repair of closed fractures
3. Ensure administration prior to inflation of tourniquet
Thoracic (non-cardiac)
1. Lung resection
Vascular Surgery
1. Abdominal Aorta Repair
2. Lower extremity revascularization involving incision in the groin
3. Amputation of ischemic lower extremity
4. Placement of prosthetic grafts
1.
2.
D.
E.
F.
G.
H.
I.
This chapter is a revision of the original chapter authored by Robert G. Sawyer, M.D.
READING LIST:
1. Antimicrobial prophylaxis for surgery. Treat Guidel Med Lett. 2006 Dec;4(52):83-8.
2. Barie PS, Eachempati SR. Surgical site infections. Surg Clin North Am. 2005 Dec;85(6):1115,35, viii-ix.
3. Burton MJ, Geraci SA. Infective endocarditis prevention: Update on 2007 guidelines. Am J Med. 2008 Jun;121(6):
484-6.
4. National Nosocomial Infections Surveillance System. National nosocomial infections surveillance (NNIS) system
report, data summary from January 1992 through June 2004, issued october 2004. Am J Infect Control. 2004 Dec;
32(8):470-85.
QUESTIONS:
32.1.
A.
B.
C.
D.
When should prophylactic antibiotics be first administered?
The night before operation
On call to the operating room
Within one hour of skin incision
In the recovery room
32.2.
A.
B.
C.
D.
Under most circumstances, for how long should prophylactic antibiotics be administered?
≤24 hours
3 days
7 days
Until skin staples are removed
32.3.
A.
B.
C.
D.
What is the expected wound infection rate for a clean wound?
0%
1-5%
5-10%
10-20%
158
ASCCA Residents Guide to the ICU
32.4.
A.
B.
C.
D.
What is the wound classification for a gunshot wound to the left colon?
Clean
Clean contaminated
Contaminated
Dirty
32.5.
A.
B.
C.
D.
What are the predominant bacteria in the large intestine?
Gram positive cocci
Gram negative rods and anaerobes
Yeast
Gram positive rods
32.6.
How often should cefazolin be redosed during an operation in the absence of severe blood
loss?
Q 30 mins
Q 2-4 hours
Q 6 hours
Q 8 hours
A.
B.
C.
D.
32.7.
A.
B.
C.
D.
For which operation is there little evidence to support the use of prophylactic antibiotics?
Thyroidectomy
Appendectomy
Ventral herniorraphy with mesh
Right hemicolectomy
159
ASCCA Residents Guide to the ICU
33. Management of the Immunocompromised Patient
Zdravka D. Zafirova, M.D
A 48-year-old male patient with end-stage liver disease due to hepatitis C has undergone orthotopic liver
transplant. On postoperative day 1, he is intubated and on mechanical ventilation with FiO2 0.8. His ABG
is pH 7.23, pCO2 42 mm Hg, PaO2 58 mm Hg. His temperature is 38.8 degrees C, his blood pressure is
87/43 mm Hg, heart rate 109/min, and pulmonary artery pressure is 51/26 mm Hg. Management
considerations: 1) Differential diagnosis, evaluation and treatment of respiratory failure; 2) Differential
diagnosis, evaluation and therapy of shock; 3) Management of infection issues; 4) Immunosuppression
management.
Medical progress in the areas of transplant medicine, oncology and management of congenital and
acquired immunodeficiency syndromes has lead to increasing numbers of immunocompromised patients
requiring hospital and ICU care. These patients pose unique medical and social challenges to the health
care team as a result of altered organ function and immune system response to illness, heightened
infection risk and effects of immunosuppressive therapy. The management of these patients is based on
an understanding of the mechanisms of their pathology and use of diagnostic and therapeutic
interventions adapted to the unique presentations of critical illness in the immunocompromised host. A
multidisciplinary approach to the care of these patients with collaboration among providers with a wide
range of expertise is key to improvement of outcomes.
I.
Components of the immune system
A. Innate immunity – mechanical, chemical and biological barriers
B. Cellular subsets – NK cells, phagocytes, macrophages, T cells, B cells
C. Immunoglobulins – structure and function of immunoglobulin molecules (i.e., IgA, IgD, IgE, IgG,
IgM)
D. Cytokines – types and role
E. Complement
II. Disorders of the immune system and associated pathogens
Aggressive infections with opportunistic pathogens, recurrent infections, failure to
clear infection.
A. Disorders of the humoral immunity
1. Congenital – agammaglobulinemia, selective immunoglobulin deficiencies, hyperIgM
syndrome, common variable immune deficiency
2. Chemotherapy – Azathioprine, Cyclophosphamide, Methotrexate
3. Chronic lymphocytic leukemia
4. Associated organisms:
a) Enteric gram-negative bacilli
b) Haemophilus influenzae
c) Neisseria meningitidis
d) Streptococcus pneumoniae
e) Staphylococcus spp.
f) Mycoplasma pneumoniae
g) Giardia
h) Enterovirus
B. Disorders of cell-mediated immunity
1. Congenital - DiGeorge syndrome
2. Immunosuppression and chemotherapy
3. Lymphoma
4. Protein-calorie malnutrition
5. Viral illnesses:
a) Cytomegalovirus (CMV)
b) Epstein-Barr virus (EBV)
c) Human immunodeficiency virus (HIV)
6. Associated organisms:
160
ASCCA Residents Guide to the ICU
a) Viral - Adenovirus, EBV, CMV, varicella, Herpes simplex virus (HSV)
b) Fungi - Candida spp., Aspergillus spp., Nocardia, Coccidiomycosis
c) Cryptococcus neoformans
d) Histoplasmosis
e) Legionella
f) Listeria monocytogenes
g) Pneumocystis carinii (PCP)
h) Toxoplasma gondii
i) Mycobacteria spp.
j) Salmonella spp.
C. Disorders of complement
Complement deficiencies may be associated with pyogenic infections with encapsulated
organisms; deficiencies of the complement regulatory proteins are associated with autoimmune
diseases.
1. Congenital deficiency
2. Multiple myeloma
3. Systemic lupus erythematosus
4. Associated organisms:
a) Neisseria gonorrheae
b) Neisseria meningitidis
c) Streptococcus
D. Disorders associated with neutropenia and phagocytic function
1. Congenital – chronic granulomatous disease, myeloperoxidase deficiency, chemotactic
disorders
2. Autoimmune
3. Drug induced – bone marrow suppression, antibody and complement-mediated destruction
a) Chemotherapy – Adriamycin, ARA-C, Cyclophosphamide
b) Other medications - procainamide, antithyroid drugs, sulfasalazine, phenothiazine,
semisynthetic penicillins, nonsteroidal anti-inflammatory agents, aminopyrine derivatives,
benzodiazepines, barbiturates, gold compounds, sulfonamides
4. Infection - varicella, measles, rubella, CMV, EBV, hepatitis, HIV, Parvovirus, influenza
5. Leukemia, lymphoproliferative disorders
6. Total body radiation
7. Associated organisms:
a) Fungi - Aspergillus spp., Mucor, Candida spp.
b) Enteric gram-negative bacilli (Klebsiella)
c) Pseudomonas aeruginosa
d) Staphylococcus aureus
e) Burkholderia cepacia
f) Nocardia
g) Mycobacterium (including atypical)
h) Serratia
E. Combined immunodeficiency
1. Congenital - severe combined immune deficiency, Wiskott-Aldrich, Ataxia-telangiectasia
2. Acquired – lymphoproliferative disorders, chemotherapy, infection (EBV, CMV, HIV)
III. Infection control for immunocompromised patients in ICU
A. Preoperative evaluation and treatment of infection
1. Evaluation of sites for chronic, latent infection/contamination – sinus, oral cavity
2. Treatment and prophylaxis – overt clinical infection, latent viral and fungal infection
B. Proper hand washing techniques
C. Isolation precautions/requirements
1. Category-specific vs. disease-specific
2. Use of barrier protection
3. Use of single, HEPA-filtered, positive-air-pressure, sealed rooms
D. Universal healthcare worker precautions
E. Environmental/endemic nosocomial infection surveillance
161
ASCCA Residents Guide to the ICU
IV. Antimicrobial therapy principles
A. Drug classes
1. Antibacterial: penicillins, cephalosporins, beta-lactam inhibitor combinations, monobactams,
aminoglycosides, macrolides/ketolides, fluoroquinolones, lincosamides, trimethoprimsulfamethoxazole, glycopeptides, streptogramins, oxazolidinones, lipopeptides, glycylcyclines
2. Antifungal: Amphotericin B, Fluconazole, Itraconazole
3. Antiparasitic: Pentamidine, Sulfadiazine/pyrethiamine/folinic acid,
4. Antiviral: Acyclovir and Gancyclovir
B. Modes of antimicrobial therapy
1. Therapeutic – treatment of established infection
2. Prophylactic – administration to entire patient population to prevent infection
3. Empiric – treatment of suspected infection with fixed antimicrobial regimen
4. Preemptive – administration to select population at risk based on epidemiologic, clinical or
laboratory markers
C. Antimicrobial prophylaxis
1. Perioperative – based on surgical site, presence of preexisting pathogen
2. Post transplant – based on timing, preexisting pathogen in donor or recipient, risk for
opportunistic organism
D. Management of drug-resistant pathogens
1. Methicillin resistant Staphylococcus aureus (MRSA)
2. Vancomycin resistant enterococcus (VRE)
3. Multidrug resistant Pseudomonas aeruginosa
4. Multidrug resistant enteric Gram-negative pathogens
5. Pathogens with limited antimicrobial drug availability – Stenotrophomonas, Acinetobacter
E. Immunizations
V. Bone marrow transplantation (BMT)
A. Combined risks of ablative chemotherapy and suppression of cell-mediated/humoral immunity
B. Pre-bone marrow engraftment infections
C. Early, post-marrow engraftment complications (i.e., 0-to-30 days)
1. Therapy-related toxicity – mucositis, cystitis, interstitial pneumonitis, alveolar hemorrhage,
veno-occlusive disease, myocarditis
2. Graft failure
3. Infection (barrier injury, neutropenia) - Gram-negative bacteremia, Gram-positive bacteremia,
fungal infections, HSV viremia, respiratory virus
D. Intermediate, post-marrow engraftment complications (i.e., 30-to-90 days)
1. Acute graft-versus-host disease (GVHD)
2. Infections (T cell dysfunction, hypogammaglobulinemia) - Aspergillus spp., Candida spp.,
Pneumocystis carinii, CMV
E. Late, postmarrow engraftment complications (i.e., >90 days)
1. Chronic GVHD
2. Hypothyroidism
3. Malignancy - relapse or secondary
4. Infection (slow T cell reconstitution, GVHD) - Streptococcus pneumoniae, Haemophilus
influenzae, PCP, CMV, Varicella-zoster
F. Therapeutic interventions
1. Immunosuppression for GVHD
2. Infection prophylaxis and therapy
3. Supportive therapy and management of complications
VI. Issues involving solid organ transplantation
A. Preoperative organ system dysfunction
B. General strategies for immunosuppression
1. Induction of immunosuppression
2. Maintenance of immunosuppression
C. Diagnosis and therapy of acute organ rejection
D. Diagnosis and therapy for chronic or recurrent organ rejection
E. Infection
162
ASCCA Residents Guide to the ICU
Early (0-30 days)
a) site – surgical wound, respiratory, bacteremia/vascular access, urinary
b) source – preexisting recipient pathogen, donor-transmitted hepatitis, HIV, nosocomial
c) pathogens – conventional nosocomial pathogens, HSV, hepatitis B
2. Intermediate (30-180 days)
a) lingering pathogens
b) opportunistic infections – CMV, EBV, varicella-zoster, Pneumocystis, Aspergillus,
Toxoplasma, Listeria
3. Late (>120-180 days)
a) Community-acquired respiratory viral and bacterial infections
b) Hepatitis B, C
c) CMV
d) Listeria
e) Cryptococcus
F. Complications of chronic immunosuppression
1. Bone marrow suppression, immunomodulation
2. Organ system toxicity – renal failure, neurotoxicity, hypertension, metabolic abnormalities,
diabetes mellitus, hyperlipidemia, osteoporosis
3. Malignancy
G. Modification of immunosuppression regimens in transplant recipients with active infections
1.
VII. Immunosuppressive agents for transplant recipients
A. Alkylating agents-chlorambucil, cyclophosphamide
B. Corticosteroids
C. Antiproliferative agents – Azathioprine, Mycophenolate
D. mTOR inhibitors – sirolimus, everolimus
E. Calcineurin Inhibitors——cyclosporine, tacrolimus
F. Antibodies - monoclonal (OKT-3), polyclonal antibodies (thymoglobulin), Anti–Interleukin-2
Receptor
VIII. HIV and the acquired immunodeficiency syndrome (AIDS)
A. Epidemiology of the disease
1. Demographics, biology, and classification of HIV infection
2. Acquisition and viral transmission
3. Risks to the health care worker. Post exposure prophylaxis.
4. Prevention and universal precautions
5. Natural history of HIV infection
B. Diagnosis of HIV infection and AIDS
1. History and physical examination
2. Definition of AIDS and diagnosis using serology testing.
3. Laboratory evaluation
C. Clinical manifestations and co-existing diseases
1. Acute Retroviral Syndrome
2. Chronic HIV infection
3. HIV wasting syndrome
4. Cutaneous complications
5. Central and peripheral neuromuscular system complications
6. Ocular complications
7. Cardiovascular complications
8. Pulmonary complications
9. Pulmonary hypertension
10. Gastrointestinal complications
a) Oropharyngeal manifestations
b) Gastrointestinal manifestations
c) Lower gastrointestinal manifestations
d) Hepatobiliary and pancreatic disease
11. Hematologic complications
12. Renal complications
163
ASCCA Residents Guide to the ICU
13. Endocrine complications
14. Autoimmune and rheumatic complications
15. Psychiatry complications
D. Management and therapeutic strategies
1. Asymptomatic HIV-positive individuals
2. Highly-active antiretroviral therapy (HAART)
a) Resistance testing and initiation of therapy
b) Nucleoside reverse transcriptase inhibitors (NRTIs), Non-nucleoside reverse
transcriptase inhibitors (NNRTIs), Protease inhibitors (PIs), Fusion inhibitors, Entry
inhibitors, Integrase inhibitors.
c) Treatment failure
d) Adverse effects
3. Bacterial infections:
a) CNS infections
b) Gastrointestinal tract infections
c) Mycobacterial infections
d) Respiratory tract infections
e) Syphilis
f) Other organ systems
4. Fungal infections:
a) Superficial mycoses
b) Systemic mycoses
5. Parasitic infections:
a) Cryptosporidium spp.
b) Isospora belli
c) Pneumocystis jiroveci
d) Toxoplasma gondii
6. Viral infections:
a) Cytomegalovirus
b) Epstein-Barr virus
c) Herpes simplex virus I and II
d) Papovavirus, JC
e) Hepatitis B and C - infection and co-infection
7. AIDS-defining malignancies
a) Kaposi’s sarcoma
b) Non-Hodgkin’s Lymphoma
c) Hodgkin’s disease
d) Cervical cancer
8. Non-AIDS-defining malignancies
9. HIV infection in women and children
10. Perinatal transmission
11. Pain management
12. Prophylaxis of opportunistic infections
E. Legal and legislative issues
1. HIV education and prevention
2. Confidentiality issues
3. Anonymous/confidential counseling and testing
4. Voluntary vs. mandatory partner notification
This chapter is a revision of the original chapter authored by Hans W. Schweiger, M.D.
READING LIST:
1. Eggimann P, Pittet D. Infection control in the ICU. Chest 2001; 120:2059-93.
Excellent review of issues in infection control in the ICU.
164
ASCCA Residents Guide to the ICU
2. Masur H. Critically ill immunosuppressed host. In: Critical care Medicine (2nd ed) (Parillo J, Dellinger RP [eds.]).
St. Louis: Mosby, Inc. 2002, 1089-1108.
3. Kirkpatrick CH, Leach S. Transplantation Immunology - Clinical Aspects. JAMA, 1982;248:2727-2733.
Historically, an important review article that stressed the clinical aspects of transplant medicine.
4. Weisdorf DJ. Complications after Hematopoietic Stem Cell Transplantation. In Hoffman: Hematology: Basic
Principles and Practice (4th ed) (Hoffman R et al. [eds.]). Philadelphia: Elsevier, 2005, ch. 104.
5. Kusne S, Krystofiak S. Infection control issues after bone marrow transplantation. Curr Opin Infect Dis 2001;
14:427-31.
Excellent review of infectious disease issues in BMT.
6. Marty FM, Rubin RH. The prevention of infection post-transplant: the role of prophylaxis, preemptive and empiric
therapy. Transplant International 2006; 19:2-11.
7. Cunha B. Antimicrobial Therapy of Multidrug-Resistant Streptococcus pneumoniae, Vancomycin-Resistant
Enterococci, and Methicillin-Resistant Staphylococcus aureus. Med Clin N Am 2006; 90:1165–1182
8. Khardori N. Antibiotics-Past, Present, and Future. Med Clin N Am 2006; 90:1049–1076.
Review of current antibiotic therapy.
9. Lopez MM et al. Long-term problems related to immunosuppression. Transplant Immunology 2006; 17: 31–35.
10.Fischl MA, Richman DD, Grieco MH, et al. The Efficacy of Azidothymidine (AZT) in the Treatment of Patients
with AIDS and AIDS-Related Complex; A Double-Blind, Placebo-Controlled Trial. N Eng J Med 1987; 317:187-192.
A landmark article describing the first major breakthrough in the battle against the HIV infection.
11.Stebbing J et al. Where does HIV live? NEJM 2004; 350:1872-80.
Review of the current understanding of the mechanisms of HIV infection.
12.Bouza E et al. Fever of Unknown Origin in Solid Organ Transplant Recipients. Infect Dis Clin N Am 2007; 21:1033–
1054.
Up to date review of fever and infection issues in solid organ transplant.
13.Mason CM et al. Pulmonary Host Defenses and Factors Predisposing to Lung Infection. Clin Chest Med 2005; 26:11
– 17.
14.Kotloff RM. Noninfectious Pulmonary Complications of Liver, Heart and Kidney Transplantation. Clin Chest Med
2005; 26:623 – 629.
QUESTIONS:
33.1.
A.
B.
C.
D.
E.
33.2.
A.
B.
C.
D.
E.
33.3.
A.
B.
C.
D.
E.
Which of the following complications IS NOT seen in the early period after bone marrow
transplantion:
Interstitial pneumonitis
Hypothyroidism
Veno-occlusive disease
Graft failure
HSV infection
All of the following statements regarding infection disease control in immunosuppressed
patients are correct EXCEPT:
Use of HEPA filters and laminar flow ventilation helps to prevent infections
Avoidance of fresh flowers and fruits and vegetables that cannot be effectively washed is
recommended to prevent Pseudomonas and Aspergillus contamination
Dry mopping of floors leads to less particle dispersion compared to wet mopping
Nosocomial legionella infections are often result of contaminated drinking water
The oral cavity is a primary site of entry of viridans streptococci
In the first month after solid organ transplant, all of the following organisms are likely causes
of infection EXCEPT:
Aspergillus
Staphylococcus
Hepatitis
Clostridium
Herpes simplex
165
ASCCA Residents Guide to the ICU
33.4.
A.
B.
C.
D.
All of the following antimicrobial mechanisms are correct EXCEPT:
Aminoglycosides inhibit protein synthesis by irreversible binding to 30S unit of the ribosome
Macrolides inhibit transpeptidase, carboxypeptidase, and endopeptidase
Fluoroquinolones inhibit topoisomerase type II (DNA gyrase) and topoisomerase type IV
Oxazolidinones inhibit ribosomal protein synthesis by interfering with initiation complex
formation
E. Lipopeptides bind to the cell membrane in a calcium-dependent manner, depolarize the
bacterial membrane potential, and terminate bacterial DNA
33.5.
A.
B.
C.
D.
E.
All of the following principles regarding antibiotic therapy in immunocompromised patients are
correct EXCEPT:
In serious infections, in the absence of a microbiologic diagnosis, the most resistant of
possible pathogens should be treated
Use of bacteriostatic rather than bactericidal agents contributes to suboptimal therapy
Use of agents with high bioavailability, higher volume of distribution, and better tissue/
phagocyte penetration among the appropriate agents available for the infecting organisms is
preferred
When the infection is microbiologically documented, use the most broad-spectrum agent
possible is recommended
Use of sub-inhibitory concentration of antibiotics for even a short period is highly likely to
induce resistance
33.6.
A.
B.
C.
D.
All of the following regarding fever in post transplant patients is correct EXCEPT:
Fever is a frequent presentation of rejection in kidney transplant
Infection is the source of fever in more than 2/3 of the patients with liver transplant
Pneumonia is the most common infection in heart transplant
Non infection fever can be caused by anti-thymocyte globulin, everolimus, sirolimus,
antimicrobials and antihypertensive drugs
E. Fever enhances several host defense mechanisms (chemotaxis, phagocytosis, opsonization)
and may not need to be systematically eliminated
33.7.
A.
B.
C.
D.
E.
33.8.
A.
B.
C.
D.
E.
All of the statements about factors influencing the development of pulmonary infection are
correct EXCEPT:
Factors predisposing to colonization of the oropharynx with pathogenic gram-negative
bacteria include acidosis, azotemia, hypotension and preceding viral infection
Duration of endotracheal tube placement and the need for reintubation or tracheostomy are
risk factors for acquiring nosocomial pneumonia
Drugs that may adversely affect pulmonary defense mechanisms include aspirin, calcium
channel blockers and macrolide antibiotics
Frequent scheduled changes of the ventilator circuit result in decreased risk of contamination
Use of noninvasive positive pressure ventilation when appropriate and avoidance of
intubation reduces the risk of ventilator-associated pneumonia.
Which association of noninfectious pulmonary complications in transplant patients with its
etiology is correct:
Interstitial pneumonitis is associated with toxic levels of sirolimus
Metastatic pulmonary calcifications are associated exclusively with renal transplant
Right-sided diaphragmatic dysfunction is a rare complication of liver transplantation
After heart transplantation, the impairment in diffusion capacity resolves rapidly and
consistently
Acute respiratory distress syndrome (ARDS) can develop as a result of sepsis, aspiration and
OKT-3 use
166
ASCCA Residents Guide to the ICU
34. Neurologic Critical Care
Khaldoun Faris, M.D., Faraz A. Syed, D.O.
A 48-year-old female helmeted bicyclist was admitted to the ICU after a fall down a 15-foot embankment.
She was intubated in the field and had a Glasgow Coma Score (GCS) of 5 upon arrival to the trauma bay.
Her initial head CT scan revealed a large subarachnoid hemorrhage. A bedside ventriculostomy was
placed by the neurosurgeon and the initial ICP was 28. Hyperosmolar therapy (mannitol alternating with
23.4 % hypertonic saline) was instituted. The patient was placed in a 30 degree head up position to
facilitate venous drainage. Cerebral perfusion pressure was maintained with fluids and vasoactive
medications.
Neurologic problems may be present in critically ill patients either as the primary diagnosis or as a
complication of the underlying process. General management principles are based on the observation
that injured brain tissue poorly tolerates subsequent insults. Even apparently small decreases in blood
pressure, arterial oxygen saturation, and serum sodium may result in clinical deterioration; therefore,
meticulous monitoring and interventions are the mainstay of neurologic critical care. Although several
monitoring techniques are listed below, the best monitor of neurologic function still consists of sequential
neurologic exams by a trained clinician.
I.
Neurophysiology
A. The cranial vault has three components - brain tissue, cerebrospinal fluid (CSF), and blood all of
which are housed in a non-compliant skull. Thus, an increase in volume of any of these
components (cerebral edema, hydrocephalus, space occupying mass, vasodilation) will lead to
increased intracranial pressure (ICP) and decreased cerebral perfusion pressure (CPP) that may,
if sustained, cause neurologic damage and even death.
B. Cerebral Blood Flow (CBF)
1. Highly regulated to match the metabolic needs of the brain.
2. Determined by PaO2, PaCO2 and autoregulation
3. Hypoxia and hypercapnia produce vasodilatation and increased CBF
4. Pressure Autoregulation: in normal subjects with intact blood brain barriers, CBF is
maintained constant over the normal range of mean arterial pressure (MAP: 50 – 150
mmHg).
C. Other factors that may have deleterious effects on neurologic outcome include
1. Hyperthermia
2. Hyperglycemia
3. Hypoxemia
4. Anemia
II. Neuromonitoring
A. Neurologic examination and the Glasgow Coma Scale (GCS)
B. Neuroimaging
1. Head CT
2. Brain MRI
3. Angiography
C. ICP monitoring
1. Types
a) Catheter-based devices (ventriculostomies) : They are more widely used and allow
continuous ICP monitoring and therapeutic CSF drainage
b) Transducer-based devices: The tip is placed in the brain parenchyma or the subdural
space
2. Indications
a) Severe head injury (GCS < 8) and abnormal head CT finding
b) Severe head injury (GCS < 8) and normal head CT if
(1) > 40 years old
167
ASCCA Residents Guide to the ICU
D.
E.
F.
G.
H.
I.
III.
(2) Posturing
(3) Systolic BP < 90 mmHg
3. Complications
a) Malfunction
b) Infection
c) Bleeding
Transcranial doppler ultrasonography (TCD)
Jugular venous oxygen saturation (SjvO2)
Brain tissue oxygen partial pressure (PbtO2)
Microdialysis
Near-infrared spectroscopy
Continuous EEG
Disease States
A. Altered consciousness
1. Differential diagnosis
a) Normal sleep
b) Pathologic: hypoglycemia, hypoxia, drug overdose, poisoning, trauma, postictal state,
meningitis, encephalitis, stroke, post cardiac arrest
c) Psychogenic coma
d) Locked-in state
2. Diagnosis
a) Detailed history and physical with thorough neurologic examination
b) Neuroimaging
c) Lumbar puncture to obtain CSF
d) EEG
3. Management
a) Supportive measures
b) “Coma Cocktail” (mixture of drugs to reverse common causes of unconsciousness dextrose, thiamine, naloxone, and flumazenil)
c) Definitive treatment
d) Induced hypothermia for post cardiac arrest cases
B. Cerebrovascular disease
1. Ischemic cerebrovascular disease (acute ischemic stroke)
a) Etiology
(1) Embolic
(2) Atherothrombotic
(3) Lacunar
(4) Watershed
b) Diagnosis
(1) History and physical exam
(2) Neuroimaging
(3) Echocardiography
(4) Carotid ultrasound
c) Management
(1) Thrombolysis within 3 hours of onset
(2) BP should not be aggressively lowered
(3) Antiplatelet therapy
(4) Supportive measures
(5) Delayed anticoagulation for embolic stroke
2. Intracerebral hemorrhage
a) Etiology
(1) Hypertension
(2) Rupture of cerebral aneurysm or arteriovenous malformation (AVM)
(3) Traumatic head injury
(4) Coagulation disorders and spontaneous bleeding
b) Diagnosis
(1) History and physical exam
168
ASCCA Residents Guide to the ICU
Neuroimaging
(a) CT scan
(b) MRI
(c) Angiography
(3) Lumbar puncture is contraindicated
(4) Coagulation profile
c) Management
(1) Supportive measures
(2) Antihypertensives
(3) ICP management
(4) Surgical intervention
3. Subarachnoid hemorrhage (SAH)
a) SAH is a systemic disease that results from a ruptured cerebral aneurysm. It carries
significant neurologic morbidity and mortality.
b) Diagnosis
(1) History “worst headache ever” and physical
(2) Head CT/ CTA
(3) Lumbar puncture
(4) MRI/ MRA
(5) Angiography
c) Associated complications
(1) Rebleeding
(2) Vasospasm
(3) Hydrocephalus
(4) Fluid and electrolytes abnormalities
(5) Cardiac and pulmonary abnormalities
(6) Seizure
d) Management
(1) Supportive measures
(2) Rebleeding
(a) Angiography for aneurysm coiling
(b) Craniotomy for aneurysm clipping
(c) Antihypertensives
(d) Antifibrinolytics
(3) Vasospasm
(a) Hypertensive hypervolemic therapy (HHT)
(b) Angioplasty
(c) Calcium antagonists
(4) Hydrocephalus
(a) Ventriculostomy
(b) Shunting
C. Traumatic brain injury
1. Pathology
a) Primary brain injury (direct traumatic insult)
b) Secondary brain injury (due to hypotension and hypoxia)
c) Intracranial hypertension
d) Brain herniation and brain death
2. Diagnosis
a) History and physical
b) GCS
c) Neuroimaging: primarily head CT
d) ICP monitoring
3. Management
a) Supportive measures to prevent secondary injury
b) ICP management
(1) mild hyperventilation for acute elevation of ICP
(2) optimize oxygenation
(3) head of bed elevation
(2)
169
ASCCA Residents Guide to the ICU
(4) sedation
(5) prevent hyperthermia and hyperglycemia
(6) hyperosmolar therapy
(7) CSF drainage
(8) induced coma
(9) surgical decompression
D. Traumatic spinal cord injury
1. Pathology
a) Primary spinal cord injury
b) Secondary spinal cord injury
c) Compete spinal cord injury
d) Incomplete spinal cord injury
e) Spinal shock
(1) associated with complete spinal cord injury at a high level
(2) hypotension
(3) bradycardia
(4) hypothermia
2. Diagnosis
a) History and physical
(1) level of injury
(2) completeness of injury
(3) presence of spinal shock
b) Neuroimaging
(1) Plain radiography
(2) CT scan
(3) MRI
3. Management
a) Spinal precautions
b) Supportive measures including effective management of spinal shock
c) Methylprednisolone (SoluMedrol) protocol
d) Decompressive and/or stabilization surgery
E. Status epilepticus
1. Etiology
a) Idiopathic
b) Stroke
c) Anoxic brain injury
d) Metabolic
e) Pharmacologic
f) Infection
2. Diagnosis
a) Neurologic exam
b) EEG
c) CT scan
3. Management
a) Evaluation and stabilization
b) cardiopulmonary support
c) glucose and thiamine
d) Seizure control
e) Benzodiazepines
f) phenytoin
g) fosphenytoin
h) Phenobarbital
i) Refractory status epilepticus
j) intubation and hemodynamic support
k) continuous EEG monitoring
l) pentobarbital, propofol or benzodiazepines to obtain burst suppression on EEG
F. Guillain-BarrГ© syndrome
1. Pathology
170
ASCCA Residents Guide to the ICU
a) Multifocal demyelination of cranial and peripheral nerves
b) Immune-mediated
c) Progressive weakness and areflexia
d) Autonomic dysfunction
e) Spontaneous recovery, weeks to months
2. Diagnosis
a) History and physical
b) CSF analysis
c) EMG and nerve conduction studies
d) Autoantibodies
3. Management
a) Close observation of respiratory status
b) Hemodynamic monitoring
c) Immunotherapy
d) Plasmapheresis
G. Myasthenia gravis
1. Pathology
a) Autoantibodies bind to acetylcholine receptors blocking the receptors and decreasing
their density
b) Muscle weakness and fatigue
2. Diagnosis
a) History and physical
b) Edrophonium test
c) Antibodies to acetylcholine receptors
d) EMG
e) Chest CT for evidence of thymoma
3. Management
a) Supportive care
b) Cholinesterase inhibitors
c) Immunotherapy
d) Plasmapheresis
e) Corticosteroids
f) Thymectomy
This chapter is a revision of the original chapter authored by C. Lee Parmley, M.D., J.D. and Steven J. Allen, M.D.
READING LIST
1. Bracken MB, Shepard MJ, Collin WF, et al. A randomized controlled trial of methylprednisolone or naloxone in the
treatment of acute spinal-cord injury. N Engl J Med 1990;322:1405.
Methylprednisolone given as a 30 mg/kg bolus followed by infusion of 5.4 mg/kg/hr for 23 hours with treatment
initiated within 8 hours of acute spinal cord injuries is associated with significant improvement in motor and
sensory function, as compared to naloxone and placebo.
2. Lassen NA. The luxury-perfusion syndrome and its possible relation to acute metabolic acidosis localized within
the brain. Lancet 1966;2:1113.
An early description of the over-abundant cerebral blood flow relative to metabolic needs following injury to the
brain; with the cerebral arteriovenous oxygen difference being abnormally small, bright red venous blood is a
cardinal feature of the luxury perfusion syndrome.
3. Obrist WD, Langfitt TW, Jaggi JL, Cruz J, Gennarelli TA. Cerebral blood flow and metabolism in comatose patients
with acute head injury: relationship to intracranial hypertension. J Neurosurg 1984;61:241.
Following traumatic brain injury, cerebral hyperemia is common despite significant reduction in CMRO2.
Hyperemia is characterized by narrow AVDO2, indicative of luxury perfusion, in association with normal or
supranormal CBF. Hyperemia occurs in approximately 55% of patients with severe head injury, and occurs in
patients of all ages, although it is more common in younger patients. Intracranial hypertension is associated with
hyperemia.
171
ASCCA Residents Guide to the ICU
4. Muizelaar JP, Marmarou A, Ward JD, et al. Adverse effects of prolonged hyperventilation in patients with severe
head injury: a randomized clinical trial. J Neurosurg 1991;75:731.
Sustained hyperventilation(PaCO2 25 В±2) for management of intracranial hypertension is associated with lower
Glasgow Outcome Scale scores as compared to patients whose PaCO2 was maintained at 35 В±2, or who received
THAM in conjunction with sustained hyperventilation. Differences in outcome were statistically significant at 3
and 6 months, but not at 12 months postinjury.
5. Sonntag VKH, Douglas RA. Management of spinal cord trauma. Neurosurg Clin North Am 1990;1:729.
The management of patients suffering spinal cord injury is reviewed.
6. Tator CH, Fehlings MG. Review of the secondary injury theory of acute spinal cord trauma with emphasis on
vascular mechanisms. J Neurosurg 1991;75:15.
Spinal cord injury often results in complete functional loss even though total transection of the cord can seldom be
demonstrated. Vascular mechanism including hypotension, reduced cardiac output, and loss of local
autoregulation in zones adjacent to injured spinal cord segments may account for neurologic deficits.
7. Temkin NR, Dikmen SS, Wilensky AJ, et al. A randomized, double-blind study of phenytoin for the prevention of
post-traumatic seizure. N Engl J Med 1990;323:497.
Anticonvulsant medications are commonly administered following head trauma to prevent post-traumatic
seizures. Phenytoin has been shown to be beneficial in preventing such seizures only in the first week following
severe head injury.
8. Bullock R, Chesnut R, Clifton G, et al. Guidelines for the management of severe head injury. Journal of
Neurotrauma 1996;13:647-734.
Based on extensive review of published studies, this writing sets forth standards, guidelines and other
recommendations for the management of severe head injury. It is the product of a joint venture of The Brain
Trauma Foundation, The American Association of Neurological Surgeons, and The Joint Section on Neurotrauma
and Critical Care.
9. Boucher BA. Fosphenytoin: a novel phenytoin prodrug. Pharmacotherapy 1996;16(5):777-791.
This article discusses the pharmacology and clinical application of Fosphenytoin, a newer anticonvulsant agent.
10.Rosner MJ, Bowner SD, Johnson AH. Cerebral perfusion pressure: management protocol and clinical results. J.
Neurosurg 1995;83:949-961.
CPP maintained at or above 70 mmHg with a protocol using volume expansion, CSF drainage, systemic
vasopressors, and mannitol is reported to have better outcome as measured by Glasgow Outcome Scale, when
compared to other comparable series of head injury patients with initial GCS of 3 - 7.
11.Suarez JI: Outcome in neurocritical care: Advances in monitoring and treatment and effect of a specialized
neurocritical team. Crit Care Med 2006; 34: S232-8.
This article is an excellent review of the current advances in the monitoring and management of the critically ill
neurologic patients.
QUESTIONS
34.1.
A.
B.
C.
D.
E.
34.2.
A.
B.
C.
D.
E.
A 27-year-old man has had a right middle cerebral artery aneurysm uneventfully clipped 2
days following a subarachnoid hemorrhage. On the third postoperative day, he develops leftsided weakness over one hour. Of the following, which statement is most likely to be true?
He should be immediately returned to the OR for replacement of the clip.
His filling pressures should be elevated.
He should be hyperventilated and sedated.
He should receive steroids and mannitol.
Thrombolysis is likely to reverse these changes.
A patient with a severe headache, fever, and stiff neck is UNLIKELY to have the following
finding on lumbar puncture:
A normal lumbar puncture
Elevated WBCs in the CSF
Blood in the CSF
Elevated CSF glucose
Elevated CSF protein
172
ASCCA Residents Guide to the ICU
34.3.
A.
B.
C.
D.
E.
Which of the following statements is most correct about the Guillain-BarrГ© Syndrome?
It usually begins rapidly and may take months to resolve.
It will affect upper extremities more frequently than legs.
Bulbar involvement is almost always fatal.
Plasmapheresis is the treatment of choice in mild cases.
Autonomic instability is common.
34.4.
A.
B.
C.
D.
E.
The malignant neuroleptic syndrome:
Is a variant of malignant hyperthermia.
Is reversible with dantrolene.
Can be triggered by succinylcholine in ICU patients.
Can be the cause of renal failure.
Is self-limited and not associated with significant morbidity.
34.5.
A.
B.
C.
D.
E.
A changing localized neurologic pattern (waxing and waning) is most consistent with:
Cerebral vasospasm
An expanding mass lesion
A peripheral neuropathy
A cerebral ischemic infarct
Increased ICP
34.6.
A.
B.
C.
D.
E.
Which of the following are concerns in the comatose, head injured patient?
Aspiration of gastric contents
Increased neurologic damage from increased ICP
Gastrointestinal hemorrhage
Deep venous thrombosis and pulmonary embolism
All of the above
34.7.
A.
B.
C.
D.
E.
All the following factors may have deleterious effects on neurologic outcome EXCEPT:
Hypoxia
Anemia
Hyperglycemia
Diminished autoregulation
Hypothermia
34.8.
A.
B.
C.
D.
E.
The following are correct about acute ischemic stroke EXCEPT:
It could be caused by an embolic event
Blood pressure should be maintained lower but close to baseline
Echocardiography may be required
Thrombolysis should be initiated within six hours of the onset of stroke
Antiplatelet therapy is recommended
34.9. Which on of the following regarding ICP is INCORRECT?
A. ICP monitoring is indicated in a 70 year old patient with normal head CT and GCS of 7.
B. ICP monitoring is not indicated in a 35 year old trauma patient with normal head CT
and GCS of 3.
C. Transducer based catheters for ICP monitoring are less widely used and do not allow CSF
drainage.
D. ICP monitoring is indicated in a 20 year old motorcycle crash patient with abnormal CT
and GCS of 12.
E. Complications of ICP monitoring include infection, hemorrhage and malfunction.
173
ASCCA Residents Guide to the ICU
34.10.
A.
B.
C.
D.
E.
All the following are used to treat myasthenia gravis EXCEPT:
Pyridostigmine
Edrophonium
Plasmapheresis
Corticosteroids
Thymectomy
174
ASCCA Residents Guide to the ICU
35. Traumatic Brain Injury
Ruben J. Azocar, M.D.
You are called to assess the airway of an 18-year-old male who was an unrestrained passenger in a
motor vehicle crash (MVC). He open his eyes to pain, does not follows any commands and has an
incoherent speech. The ED physician wants him to have a head CT SCAN first to rule out an intracranial
injury as he believes the patient is just intoxicated and will get better as his EtOH levels decrease.
I.
Introduction
Severe traumatic brain injury (TBI) is a serious health problem in the US with an incidence of 1.6
million victim/yr.1 Furthermore, TBI is associated with:
A. 60,000 deaths /year
B. 70-90,000 victims left with permanent neuro damage
C. Most common in males aged 15 to 24
D. 50% of victims have significant concurrent injuries
E. ETOH is a factor in 40% cases
F. MVCs are the most common cause in teenagers and young adults whereas falls are the most
common in the extremes of ages
G. Elderly patients carry worse prognosis
H. Survivors are usually disabled and constitute a burden to their families and society
I. $100 billion/yr are expended in medical care
II.
Guidelines
A. No standardization of treatment before 1995
B. Neurosurgery Guidelines Committee practice parameters (1996)2
C. Revised Guidelines (2000)3
D. Unfortunately, the guidelines have only 3 Class 1 recommendations (all 3 showing ineffectiveness
of long-standing practices) and only 8 guidelines based on Class II evidence
III.
Initial Assessment
A. As with any trauma patient, evaluation of airway, breathing and circulation takes precedence
B. The severity of the TBI is classified clinically by Glasgow Coma Score (GCS) - seeTable 35.1
1. Mild 13-15
2. Moderate 9-12
3. Severe <8 (Mortality of 33%)
C. Careful assessment of the intoxicated patient is most important to prevent missing injuries
IV.
Primary neurological injury
The primary brain injury sustained at the time of the trauma cannot be reversed. A variety of injuries
can occur and will be briefly described
A. Skull fractures (Fxs)
1. Located at cranial vault or skull base.
2. Could be linear or stellate, depressed or not depressed
3. Indicate that a large amount of force was applied to the patient’s head
4. Linear vault fractures are usually associated with intracranial hematomas (ICH)
5. Basilar Fxs clinical signs: hemotympanum, Battle’s sign, periorbital ecchymoses, CN palsies
B. Epidural hematomas
1. Occur in less than 1% of all head traumas and less than 10% of comatose patients have
epidural hematomas
2. Located outside the dura and typically biconvex shape on CT scan
3. Temporoparietal location frequent due to rupture of middle meningeal artery
4. Initial loss of consciousness may be followed by lucent period and then late neurological
deterioration
5. Prompt intervention leads to favorable outcome
C. Subdural hematomas
175
ASCCA Residents Guide to the ICU
1. Incidence of 30% in severe TBI
2. Results form the tearing of a bridging vein between the cerebral cortex and draining venous
sinus
3. In 80% of SDH, the outcome depends on the underlying brain injury
4. CT Scan: Crescent shape blood collection with associated parenchymal contusion and
midline shift
D. Intracerebral hematomas
1. Associated with moderate or severe injury
2. Frontal and temporal location most frequent
3. Produce mass effect
4. Hyperdense areas on the CT scan
5. May be a delayed injury appearing 24 hrs after insult. Late neurological deterioration
mandates immediate CT scan
E. Diffuse axonal injury (DAI)
1. Shearing forces affect axons that transverse large areas of the brainstem leading to Reticular
Activating System dysfunction
2. Axons are not torn at injury but suffer from sequential focal changes leading to swelling and
disconnection1
a) Axolemma damage allows CA+2 influx triggering local intraxonal cytoskeletal and
mitochondrial damage
b) Presence of intra-axonal caspase 3 suggests apoptosis
c) Disconnection downstream: disconnected fibers degenerate leading to diffuse
deafferentation of target sites
3. Affected patients have a high morbidity and mortality
a) DAI can cause immediate and prolonged unconsciousness.
b) Persistent vegetative state is a common outcome
F. Brain edema
1. Clearly associated with acute injury
2. May worsen with surgery
3. Results in vasocongestion and mass effect
V. Management of Primary Injury
A. Rapid identification of injury by imaging and neurosurgical consultation
B. Surgery should be performed if necessary as soon as possible
C. As soon as the dura is open the ICP=0
VI. Secondary Brain Injury
A. Injury not related to the disruptive force
B. The postraumatic loss of cerebral circulation autoregulation, cerebral vasculature
vasoconstriction, systemic hypotension, intracranial hypertension and hypoxia result in further
ischemia and infarction
C. Other mechanisms involve include free radical formation, lipid peroxidation as the result of
spillage of intracellular contents and inappropriate neurotransmitter release (glutamate, aspartate)
D. Hyperglycemia and anemia may exacerbate injury
E. Later, seizures, infection and sepsis complicates patient outcome
F. The care of TBI patients is oriented to limit the secondary brain injury
VII. Areas of Management for prevention of secondary injury
A. Hemodynamic stability - Avoid Hypotension
1. Chestnut, et al demonstrated that hypotension was associated with dramatic deterioration of
favorable outcome4
2. Both early and late epidodes of hypotension affect outcome
3. Arterial catheter ideal for monitoring
4. Use of vasopressors while fluid resuscitation is in process might be indicated
5. Search for associated injuries that may explain the hypotension
6. Which fluid resuscitation is ideal?
a) Avoidance of hypotension is more important than the type of fluid. Fluid restriction in an
unstable trauma patient who sustained TBI is an archaic concept
176
ASCCA Residents Guide to the ICU
b) There are data supporting aggressive resuscitation (fluid and vasopressors) does not
worsen neurological injury.5
c) Avoidance of hypotonic fluids is optimal but no differences between NS and LR
d) There is some evidence that hypertonic saline (7.5%) with or without dextran improves
outcome.6
e) A cohort analysis of patients from double blind trials suggests that hypertonic solution
significantly improves outcome. 7
f) Recent publication comparing hypertonic saline with LR/NS showed identical neurological
function at 6 months18
g) If ongoing blood loss is an issue, blood might be the ideal fluid
h) Glucose containing fluids should be avoided.
i) Recent data suggest that the use of albumin in these patients increases mortality.
B. Avoidance of hypoxemia.
1. 50% of patients are hypoxic in the field. This finding is associated with increased mortality
2. Hypotension and hypoxemia are not a good mix
3. There are multiple causes of hypoxemia:
a) Airway control may be difficult
b) Concomitant thoracic injuries (pneumothorax, pulmonary contusion)
c) Trauma patients have high incidence of aspiration
d) Neurogenic pulmonary edema
e) ARDS
f) Airway management
(1) All patient with a GCS less than 8 must be intubated for airway protection.
Hyperventilation will be discussed below.
(2) Also patients with multiple trauma and those requiring multiple diagnostic procedures
should be intubated
(3) Difficulties: Elevated ICP, full stomach, C-Spine injury, airway trauma, uncertain
volume status, uncooperative/combative patient, hypoxemia
(4) Rapid sequence with in-line axial stabilization
(5) Consider using Etomidate to prevent hemodynamic complications
(6) Lidocaine IV may decrease the sympathetic response and the associated ICP
increase
(7) Succinylcholine is safe to use despite mild and transient
increases in ICP
(8) Awake nasal intubation: Base of skull fractures may result in an intracranial
intubation
(9) A note about C-spine injuries
(a) Incidence of C-spine injury is 1-3% in adults and 0.5% in children
(b) Head-first falls and high speed MVC have 10% greater risk of C-Spine injury
(c) Cross-table X-Rays may miss 20% fractures and with the addition of AP and
odontoid views “only” 7%” are missed.
(10) If Mechanical Ventilation is initiated:
(a) Judicious use of PEEP
(b) Titrate FiO2 to a saturation above 95% to avoid deleterious effect of oxygen
therapy
C. Management of cerebral circulation
1. Cerebral circulation physiology after TBI is characterized by:
a) Reduced CBF
(1) In a series of 106 patients with TBI within 6 hrs, 1/3 had CBF<18mlx100g-1xmin-1
(ischemic threshold)
(2) The cerebral Arterio-Venous difference is high in the first hrs and then progressively
decreases
b) Impaired cerebral pressure auto regulation
(1) In 1/3 patients CBF passively changed as CPP changed
(2) In an animal model, CBF was poor with hemorrhagic hypotension but improved with
phenylephrine infusion
c) Increased ICP
2. Maintenance of Cerebral Blood flow
a) Goal: Maintain CBF=Preserve CPP
177
ASCCA Residents Guide to the ICU
b) CPP=MAP-ICP
c) CPP> 60-70 mm Hg is ideal
d) Traditional treatment of TBI has focused in ICP reduction
3. Reduction of ICP
a) ICP Monitoring
(1) In cases of severe TBI, measurement of ICP is essential to monitor CPP
(2) An ICP above 20 is considered high.
(3) There are different methods to measure ICP and all of them have advantages or
disadvantages
(a) Ventriculostomy: Allows CSF drainage if ICP is high but it is difficult to place
and carries a higher risk of infection
(b) Intraparenchymal monitor: Easy to place, low risk of injury/infection but
questionable measurement of global ICP
(c) Subarachnoid Bolt: Easy to place, low risk of injury/infection, but frequent
occlusion with brain tissue
b) How to lower ICP
(1) Therapies to decrease ICP are added in an order that reflects the risk of
complications associated with each therapy.
(2) First line of treatment:
(a) Intubation
(b) Positioning: 30 degrees of head up elevation, head in neutral position to avoid
venous congestion
(c) Sedation/Paralysis to avoid valsalva maneuver
(d) Mild Hyperventilation: see note below
(3) Second line of treatment
(a) CSF drainage: only possible with ventriculostomy drains
(b) Osmotic therapy: Mannitol (0.25 -1 g/kg and titrated to specific measured
osmolality or osmolar gap [measured – calculated osmolalilty]
(4) Third line of treatment
(a) Pentobarbital coma
(b) Surgical decompression
(5) The use of chronic and profound (25 mmHg) hyperventilation is discouraged.2
Hyperventilation may decrease CBF and reduce brain oxygenation. Skippen et al
using xeno-enhaced CT and CBF studies demonstrated a 2.5 fold increase in the
number of regions of brain ischemia in children with TBI who were hyperventilated.8
Muizellar et al published a prospective randomized study that demonstrated worse
neurological outcome in patients that were hyperventilated. 9 A PaCO2 between
30-35 seems appropriate. It should also be noted that the effects of hyperventilation
are self limited and disappeared after 8-12 hours. However, it could be used in a
critical situation as a lifesaving maneuver.
4. Novel theories of CPP preservation10
a) Different strategies for the management of cerebral circulation during TBI have been
recently advocated. They have not proved to improve outcome after TBI when compared
with the traditional approach described above.
(1) The “CPP management”, has been advocated by Rosner et al.11 According to this
hypothesis, a reduction in CPP—either a decrease in MAP, an increase in ICP, or
both—stimulates the cerebral vessels to dilate in an attempt to maintain CBF. This is
the normal pressure autoregulatory response to a decrease in CPP. Because the
increase in cerebral blood volume that accompanies the vasodilation further reduces
CPP by increasing ICP, this sets up a cycle that leads to reducing CPP even more.
An increase in arterial blood pressure under this circumstance has been observed to
break the cycle and reduce ICP. A detailed description of this approach is given in a
recent report of a clinical series. In this series of 158 patients admitted with a
Glasgow Coma Scale score less than 7, mortality was only 29%, and 59% achieved
a good recovery or moderate disability by 6 months postinjury. This approach was
believed to be of sufficient value that it was included in the 1996 and 2000 Head
Injury Guidelines as a treatment option.
178
ASCCA Residents Guide to the ICU
D.
E.
F.
G.
H.
(2) The “Lund” 12 therapy emphasizes reduction in microvascular pressures to minimize
edema formation in the brain. The goals of this approach are to preserve a normal
colloid osmotic pressure (infusion of albumin and erythrocytes), to reduce capillary
hydrostatic pressures by reducing systemic blood pressures (metoprolol and
clonidine), and to reduce cerebral blood volume by vasoconstricting precapillary
resistance vessels (low-dose thiopental and dihydroergotamine). Treatments that
would favor increasing transcapillary filtration of fluid are avoided, including
cerebrospinal fluid drainage, high-dose (to burst suppression) barbiturates, osmotic
diuretics, and high CPP. Decompressive craniectomy, which can also increase
edema formation, is reserved as a last resort
Monitor brain oxygenation
1. Measurement of the oxygen content indirectly by jugular venous oxygen saturation (SjvO2) or
directly by brain tissue PO2 has shown to be good indicators of an injury that is worsening
2. Jugular venous oxygen saturation
a) Utilizes “mixed” cerebral blood that may reflect ischemia
b) There is evidence that a decrease in (SjVO2) is associated with worse outcome.13
c) Limitations
(1) Invasive
(2) Possible malposition
(3) Unilateral
(4) Global (not focal) ischemia detected
(5) Anemia may cause false negative
3. Brain Tissue PO2 provides evidence of ischemia that also correlates with neurological
outcome and reflect changes in CPP
Neuroprotective agents/Seizure prophylaxis
1. Phenytoin or carbamazepine are useful to prevent early postraumatic seizures (<7 days)
a) No evidence for chronic use of anticonvulsants to prevent seizures2
2. As free radicals are involved in secondary injury, use of free radical scavengers may be
beneficial (i.e. Mannitol)
3. There is no benefit in administering steroids.2
a) These drugs may alter nutrition and metabolic profile (Hyperglycemia)
Cardiovascular system
1. Massive catecholamine response and aggressive fluid resuscitation may unmask or worsen
cardiac disease
2. B-Blockers (labetalol/esmolol) may be used to control hypertension and tachycardia
3. Arrhythmias are not uncommon and may be unrelated to the heart (“Brain arrhythmias”)
4. Severe ICP precipitates reflex hypertension and bradycardia (Cushing’s triad)
5. Venous thrombosis prophylaxis
a) Anticoagulants may be contraindicated
b) Venous compressive devices
c) IVC filters
Nutrition/ Gastrointestinal
1. TBI results in a hypermetabolic and catabolic state
2. Caloric replacement should be at least 140% (non-paralyzed patients) and 100% (paralyzed
patients) of RME by day 7. Start feedings by 72 hr
3. Enteral feeding is the preferred route
4. Stress gastritis protection should be provided to patients requiring ventilatory support and
who are not being enterally fed. Double coverage is the norm
Endocrine and metabolic
1. Avoid hyperglycemia14
a) Brain glucose concentrations parallel blood levels
b) Hypoxia or anoxia results in anaerobic metabolism and production of lactic acid
c) The associated H+ produce cellular damage by:
d) Altering neurotransmitter release and uptake
e) Alteration of ion homeostasis
f) Changes in protein synthesis and activity
g) Hyperglycemia increases mortality in the ICU14
h) Avoid glucose containing fluids for resuscitation
179
ASCCA Residents Guide to the ICU
I.
J.
i) Consider insulin infusion if hyperglycemia is a problem
2. Electrolyte abnormalities
a) SIADH: Hyponatremia, UNA>30mEq/lt., Uosm>300mOsm/kg
b) Diabetes insipidus: Hypernatremia, Uosm<290mOsm/kg
c) Cerebral “Salt wasting syndrome”: Intravascular volume contraction and negative NA+
balance
Hematological
1. Coagulopathies and DIC may be present due to liberation of large amounts of brain
thromboplastin
2. Hypothermia, massive blood transfusion are also possible causes of coagulopathy in any
trauma patient
3. Monitoring of coagulation parameters is very important intraoperatively
Temperature
1. Passive cooling to mild hypothermia (to about 34-35o C) decreases cerebral metabolic rate
(CMR).
2. A 7% reduction in CMR per degree Celsius decrease has been described.
3. Reductions of CMR result in decreases in oxygen consumption and therefore hypothermia is
thought to be protective of the brain. Two large studies looked into this hypothesis.
4. Treatment of Traumatic Brain Injury with Moderate Hypothermia. Marion et al 16
Treatment with moderate hypothermia for 24 hours in patients with severe traumatic brain
injury and GCS of 5 to 7 on admission hastened neurologic recovery and may have improved
the outcome.
5. Lack of Effect of Induction of Hypothermia after Acute Brain Injury Clifton, et al.17
Treatment with hypothermia, with the body temperature reaching 33В°C within eight hours
after injury, is not effective in improving outcomes in patients with severe brain injury.
However patients that were hypothermic upon arrival had a better outcome.
6. It seems that hypothermia has a protective effect if it occurs at the time of the primary injury.
Severe hypothermia should be avoided due its deleterious systemic effects.
Table 35.1. Glasgow Coma Scale
Signs
Eye opening
Spontaneous
To verbal Command
To pain
No response
Best motor response
Obeys verbal commands
Localizes pain
Withdraws to pain
Flexion response to pain
(decorticate)
Extension response to pain
(decerebrate)
No response
Best verbal response
Oriented
Confused
Inappropriate words
Nonspecific sounds
No response
Score
4
3
2
1
6
5
4
3
2
1
5
4
3
2
1
180
ASCCA Residents Guide to the ICU
REFERENCES:
1. Marik PE, Varon J, Trask T: Management of head trauma. CHEST 2002; 122:699-711.
2. Brain Trauma Foundation, American Association of Neurological Surgeons, Joint Section on Neurotrauma and
Critical Care: Guidelines for the management of severe head injury. J Neurotrauma 1996; 13: 641-734.
3. Bulllock RM, Chestnut R, Clifton GL et al: Management and prognosis of severe traumatic brain injury, part 1:
Guidelines for the management of severe traumatic brain injury. J Neurotrauma 2000; 17:451-553.
4. Chesnut RM: Avoidance of hypotension: conditio sine qua non of successful severe head-injury management. J
Trauma 1997; 42: S4-S9.
5. Scalea TM, Maltz S Yelon J et al: Resuscitation of multiple trauma and head injury: Role of crystalloids fluids and
inotropes. Crit Care Med 1994; 22:1610-5.
6. Vassar MJ, Perry CA, Holcroft JW. Prehospital resuscitation of hypotensive trauma patients with 7.5% NACL
versus 7.5% NACL with added Dextran: a controlled trial. J Trauma 1993; 34:622-33.
7. Wade CE, Grady JJ, Kramer GC et al. Individual patient cohort analysis of efficacy of hypertonic saline/dextran in
patients with traumatic brain injury and hypotension. J Trauma1997; 42 (%S): S61-S65.
8. Skippen P, Seer M, Poskitt K, et al. Effects of hyperventilation on cerebral flood flow in head-injures children. Crit
Care Med 1997; 25:1402-1409.
9. Muizelaar JP, Marmarou A, Ward JD, et al: Adverse effects of prolonged hyperventilation in patient with severe
head injury: a randomized control trial. J Neurosurg 1991; 75: 731-739.
10. Robertson C: Management of Cerebral Perfusion Pressure after Traumatic Brain injury. Anesthesiology 2001;
95:1513-17.
11. Rosner MJ, Rosner SD, Johnson AH: Cerebral perfusion pressure: Management protocol and clinical results. J
Neurosurg 1995; 83:949-62.
12. Eker C, Asgeirsson B, Grande PO, et al: Improved outcome after severe head injury with a new therapy based on
principles for brain volume regulation and preserved microcirculation. Crit Care Med 1998; 26: 1881-6.
13. Gopinath et al: Jugular venous desaturation and outcome after head injury. J Neurol Neurosurg and Psychiatry.
1994 57:717-723.
14. Lam AM, Winn HR, Cullen BF et al: Hyperglycemia and neurological outcome in patients with head injury. J
Neurosurg 1991; 75:545-51.
15. van den Berghe G, Wouters P, Weekers F, et al Intensive insulin therapy in the critically ill patients. N Engl J Med.
2001 Nov 8;345(19):1359-67.
16. Marion et al: Treatment of Traumatic Brain Injury with Moderate Hypothermia. New Engl J Med 1997;
336:540-546.
17.Clifton, et al: Lack of Effect of Induction of Hypothermia after Acute Brain Injury New Engl J Med 2001;
344:556-563.
18. Copper et al: Prehospital hypertonic saline resuscitation of patients with hypotension and sever traumatic brain
injury: a randomized control trial. JAMA 2004; 291: 1350-7.
19.The SAFE Study Investigators. Saline or albumin for fluid resuscitation in patients with traumatic brain injury.
New Engl J Med 2007: 357-874-884.
QUESTIONS:
35.1.
A.
B.
C.
D.
The CORRECT statement regarding the fluid resuscitation in the patient with traumatic brain
injury is:
Crystalloids are better than colloids
Hypertonic saline is clearly superior
Vasopressors are clearly contraindicated
There is no difference in outcome at 6 months in relation to the type of crystalloid fluid
181
ASCCA Residents Guide to the ICU
35.2.
A.
B.
C.
D.
Current concepts in relation to the use of hyperventilation in the TBI patient include the
following EXCEPT:
Prophylactic hypercarbia is contraindicated
Hypercarbia might worsen outcome
Ischemia might result from the use of hypercarbia
A PaCO2 of 25mmHG is indicated in most TBI patients
35.3.
A.
B.
C.
D.
Airway management in the TBI patient might be complicated by the following EXCEPT:
Hypoxemia
Unknown neck injuries
Unknown intravascular volume
Non combative or non intoxicated patient
35.4.
A.
B.
C.
D.
Prevention of secondary injury in the TBI patient includes the following EXCEPT:
CPP management
Prevention of hypoxemia
Immediate surgical decompression
Prevention of hyperglycemia
35.5.
A.
B.
C.
D.
None of the following are accepted means to care for the TBI victim EXCEPT:
Aggressive hypocarbia
Provide prophylactic anti-seizure drugs for 7 days
Use steroids to minimize brain swelling
Delay enteral feeding for 5-7 days
182
ASCCA Residents Guide to the ICU
36. Management of Increased Intracranial Pressure
Jonathan R. Jagid, M.D., Matthew M. Ruel, M.D., Leo Harris, P.A, M.P.H., Miguel Cobas, M.D.
A 32-year-old male is brought to the ED after being found unresponsive at the scene after an MVA. EMS
reports that he was an unrestrained driver and was ejected from the vehicle. The patient arrives
intubated, his vital signs are: HR = 52, BP = 168/95, SpO2 = 100%. Injuries include multiple facial
fractures, and head CT reveals a bilateral frontal contusion with diffuse brain edema. Chest and C-spine
x-rays are unremarkable, and CT of abdomen/pelvis are negative for trauma related injuries. On exam,
the patient is nonfocal and demonstrates decerebrate posturing to deep pain.
I.
Key Concepts / Definitions
A. Anatomical/Physiological concepts unique to the brain
1. The brain is enclosed in a confined compartment, the cranium.
(1) Monroe-Kellie Doctrine states that the cranium is a rigid compartment.
(a) Three non-compressible components:
(1) Brain Parenchyma
80%
(2) Cerebral Spinal Fluid
10%
(3) Intracranial Blood
10%
2. Any increase in volume of any of these three components, or the addition of a mass, past a
threshold leads to an increase in intracranial pressure.
3. Any increase above and beyond this threshold will result in a disproportionate increase in
pressure (ICP) with respect to the increase in volume. Even minute volumes (1-2 mls) can
cause an exponential rise in ICP’s. This is considered to be life threatening and potentially
leading to herniation syndrome or “herniation.”
4. The Monroe-Kellie Doctrine is the strategic foundation for the management of increased
ICP’s
B. Cerebral autoregulation (see Figure 36.1)
1. The ability of the brain to maintain a constant critical level of cerebral blood flow (CBF)
{45-50ml /100g/min , 20ml/100g/ min in white matter to 70ml/100g/min in grey matter}over a
wide range of mean arterial pressures (MAP) {60-150mmHg} in a sigmoidal pattern.
2. In the severely injured brain, there is malfunction of autoregulation termed disautoregulation.
The autoregulatatory mechanism does not function and the relationship is converted to a
linear pattern.
3.Exact physiological
mechanism are not well
understood, studies
Figure 36.1 - Cerebral Autoregulation
suggest that the cerebral
vascular bed exhibits an
intrinsic myogenic
regulation of vascular
tone which plays a role.
4.Autoregulation is
disrupted by extremes in
MAP, traumatic brain
injury; severe metabolic
disturbances, disruption of
blood brain barrier; severe
hypercarbia and/or
hypoxemia (see figure
36.2).
5.Chronic hypertension
shifts the autoregulatory
curve to the right,
increasing the risk of
183
ASCCA Residents Guide to the ICU
ischemia with relative hypotension that may fall within the normal acceptable range (MAP
60-150mmHg).
6. Disautoregulation places the onus upon the intensivist and or anesthetist to reestablish
cerebral blood flow by appropriate and aggressive management of the mean arterial
pressure.
C. Elastance, Compliance and the Intracranial Pressure-Volume curve
1. The curve is a graphical representation of the nature of intracranial elastance, or the change
in pressure for a given change in volume (delta P/ delta V).
2. The concept of elastance needs to be distinguished from compliance, which is change in
volume per unit pressure (think of chest wall/pleura/lung model). In the lung model we are
interested in volume change per unit pressure, whereas in the cranium the change in
pressure per unit volume is more relevant.
3. An easy way to visualize these two reciprocal concepts is via a balloon analogy. Compliance
relates to how easily a balloon expands when pressure is introduced, whereas elastance
refers to the force the balloon exerts against a specific volume when filled.
4. The flat portion of the curve illustrates low-elastance (high compliance), a state where
incremental changes in volume are accompanied by relatively small increases in pressure.
This low elastance state reflects shunting of CSF and blood to extracranial locations down a
developing pressure gradient.
5. The steeper portion of the curve is reached once this shunting mechanism is overcome and
small increases in volume lead to large increases in ICP. Clinically, his high-elastance (low
compliance) state can rapidly lead to catastrophic cerebral ischemia and herniation if not
treated expeditiously.
D. Cerebral Perfusion Pressure (CPP).
1. An indirect measurement of cerebral blood flow (CBF)
2. Cerebral Perfusion Pressure = Mean Arterial Pressure- Intracranial Pressure
3. Studies indicate that CPP should be kept above 70mmHg to ensure adequate cerebral
perfusion.
4. Recent studies have shown that if a patient’s CMRO is decreased (paralytics/sedation/
barbiturates) CPP’s may be allowed to drift to below 70mmHg but no less than 60mmHg.
Figure 36. 2. The proposed vasodilatory and vasoconstriction cascades model:
As CPP increases, cerebral vasoconstriction limits CBV and ICP. Conversely, a reduction in CPP may stimulate
cerebral autoregulatory vasodilation with an increase in CBV and ICP. (Reproduced with permission from: “Rosner
MJ, Rosner SD, Johnson AH: Cerebral perfusion pressure: management protocol and clinical results. J Neurosurg
83:949-962, 1995).
II. Etiology of intracranial hypertension
A. As stated in the Monroe-Kellie doctrine, described above, an increase in volume of any of the
184
ASCCA Residents Guide to the ICU
three intracranial components, or the addition of a mass occupying lesion (tumor, epidural/
subdural hematoma), will lead to increased intracranial pressures.
B. Increased CSF volume occurs via decreased absorption (compression of foramen of Monroe,
sinus injury) or increased production.
C. Increased cerebral blood volume (CBV) may occur via vasodilation or intracranial hemorrhage
D. Increased brain tissue volume can occur in the context of enlarging neoplastic lesions, expanding
intracerebral/epidural/subdural hematoma or edema.
III. Indications for ICP monitoring
A. Severe Brain Injury or Glasgow coma score ≤ 8 (after resuscitation) and abnormal CT
B. Severe Brain Injury or Glasgow coma score ≤ 8 (after resuscitation) and normal CT with ≥2 of the
following:
1. age 40
2. SBP 90mmHg
3. decerbrate on motor exam
4. decorticate posturing on motor exam
IV. Methods of monitoring ICP
Continuous ICP monitoring in an ICU setting allows optimization of CPP in critically ill patients with
intracranial hypertension. In present TBI practice, the intraparenchymal and ventricular catheters are
considered the standard of care. A normal value for ICP in an adult is 8-15mmHg, and 3-7mmHg in
pediatric patients.
A. Intraventricular (ventriculostomy) catheter, fluid coupled system
1. Gold standard for ICP monitoring
2. Allows for continuous ICP monitoring, therapeutic/diagnostic CSF drainage.
3. Risk of infection, tissue damage during placement, hematoma formation.
4. Placement can be technically challenging, as insertion is a “blind” procedure.
5. Antibiotic impregnated catheters have significantly lowered the infection risk
B. Intraparenchymal monitors
1. Directly measure brain tissue pressure using fiberoptic or strain gauge sensors.
2. Placed in cortical gray matter, risk of tissue damage during placement.
3. Solid-state technology, no tubing to kink or clog
4. Cannot drain CSF, cannot be recalibrated once placed in brain.
5. Acceptable drift that increases each day
6. Used in scenario of diffuse edema where direct ventricular access is difficult
7. Lower infection rate
8. Requires frequent re-zeroing
C. Subdural bolt
1. Hollow screw threaded into skull with tip passing into subdural space.
2. No parenchymal penetration may be placed in any location which avoids venous sinuses.
3. Cannot relieve ICP by draining CSF, infection risk similar to ventriculostomy catheter.
D. Epidural transducer
1. Places pressure-sensitive membrane in contact with dura.
2. Decreased risk of infection since no dural puncture.
3. Cannot therapeutically drain CSF
4. Placement in epidural space difficult, risk of hematoma due to venous plexus.
V. Pathophysiology
A. Ischemia is defined as a localized insufficiency of blood flow, delivering oxygen and nutrients or a
mismatch of tissue demands versus blood delivery
B. The brain is the organ most sensitive to ischemic insult.
C. Ischemia results in damage to neurons as a result of sustained intracranial hypertension and is
the pathopysiological mechanism by which cell death ensues.
D. Cellular metabolic consequences of ischemia
1. Sustained ischemia compromises ATP production by disrupting oxidative phosphorylation.
2. ATP deficit causes shut-down of ATP dependent ion pumps, leading to increased intracellular
sodium and calcium and decreased potassium.
3. Membrane depolarization occurs which in turn causes release of excitatory amino acids
185
ASCCA Residents Guide to the ICU
(glutamate) and oxygen free radicals.
This cascade of events leads to cellular necrosis and brain edema, which in turn increases
ICP further and sets up a catastrophic clinical course if not managed expeditiously.
Precipitating factors for ischemia also know as secondary brain injury
1. Hypotension
2. Hypoxia
3. Hyperthermia
4. High ICP’s
Hypotension and Hypoxia are two most common reasons for ischemia
Ischemia is not a singular event but rather cumulative, progressive event
Without aggressive and rapid reduction in ICP ultimately brain herniation and death will ensue.
4.
E.
F.
G.
H.
VI. Rationale and Strategies:
A. MAINSTAY OF ICP MANAGEMENT IS PREVENTION OF ISCHEMIA
1. Maintain CBF
a) Difficult to monitor real time
(1) limited practical/reliable technology (Blood flow monitors, Xenon scans)
(2) CPP acceptable real time measure of cerebral blood flow
(a) CPP cannot be measured directly
(b) CPP has direct relationship to CBF
(c) CPP is PARAMOUNT when monitoring increased ICP’s
(d) CPP = MAP – ICP
b) Arterial line, central venous access and ICP monitor are strongly urged in the moderate/
severe brain injury patient.
(1) Normal MAP – 90-100mmHg
(2) NEVER lower MAP artificially in face of increased ICP’s
(3) Normal ICP’s
(a) Adult 8-15mmHg
(b) Pediatric 3-7mmHg
(4) Maintanence of CPP’s
(a) ≥70mmHg
(b) ≥60mmHg if CMRO² (Cerebral Metabolic Rate of Oxygen Consumption)
decreased with
i) Hypothermia
ii) Sedation/Paralytic
iii) Barbiturate
(5) The NeuroIntensivist and the Neurosurgeon should always devise a plan before
initiating management of high ICP’s
VII. Therapeutic Interventions
A. Positioning of patient
1. Head of Bed 30-45Вє
a) Promotes venous outflow of intracranial drainage (CBV) in periphery, promotes CSF flow
from intracranial vault to spinal vault, ↓ ICP’s
2. Neck in midline so as to not occlude carotid/jugulars vessels
3. Care must be taken to ensure MAP is maintained when in head up position.
B. Intravascular Solution
1. AVOID HYPOTONIC SOLUTIONS IN TBI patients.
2. Ideal state is EUVOLEMIA (maintain SBP) with HYPERTONICITY (↓ edema)
3. Normal Saline most frequent solution
4. Hypernatremic solutions are gaining popularity (3%-23.4%)
a) Rapid volume expansion
b) Increase tonicity of blood decreasing edema
C. Diuretics
1. Mannitol 20% (0.25-1g/kg IV) is an osmotic diuretic.
a) Should be initiated as soon as possible after neurosurgical evaluation
(1) Rapid plasma expander by increasing tonicity of blood
(a) hematocrit and viscosity improving CBF and Oxygen delivery
186
ASCCA Residents Guide to the ICU
(b) Increased tonicity draws edema from brain tissue
15-30 minutes to take effect.
Lasts 1.5-6 hours
Supports microcirculation by rheology properties
Therapeutic range in TBI by keeping serum 300-320 mOsm/L
Bolus and Maintenance doses differ,
(a) Load is 1gm/kg, maintenance 0.5 gm/kg
(b) Frequency is usually Q4В°
(7) Serum osmolarity and plasma sodium concentration should be monitored Q4,
staggered with Mannitol infusion to allow for dosing changes. Renal toxicity is
common with serum osmolarity greater than 320, and hypernatremia is an acceptable
side effect.
(8) Must continue IVF, enteric feeding alone is not a substitute when utilizing
hyperosmolar therapy
(9) Frequently the blood brain barrier is violated in brain injury patients, making it
essential to maintain mannitol within the therapeutic range. If mannitol is allowed to
drift to subtheraptic levels it may worsen ICP by drawing water into the cranial vault.
(10) Mannitol should be weaned to prevent rebound intracranial hypertension
2. Furosemide (0.5-1mg/kg IV alone; 0.15-0.3mg/kg in comb. with mannitol) acts at the level of
the glomerulus (loop diuretic).
a) Decreases CSF production.
b) Not as effective in emergent setting as mannitol
c) Synergistic effect with Mannitol
d) Necessary to follow electrolytes closely, hypocalcemia and hypokalemia common.
e) Hold if Serum Osm ≥ 320 to prevent hyperosmolar ketoacidosis
Hyperventilation- PaC02 is the most potent determinant of CBF.
1. PROLONGED HYPERVENTILATION IS CONTRAINDICATED
2. HYPERCAPNIA (PaCO2 ≥46) AND severe HYPOCAPNIA (≤30) SHOULD ALWAYS BE
AVOIDED
3. Vasocontriction of intracranial vessels ↓CBF and CBV
4. Hyperventilation therapy is effective for acute ICP control only. After 12-24 hours
compensatory metabolic mechanisms lead to decreased bicarbonate in the CSF,
normalization of pH, and resolution of induced vasoconstriction.
5. For every mmHg change in PaC02, CBF changes 1-2ml/100g/min.
6. Target PaCO2 35mmHg, at partial pressures less than 25 the O2-Hb dissociation curve shifts
to the left and O2 delivery may be compromised.
7. Hypoxemia must be avoided, at PaO2 less than 50 mmHg CBF and therefore ICP will
increase.
8. Be aware that PEEP may decrease venous outflow from cranial vault via increased
intrathoracic pressures.
9. CBF varies directly with PaCO2 over a range of 20-80mmHg.
CSF Diversion
1. Very Effective
2. Immediate result
3. Small volumes (5-10cc/hr) for ICP’s sustained for ≥5 minutes
4. Large volumes (>10cc/hr) may result in new or increase in extra axial collections
Corticosteroids
1. STEROIDS ARE AN ABSOLUTE CONTRAINDICATION IN THE SEVERE BRAIN INJURY
PATIENT
2. Dexamethasone requires 12-36 hours to take effect, has not been shown to be effective in
the acute setting.
3. Appropriate in the treatment of vasogenic edema (tumors, abscesses) steroids have been
shown to decrease vasogenic edema formation around neoplasms.
4. Edema of TBI is frequently a combination of cytotoxic, vasogenic, and interstitial
5. May help repair/stabilize disrupted blood brain barrier via exocytosis.
6. Must weigh use against risk profile in non TBI patients: hyperglycemia, GI bleed, electrolyte
disturbances, increased infection risk.
Surgical decompression
(2)
(3)
(4)
(5)
(6)
D.
E.
F.
G.
187
ASCCA Residents Guide to the ICU
1. Should be reserved for patients with intractable ICP’s.
2. Involves removal of a portion of the skull +/- resection mass occupying lesion.
3. Studies show promise in early decompression
4. Decompressions becoming larger in size with advances in technology
H. Hypothermia
1. Studies inconclusive but subgroups have shown some benefit
2. Investigations ongoing
3. Leads to decreased CMR02 and resultant cerebral vasoconstriction
4. Target temperature should be between 33 and 35 degrees C, significant side effects with
Temperatures < 33ВєC.
I. Barbiturate/Sedative therapy
1. Barbiturates not commonly utilized today
a) Multi-system side effects
b) Usually require PA catheter
2. Sedatives-Benzodiazepines and propofol
a) Useful in lowering ICP’s
b) Lowers CMRO
c) Shorter half life preferred to preserve rapid neurological exam
3. Paralytics
a) Lowers CMRO
b) Should never be used without sedative
188
ASCCA Residents Guide to the ICU
Table 36.1: Tiered Management of Intracranial Hypertension
Treatment
Effect
Comments
Head elevation
• Decreased CBV and capillary
leak
• CSF to lumbar cistern
•Decreased CSF volume
• Need to maintain MAP with head
• Decreased brain tissue water
and CSF formation
• Decreased blood viscosity →
decreased CBV
• Decreased CBV and capillary
leak
• Hypotension and hypovolemia
• Electrolyte disturbances and possibly
renal failure
•Mannitol 20% agent of choice
• No role for prolonged hyperventilation
(PaCO2 ≤ 30mmHg)*
• Improved CSF outflow
Decreases CBF and may exacerbate
cerebral ischemia.
• Decreased brain CMRO2
• Untoward systemic side effects
• Cerebral vasoconstriction
• Less effective in GCS 3 and 4
Decreased CMRO2 в†’ decreased
CBV
• Decreased capillary leak
• Decreased brain tissue volume
• Immunosuppression; needs
hemodynamic support and monitoring
CSF drainage
Osmotherapy
Mild
Hyperventilation
(pCO2 30-35)
Hypothermia
Barbiturates
Operative
decompression
Steroids
Endothelial cell membrane
stabilization
• Decreased release of cytokines
* Brain Trauma Foundation Standards
up
•Infection, formation of extraxial
collections from overdrainage
• Brain tissue loss and damage
• No role in TBI *
• Hyperglycemia, GI bleeding
• Immunosuppression
This chapter is a revision of the original chapter authored by Ahmed E. Badr, M.D., M.Sc.
READING LIST:
1. Rosner MJ, Rosner SD, Johnson AH. Cerebral perfusion pressure: management protocol and clinical results. J
Neurosurg 1995; 83(6):949-62.
Excellent presentation of the importance of CPP management in severe head injury. See comments also.
2. Brain Trauma Foundation.
Critical pathway for the treatment of established intracranial hypertension. J Neurotrauma 1996;13(11):719-20. Upto-date data on management of increased ICP in head injury.
3. Bullock R, et al. Guidelines for the management of severe head injury. Eur J Emerg Med 1996;3(2):109-27.
4. Brain Trauma Foundation. The use of mannitol in severe head injury. J Neurotrauma 1996;13(11):705-9.
5. Brain Trauma Foundation. The use of barbiturates in the control of intracranial hypertension. J Neurotrauma
1996;13(11):711-4.
6. Brain Trauma Foundation. The role of glucocorticoids in the treatment of severe head injury. J Neurotrauma
1996;13(11):715-8.
7. Brain Trauma Foundation. Indications for intracranial pressure monitoring. J Neurotrauma 1996;13(11):667-79.
8. Gray MJ, Rosner MJ. Pressure-volume index as a function of cerebral perfusion pressure. Part 2: the effects of low
cerebral prefusion pressure and autoregulation. J Neurosurg 1987;67(3):377-80.
9. Wilberger JE, Cantella D. High-dose barbiturates for intracranial pressure control. New Horizons 1995;3(3):469-73.
189
ASCCA Residents Guide to the ICU
10.Muizelaar JP, et al. Adverse effects of prolonged hyperventilation in patients with severe head injury: a
randomized clinical trial. J Neurosurg 1991;75(5):731-9.
11.Marion DW, et al. Treatment of traumatic brain injury with moderate hypothermia. New Engl J Med 1997;336(8):
540-6.
QUESTIONS:
36.1.
A.
B.
C.
D.
E.
Desirable goals for management of acute severe head injury include all EXCEPT:
Maintenance of CPP >70 mmHg
Initiation of ICP treatment at an upper threshold of 20-25 mmHg
Prophylactic hyperventilation to a PaCO2 of ≤35 mmHg
Maintenance of adequate oxygenation with a PaO2 >60 mmHg
Maintenance of euvolemia
36.2.
A.
B.
C.
Appropriate indications for ICP monitoring include all EXCEPT:
Patients with mild to moderate head injury
Patients with severe head injury (GCS of 3-8) and abnormal head CT scans
Patients with severe head injury and normal head CT scans if unilateral or bilateral motor
posturing is present
D. Patients in whom barbiturate coma is used to control ICP
E. Patients with severe head injury in whom sedation, analgesia, and paralysis are instituted
36.3.
A.
B.
C.
D.
The most common electrolyte abnormality encountered with Mannitol is:
Hyponatremia
Hypokalemia
Hypermagnesemia
Hypernatremia
36.4.
Upon arrival to the emergency room, the blood pressure of the patient described at the
beginning of the chapter should be:
Not treated
Raised
Lowered
None of the above
A.
B.
C.
D.
36.5.
A.
B.
C.
D.
E.
Cerebral Compliance is defined as:
CPP/ICP
∆V/∆P
∆P/∆V
MAP/CPP
None of the above
36.6.
All of the following treatment strategies may be employed to treat intracranial hypertension
EXCEPT:
Mild hyperventilation
Osmotic diuresis
Elevation of head of bed
Steroids
Cerebrospinal fluid diversion
A.
B.
C.
D.
E.
36.7.
A.
B.
C.
D.
E.
All of the following are true of Hypothermia, EXCEPT:
Decreases the metabolic demand of oxygen
Improves functional outcome in head injury
It is associated with lowered ICP
Severe systemic side effects may ensue at temperatures lower than 33oC
It may be less effective in patients with GCS of 3-4
190
ASCCA Residents Guide to the ICU
36.8.
A.
B.
C.
D.
E.
All of the following are true of Cerebral Autoregulation EXCEPT:
Autoregulation is effective in the injured brain
It maintains adequate cerebral blood flow over a wide range of MAP
Arterial CO2 is an important autoregulatory factor
It is unique to the brain
None of the above
36.9.
A.
B.
C.
D.
E.
Which of the following is true of mild hyperventilation:
A.It should be used for long intervals of time to maintain ICP
The target is a venous CO2 of 28-32
The target is an arterial CO2 of 30-35
The target is a venous CO2 of 30-35
It has no effect on cerebral ischemia
191
ASCCA Residents Guide to the ICU
37. Renal Protection
Daniel R. Brown, M.D., Ph.D.
A 74-year-old woman with a history of severe, poorly controlled hypertension presents for urgent coronary
artery revascularization. One week prior to admission she suffered an acute myocardial infarction
complicated by acute pulmonary edema requiring tracheal intubation and mechanical ventilation. Her
preoperative serum creatinine is 2.2 mg/dL. Cardiac surgery is uneventful except for a low urine output
throughout cardiopulmonary bypass (CPB) which persists into the postoperative period. In the ICU her
arterial blood gases and lung mechanics permit extubation the morning after surgery. However, oliguria
persists despite treatment with furosemide, ethacrynic acid, and bumetanide. Rising pulmonary artery
pressures and metabolic acidosis necessitates the institution of continuous veno-venous hemodialysis.
Her subsequent postoperative course is complicated by persistent acute renal failure, intermittent
pulmonary edema requiring reintubation, rapid atrial fibrillation, pneumonia, recurrent gastrointestinal
bleeding, and ileus. One month after surgery she remains in the ICU, having undergone a Billroth II
partial gastrectomy for a bleeding peptic ulcer and a tracheostomy to facilitate slow ventilatory weaning.
Acute renal failure is a devastating complication of surgery or trauma. As illustrated by the case above, it
profoundly increases the duration of ICU and hospital stay, and is associated with an in-hospital mortality
of 60-90%. Dialysis is able to control the immediate threats to life from hyperkalemia, metabolic acidosis,
fluid overload and acute uremia, but patients may have protracted ICU courses because of weakness and
debilitation, impaired wound healing, and multiorgan dysfunction, and may die from sepsis, bleeding, or
cardiovascular complications. Although full recovery from acute renal failure is possible, continuing
insults such as sepsis, bleeding, or hypotension (which may occur as a consequence of dialysis)
represent secondary ischemic insults to the kidney that delay or halt its ultimate recovery. The following
is an outline of topics related to the etiology and prevention of perioperative acute renal failure.
I.
Etiology of Acute Renal Failure (ARF)
A. Acute tubular necrosis (ATN)–90% of cases
1. Ischemic–shock, sepsis, aortic cross-clamping
2. Nephrotoxic
a) Endogenous–hemolysis, bilirubin, myoglobin, urate
b) Exogenous–aminoglycosides, cyclosporin A, amphotericin, NSAIDs, cisplatinum,
radiocontrast dye
B. Vascular injury
1. Arterial–embolism, thrombosis, trauma
2. Venous–thrombosis, elevated (intraabdominal) pressure
C. Vasomotor nephropathy (prerenal)–sepsis, liver failure, CHF
D. Abdominal compartment syndrome–intraperitoneal pressures ≥20 mmHg
E. Acute on chronic renal insufficiency
F. Systemic diseases–SLE, scleroderma, vasculitis, sickle cell crisis
G. Interstitial nephritis–allergy to penicillin antibiotics
H. Acute glomerulonephritis
II. Risk factors for perioperative ARF
A. High risk procedures
1. Cardiac surgery with cardiopulmonary bypass (CPB)
a) Effects of CPB on renal function
b) Effects of cardiac function
2. Major vascular surgery
a) Effects of suprarenal aortic cross-clamping
b) Effects of infrarenal aortic cross-claiming
c) Other complicating factors
3. Intraabdominal sepsis
4. Biliary surgery (obstructive jaundice)
5. Genitourinary surgery (obstructive uropathy)
192
ASCCA Residents Guide to the ICU
Trauma
a) Increased intraabdominal pressure
b) Rhabdomyolysis
c) Direct renal/urinary tract injury
7. Complicated obstetrics
8. Radiologic procedures using radiocontrast dye
B. Patient risk factors and predisposing surgical complications:
1. Preexisting renal insufficiency (serum creatinine >2 mg/dL)
2. Advanced age
3. Diabetes, hypertension, abnormal lipids (diffuse atherosclerosis)
4. Hypovolemia (acute, chronic)
5. Sepsis
6. Hemolysis, rhabdomyolysis, jaundice
7. Coagulopathy, DIC, bleeding
6.
III. Diagnosis of Renal Failure
A. Urine flow rate (but 75% of ARF is nonoliguric)
B. Urinalysis–oliguria
1. Features of urine osmolality, sodium, FENa (in prerenal syndrome)
2. Features of urine osmolality, sodium, FENa (in ARF)
C. BUN
1. Causes of elevated BUN without ARF
2. Causes of low BUN with ARF
D. Serum creatinine
1. Relationship to GFR
2. Causes of low serum creatinine with ARF
E. 2 hr creatinine clearance
F. Renal ultrasound
G. Renal scintiscan
IV. Prevention of ARF
A. Maintenance of renal blood flow (RBF)
1. Maintenance of preload (intravascular volume)
2. Maintenance of cardiac output
a) Use of inotropic agents
b) Use of afterload reduction–effects of vasodilators on renal function
B. Prevention of tubular obstruction (pigment nephropathy)
1. Use of volume loading
2. Mannitol, furosemide (controversial)
C. Maintenance of renal perfusion pressure
1. Situations with loss of renal autoregulation:
a) Sepsis
b) ARF
c) CPB (?)
d) Spinal/epidural anesthesia (?)
2. Interventions to normalize renal perfusion pressure
a) Fluid administration
b) Pressors-norepinephrine, vasopressin
D. Pharmacologic protection
1. Dopaminergic agonists–dopamine, fenoldopam
a) Effects on renal blood flow (renal protection)
b) Effects on sodium reabsorption (diuresis)
2. Mannitol
a) Dosing and administration
b) Actions and mechanisms
c) Side-effects
3. Loop diuretics
a) Dosing and administration
193
ASCCA Residents Guide to the ICU
4.
5.
6.
b) Actions and mechanisms
c) Side-effects
Calcium channel blockers
a) Role in hypertension
b) Role in protection against nephrotoxic insults
N-Acetylcysteine
a) Dosing and proposed mechanisms
b) Role in contrast-mediated nephrotoxicity
Sodium bicarbonate
a) Dosing and proposed mechanisms
b) Role in contrast-mediated nephrotoxicity
V. Treatment of oliguria
A. Examination of urine (see above) and collecting system
B. Restore intravascular volume and renal perfusion pressure
C. Diuretic therapy in the setting of hypervolemia considered if increased urine output would aid in
patient management
1. Furosemide bolus
2. Furosemide infusion
3. Combination therapy (furosemide + others)
a) Dopamine, fenoldopam
b) Bumetanide, hydrodiuril, metolazone
This chapter is a revision of the original chapter authored by Rob Sladen, M.D.
READING LIST:
1. Gribes AR. Prevention of acute renal failure: role of .vaso-active durgs, mannitol and diuretics. In J Artif Organ
2004; 27: 1049-53.
A comprehensive review of the role of various medications in preventing acute renal failure.
2. Kelly AM, Dwawena B, Cronin P, Bernstein SJ, Carlos RC. Meta-analysis: effectiveness of drugs for preventing
contrast-induced nephropathy. Ann Int Med 2008; 148; 284-94.
The authors conclude that N-acetylcysteine is effective for the prevention of contrast-induced nephropathy.
3. Karthik S and Lisbon A. Low-dose dopamine in the intensive care unit. Semin Dial 2006; 19: 465-71.
The use of low-dose dopamine should be abandoned.
4. Benedetto U, Sciarreta S, Roscitano A, et al. Preoperative angiotensin-converting enzyme inhibitors and acute
kidney injury after coronary artery bypass grafting. Ann Thorac Surg 2008; 86: 1160-5.
A propensity bases analysis of over 500 patients undergoing CABG with bypass showed that administration of
preoperative ACE inhibitors was associated with a reduced rate of acute kidney injury.
5. Mehta RL, Pascual MT, Soroko S, et al. Diuretics, mortality and nonrecovery of renal function in acute renal failure.
JAMA 2002; 288; 2547-601.
The use of diuretics in critically ill patients was associated with a higher risk of death.
6. Jones DR and Lee HT. Perioperative renal protection. Best Pract Res Clin Anaesthesiol 2008; 22: 193-208.
A nice review of the subject.
194
ASCCA Residents Guide to the ICU
QUESTIONS:
37.1.
A.
B.
C.
D.
E.
37.2.
A.
B.
C.
D.
37.3.
A.
B.
C.
D.
E.
37.4.
A.
B.
C.
D.
E.
A 28-year-old man is admitted to the ICU following emergency exploratory laparotomy for
injuries sustained in a motor vehicle accident. At laparotomy he was found to have splenic
and hepatic lacerations, which were repaired, and a large intraabdominal hematoma, which
was evacuated. He also sustained a fractured left femur, which was immobilized by external
fixation. Two hours after admission to the ICU his urine flow remains only 15 ML/hr. Which of
the following urinary findings is most consistent with a diagnosis of a prerenal state?
Fractional excretion of sodium of 3.5%
Urine specific gravity of 1.010
Urine sodium of 12 mEq/L
Urine to plasma osmolar ration of 1.0
Urine to plasma creatinine ration of 15
The urine is tested for myoglobin and is found to be positive. Appropriate interventions
include:
Send blood for creatinine phosphokinase (CPK) level
Add 50 mEq/L sodium bicarbonate to maintenance IV fluid
Increase maintenance IV fluid to 200 mL/hr
All of the above
Over the next 6 hours the patient’s abdomen is noted to become progressively more
distended. HR = 110/min, BP = 115/66, pulmonary artery pressure (PAP) = 34/18, pulmonary
artery occlusion pressure (PAOP) = 16 mmHg, cardiac output = 6.8 L/min. Indirect
measurement of intraabdominal pressure via the urinary catheter reveals it to be 24 mmHg.
Urine flow has declined from 80 to 25 mL/hr. The most appropriate intervention to restore
urine flow at this point is:
Furosemide 20 mg IV
Mannitol 6.25 g IV
Dopamine 3 mcg/kg/min infusion
Urgent return to the OR for exploratory laparotomy
All of the above
During infrarenal cross-clamping for aortic aneurysm resection in a 54-year-old, 78 kg man,
the urine flow rate declines to 15 mL/hr. HR = 92 beats/min, BP = 145/90, PAP = 25/14,
PAOP = 12 mmHg, cardiac output = 4.6 L/min. Dopamine is started at 3 mcg/kg/min and
urine flow subsequently increases to 55 mL/hr. The most likely explanation for this response
to dopamine is:
Increase in cardiac output
Increase in renal blood flow
Increase in glomerular flow rate
Decrease in tubular sodium reabsorption
Decrease in presynaptic norepinephrine release
195
ASCCA Residents Guide to the ICU
38. Renal Replacement Therapy
Ruben J. Azocar, M.D.
A 65-year-old female presents to the ER with nausea and vomiting and abdominal distention. A CT scan
reveals a pelvic mass and a distended cecum. She is emergently taken to the OR for an exploratory
laparotomy with en block removal of pelvic contents. However it is noted that her cecum is perforated. A
diverting ileostomy is performed and the patient is taken to the ICU. In the ICU she remains hypotensive
and a diagnosis of septic shock is made. A few hours after admission, it is noted that he urine output
drope to less than 0.5cc/kg/hr and that her baseline creatinine has doubled.
Among critically ill patients, acute kidney injury (AKI) requiring dialysis is associated with mortality rates
generally in excess of 50%.1 Besides longer hospitalization, higher cost of therapy are associated with
this entity.2 Furthermore, some literature supports the aggressive use of renal replacement therapy in the
critically ill that develops AKI. 3 One prospective study was performed to determine if daily dialysis results
in better survival than every other day. One hundred sixty patients with acute renal failure were assigned
to receive daily or conventional intermittent hemodialysis. Patients in the daily dialysis group had better
uremia control, lower mortality (22 vs. 38%) and shorter time to renal recovery (9 vs. 16 days compared to
the conventional group).
I.
II.
Indications for Renal Replacement Therapy
A. Fluid Overload
B. Hyperkalemia
C. Hyper/Hyponatremia
D. Metabolic Acidosis
E. Uremia (BUN > 100)
F. Progressive Azotemia without signs of uremia
G. Dialyzable toxin overdose
H. Oliguria/Anuria
I. Fluid Overload
J. Sepsis?
Classification
Dialysis is a generic name used commonly to refer to renal replacement therapy (RRT). There are
multiple types of RRT. Intermittent hemodialysis (IHD), peritoneal dialysis (PD or CAPD) and
continuous renal replacement therapy (CRRT) will be discussed. All of them have advantages and
disadvantages. An excellent review of these modalities can be found on-line.4 Before entering the
discussion about the different types of RRT, there are some basic differences between hemodialysis
(dialysis by diffusion) and ultrafiltration that are important to understand.
A. Diffusion dialysis
1. Solute diffuses from high to low concentration along an electrochemical gradient
2. Electrolyte solution runs countercurrent to opposite side of semi-permeable membrane filter
3. Larger molecules poorly removed/Better for smaller molecules
4. Solute removal proportional to dialysate rate
B. Ultrafiltration (UF)
1. Ultrafiltration works via Convection instead of Diffusion
2. Solute carried across membrane in response to a pressure gradient called a “solvent drag”
3. Mimics normal kidney function
4. Rate of UF proportional to hydrostatic pressure and blood flow
5. Very effective for fluid removal and middle molecules (i.e. cytokines in septic patients)
C. Semi-Permeable Membranes
In addition, there are two different types of semi-permeable membranes used for either dialysis or
CRRT:
196
ASCCA Residents Guide to the ICU
1.
The Cellulose-based
a) Thinner membrane/ Low permeability
b) Activates Inflammatory Response Cascade including Complement
c) Not typically used in critically ill
2.
The Synthetic
a) Thicker
b) Higher Sieving Coefficient = Better for convective clearance
c) Better for UF than diffusion dialysis
d) If used on dialysis some UF will still occur
e) Used in ICU patients
III. Intermittent Hemodialysis (IHD)
A. Most traditional approach
B. Requires central vein access or arteriovenous fistula
C. A specialized pump provides the driving force
D. Eliminates–solutes very effectively and fluid
E. Hemodynamic instability related to impaired sympathetic response, volume and solute shifts, and
buffers, therefore not suitable for many ICU patients as it may worsen ischemic injury to kidneys
or other organs
F. Therefore daily IHD typically used for ICU patients that can tolerate fluid shifts.
G. Requires specialized personnel and machinery
IV. Peritoneal dialysis
A. Represents an alternative to IHD because of patient preference in the outpatient setting, and
because of less hemodynamic fluctuations in the ICU setting when continuous RRT is not
available
B. Cost effective and simple
C. No need for heparin or venous access
D. Requires peritoneal access (surgical or percutaneous)
E. An osmotic gradient acts as driving force
F. Eliminates–fluids and, to a lesser extent, solutes resulting in poor solute and uremic control
G. May compromise diaphragmatic excursion
H. Increased potential for infection
V. Continuous Renal Replacement therapy (CRRT)
A. More suitable for the hemodynamically unstable patients
B. Less drastic fluid shifts and precise volume control, which is immediately adaptable to changing
circumstances.
C. Very effective control of uremia, hypophosphatemia and hyperkalemia.
D. Rapid control of metabolic acidosis
E. Improved nutritional support (full protein diet).
F. May have an effect as an adjuvant therapy in sepsis.
G. Probable advantage in terms of renal recovery.
H. Diffusion or Convection or both
I. Effective at regulating very specific fluid losses/gains throughout a specific period of time (i.e. -1.5
L/day)
J. Requires venous access and anticoagulation
K. Available 24 hours a day with minimal training.
L. Hypothermia.and severe depletion of electrolytes – particularly K+ and PO4, where care is not
taken.
M. Three major subtypes
1. Continuous veno-venous hemofiltration (CVVH)
2. Continuous veno-venous hemodialysis (CVVHD)
3. Continuous veno-venous hemodiafiltration (CVVHDF)
4. Continuous arteriovenous hemofiltration (CAVH)
5. Continuous arteriovenous hemodiafiltration (CAVHD)
197
ASCCA Residents Guide to the ICU
VI.
CVVH
A. Convection Dialysis
B. UF rate high
C. Replacement electrolyte solution is required to maintain hemodynamic stability
D. This mode is also very effective for clearing mid sized molecules, such as inflammatory cytokines.
E. It is hypothesized that removal of such mediators may play a role in improving outcome in sepsis.
F. A very simple version of this is slow continuous ultrafiltration, (SCUF) which is used for volume
control in fluid overloaded patients.
G. SCUF does not require the use of replacement fluid, and fluid removal is 300ml to 500ml per
hour.
VII. CVVHD
A. Continuous diffusive dialysis,
B. The dialysate is driven in a direction countercurrent to the blood.
C. This provides reasonably effective solute clearance, although mostly small molecules are
removed.
VIII. CVVHDF
A. Better solute clearance than CVVH
B. Most popular in ICU
C. Combines Convection and Diffusion
D. Medium and Small solutes removed
E. Dialysate and Replacement fluid used
IX.
CAVH
A. Access–artery and vein
B. Driving force – arterial blood pressure
C. Eliminates – fluid and solutes
D. Prevents hemodynamic instability
E. Requires arterial access and heparinization
F. Filtration rates may decrease to less than 20 mL/minute
X.
CAVHD
A. Access–artery and vein
B. Driving force – arterial pressure
C. Eliminates–fluid and solutes to a greater degree than CAVH
D. Prevents hemodynamic instability
E. Disadvantages–requires arterial access and heparinization
XI.
IHD vs CRRT
Mortality in the patients that develop AKI remains high. The literature suggests an aggressive
approach may be beneficial to these patients in terms of the use of RRT. In recent years use of
CRRT has become more frequent in the critically ill. One of the reasons for this observation
includes the fact that CRRT ensures adequate creatinine clearance while preserving hemodynamic
stability. This fact may prevent secondary ischemic injury to the kidneys or other organs that could
occur during hypotension related to hemodialysis. In addition CRRT is superior to intermittent
hemodialysis for volume control. Although still controversial, CRRT with UF might clear or filter the
plasma of pro-inflammatory cytokines modulating the immune response during SIRS or sepsis.
There are not strong data that support that this property alters outcomes in humans. Furthermore,
there is no strong evidence that CRRT is superior that HD. A recent study found worse outcomes
with CRRT when compared with IHD but the sample was small and the authors recommended
larger, prospective, randomized clinical trials to compare these modalities in severe AKI. 2 In
addition, meta-analysis comparing the two modalities did not found a difference.5 It seems for the
actual evidence that what is important is to aggressively initiate RRT in patient that develop AKI.
This chapter is a revision of the original chapter authored by C. William Hanson, III, M.D.
198
198
ASCCA Residents Guide to the ICU
REFERENCES:
1. Cho K, Himmelfarb J, Paganini E Survival by Dialysis Modality in Critically Ill Patients with Acute Kidney
Injury .J Am Soc Nephrol 17: 3132-3138, 2006.
2. Fuiano G, DiFillippo S, Memoli B: Guidelines for dialysis. Replacement therapy for acute renal failure in critically
ill patients. G Ital Nefrol 2004; 21:S28: S1-S10
3. Schiffl H, Lang S, Fischer R: Daily Hemodialysis and the Outcome of Acute Renal Failure NEJM 2002; 346:305-310.
4. http://www.ccmtutorials.com/renal/RRT/index.htm
5. Kellum JA, Angus DC, Johnson JP: Continuous versus intermittent renal replacement therapy: a meta-analysis.
Intensive Care Med 2002; 28:29-37.
QUESTIONS:
38.1.
A.
B.
C.
D.
Acute kidney injury in the critically ill:
It is a relatively infrequent entity
It is better treated with every other day diffusion dialysis
It is associated with a mortality between 40-65%
It has no impact in ICU or Hospital stays
38.2.
A.
B.
C.
D.
The following are true in regards to CRRT EXCEPT:
May be of benefit in the septic patient
A better control of the hemodynamics and fluid shift is possible
It may lead to hypothermia
It is better than intermittent hemodialysis in the critically ill
38.3.
A.
B.
C.
D.
The following are true in regards to peritoneal dialysis EXCEPT:
Has lower costs
It is very effective in solute removal
Has been largely replaced by CRRT in the ICU setting
Does not require anticoagulation
38.4.
A.
B.
C.
D.
Of the options for CRRT in the ICU the one that is more popular is
CVVH
CVVHD
CVAHDF
CVVHDF
38.5.
A.
B.
C.
D.
In relation to the membranes use during dialysis, the following are incorrect EXCEPT:
There is no difference among the membranes used
The natural cellulose based membrane is more favorable for the critically ill
The synthetic membrane is thicker
The synthetic membrane is better for ultrafiltration purposes
199
ASCCA Residents Guide to the ICU
39. Toxicology and Support of Patients with Drug
Overdoses
Matthew D. Koff, M.D., M.S.
A 26-year-old male is found down at home by his girlfriend after they had an argument over the phone.
EMS was called and transported the patient to a local emergency department. He is spontaneously
breathing with a non-rebreather mask over his face collapsing the reservoir bag and breathing.at a rate of
40 breaths/min, HR is 120, BP is 138/82, He has an altered level of consciousness. A finger stick in the
ED shows glucose of 186. ECG shows sinus tachycardia with normal QRS intervals and with a normal
axis. An ABG performed shows a pH of 6.9 pCO2 21 pO2 420 HCO3 5 and an anion gap of 22. A serum
osmolarity, aspirin level and acetaminophen is pending. How should this patient be immediately
managed.
Critically ill patients who are admitted to the intensive care unit with drug-related events test the
understanding and application of pharmacology and pathophysiology and medical history assessment.
Patients can present with drug toxicity secondary to side-effects, allergic reactions, or overdoses.
Additionally, toxic reaction to commonly encountered plants and animals can produce life-threatening
reactions. Occasionally, some patients in the ICU and even the OR will develop drug toxicity as a sequel
to iatrogenically administered medications. The initial step in management, irrespective of initiating
factors, is most often that of supportive care, which includes control of airway patency, oxygenation/
ventilation, and maintenance of circulation. Following initial assessment, these patients may require
monitoring and physiologic support in the ICU. The following is an outline of important topics for
management of a patient suffering from a toxicological emergency. Use of internet based resources such
as Micromedex and Clinical Pharmacology on-line is helpful to guide current treatment and management.
Contact with the local poison center can also offer immediate assistance and should be readily utilized.
I.
General management
A. Basic life support with careful and continuous evaluation of airway protection
B. Neurologic evaluation
1. Glasgow coma scale
C. Antagonist administration
1. Glucose
2. Naloxone
3. Thiamine
4. Flumazenil
5. Fomepazole
D. Laboratory evaluation
1. Acid-base disorders
2. Co-oximetry
3. Osmolar Gap
4. Toxicology screens
5. Electrocardiogram
E. Prevention of further absorption
1. Gastric lavage rarely used unless immediately after life threatening ingestion
2. Activated charcoal with cathartics single and multidose for specific ingestions
3. Emetics rarely indicated and used.
4. Whole bowel irrigation for extended release medications ie. bupropion
F. Enhanced elimination
1. Solute diuresis
2. Alkaline diuresis
3. Hemodialysis
4. Charcoal/resin hemoperfusion
200
ASCCA Residents Guide to the ICU
II. Pharmacokinetics
A. Absorption
B. Metabolism
C. Distribution
D. Excretion
III. Substances
A. Alcohol
1. Ethanol
a) Wernicke’s encephalopathy
b) Withdrawal/Delirium Tremens
c) Korsakoff’s psychosis
d) Alcoholic Ketoacidosis
2. Methanol
3. Ethylene Glycol
4. Isopropanol
B. Sedative-hypnotics
1. Barbiturates
a) Receptor physiology
b) Thio/oxybarbiturates
2. Gamma Hydroxy Butyrate (GHB)
C. Benzodiazepines
1. Ultra-short acting vs. short-acting vs. long-acting
2. Receptor physiology
3. Withdrawal
4. Antagonism
D. Cyclic Antidepressants
1. Classes
2. Cardiovascular toxicity
3. Treatment
4. CNS toxicity
5. Monitoring
E. Narcotics
1. Receptors
2. Physical exam
3. Drugs
4. Withdrawal
5. Antagonists
6. Agonist/antagonists
F. Salicylates
1. Acid-base disorders
2. Coagulopathy
3. ARDS
4. Fever
G. Acetaminophen
1. Rumack-Matthew nomogram
2. Hepatotoxicity
a) NAPQI
b) Glutathione
c) N-acetylcysteine
H. Cocaine
1. Organ Effects
a) Central nervous system effects
b) Cardiovascular effects
c) Respiratory effects
d) Gastrointestinal effects
e) Obstetrical considerations
2. Treatment
201
ASCCA Residents Guide to the ICU
I.
J.
a) Benzodiazepines
b) Beta and Alpha blockade
c) Sympatholytics
Cyanide
1. Nitroprusside
a) Physiochemistry
b) Dose
c) Acid-Base effects
d) Mixed venous saturation
e) Thiocyanate
f) Treatment
Carbon monoxide
1. Smoke inhalation
2. Non-invasive co-oximetry
IV. Iatrogenic
A. Malignant hyperthermia
B. Neuroleptic malignant syndrome
C. Chemotherapeutics
D. Radiation effects
E. Pro-arrhythmic drugs
F. Renal toxins
G. Hepatotoxins
H. Dermatologic prescriptions
V. Pediatric (One pill or mouthful can kill a 10 kg child)
A. Iron.
B. Chloroquine, quinine, quinidine and hydroxychloroquine
C. Clonidine
D. Sulfonylureas.
E. Tricyclic antidepressants
F. Lindane.
G. Diphenoxylate/atropine.
H. Beta blockers.
I. Theophylline.
J. Calcium channel blockers.
K. Camphor
L. Oil of wintergreen/salicylates
M. Ethylene glycol
N. Nose sprays and eyedrops.
O. Benzocaine
P. Opioids ie. methadone
READING LIST:
1. Hall JB, Schmidt GA, Wood LDH, editors. Principles of critical care. Toxicology in Adults Third Edition New York:
McGaw-Hill, Inc.; 2005 . pp. 1499-1545.
2. Hall JB, Schmidt GA, Wood LDH, editors. Principles of critical care. Critical Care Pharmacology Third Edition
New York: McGaw-Hill, Inc.; 2005 . pp. 1547-1571.
3. Mokhlesi B, Leiken JB, Murray P, Corbridge TC. Adult toxicology in critical care: part I: general approach to the
intoxicated patient. Chest 2003; 123(2):577-592.
4. Mokhlesi B, Leikin JB, Murray P, Corbridge TC. Adult toxicology in critical care: Part II: specific poisonings. Chest
2003; 123(3):897-922.
5. Baumgartner GR, Rowen RC. Clonidine vs. chlordiazepoxide in the management o acute alcohol withdrawal
syndrome. Arch Intern Med 1987;147:1223.
202
ASCCA Residents Guide to the ICU
6. Hakim AM, Pappius HM. Sequence of metabolic, clinical, and histological events in experimental thiamine
deficiency. Ann Neurol 1983;13:365.
7. Jacobsen D, McMartin KE. Methanol and ethylene glycol poisonings. Mechanism of toxicity, clinical course,
diagnosis, and treatment. Med Toxicol 1986;1:309.
8. Brent J, McMartin K, Phillips S, Aaron C, Kulig K. Fomepizole for the treatment of methanol poisoning. N Engl J
Med 2001; 344(6):424-429.
9. Brent J, McMartin K, Phillips S, Burkhart KK, Donovan JW, Wells M et al. Fomepizole for the treatment of ethylene
glycol poisoning. Methylpyrazole for Toxic Alcohols Study Group. N Engl J Med 1999; 340(11):832-838.
10.Skolnick P. The (gamma)-aminobutyric acid A (GABA) receptor complex and hepatic encephalopathy: some
recent advances. Ann Intern Med 1989;110:532.
11.Khantzian EJ, McKenna GJ. Acute toxic and withdrawal reactions associated with drug use and abuse. Ann Intern
Med 1979;90:361.
12.Frommer DA, Kulig KW, Marx JA, Rumack B. Tricyclic antidepressant overdose: a review. JAMA 1987;257:521.
13.Henry K – Deadly Ingestions. Pediatr Clin North Am - 01-APR-2006; 53(2): 293-315
This chapter is a revision of the original chapter authored by Todd Dorman, M.D.
QUESTIONS:
39.1.
A.
B.
C.
D.
Signs of cyanide toxicity include all EXCEPT which of the following:
Metabolic acidosis
Tachycardia
Seizures
Decreased mixed venous saturation
39.2.
A 31-year-old patient is brought to the emergency department after being found at home,
unconscious, lying next to a bottle of Tylox. Upon arrival to the ED, she is given naloxone
and glucose and promptly awakens. No other abnormalities are noted. What other drug
should be most likely administered to this patient?
Flumazenil
Pyridostigmine
Acetylcysteine
Ipecac
A.
B.
C.
D.
39.3. Isopropyl alcohol ingestion leads to an anion gap metabolic acidosis.
A. True
B. False
39.4.
A.
B.
C.
D.
E.
A comatose patient is admitted to the ICU and responds to sternal rub with withdrawal.
Neurologic evaluation shows increased deep tendon reflexes. Her ECG is interpreted as
sinus tachycardia with first degree heart lock and prolonged QRS and QTc with frequent
ventricular ectopic beats. The most likely cause of her symptoms is:
Aspirin
Heroin
Diazepam
Acetaminophen
Nortriptyline
203
ASCCA Residents Guide to the ICU
39.5.
A.
B.
C.
D.
E.
Prior to administering glucose to a patient with alcohol intoxication, what drug should be
administered?
Folate
Magnesium
Multivitamins
Thiamine
Clonidine
39.6. The benzodiazepine receptor is an integral part of the serotonin receptor complex.
A. True
B. False
39.7.
A.
B.
C.
D.
The most common symptom associated with an aspirin overdose in pediatric patients is:
Headache
Respiratory acidosis
Fever
Respiratory alkalosis
204
40. Solid Organ Transplantation
Zdravka D. Zafirova, M.D., Michael C. Woo, M.D.
A 48-year-old male patient with end-stage liver disease due to hepatitis C has undergone
orthotopic liver transplantation. On postoperative day 1, he is intubated and on mechanical
ventilation with FiO2 0.8. His ABG is pH 7.23, pCO2 42, pO2 58. His temperature is 38.8 В° C,
blood pressure 87/43 mmHg, heart rate 109 beats/min, and pulmonary artery pressure 51/26
mmHg. ALT and AST are 1780 and 1975 IU/L. Management considerations: 1) Differential
diagnosis, evaluation and treatment of respiratory failure; 2) Differential diagnosis, evaluation and
therapy of shock; 3) Differential diagnosis and evaluation of elevated liver enzymes; 4)
Management of infection issues; 5) Immunosuppression therapy.
The field of organ transplantation has witnessed an astounding growth in recent years as the
indications for transplant and the therapeutic options have expanded dramatically. Better
understanding of immune mechanisms has led to an array of new immunosuppression therapies
and has improved survival. However, transplant patients continue to present challenges to the
health care team due to complications related to their underlying disease processes, transplant
procedure, immunosuppression and impaired host defense mechanisms. The critical care
physician plays a pivotal role in the management of transplant patients and comprehensive
expertise in the issues facing these patients is essential in improving their life expectancy and
quality of life.
I.
Immunosuppression
A. Steroids
1. Mechanisms – lymphotoxicity, alteration of the distribution of lymphocytes, inhibition
of IL-1 production by macrophages
2. Toxicity – adrenal suppression, hypertension, diabetes, acne, hyperlipidemia, obesity,
osteoporosis, cataracts
B. Calcineurin inhibitors - Tacrolimus, Cyclosporine
1. Mechanism – receptors (cyclophilin, FK binding protein (FKBP), inhibition of
calcineurin (calcium-dependent phosphatase)
2. Toxicity
a) neurotoxicity – headache, tremor, seizures, delirium, coma
b) nephrotoxicity
c) hypertension
d) diabetes mellitus
e) lipid abnormalities
f) gingival hyperplasia
g) osteoporosis
h) alopecia
i) hyperkalemia, hypomagnesemia
C. Inhibitors of Purine and Pyrimidine synthesis
1. Mycophenolate mofetil
a) Mechanism – inhibition of inosine monophosphate dehydrogenase, guanosine
nucleotide synthesis, cytostatic effect on T, B lymphocytes
b) Toxicity - upper GI complaints, diarrhea, hematologic (leukopenia,
thrombocytopenia, anemia), teratogenic
2. Azathioprine
a) Mechanism - antagonizes purine metabolism
b) Toxicity – leukopenia, GI adverse effects (nausea and vomiting, hepatitis,
cholestasis, pancreatitis)
D. Inhibitors of Mammalian Target of Rapamycin (mTOR) – sirolimus, everolimus
1. Mechanism – inhibition of mTOR kinase, arrest in the cell cycle related to a block in
the G1-to-S phase transition
2. Toxicity – hyperlipidemia, hepatic artery thrombosis, thrombotic microangiopathy,
205
wound healing, dehiscence of airway anastomosis
E. Antilymphocyte antibodies
1. OKT 3
a) Mechanism - murine monoclonal antibody directed against the CD3 complex,
depletion of CD3 T cells
b) Toxicity - cytokine release syndrome (fever, rigors, headache, dyspnea,
pulmonary edema, GI side effects), post-transplantation lymphoproliferative
disease
2. Antithymocyte globulin
a) Mechanism – antithymocyte polyclonal antibody, lymphopenia via complement
mediated cell lysis
b) Toxicity – cytokine release syndrome, anemia, thrombocytopenia, leucopenia,
serum sickness, nephritis, post-transplantation lymphoproliferative disease
3. Anti–Interleukin-2 (IL-2) receptor antibody – basiliximab, daclizumab
a) Mechanism - chimeric and humanized monoclonal anti-CD25 antibodies
targeting IL-2 receptor on activated T cells
b) Toxicity – minimal
F. Novel agents – potential mechanisms include inhibition of T cell proliferation, interference
with cell-surface molecules regulating immune cell interactions, inhibition of signaling
mechanisms, altered trafficking and recruitment of immune cells responsible for rejection
G. Modes of therapy
1. Induction immunosuppression - intense prophylactic therapy used at the time of
transplantation, short term, high toxicity potential
2. Maintenance immunosuppression – chronic therapy of lesser potency, more
tolerability
3. Rescue therapy - intense, chronically intolerable therapy applied in response to a
rejection episode
II. Heart transplant
A. Indications and contraindications
1. Indications
a) Severe heart failure NYHA class III-IV uncontrolled on maximal medical therapy,
requiring continuous intravenous inotropes
b) Hypertrophic cardiomyopathy, congenital heart disease, valvular heart disease
c) Severe coronary artery disease untreatable by medical therapy or
revascularization
d) Refractory ventricular arrhythmias not controlled with medical or device therapy
2. Contraindications
a) absolute
(1) Malignancy with high recurrence potential
(2) Irreversible severe pulmonary hypertension
(3) Active infection (including HIV)
(4) Irreversible severe neurologic, hepatic, renal, pulmonary dysfunction
(combined transplant selectively) or peripheral vascular disease
(5) Severe mental or psychological defects or substance abuse
b) relative
(1) Advanced age
(2) Unfavorable socioeconomic factors
(3) Active systemic illness that would limit long-term survival
3. List categories
a) Status 1A
(1) Mechanical devices (VAD for < 30 days, ECMO, IABP, ventilator)
(2) VADs > 30 days with device related complications
(3) High dose single inotropes with pulmonary artery catheter, dobutamine at 7.5
mcg, milrinone at .5 mcg
(4) Multiple inotropes with pulmonary artery catheter
b) Status 1B
(1) VAD for > 30 days
206
(2) Inotropic support (regardless of location of patient-ICU without PA catheter,
floor or home)
c) Status 2
(1) All other patients actively listed for heart transplant
B. Preoperative evaluation
1. Cardiac function
a) medications
b) devices – ventricular assist device (VAD), intraaortic balloon counterpulsation
(IABP), pacemaker/AICD
2. Pulmonary function
3. CNS function
4. Renal function
5. Coagulation status
6. Infectious issues
7. Socioeconomic factors
C. Perioperative management
1. Surgical procedure – biatrial or bicaval orthotopic, heterotopic
2. Intraoperative considerations
a) Anesthetic plan
b) Intraoperative monitoring – invasive arterial, central venous and pulmonary artery
pressure, echocardiography
c) Respiratory management
d) Hemodynamic support – inotropes, vasoactive therapy, rhythm control
e) Fluid, blood and coagulation management
f) Cardiopulmonary bypass – mechanics, physiologic alterations, anticoagulation
3. Postoperative management
a) Ventilatory support
(1) optimization of oxygenation and ventilation, effects on hemodynamics
(2)
ventilatory therapy – volume control vs. pressure control, PEEP,
noninvasive ventilation, inhaled bronchodilator and pulmonary
vasodilator therapy
(3) baseline pulmonary dysfunction
(4) perioperative lung injury
b) Hemodynamic management
(1) physiology of the denervated heart
(2) evaluation of cardiovascular function – perfusion, preload, contractility,
rhythm
(3) filling pressures, pulse pressure variation
(4) thermodilution monitoring
(5) mixed venous oxygen saturation
(6) echocardiography
(7) hemodynamic support
(a) inotropes, vasopressors, vasodilators
(b) IABP
c) Coagulopathy
d) Immunosuppression
(1) induction, maintenance, need for rescue
(2) choice of therapy, monitoring
e) Infection control
(1) antimicrobial prophylaxis and therapy
(2) strict antisepsis in care for the patient and devices
(3) isolation as indicated
D. Complications
1. Right ventricular failure
a) Inotropic support
b) Control of pulmonary vascular resistance
c) Respiratory considerations
2. Rejection
207
Timing, type, grade - hyperacute, acute cellular, acute humoral (vascular), or
chronic
b) Diagnosis
(1) clinical presentation
(2) biochemical markers
(3) imaging – echocardiography and cardiac MRI
(4) endomyocardial biopsy
c) Long-term monitoring
d) Therapy
Primary graft dysfunction
Coronary artery vasculopathy
Respiratory failure
CNS complications
Complications related to immunosuppression
a)
3.
4.
5.
6.
7.
III. Lung transplantation
A.Indications
1. Cystic fibrosis
2. Bilateral bronchiectasis
3. Pulmonary vascular disease
a) Primary pulmonary hypertension
b) Cyanotic congenital heart disease, Eisenmenger’s complex (heart-lung)
4. Obstructive lung disease – emphysema, α1-Antitrypsin deficiency
5. Restrictive lung disease
a) Idiopathic pulmonary fibrosis
b) Sarcoidosis
c) Occupational lung disease
6. Lymphangioleiomyomatosis
B. Preoperative evaluation
1. Respiratory system
a) Clinical status
b) Imaging and function testing – CXR, CT scan, pulmonary function tests,
cardiorespiratory exercise test
2. Cardiovascular system
a) Clinical status
b) Tests – ECG, echocardiography, stress test, cardiac catheterization
3. Medical therapy
C. Perioperative care
1. Surgical procedure
a) Single-lung vs. double-lung vs. lung-heart
b) End-to-end vs. telescopic anastomosis
c) Cardiopulmonary bypass
2. Anesthetic considerations
a) Regional anesthesia for postoperative pain control
b) Monitoring – invasive pressures, echocardiography
c) Respiratory management – double lumen ETT, ventilation modes
d) Hemodynamic support
e) Fluid and blood product strategy
3. Postoperative management
a) Respiratory management
(1) Ventilation modes and weaning strategy
(2) Airway management
(3) Monitoring – lung mechanics and gas exchange, bronchoscopy, biopsy
(4) Chest therapy
(5) ECMO
b) Hemodynamic support
(1) Intravenous fluid therapy and diuresis
(2) Immunosuppression
208
c) Infection control
D. Complications
1. Pulmonary
a) Airway – ischemia/necrosis, anastomostic dehiscence, strictures, obstruction
b) Reperfusion injury
c) Acute rejection
d) Diaphragmatic dysfunction, phrenic nerve injury
e) Hemorrhage
f) Bronchiolitis obliterans
g) Acute and chronic native lung hyperinflation
h) Pleural complications- pneumothorax, air leak, effusion
i) Lung cancer
2. Cardiovascular
a) Shock
b) Pulmonary embolism
c) Arrhythmias
3. Renal
a) Infection
b) Posttransplantation lymphoproliferative disease
IV. Hepatic transplantation
A. Indications
1. Chronic or progressive hepatic failure
a) Cirrhosis
(1) Primary biliary cirrhosis
(2) Alcoholic liver disease
b) Primary sclerosing cholangitis
c) Fulminant viral hepatitis/hepatic failure
d) Alpha-1 antitrypsin deficiency
e) Budd-Chiari syndrome
f) Wilson’s disease
g) Drug/toxin related
h) Idiopathic
2. Conditions with normal hepatic function
a) Hemophilia
b) Protein C deficiency
c) Oxalosis
3. MELD score for organ allocation, est. 2002
a) 3.8*[loge(bilirubin)] + 11.2*[loge(INR]) + 9.6*[loge(creatinine)] + 6.4*[etiology score]
where bilirubin and creatinine are in mg/dL, etiology score is 0 if cholestatic or
alcoholic, 1 if other
b) fallacies
(1) designed from TIPS outcomes, adopted into organ transplantation
(2) negative score is possible
(3) does not address the patient with multiple etiology hepatic failure
(4) variability in serum creatinine measurement in the setting of
hyperbilirubinemia may affect score
c) Modified MELD score (6-40)
(1) Avoid unfair advantage of intrinsic renal disease
(2) Modifiers
(a) time on waiting list used as tiebreaker
(b) special conditions
i) acute liver failure (status 1)
(1) fulminant failure
(2) early graft failure - graft primary non-function, hepatic artery
thrombosis
ii) hepatocellular carcinoma
(1) tumor burden
209
(2) predicted survival
iii) hepatopulmonary syndrome
(1) PaO2 <60 mmHg on RA
iv) metabolic diseases
(1) familial amyloidosis
(2) primary oxaluria
B. Preoperative care
1. Central Nervous System – cerebral edema, hepatic encephalopathy
2. Cardiovascular – low systemic vascular resistance, increased cardiac output
3. Respiratory – hepatopulmonary syndrome, portopulmonary hypertension
4. Renal – hepatorenal syndrome
5. Metabolic – hypoglycemia, metabolic acidosis
6. Hematological – thrombocytopenia and thrombocyte dysfunction, coagulopathy
7. Gastrointestinal- ascites, esophageal varices, hyperbilirubinemia
8. Immune dysfunction – infection, sepsis
9. Liver support devices – bioartificial, artificial
C. Intraoperative considerations
1. Surgical procedure – orthotopic, living-related, end-to-side anastomosis of donor
hepatic vein, venovenous bypass
a) Preanhepatic phase
b) Anhepatic phase
c) Reperfusion – hemodynamic, metabolic disturbances, reperfusion syndrome
d) Neohepatic phase
(1) Anesthetic plan – aspiration risk, pharmacokinetics alterations (volume of
distribution, metabolism, protein binding, encephalopathy
(2) Monitoring – invasive pressure monitoring (arterial venous, pulmonary
artery), cardiac output, ABG
(3) Fluid and blood management
D. Postoperative care
1. Respiratory support and weaning
a) Modes of mechanical ventilation
b) Oxygen delivery and consumption
c) Perioperative lung injury – TRALI, sepsis
2. Circulation support
3. Coagulopathy management
a) Monitoring – routine coagulation tests, thromboelastogram
b) Treatment – replacement of factors with plasma, cryoprecipitate; recombinant
factor VII, antifibrinolytics; DDAVP; platelet transfusion
4. Monitoring graft function
a) Biochemical markers
b) Ultrasound evaluation
c) Angiography
d) Liver biopsy
e) Early poor function vs. primary non-function
f) Predictors of poor graft function
g) Indicators of poor graft function
5. Immunosuppression
6. Infection control and prophylaxis
E. Complications
1. Cardiovascular – hemodynamic instability, hypertension
2. Pulmonary – pulmonary edema, pleural effusions, infection, hepatopulmonary
syndrome, pulmonary hypertension
3. Neurological – immunosuppression, fluid and metabolic disturbances
a) Seizures
b) Encephalopathy
c) Cerebral edema with increased ICP
d) Central pontine myelinolysis
e) CNS infections
210
4.
5.
6.
7.
8.
f) Intracranial hemorrhage
Graft dysfunction
a) Markers
b) Failure of the biliary system - leaks vs. obstruction
c) Rejection – acute, chronic
d) Vascular thrombosis – arterial, venous
Renal dysfunction
a) Effect on outcome
b) Acute Tubular Necrosis
c) Hepatorenal syndrome
d) Nephrotoxicity of immunosuppressive drugs
e) Possible interventions
Bleeding
Infection
Nutritional complications
V.
Renal Transplantation
A. Renal Failure–etiologies and co-morbidities
1. Cardiovascular
a) Hypertension
b) Coronary artery disease
c) Heart failure
2. Pulmonary
a) Pleural effusions
3. Metabolic and electrolyte alterations
4. Endocrine
a) Diabetes mellitus
b) Hyperparathyroidism
5. Platelet function and hematopoiesis
6. Perioperative renal replacement therapy
B. Intraoperative considerations
1. Surgery – cadaveric vs. living-related, orthotopic vs heterotopic
2. Anesthetic plan – induction, pharmacokinetic alterations
3. Monitoring – invasive pressures, urine output, cardiac
4. Hemodynamic support
5. Fluid and electrolyte issues
C. Postoperative care
1. Hemodynamic support - treatment of hypo- and hypertension
2. Replacement of fluid, correction of electrolyte and acid-base abnormalities
3. Immunosuppression – monitor drugs
4. Infection control
D. Complications
1. Assessment and treatment of oliguria
2. Graft dysfunction
a) Acute tubular necrosis
b) Rejection – hyperacute or accelerated acute
c) Drug toxicity
d) Vascular thrombosis – arterial or venous
e) Anastomotic leaks
3. Infection
VI.
Pancreas Transplantation
A. Co-morbidities of diabetes mellitus
B. Surgical technique
1. Placement of graft
2. Drainage of exocrine secretions
C. Postoperative care
1. Difficulties related to preexisting co-morbidities
211
2.
3.
Monitoring of graft function
Blood glucose and urinary amylase
VII. Intestinal Transplantation
A. Indications
1. Gastroschisis
2. Jejunoileal atresia
3. Midgut volvulus
4. Necrotizing enterocolitis
5. Resection for intestinal or other extensive abdominal tumor
6. Mesenteric artery or vein occlusion/thrombosis
7. Extensive resection of bowel for IBD, trauma, obstruction
8. Preoperative evaluation – status of other organs, i.e. liver
9. Perioperative considerations
a) Fluid and electrolyte management
b) Metabolic abnormalities
c) Nutritional issues
10. Complications
a) Graft dysfunction and rejection
b) Parenteral nutrition-related complications
c) Infection
This chapter is a revision of the original chapter authored by Jonathan T. Ketzler, M.D.
READING LIST:
1. Hirose R, Vincenti F. Immunosuppression: Today, Tomorrow, and Withdrawal. Semin Liver Dis
2006;26:201–210.
Review of the current state of immunosuppression therapy.
2. Kirk A. Induction Immunosuppression. Transplantation 2006; 82: 593–602.
Detailed view on induction immunotherapy.
3. Lin S, Cosgrove CJ. Perioperative Management of Immunosuppression. Surg Clin N Am 2006; 86:1167–
1183.
Excellent discussion of pharmacodynamics, pharmacokinetics and adverse effects of perioperative
immunosuppression.
4. Miller LW. Heart transplantation. In: Civetta JM, Taylor RW, Kirby RR et al., eds. Critical Care. 3rd edition.
Philadelphia: Lippincott-Raven Publishers, 1997, pp.1333-1340.
A comprehensive review of the care of the heart transplant recipient.
5. Lindenfeld J et al. Drug Therapy in the Heart Transplant Recipient Part I: Cardiac Rejection and
Immunosuppressive Drugs. Circulation. 2004;110:3734-3740
6. Miyoshi S et al. Physiologic aspects in human lung transplantation. Ann Thorac Cardiovasc Surg 2005;
11(2):73-79.
7. Pierre AF et al. Lung transplantation: donor and recipient critical care aspects. Current Opinion in Critical
Care 2005, 11:339—344.
8. Granton J. Update of early respiratory failure in the lung transplant recipient. Current Opinion in Critical
Care 2006, 12:19–24
9. Khan SA et al. Acute Liver Failure: a Review. Clin Liver Dis 2006; 10:239–258. Review of acute liver failure
management.
10. Brunson ME, Howard RJ. Orthotopic liver transplant. In: Civetta JM, Taylor RW, Kirby RR et al., eds.
Critical Care. 3rd edition. Philadelphia: Lippincott-Raven Publishers, 1997, pp.1341-131352.
A comprehensive review of the care of the liver transplant recipient.
212
QUESTIONS:
40.1. Which of the following mechanism of action of immunosuppression drug is correct:
A.
Tacrolimus inhibits mTOR
B.
Sirolimus inhibits calcineurin
C.
Mycophenolate mofetil inhibits inosine monophosphate dehydrogenase
D.
Azathioprine inhibits CD3 complex on the T cells
E.
Basiliximab inhibits purine synthesis
40.2.
A.
B.
C.
D.
E.
40.3.
A.
B.
C.
D.
E.
40.4.
A.
B.
C.
D.
E.
40.5.
A.
B.
C.
D.
E.
40.6.
A.
B.
C.
D.
E.
All of the following adverse effects of immunosuppression are correctly matched with
the causative agent EXCEPT:
Cyclosporine – renal dysfunction
Sirolimus – hepatic artery thrombosis
Tacrolimus – seizure
Mycophenolate – diabetes
Antithymocyte globulin – cytokine release syndrome
What is the least likely cause of postoperative cardiac failure on day 2 after heart
transplant:
Acute rejection
Right ventricular failure
Primary graft dysfunction
Coronary artery vasculopathy
Sepsis
All of the following are causes of early acute respiratory failure after lung transplant
EXCEPT:
Graft failure
Bronchiolitis obliterans
Reperfusion injury
Infection
Diaphragmatic dysfunction
All of the following are likely causes of neurologic dysfunction in the first
postoperative days after liver transplant EXCEPT:
Seizure
Cerebral edema
Central pontine myelinolysis
Intracranial hemorrhage
Disseminated aspergillosis
All of the following are likely causes of renal dysfunction after kidney transplant
EXCEPT:
Mycophenolate mofetil
Acute rejection
Anastomotic leak
Vascular thrombosis
Acute tubular necrosis
213
41. Organ Donation and Procurement in the ICU
Jocelyn A. Park, M.D.
A 35-year-old male was admitted to the ICU via the emergency department after a single vehicle
unhelmeted motorcycle accident. He was intubated at the scene after an undetermined period of
hypoventilation and was found to have fixed and dilated pupils. In the emergency department
trauma lines were placed for resuscitation and he was diagnosed with a closed head injury, rib
fractures and a femur fracture. He had an intracranial pressure (ICP) monitor placed in the ICU
for control and monitoring of his intracranial pressures. One day later, after aggressive medical
therapy, he remains with unreactive pupils. Brain death determination is undertaken. The ICU
team discusses the patient’s poor prognosis with the family and contacts the organ donation
coordinator that brain death determination is being undertaken.
Organ donation in the United States is of growing importance and both correct identification and
management of potential donor patients is critical to organ salvage. As of 2008 there were more
than 98,000 patients on the waiting list for organ transplantation. From January-September 2007,
more than 20,000 organ transplants were performed in the United States, four-fifths of which
came from deceased donors.
Historically, most donors in the past twenty years have been declared brain dead in the ICU and
then stabilized and optimized while the transplant team is mobilized. However, due to the
shortage in organs and overwhelming need, there has been a renewed interest in the use of
donation after cardiac death.
In the brain dead donor, pre-donation care in the intensive care should focus on optimizing organ
function. This approach includes maintaining perfusion and oxygenation and treatment of
common complications.
I.
Importance in US Medicine
A. Waiting list for transplant, greater than 98,000
1. Waiting list for kidney: nearly 75,000
B. Widening gap between number of donors and recipients
1. Only 15-20% of potential candidates become donors
2. Estimated 25% of potential donors have perfusion events causing them to be
unsuitable for donation
C. Consent issues
1. Rates from 33-45%
2. Consider decoupling—separation of notification of death and request for donation to
increase rates
3. Critical pathways published by United Network for Organ Sharing to increase rates
both through consent and ICU management standardization
II. Recognition of Donors/Inclusionary and Exclusionary Criteria
A. Flexible criteria of inclusion including no absolute age cut-off
B. Absolute exclusion criteria
1. HIV infection
2. Human T-cell leukemia-lymphoma virus
3. Systemic viral diseases
4. Prion-related disease
5. Herpetic meningoencepalitis
6. Active malignant disease with exceptions of non-melanoma skin cancers and certain
primary brain tumors
C. All other criteria are relative
214
III. Diagnosis of Brain Death/Donation after Cardiac Death
A. Declaration of brain death
1. Total lack of function of the whole brain including the brain stem—absent cerebral
and brain stem functions
2. Protocol for declaration varies by hospital
3. Methods of examination
a) No spontaneous movement
b) No pupillary reflexes, direct and consensual
c) Negative corneal, oculocephalic, cough, and gag reflexes
d) Absent occulovestibular reflex tested with ice water irrigation
(1) Irreversibility
(2) Establish root cause of the patient’s comatose state
(3) Exclusion of any chance of recovery
(4) Persistence of loss of brain function
(5) Must exclude prior to declaration: drug intoxication, neuromuscular blockers,
metabolic abnormalities, hypothermia, hypotension
e) Apnea test
(1) Ventilate with 100% oxygen prior
(2) Adjust minute ventilation to create pCO2 > 40 mm Hg
(3) Disconnect ventilatorO2 via cannula to trachea at 6 L/min
(4) If no adverse events, continue trial for 10 minutes
(a) If spontaneous breathing starts, test is aborted
(5) After 10 minutes if no spontaneous breathing, check arterial blood gas
(a) If pCO2 > 60 mm Hg without spontaneous breathing, apnea test
confirmed
(b) Continue test if pCO2 threshold is not met.
4. Donation after cardiac death
a) Expected to be 10% of all donors, some centers up to 50% already
b) Death certified on basis of no cardiopulmonary activity before organ recovery can
begin
c) Different management issues: coordination, waiting period, often five minutes
d) Longer warm ischemic time, same ultimate success rates for kidneys.
IV. Management of Donors in the ICU
A. Continue routine nursing
B. Continue aggressive cardiovascular and ventilatory management
1. Consider PA catheter or esophageal doppler for optimization of fluid management
C. Physiologic issues after brain death
1. Diffuse vascular regulation injury
2. Diffuse metabolic cellular injury
D. Common complications after brain death include:
1. Cardiac
a) Autonomic storm followed by reduction in sympathetic outflow causing:
(1) Hypotension—most common complication
(2) Arrhythmias
(3) Cardiac Arrest
(4) Pulmonary Edema
2. Endocrine-hypothalamic-pituitary dysfunction
a) Diabetes Insipidus
(1) Antidiuretic hormone deficiency
b) Anterior pituitary dysfunction
(1) Hypothyroidism
(2) Depressed glucocorticoids
c) Coagulopathy
(1) Initially hypercoagulable, followed by consumptive coagulopathy
(2) Elevated prothombin time and thrombocytopenia
3. Hypothermia—central dysregulation
215
4. Infection
5. Electrolyte abnormalities
READING LIST
1. Abt, P., Fisher, C., Singhal, A., “Donation After Cardiac Death in the US: History of Use.” Journal of the
American College of Surgeons, Volume 203, Issue 2, August 2006, pgs 208-225.
2. Critical Pathway for the Adult Organ Donor. United Network for Organ Sharing. 27 February, 2008.
http://www.unos.org/SharedContentDocuments/Critical_Pathway_DCD_Donor.pdf
3. Critical Pathway for Donation After Cardiac Death. United Network for Organ Sharing. 27 February,
2008. http://www.unos.org/SharedContentDocuments/Critical_Pathway_DCD_Donor.pdf
4. Grenvik, A., Darby, J., Broznick, B., “Organ Transplantation: A Review of Problems and Concerns.”
Critical Care. Ed. J. Civetta. 3rd ed. Philadelphia: Lipincott-Raven, 1997. 1289-1299.
5. Heffron, T., “Care of the Multiorgan Donor.” Principles of Critical Care. Ed. J. Halle. 3rd ed. New
York:McGraw-Hill, 1998. 1345-1350.
6. Jenkins, D., Reilly, P., Schwab, W., “Improving the Approach to Organ Donation: A Review” World J Surg.
23: 644-649, 1999.
7. Powner, D., DeJoya, G., Darby, J., “Brain Death-Definition, Determination, and Physiologic Effects on
Donor Organs.” Textbook of Critical Care. Ed. A. Grenvik. 4th ed. Philadelphia: W.B. Saunders, 2000.
1894-1899.
8. Wood KE, Becker BN, McCartney JG, et al.: Care of the potential organ donor. New Engl J Med 2004; 351:
2730-9
QUESTIONS:
41.1.
A.
B.
C.
D.
E.
The exclusion criteria for organ donation include:
Lymphoma
Septic Shock
AIDS
Measles
All of the above
41.2.
A trauma 65-year-old man in the ICU s/p trauma who is awaiting donor harvesting
has a urine output of 1000 ml/hr for the past two hours with hypotension and a low
central venous pressure. His serum sodium is 152 mEq/L. He responded minimally to
a single one liter fluid bolus. What is the next most appropriate treatment?
Vasopressin infusion
IV fluid bolus with D5W.
Insulin infusion
Transfuse one unit of PRBC
Free water via the NG tube
A.
B.
C.
D.
E.
41.3.
The same patient then begins to ooze from his catheter insertion sites and his urine
turns blood-tinged. A coagulation panel was sent to the lab, what would you expect
the prothrombin time to be?
A. Normal
B. Elevated
C. Low
41.4.
A.
B.
C.
D.
The most common complication seen in patients after brain death is
Hypercoagulability
Hypothermia
Hypotension
Cardiac arrest
216
42. Trauma Management
Edgar J Pierre, M.D., David A. Riesco, M.D., Miguel A. Cobas, M.D.
A 26-year-old man sustained multiple gunshot wounds to the chest, abdomen and lower
extremities. Vital signs: blood pressure 75/55 torr, heart rate 130 beats per minute, respiratory
rate of 35/min, temperature 35.2ЛљC. He is pale, diaphoretic and unresponsive to verbal command.
Traumatic injuries are the most common cause of death in Americans under the age of 45.
Although the number of deaths from motor vehicle accidents ahs decreased, there has been an
alarming increase in firearm related deaths. The immediate care required by most severely
injured patients is best managed with a multidisciplinary team. The Advanced Trauma Life
Support (ATLS) Course developed by the Committee on Trauma of the American College of
Surgeons helps physicians maximize their resuscitative efforts and avoid missing life-threatening
injuries by using an organized approach in trauma care.
I.
Shock
Shock is defined as a clinical condition in which cardiac output is insufficient to fill the arterial
system and provide organs and tissues with adequate blood flow. Signs and symptoms of
shock consist of tachycardia, hypotension, cool extremities, pallor, oliguria, tachypnea,
decreased capillary refill, anxiety, restlessness and a loss of consciousness.
There are four types of shock:
A. Hypovolemic shock (hypovolemia, hemorrhagic)- is characterized by a loss of
circulatory blood volume. The pathophysiology of hypovolemic shock consists of two
derangements: cardiovascular and acid-base
1. Cardiovascular
An acute decrease in circulating blood volume leads to an increase in sympathetic
activity and an outpouring of vasopressors from the adrenal gland. The alphaadrenergic response that ensues causes vasoconstriction. The resulting
vasoconstriction shunts blood from the skin and viscera and thereby preserves the
coronary and cerebral circulation.
2. Acid-Base Disturbance
Metabolic acidosis is almost always seen in association with the shock state. As a
result of low flow state, there is a reduction in oxygen delivered to vital organs and
consequently a mandatory change from aerobic to anaerobic metabolism. This shift
will lead to the production of lactic acidosis.
B. Cardiogenic shock- is characterized by an inability of the myocardium to pump blood
C. Vasogenic shock (septic)- is characterized by an infection which causes a decrease in
peripheral vascular resistance This is also called distributive shock.
D. Neurogenic shock- is characterized by an impairment of the central nervous systemmediated control of vascular tone.
II. Initial assessment
The goal for the trauma patient is to re-establish blood volume and to control hemorrhage.
Approximately 60% of all hospital deaths related to trauma occur during the first hour. In
trauma patients whose injuries are deemed to be life-threatening, a rapid systemic approach
to their initial assessment and resuscitation will improve survival. The initial assessment of
any trauma victim consists of the primary, secondary, and tertiary surveys.
A. Primary Survey- This initial phase consists of following the ABCDE algorithm for the
resuscitation of trauma patients.
1. Airway
a) Your first responsibility is to establish and maintain the airway.
b) Endotracheal intubation is the most common form of establishing an adequate
airway.
c) In patients with a GCS <9 maintaining a patent airway is critical.
d) Management of the airway:
217
2.
3.
(1) The trauma patient is almost always considered a full stomach. This
requires an induction technique before direct laryngoscopy known as a rapid
sequence induction. With this technique an induction agent is given followed
immediately by a neuromuscular blocking agent. Cricoid pressure is then
held by an assistant to prevent aspiration of gastric contents into the lungs.
(2) Trauma patients may also present with cervical spine injuries making direct
laryngoscopy more difficult. All patients with blunt trauma are at risk for
cervical spine injury. A patient that is awake and does not complain of neck
pain is unlikely to have a cervical spine injury. If a cervical spine injury is
suspected any exaggerated neck extension or flexion must be avoided.
(a) In-line immobilization is applied. In this technique, an assistant places
their hands on the sides of the patients head and holds down their
occiput to minimize any neck rotation. Laryngoscopy (either direct or
utilizing a device such as a GlidescopeВ®) is then performed with care
not to hyperextend the neck.
(b) Acute trauma patients may also present with facial or neck injuries
preventing the use of direct laryngoscopy. Although each treatment
should be individualized to each patient’s situation, the use of the
fiberoptic bronchoscope and cricothyrotomy should be considered.
Breathing
After the airway is secured, breathing should be assessed. This involves a physical
examination of the chest, including palpitation for crepitus or chest wall deformity, and
auscultation. Several thoracic injuries that can be fatal and must be recognized
immediately include: open pneumothorax, tension pneumothorax, and flail chest.
a) Tension pneumothorax
(1) Most common life-threatening injury identified during the breathing
assessment
(2) Typically occurs with a parynchemal lung injury in which air cannot escape
the pleural space
(3) Increasing pressure within the pleural space causes inadequate ventilation
(4) With increased pressure in the mediastinum, mediastinal structures shift to
the contralateral side causing tension pneumothorax
(5) The result is a lower cardiac output and hypotension
(6) Diagnosis is based on clinical findings because there is often no time to
obtain a chest radiograph
(7) The quickest way to relieve a tension pneumothorax is to insert a 14 or 16
gauge IV catheter into the second intercostal space mid clavicular line, a
sudden rush of air confirms the diagnosis of tension pneumothorax
(8) A chest tube should be placed once the diagnosis is made followed by a
chest radiograph to confirm the location of the tube
b) Flail chest
(1) Presents with signs of respiratory difficulty requiring immediate therapy
(2) Caused by multiple rib fractures in three or more adjacent ribs which leads to
paradoxical chest wall movements
(3) Problems associated with flail chest include: rib fractures, pneumothorax,
pain, and pulmonary contusion. This can lead to severe hypoxemia
c) Open pneumothorax
(1) “Sucking chest wound” allowing air to enter and exist the chest during
respiration
(2) Wound is covered with a Vaseline gauze or other occlusive dressing on three
sides, so that air may exit but not enter the chest
(3) Tube thoracostomy is the treatment of choice
Circulation
Restoration of an adequate circulating intravascular volume is the third step in the
ABCDE algorithm.
a) Signs of inadequate intravascular volume include:
(1) Hypotension, tachycardia, diminished peripheral pulses and extremities that
are pale, cool, or appear cyanotic.
218
4.
5.
(2) In many trauma patients, the cause of hypovolemia is secondary to
hemorrhage. It is important to first find and control the source of bleeding
before attempting to restore intravascular volume.
(3) Adequate perfusion can also be assessed by determining the quality, rate,
and regularity of the pulse.
(4) The pulse is a sensitive indicator of hypovolemia: the blood pressure remains
normal even with the loss of up to 30% of the blood volume.
(5) It is often appropriate to assume that hypovolemia is present if there is a
pulse rate faster than 120 bpm in an adult.
(6) Altered mental status can be used as an indicator to identify the hypovolemic
patient. Signs maybe unreliable because patients may have central nervous
system injury, be intoxicated with drugs or alcohol, or have some degree of
hypoxemia that may contribute to change in mental status.
b) Resuscitation
(1) The restoration of circulating intravascular volume is best accomplished by
the use of intravenous fluids and blood transfusions when necessary.
(2) For the initial resuscitation peripheral IV’s are usually all that is needed. (16
or 14 gauge) or with a 7-9 F introducer in either the internal jugular or
subclavian vein.
(3) The placement of the catheters should be dictated by the location of the
injuries.
(a) In patients with upper torso injuries, catheters may be placed in the
lower extremities
(b) In patients with abdominal trauma, however, it may be not appropriate
to place IV catheters in the lower extremities.
c) Fluid resuscitation
(1) The initial fluid of choice in trauma patients is Lactated Ringer’s solution. A
bolus of 10-20 ml/kg which is roughly equivalent to one liter is usually given.
As with any treatment the need for further volume resuscitation is always
important to reassess the patient to see if they respond to the initial fluid
bolus.
(2) Blood products are often needed in the treatment of hypovolemic shock. If
type-specific blood is not available initially and the patient is unresponsive to
crystalloid infusions then the use of uncrossed O-negative blood is required.
Once type-specific blood is available then the use of uncrossed O-negative
blood is discontinued. Usage of blood must be monitored closely in order to
have blood available at all times for the hemorrhagic trauma patient.
(3) Colloid solutions are expensive and may not be necessary as many studies
show adequate restoration of intravascular volume can be accomplished with
the use of crystalloid solutions or blood products.
(4) Rapid infusion systems are available for massive amounts of blood loss. It is
essential to warm all IV fluids and blood products prior to transfusion as
hypothermia can cause and perpetuate coagulopathy that can make the
resuscitation of trauma patients inadequate and much more difficult.
(5) Other useful indicator of adequate resuscitation after the blood pressure and
pulse rate have normalized is the base deficit as determined by arterial blood
gas analysis as well as clearance of lactic acid.
Disability
At this stage a quick neurologic examination is appropriate. It is important to first
evaluate the papillary size and activity, motor and sensory responsiveness, rectal
tone and level of consciousness.
Exposure
a) The complete assessment of the trauma patient involves a thorough examination
for occult injury. Patients should have all garments removed to look for nonobvious injuries.
219
b) Patients should be rolled with care to protect the spine so that the patient’s back
can be examined. After the patients have been examined thoroughly, cover the
patient to prevent hypothermia.
B. Secondary Survey:
The goal of the secondary survey is to follow up the primary survey with a complete head
to toe examination while there is an ongoing reevaluation of the adequacy of
resuscitation.
1. Initial Therapy
a) Therapy and diagnosis should be carried out simultaneously. Prioritize treatment
and management to the greatest threat to life or limb and the stability of the
patient.
b) Airway maintenance is crucial, because reintubation may be difficult owing to
injuries and airway edema.
c) Blood and crystalloid solutions are both necessary to resuscitate severely injured
patients.
d) Blunt cardiac trauma is significant only if it results in arrhythmias or
hemodynamic instability.
e) Rewarming should be the initial treatment for most coagulopathic trauma patients
because of hypothermia induced thrombocytopenia.
f) The first step in limiting blood loss in patients with pelvic fractures is application
of an external fixator. If a patient continues to bleed, patient should undergo
angiography and embolization of any bleeding sites that are identified.
C. Specific Traumatic Problems:
1. Undetected hemorrhage
This injury is usually overlooked during the primary and secondary survey. Abdominal
sources of hemorrhage must be addressed. Ultrasound, CT scan of the abdomen
and DPL are the most common methods used to evaluate abdominal trauma.
Ultrasound is preferred for the unstable patient and CT scan is often reserved for
evaluation of hemodynamically stable patients. The best technique to rule out
abdominal injury is the exploratory laparotomy.
2. Pneumothorax or hemothorax
Tension pneumothorax and a massive hemothorax may both present as persistent
hypotension.
3. Cardiac Tamponade
Patients may present with hypotension, tachycardia, and an elevated central venous
pressure. Beck’s triad (distant heart sounds, distended neck veins, and
hypotension) is often not presented in the acute setting.
4. Air Embolism
Venous air embolism occurs if a significant amount of air is introduced into the right
heart circulation, usually through a central venous pressure catheter. Systemic air
embolism occurs with smaller amount of air and presents as a sudden onset of shock
in a patient who has sustained a lung injury.
5. Acidosis/ Hypothermia
Present in massively injured and resuscitated patients. Acidosis can result in
persistent hypotension despite adequate resuscitation. Primary treatment is fluid
resuscitation and restoration of blood volume with fluids and blood products and
improvement of oxygen transport.
6. Blunt Cardiac and Pulmonary Injury
Formerly known as “myocardial contusion” it is usually caused by blunt chest trauma
and is frequently produced by a deceleration injury in a motor vehicle accident.
Cardiac contusion presents as a spectrum of injuries including direct myocardial
damage, injury to coronary arteries, rupture of the cardiac free wall and septum.
Injuries to cardiac valves including rupture are rare but can occur.
Cardiac contusion can clinically present as angina, difficulty breathing, chest wall
ecchymosis, arrhythmias, and heart failure. The most common arrhythmia is sinus
tachycardia, which is a common, nonspecific finding in the trauma victim.
Studies used to diagnose cardiac contusion include: 12 lead ECG, troponin I levels
and echocardiogram. Some common findings on 12 lead ECG include: arrhythmias,
220
7.
8.
9.
conduction delays, and ST-T wave changes. Treatment options will vary according to
the specific diagnoses.
Pulmonary contusions may manifest hours to days after the initial injury with diffuse
alveolar infiltrates and ARDS.
Intra-abdominal Hypertension
Elevated Intra-abdominal pressure carries significant complications. The most
common cause is hemoperitoneum, which typically occurs postoperatively. Other
causes include: peritoneal tissue edema, fluid overload secondary to hemorrhagic or
septic shock, retroperitoneal hematoma, abdominal packing from hemorrhage and
closure of abdomen under tension.
Intra-abdominal pressure (IAP) can be measured using an indwelling Foley catheter.
The tubing is clamped just distal to the aspiration port. Abdominal compartment
syndrome is defined by sustained or repeated IAP > 25 mm Hg in association with
new onset single or multiple organ failure.
a) Manifestations of elevated intra-abdominal pressures
(1) Cardiac output may decrease dramatically because of a decrease in venous
return and elevated systemic vascular resistance.
(2) Peak airway pressures rise to and there is a marked decrease in tidal volume
secondary to decreases in compliance.
(3) Renal failure as a result of a decrease in cardiac output as well as direct
renal compression.
Massive Transfusion
Defined as a replacement of more than one blood volume, which is usually greater
than 10 units packed red blood cells. Complications that can arise include:
a) Thrombocytopenia: from dilution and consumption.
(1) Hypothermia-induced platelet dysfunction leads to coagulopathy despite
adequate platelet numbers and can only be corrected by re-warming the
patient
(2) In actively bleeding patients, platelets should be maintained to a level greater
than 75,000 cubic millimeters.
b) Hyperkalemia
(1) Potassium leaks out of red blood cells during storage
(2) In a unit of packed red blood cells there is a high concentration of potassium;
it is usually such a small volume that the potassium is rapidly diluted in the
patient’s circulation.
c) Hypothermia
(1) When room temperature fluid or blood stored at 4ЛљC is administered at high
volumes to patients, the body temperature may drop to 32ЛљC or lower.
(2) All fluids and blood products should be warmed before administration. Rapid
infusion devices act by countercurrent heat exchange to warm fluid and
blood products.
d) Citrate toxicity
(1) Citrate is used as an anticoagulant in banked blood and works by binding to
calcium. Normally citrate in metabolized in the liver to bicarbonate. In a
massive transfusion, the citrate continues to circulate and binds to calcium.
(2) This results in severe decrease in ionized calcium that may result in
myocardial dysfunction and a decrease in vascular tone.
(3) Arrhythmias and prolonged QT intervals can also be observed
(4) Additional calcium may be administered as necessary to maintain
hemodynamic stability
e) Decrease in 2,3 DPG
(1) The 2, 3 DPG levels in stored blood decline but in stable patients these
levels are restored within 24 hours.
(2) A shift in the oxyhemoglobin dissociation curve as a result of hypothermia
and a decrease in 2, 3 DPG levels increases the affinity of hemoglobin for
oxygen.
Pelvic Fractures:
221
Pelvic fractures constitute 3-5% of all fractures, account for 1 in every 1000
hospital admissions and is the third leading injury in victims of a motor vehicle
crash.
b) Morbidity rates range from 33% to 75%. Open pelvic fractures carry a high
mortality rate ( > 50%) and require operative intervention to prevent the
development of multiple organ failure and sepsis.
c) Pelvic fractures are often associated with significant blood loss and
hemodynamic instability. It is essential to rule out significant intraabdominal
injury. Abdominal CT scan with contrast may be used in hemodynamically stable
patients.
d) The most common associated injuries include urethral and bladder injuries. If an
anorectal injury is found, the patient must have a diverting colostomy. Vaginal
lacerations must be repaired after thorough irrigation and debridement.
e) Early reduction and fixation of pelvic fractures reduces the incidence of
hemorrhagic shock and sepsis. The potential sources of hemorrhage in pelvic
fractures include: fracture sites, deep pelvic arterial bleeds, and major arterial
injuries.
f) The most important predictor of outcome is hemodynamic status. The mortality
rate of stable patients is reported to be 3.4%, and the mortality of unstable
patients reaches 42%.
g) Different therapeutic interventions for pelvic fractures
(1) External fixation
(a) The main goal is to control hemorrhage and provide functional
stabilization of the injury. This is accomplished by restoring the normal
pelvic circumference by decreasing anterior widening.
(b) A more definitive stabilization requires open reduction and internal
fixation of both the anterior and posterior elements.
(2) MAST ( Military Antishock Trousers )
(a) Used to achieve temporary stabilization and to aid in controlling the
hemorrhage.
(b) The inflation pressure should begin at 40 mm Hg. Prolonged inflation
pressure above the diastolic pressure must be avoided to minimize
complications.
(c) Deflation of the MAST is accomplished stepwise (10 mm Hg every 4-6
hours)
(d) Angiography and external fixation may be attempted as a primary mode
of treatment before the MAST
i) Angiographic embolization
ii) Posterior component fractures that do not stabilize with external
fixation or application of the MAST may have lacerated deep
pelvic arteries. These arteries must be located by angiography.
Indications include:
(1) More than 6 units of blood transfused in 24 hours.
(2) Expanding pelvic hematoma visualized on CT scan or during
surgery.
(3) Open pelvic fracture
(4) Major pelvic fracture in a patient undergoing angiography for
associated injuries.
(5) 85-95% success rate in controlling hemorrhage can be
achieved.
a)
This chapter is a revision of the original chapter authored by Douglas B. Coursin, M.D. and Lisa Connery, M.D.
REFERENCES:
1. Trunkey DD, Blaisdell FW: Epidemiology of trauma. Sci Ame 4:1, 1998
222
2. Schweiger JW:The pathophysiology, diagnosis, and management stratergies for flail chest injury and
pulmonary contusion.Anesth Analg 92 (Suppl): 86. 2001
3. Orliaguet G, Ferjani M, Riou B: The heart in blunt trauma.Anesthesiology 95:544, 2001
4. Wood RP, Lawler PGP: managing the airway in cervicalspine injury: A review of the Advanced Trauma
Life Support protocol. Anesthesia 1992; 47(9): 792-797
QUESTIONS:
42.1.
A.
B.
C.
D.
The order of the algorithm for the resuscitation of trauma patients during the primary
survey is:
breathing, circulation, exposure, airway, disability
airway, breathing, circulation, disability, exposure
circulation, disability, airway, breathing, exposure
exposure, disability, airway, circulation, breathing
42.2.
A.
B.
C.
D.
E.
Complications that can arise after massive transfusion include:
thrombocytopenia
hypothermia
citrate toxicity
decrease in 2,3 dpg
all of the above
42.3.
A.
B.
C.
D.
The fluid of choice in trauma patient’s is:
0.9% normal saline
0.45% normal saline
Lactated Ringers
D5W
42.4. The trauma patient is almost always considered a full stomach
A. True
B. False
42.5.
A.
B.
C.
D.
The most common life-threatening injury during the breathing assessment of the
ABCDE algorithm is:
cardiac tamponade
myocardial infarction
tension pneumothorax
flail chest
42.6.
In a hypovolemic trauma patient unresponsive to crystalloid infusions uncrossed O
negative blood is required for transfusion if type specific blood is not available
A. True
B. False
42.7.
A.
B.
C.
D.
Beck’s triad consists of:
distant heart sounds, distended neck veins, hypertension
distant heart sounds, distended neck veins, hypotension
“machine like murmur”, hypotension, wheezing
pulsus parodoxus, elevated ICP, nausea
223
42.8.
It is often appropriate to assume that hypovolemia is present if there is a pulse rate
faster than 120 bpm in an adult.
A. True
B. False
42.9.
A.
B.
C.
D.
The most common arrhythmia after blunt cardiac injury is
multifocal atrial tachycardia
ventricular fibrillation
torsades de pointe
sinus tachycardia
42.10. The first step in limiting blood loss in patients with pelvic fractures is application of an
external fixator.
A. True
B. False
224
43. Burn Management
Edgard J. Pierre, M.D., Joseph Borau, M.D., Miguel A. Cobas, M.D.
A 28-year-old male presents to the Emergency Department after sustaining second and third
degree burns over the upper extremities and trunk, neck and abdomen (65% total body surface
area (BSA)). He is alert and oriented times three. Blood pressure is 155/90 mm Hg, pulse rate is
110/minute and regular and respiratory rate is 32/minute. He is 5 feet 10 inches tall and weighs
88kg.
In the United States 2 to 2.5 million people suffer major burn injuries every year resulting in
significant morbidity and mortality. Death rates are highest in the very young and the very old.
Thirty-eight percent of the victims are <15 years of age. Increasing thickness and surface area of
burn injury adds to the morbidity and mortality. Scalds are the most common form of childhood
thermal injuries while electrical and chemical injuries affect adults in the workplace. High voltage
electrical burns pose additional challenges as mortality is high compared to percentage of skin
surface affected. Over the last twenty years mortality decreased significantly, although burns
associated with inhalation injury still pose significant challenges in management. The needs of
the burn patient are unique with regard to the treatment of their altered respiratory and
cardiovascular physiology, and the concomitant fluid/electrolyte imbalances.
I.
Initial Evaluation
A. Neutralize the burning process
1. Extinguish and remove clothing
2. Neutralize chemical burns
First and foremost, it is important that the clinician not attempt to neutralize a
chemical unless the patient or clinician is accurately able to identify the substance.
Applying the wrong neutralizing agent to a chemical burn could aggravate the burn.
The chemical destruction of an alkali burn is more severe than an acid in that it
penetrates deeper and has a greater duration of action. The first step in treating
chemical burns is to thoroughly flush the burn with water1. If a neutralizing substance
specific to the chemical agent is present then it can be combined with the water.
Despite the presence of a neutralizing substance, water alone is still the best
irrigating solution. Below are some typical neutralizing substances utilized in the field
by emergency medical services and in the Burn Unit setting.
a) Acid Burns: 1 teaspoon Baking soda in combination with one pint of water.
Chemical burns caused by phenol (carbolic acid) must be irrigated with alcohol
secondary to the insoluble nature of phenol in water2.
b) Alkali Substance: Alkali burns are usually more serious ; they can be treated with
1-2 teaspoons of vinegar in combination with a pint of water2.
3. Cleaning a Burn Wound3
Recent literature strongly supports the use of soap and water to cleanse burn
wounds. The cleansing and irrigation of the wound gently debrides the necrotic tissue
and reveals healthy vasogenic tissue capable of initiating the healing process.
Irrigation also mechanically removes foreign objects such as clothing and other
debris that may interfere with healing. Artificial antiseptic topicals such as
chlorhexidine gluconate and povidone-iodine solution are not recommended as they
may interfere with healing. Burns containing tar and asphalt can be treated with
regular polymyxin Bacitracin Zinc Ointment applications. This treatment regimen
should emulsify the tar without having to mechanically debride the wound.
B. Airway Management/Respiratory Dysfunction
1. Inhalation injury4
Associated inhalation injury can increase mortality rates 30-40%.
a) Upper airway burns
Upper airway injury is frequently due to direct heat exposure; laryngeal reflexes
protect the lung from thermal injury in all cases except possible high-pressure
225
steam exposure. Lower airway injury is predominantly from chemical products of
combustion carried to the lung on particles of soot. Inhalation injury causes
several physiologic derangements including loss of airway patency secondary to
mucosal damage and edema, intrapulmonary shunting from small airway
collapse caused by mucosal edema, and decreased lung compliance because of
alveolar flooding and collapse with mismatching of ventilation and perfusion.
b) Carbon monoxide/cyanide
Carbon monoxide intoxication often occurs in patients involved in structural fires.
The most likely cause of early mortality secondary to flame injury is anoxia.
Carbon monoxide has a higher affinity for hemoglobin than oxygen with the
severity of anoxia depending on the percentage of carboxyhemoglobin.
Symptoms include a throbbing headache with nausea and vomiting (<30%),
slight change in mental status to coma, convulsions and cardiac arrest (>60%).
Cyanide toxicity may coexist with carbon monoxide, particularly when inhalation
injury occurs indoors. Symptoms are similar, and cyanide toxicity may
significantly potentiate carbon monoxide toxicity.
c) Subglottic injury
Immediate increased airway resistance
2. Diagnosis
Diagnosis of inhalation injury is most commonly based on a history of a closed-space
exposure with soot in the nares, hoarse voice, facial burns. The diagnosis is
confirmed with bronchoscopy which reveals airway edema, erythema, soot
accumulation and mucosal sloughing. Chest radiography is frequently normal on
hospital admission and hypoxia on blood gases is not frequently seen.
Carboxyhemoglobin levels are often deceptive in those patients who have received
oxygen therapy.
C. Treatment
1. High flow 100% oxygen
2. Intubate early
3. Consider bronchoscopy with removal of particulate material
4. Increased metabolic rate translates into increased minute ventilation
5. Escharotomy of circumferential chest burns
D. Estimating Burn Size
1. Rule of Nines: Initial management requires estimation of the percentage body surface
area burned. The rule of nines is applied. Each upper extremity accounts for 9% total
BSA, lower extremities are 18% each, trunk is 18% anterior and 18% posterior, head
is 9% and the genitals are 1%.
2. Lund-Browder Diagram: age-specific chart that compensates for body proportions
with growth and translates this information into TBSA.
3. Palmar surface = area of the palm without the fingers represents 1% body surface
area6.
4. Recognition of the deep injury associated with electrical burns
E. Fluid Resuscitation
1. Adequate vascular access
2. Parkland formula7
a) 3-4 ml/kg/% body surface burn in the first 24 hours.
b) half of the calculated 24 hour volume given in the first 8 hours.
3. A variety of other formulas (Brooke or Moore) may be used to calculate resuscitation
rates.
4. Example of fluid management
a) Our 88kg patient with 65% TBSA burn would require, using the Parkland formula:
4 x 88 x 65 = 22880mls of Lactated Ringers solution over 24 hours.
Therefore 11.5 liters should be given in the first 8 hours and 11.5 over the
following 16 hours
5. Inhalation injury increases the overall fluid requirement of burned patients’ from
volume and total salt requirement standpoints.
6. Pediatric considerations
Gluconeogenetic capacity has not fully developed in children, hypoglycemia is
226
common and a Ringer lactate solution with 5% dextrose should be added at a
maintenance rate.
7. Type of fluid
a) Isotonic vs. hypertonic
b) Crystalloid vs. colloid
Hypoproteinemia and edema formation complicate the use of isotonic crystalloids
for resuscitation. Hypertonic saline has a theoretical advantage of improved
hemodynamic response and diminished overall fluid needs as intracellular water
is shifted into extracellular space by the hyperosmolar solution. A clear role for
hypertonic saline has not yet been identified. Some groups add colloid to
resuscitation fluids as protein formulations or dextran after the first 8 hours of
resuscitation. Groups most likely to benefit from colloid administration are the
elderly, those patients with large burns and /or patients with inhalation injury.
8. End points of fluid resuscitation
a) Vital signs
b) Urine volume: 0.5 milliliter per kilogram per hour.
c) Cardiac parameters: With adequate fluid resuscitation, cardiac output increases
dramatically. This is in contrast to a decrease in cardiac output in the 18 hours
post-burn injury during which time there is continued capillary leak.
d) Normalization of acid-base abnormalities.
e) Energy Requirements: A hypermetabolic (increased resting oxygen consumption)
cascade is initiated, accompanied by accelerated gluconeogenesis, insulin
resistance, and increased protein catabolism (excess nitrogen loss).
f) If fluid volume seems adequate and no urine output is observed, consider
abdominal compartment syndrome. Bladder pressure measurements >20 mmHg
often signify dangerously elevated intraabdominal pressures.
F. Wound Management
1. Estimate extent and depth.
Characteristics of skin affect patterns of cutaneous injury. Males have thicker skin
than females. Average skin thickness is 1 to 2 mm. In general the dermis is ten times
thicker than associated epidermis. Skin areas with high blood flow, such as the face,
are also less likely to develop full thickness burns.
a) First-degree burns involve only the epithelial layer of the skin. They appear
erythematous and are painful, classically described as sunburn. No scar should
develop.
b) Second-degree burns involve epithelium and dermis. Wounds characteristically
appear red, wet, or blistered, blanchable, and extremely painful. Reepithelialization will occur due to sparing of hair follicles and other dermal
appendages. Pain and scarring vary with depth of the burn.
c) Third degree burns involve the entire skin thickness. They appear white and are
painless. Scarring is definite. This wound will not heal without surgical
intervention.
d) In thermal injury, damage to the skin results from temperature of the thermal
source and the duration of exposure. At 40 to 44ЛљC, enzymatic failure occurs
within the cell with rising intracellular sodium concentration and swelling due to
failure of the membrane sodium pump. At exposure to 60ЛљC, necrosis occurs in
one hour with release of oxygen radicals.
2. Clean with Dreft soap and water
3. Apply topical antimicrobials (2nd and 3rd degree burns)
a) Silver sulfadiazine
b) Mafenide
4. Escharotomy/Fasciotomy3
Note circumferential, or near-circumferential, burn wounds may function as
tourniquets with increased swelling. Perform extremity escharotomies as soon as
peripheral perfusion is threatened. Perform torso escharotomies as soon as
ventilation appears compromised. Electrical burns often result in significant deep
tissue necrosis and inflammation increasing the risk of a compartment syndrome.
Fasciotomy must be performed as soon as peripheral perfusion is threatened.
227
G. Tetanus prophylaxis
H. Pain Management
One of the most common fears of a burn victim is pain. Scheduled wound treatments and
dressing changes are extremely painful experiences.
1. Opiates such as morphine, and fentanyl can be used to treat pain. Narcotics have a
poor side effect profile in that they reduce GI motility and subsequently interfere with
nutritional support. This prevents early enteric feeding.
2. Regional anesthesia has proven efficacious when a single extremity is involved and
avoids the negative side effects of narcotics. Local anesthetics should never be
applied directly to the wound.
3. Dexmedetomidine may also be a valuable adjunct in pain management for dressing
changes.
4. Wound coverage is ultimately the most effective form of pain management, as it
protects the exposed nerve endings.
II. Post-resuscitation Care
A. Respiratory Support
Delayed capillary leak and epithelial injury = ARDS
B. Cardiovascular Support
1. Continued volume needs
Initially there is a fall in cardiac output that is mainly due to hypovolemia, although
rarely myocardial contractility is reduced. Eventually with adequate resuscitation, the
cardiac output returns to normal and more frequently supranormal; this is primarily a
reflection of the massive increase in blood flow found in burned tissues.
2. Transfusion requirements
Two to five days after the burn injury, the hematocrit predictably decreases secondary
to intravascular reabsorption of third space fluids and thermally injured red blood
cells.
C. Nutritional Requirements
1. Caloric requirements
a) Nutritional support minimizes protein losses during the early hypermetabolic
phase and to maximize protein synthesis during the healing phase.
b) Feeding should be accomplished by the enteral route, as soon as possible. If
gastrointestinal tract is unable to cope with large volumes of hyperosmolar fluid,
then it may be necessary to use the enteral and parenteral routes
simultaneously.
c) Daily caloric requirements for adult patients with burns greater than 20% of the
TBSA are estimated using the following formula.
(1) 25 kcal/kg body weight + 40 kcal/% burn
(2) Nitrogen is given in quantities that give a caloric-to-nitrogen ratio of 150:1.
(3) Calories are given in the form of glucose up to a dose of 5mg/kg/min, which
is supplemented with a fat emulsion. Mineral and vitamin supplements are
essential
d) Vitamins A and C
e) Zinc
f) Early enteral feeds
(1) Because sepsis commonly complicates burn care and results in frequent
gastric ileus, post-pyloric feeding is often more successful.
D. Renal Function4
1. Rhabdomyolysis and Myoglobinuria
a) Commonly found in deep thermal or high voltage burns. Requires prompt
treatment to avoid acute tubular necrosis.
b) Patients are often volume contracted and therefore require continued fluid
resuscitation. Urine alkalinization with the administration of bicarbonate (100
mEq of NaHCO3 in each liter of LR [hypertonic solution] or 150 mEq of NaHCO3
in sterile water [prepared by the pharmacy] ) may facilitate clearance of
myoglobin by preventing its entry into the tubular cells. Mannitol (100mg/day) has
also resulted in improved outcomes. The risk is reduced if urine output is greater
228
than 1ml/kg
The hypermetabolic state of the burned patient can cause rapid rises in serum
urea and creatinine if renal function is suboptimal.
d) Continuous arteriovenous hemofiltration is the technique of choice to remove
urea effectively and water balance is easily maintained.
Hypoperfusion4
If the patient is inadequately resuscitated then oliguric renal failure is possible.
Septicemia carries the same risks of renal failure. Renal failure in the ICU setting
greatly increases the patients’ morbidity and mortality.
c)
2.
E. Infection
1. Burn wound
The damaged epithelium and dermis creates an avenue for invasion by
microorganisms and subsequent infection. Burn patients are also susceptible to
bacteremia and complications secondary to a poor immunologic response. Early
sepsis (24-48 hours) is usually caused streptococcal or staphylococcal bacteria. Late
sepsis is usually due to Pseudomonas, Acinetobacter and fungi. Nearly all burn
patients are febrile and exposing these patients to antibiotics may facilitate resistant
bacterial strains or fungi. Antibiotic prophylaxis is not recommended, but instead
culture specific antimicrobial therapy should be judiciously implemented.
2. Burn Wound Infection
a) Burn impetigo (superficial infection with loss of epithelium). Associated with S.
aureus and Streptococcus Pyogenes. Treatment includes thorough cleansing of
wound; topical anti-staphylococcal medications such as mupirocin, and
occasionally grafting of fragmented areas of epithelium.
b) Open Burn Related Surgical Wound Infection describes purulent infection that
develops in surgically manipulated wounds. These types of wounds can flourish
into sepsis and often offer a poor prognosis for graft survival. Treatment requires
incision and debridement with delayed wound closure.
c) Burn Wound Cellulitis refers to an infection that has spread to adjacent healthy
tissue via the dermal layer. Treatment involves early recognition and prompt
institution of antibiotics. The causative organism is commonly S. pyogenes.
d) Invasive Burn Wound Infection signifies a heavily colonized and rapidly
progressive burn wound infection that has spread to adjacent healthy tissue with
concomitant sepsis. The wound can include punctuate hemorrhages, rapidly
progressive liquefaction, and changes in color. Treatment involves aggressive
debridement, wound excision and closure, resuscitation and parenteral
antibiotics.
3. Catheter-related bloodstream infections
4. Pneumonia
5. Infection Control
a) It can not be stressed enough the importance of washing your hands before and
after examining each patient. It is imperative that clinicians strictly adhere to
sterile technique when performing procedures. Patients are often placed in
rooms with special air filtration systems to reduce exposure to pathogens and if
infected they are placed in isolation in negative pressure rooms.
b) Topical broad spectrum antimicrobials designed to reduce colonization are also
popular but do not replace early excision of deep burns in preventing infection.
Lastly, early nutritional support will help bolster the patients’ immune system.
F. Temperature Control
The skin is the largest organ in the body and comprises approximately 15% of body
weight. The skin serves many functions, including thermostatic regulation. The increased
metabolic rate results in increased temperatures, therefore, temperature must be closely
monitored.
G. Pain Management
1. Opioids
2. Other analgesic adjuncts
a) Anticonvulsants
229
b) Antidepressants
H. Wound Management8
1. Early excision and grafting
a) Usually performed within 3-5 days of injury. Wounds are covered with sheet or
meshed autograft, harvested 3/to 10/100-inch thickness with a power dermatone
from unburned sites..
b) Grafts can be meshed onto the burn area in a ratio from 1:1 to 1:9 to increase
coverage with the assumption that the wound will reepithelialize within the mesh
network.
c) Early excision of necrotic tissue and immediate coverage of excised open
wounds is associated with less blood loss and a decrease in mortality and
morbidity.
2. Skin Substitutes
a) Commercially available cross linked collagen and chondroitin sulfate dermal
replacement covered by a temporary silicone epidermal substitute.
b) Within 2 to 3 weeks of application of this material, a new dermis forms and
ultrathin autografts may then be placed over the neodermis.
c) After healing, the synthetic dermis-supported skin graft appears to have
histological structure and physical properties similar to those of normal skin
3. Allograft
I. Physical Therapy
1. Splinting
2. Passive range of motion
3. Hydrotherapy
4. Compression garments
Pressure therapy is an important component of a recovering burn patient’s
rehabilitation. Compression can be achieved with elastic wrap bandages, tube
compression garments, splits, orthoses and casts. Compression garments provide
constant and equal pressure over the healed burn, which minimizes development of
scarring and deformity. The pressure exerted helps lengthen the skin as well as
retain any moisturizers that were applied to the damaged skin.
J. Psychological Support
K. Electrical burns:
The severity of the injury depends on the amperage, voltage, resistance, type of current,
duration of current flow and area of contact with tissues. Nerves are the best conductors,
followed by blood, muscle, skin and bone, respectively. Thus electrical burns tend to be
more extensive than the area of the skin burn suggests. Cutaneous burns may also occur
if clothes catch fire.
This chapter is a revision of the original chapter authored by Madelyn Kahana, M.D. and Aisling Conran, M.D.
REFERENCES:
1. Baxter CR. Management of burn wounds. Dermatol Clin 1993;11:709-14.
2. Waitzman AA, Neligan PC. How to manage burns in primary care. Can Fam Physician 1993;39:2394-400.
3. Papini R: Management of burn injuries of various depths. BMJ 2004 Jul 17; 329(7458): 158-60
4. MacLennan, Neil MBChB; Heimbach, David M. MD; Cullen, Bruce F. MD Anesthesia for Major Thermal
Injury.Anesthesiology. 89(3):749-770, September 1998.
5. Walker AR: Emergency department management of house fire burns and carbon monoxide poisoning in
children. Curr Opin Pediatr 1996 Jun; 8(3): 239-42 Milner
6. Sheridan RL, Petras L, Basha G, et al: Planimetry study of the percent of body surface represented by the
hand and palm: sizing irregular burns is more accurately done with the palm. J Burn Care Rehabil 1995
Nov-Dec; 16(6): 605-6
230
7. SM, Hodgetts TJ, Rylah LT: The Burns Calculator: a simple proposed guide for fluid resuscitation. Lancet
1993 Oct 30; 342(8879): 1089-91[Medline].
8. Sheridan RL, Tompkins RG: Skin substitutes in burns. Burns 1999 Mar; 25(2): 97-103
QUESTIONS:
43.1.
A.
B.
C.
D.
E.
The preferred method of cleansing a burn wound is:
Chlorhexidene Gluconate
Povidone-Iodine Solution
Alcohol
Soap and water
Hydrogen Peroxide
43.2.
A 50 kg male who has sustained a 40% flame burn would require the following fluids
over the first 8 hours:
8 liters of lactated Ringers in the first 8 hours
5 liters of D5 ВЅ NS in the first 8 hours
4 liters of lactated Ringers in the first 8 hours
6 liters of 5% albumin in the first 8 hours
4 liters of 3% saline in the first 8 hours
A.
B.
C.
D.
E.
43.3.
A.
B.
C.
D.
E.
43.4.
A.
B.
C.
D.
E.
43.5.
A 35-year-old adult male with a 50% flame burn has been receiving routine wound
care for 3 days with a topical antimicrobial. His exam and other vital signs are
unremarkable with the exception of a hyperchloremic acidosis. What is the likely
explanation?
Sepsis
Mafenide
Silver nitrate
Cyclic neutropenia
Silver sulfadiazine
A 2-year-old with 60% flame burn injury is being treated with silver nitrate for
prevention of burn wound infection? What are some of the complications of using
silver nitrate?
Decreased serum sodium levels
Methemogobinemia
Decreased serum potassium levels
Decreased serum chloride levels
All the above
A 60 year old male presents to your Trauma Department with flame burns to the face
and upper extremity estimated to be 45% of his total body surface area. Pt presents
with mild symptoms and all vital signs are stable, although on physical exam he is
noted to have singed nasal hair. What is the next step in treatment of this patient?
A. Intubation
B. Hyperbaric oxygen therapy
C. Begin fluid resuscitation using the Parkland Formula
D. Mechanical debridement of burn wounds
E. Antimicrobial therapy
231
43.6.
A.
B.
C.
D.
E.
What are the nutritional requirements for adult patients with >20% TBSA burns?
15 kcal/kg body weight + 25kcal/% burn
caloric to nitrogen ratio of 150:1
NPO until patient is able to pass a speech and swallow study
calories in the form of glucose up to a dose of 15mg/kg/min
D51/2NS @ 125cc/hr
43.7.
A 45-year-old intubated victim of a house fire develops collapse of bilateral lung
bases with concomitant hypoxia on hospital day #6. Pt has been aggressively fluid
resuscitated over the past several days. Pt suffered a near circumferential chemical
burn injuries to chest and back while at work. Which is the most reasonable
intervention?
Repeat the chest x-ray
Increase PEEP
Lasix 40mg IV x 1
Start inhaled antibiotics
Torso escharotomy
A.
B.
C.
D.
E.
43.8.
A.
B.
C.
D.
E.
A patient suffers a high voltage electrical burn to the lower extremity. Serum urea and
creatinine have been rising. Urine output has been greater than 1 ml/kg. What is the
next best intervention?
Hemodialysis
Mannitol 1000mg/day.
Bicarbonate added to IV fluids
Discontinue fluid resuscitation
lower extremity fasciotomy
232
ASCCA ResidentКјs Guide to the ICU
44. Obstetric Critical Care
Gustavo Angaramo, M.D.
You arrive early in the morning of your first day in the ICU to review the charts of your new
patients. Before sitting down with your first chart, you are distracted by a patient in bed 6. She is
a 29-year-old female, approximately 31 weeks gestation, who appears to be in moderate
respiratory distress. You proceed to the bedside for further investigation when, suddenly, she
begins to have a seizure. The only person to respond to your call for help is the third-year
medical student. He blurts out something about high blood pressure, bleeding from her IV sites,
and asks you if he can intubate her. As you ponder the wisdom of your decision to skip breakfast
and arrive early, you try to remember something about last month’s lecture regarding
hypertensive disease of pregnancy.
Pregnancy places unique demands on the skills of critical care practitioners, as underlying
disease processes are impacted by the physiology of pregnancy and vice versa. Additionally,
pregnancy is the only medical condition in which a single management plan affects two
individuals simultaneously, namely the mother and the fetus. As illustrated in the case above, a
working knowledge of the uniqueness of these processes is essential to avoid morbidity. Goals
for obstetric knowledge fundamentals include: (1) an understanding of how disease processes
impact pregnancy and vice versa, (2) an ability to communicate the anesthetic implications of the
disease to non-anesthesiologist colleagues attending the patient, (3) an ability to assess the
severity of the disease and evaluate the need for patient transfer to a high-risk care area, and (4)
an understanding of the anesthetic management of a patient for vaginal or cesarean delivery.
I.
Hypertensive Disorders of Pregnancy
A. Knowledge
1. Classification of hypertensive disorders during pregnancy
2. Epidemiology of preeclampsia–risk factors
3. Pathophysiology of preeclampsia as a multisystem disease
a) Recognition of sequelae:
(1) Seizures
(2) Coagulopathy
(3) Hepatic dysfunction
(4) HELLP syndrome
4. Medical/obstetric management of preeclampsia
a) Term vs. preterm fetus
b) Mild vs. severe disease
c) Assessment of fetal well being
d) Seizure prophylaxis; magnesium sulfate effects
e) Antihypertensive therapy
f) Management of oliguria
g) Indications for invasive monitoring
5. Anesthetic selection for and management of the preeclamptic parturient
a) Labor and vaginal delivery
b) Abdominal delivery–non-urgent
c) Abdominal delivery–urgent
B. Skills:
1. Ability to diagnose the parturient with preeclampsia
2. Ability to identify the patient requiring invasive monitoring
3. Ability to effectively treat the patient with an eclamptic seizure
II.
Morbid Obesity
A. Understanding of effects of morbid obesity on pulmonary and cardiac physiology В±
pregnancy
B. Ability to perform successful regional anesthesia techniques in morbidly obese patients
233
ASCCA ResidentКјs Guide to the ICU
III.
Respiratory Disease
A. Asthma
B. ARDS
IV. Cardiac Disease
A. Knowledge–understand when invasive monitors are needed for delivery and postpartum
care
1. Congenital heart disease
a) Left-to-right shunt
b) Right-to-left shunts (Tetrology of Fallot)
c) Pulmonary hypertension (Eisenmenger’s Syndrome)
d) Coarctation of aorta
2. IHSS/SAM
3. Ischemic heart disease
4. Valvular heart disease
a) Aortic stenosis
b) Aortic insufficiency
c) Mitral stenosis
d) Mitral regurgitation
5. Peripartum cardiomyopathy
V. Endocrine Disease
A. Knowledge:
1. Diabetes mellitus
2. Thyroid disease
a) Hyperthyroidism
b) Hypothyroidism
3. Skills:
a) Ability to manage glucose control in the parturient during cesarean or vaginal
delivery
VI. Hematologic and Coagulation Disorders
A. Anemia
B. Coagulation disorders
VII. Neurologic Disease
A. Multiple sclerosis
B. Spinal cord injury and closed head injury
C. Myasthenia gravis
D. Seizure disorders
E. Subarachnoid hemorrhage or vascular malformations
VIII.Substance Abuse and HIV Infections
A. Substance abuse
1. Ethanol abuse
2. Opioid abuse and barbiturate use
3. Cocaine abuse
B. HIV infection
IX. Miscellaneous Disorders
A. Renal Disease
B. Liver Disease (e.g., acute fatty liver of pregnancy)
C. Musculoskeletal disorders
D. Scoliosis
E. Rheumatoid arthritis
F. Spina bifida
G. Autoimmune Disorders
234
ASCCA ResidentКјs Guide to the ICU
H.
I.
J.
K.
Prior back injury and surgery, including Harrington rod placement
Antepartum and postpartum hemorrhage
Amniotic fluid embolism
Blunt and penetrating trauma
This chapter is a revision of the original chapter authored by Michael Foss, M.D.
REFERENCES
1. Writer D. Hypertensive disorders. Obstetric anesthesia, 3rd edition, 2004: 846-882.
Overall, one of the best chapters on hypertensive disorders. It provides easy reading from beginning to
end. The tables and graphs help to simplify concepts.
2. Hood D. Preeclampsia. Practical obstetric anesthesia, 1997: 211-237.
This resource has an excellent chapter on preeclampsia. It is well written from the beginning
(epidemiology, morbidity, and risk) through the middle (pathophysiology and anesthetic management)
and the end (“ten practical points”).
3. Gutsche B, Cheek T. Anesthetic considerations in preeclampsia-eclampsia. Anesthesia for obstetrics, 4th
edition, 2004: 305-336.
A very good chapter, with figures to assist in understanding. Has a good table on antihypertensive drugs
used to treat hypertensive episodes during general anesthesia.
4. Hawkins J, Jones M, Joyce T. Critical care of obstetric patients. Critical Care Practice, 1991.
A very good resource for critical care management of obstetric patients.
5. Lechner R. The critically ill obstetric patient. Handbook of critical care pain management, 1994.
This chapter begins with a case discussion. An overview of the physiology of the parturient and
management of labor follows. The following section gives a good treatment of the management of the
critically ill pregnant patient.
6. Birnbach D, Browne I. Anesthesia for obstetrics. Miller’s Anesthesia, 6th edition, 2005: 2329-2333.
This chapter is critical overview of what the anesthesiologist and critical care physician should know
regarding complicated obstetric conditions.
7. Cheek T, Samuels P.Pregnancy induced hypertension. Anesthetic and obstetric management of high risk
pregnancy, 3rd edition, 2004: 386-411.
Both an anesthesiologist and an obstetrician write this chapter.Overall, an excellent resource.
QUESTIONS:
44.1.
A.
B.
C.
D.
E.
Which of the following is true regarding hypertensive disease of pregnancy?
It affects 6-8% of all pregnancies.
It affects multiparous patients approximately 3 to 1.
It accounts for almost 60% of all maternal deaths in the United States.
The incidence has fallen dramatically since the discovery of its etiology.
It is restricted, almost exclusively, to older parturients.
44.2. Which of the following would indicate a diagnosis of severe preeclampsia?
A. A diastolic blood pressure of at least 100 mmHg on two occasions at least 6 hours
apart.
B. More than 300 mg protein in a 24-hour urine collection.
C. A systolic blood pressure of at least 140 mmHg on two occasions at least 6 hours
apart.
D. Urine output of less than 300 mL in a 24-hour period.
E. A diastolic blood pressure of at least 100 mmHg AND a systolic blood pressure of at
least 140 mmHg on two occasions at least 6 hours apart.
235
ASCCA ResidentКјs Guide to the ICU
44.3.
A.
B.
C.
D.
E.
The cardiovascular changes associated with hypertensive disease of pregnancy
include all of the following EXCEPT:
An increased sensitivity to endogenous pressor hormones.
Blood volume may be reduced 30-40% in women with severe disease.
There is a poor correlation between CVP and PCWP in many of these patients.
The colloid oncotic pressure usually increases 4-6 mmHg postpartum in women with
severe disease.
The majority of patients have a hyperdynamic myocardium.
44.4. Which of the following statements regarding preeclampsia are true?
A. Thrombocytopenia occurs in 2-4% of women with preeclampsia or eclampsia.
B. The classic manifestation of severe preeclampsia include severe headache, visual
disturbances, CNS hypoexcitability, and hyporeflexia.
C. Pulmonary edema remains the leading cause of death in women with severe
preeclampsia.
D. Levels of angiotensinogen, angiotensin I, angiotensin II, and aldosterone increase
markedly in preeclamptic women, which aids in the diagnosis as levels usually fall in
normal pregnancy.
E. The GFR is, on average, 25% below that in normal pregnancy.
44.5. Which of the following statements regarding the management of PIH are true:
A. MgSO4 exerts a peripheral effect at the neuromuscular junction.
B. Hydralazine reduces systolic blood pressure primarily as a result of its myocardial
depressant effects.
C. ACE inhibitors are indicated when hydralazine fails to adequately control maternal
blood pressure in the antepartum period.
D. Sodium nitroprusside is a powerful smooth muscle vasodilator but, since it results in
the release of nitric oxide, it is contraindicated during pregnancy.
44.6. All of the following statements regarding eclampsia are true EXCEPT:
A. The classical triad of hypertension, proteinuria, and edema may be absent or only
mildly abnormal in 20% of eclamptic women.
B. The onset of seizures in a term parturient should be considered to be eclampsia until
proven otherwise.
C. Fetal bradycardia typically results during and immediately after a seizure.
D. Approximately 20% of seizures precede delivery.
E. The risk of recurrent seizures after appropriate anticonvulsant therapy is
approximately 10-15%.
236
ASCCA Residents Guide to the ICU
Answers:
1. The ICU Experience
1.1.
1.2.
1.3.
1.4.
[D]
[D]
[B]
[A] The ACGME has identified 6 areas defined as the “core competencies”. They are: Patient
Care, Medical Knowledge, Practice-Based Learning and Improvement, Interpersonal and
Communication Skills, Professionalism and Systems Based Practice. For more details about
this and the Outcome Project (a project to help training programs achieve the competencies),
see their website, http://www.acgme.org/outcome/comp/compMin.asp
2. ICU Management
2.1.
2.2.
2.3.
2.4.
2.5.
[C] Open communication with the primary physician is key to avoid conflicts at the time of
discharge from the ICU.
[C] Respiratory therapy and nursing can accelerate weaning base on an established protocol.
[C] Only C is Correct
[B] Large nursing ratios worsen outcomes in the ICU.
[D] The APACHE (Acute Physiology, Age, Chronic Health Evaluation) Scoring system helps to
predict patient mortality and helps to compare institutions.
3. Family Support and Ethics Issues
3.1.
3.2.
3.3.
3.4.
3.5.
3.6.
[E]
[B]
[B]
[C]
[B]
[C]
4. Cardiopulmonary Resuscitation (CPR)
4.1.
4.2.
4.3.
[D] Since all patients in an ICU have continuous ECG monitoring, development of cardiac
arrest in this setting should be witnessed. A defibrillator should be immediately available in the
ICU. Under such circumstances, defibrillation should be the first intervention and should not be
delayed for any other therapy. If it can be applied within one minute of arrest, even basic life
support may be unnecessary.
[D] Using the least necessary energy levels lessens the myocardial damage and arrhythmias
that may occur with cardioversion/defibrillation. The maximum energy should be 120 to 200
Joules for biphasic defibrillators and 360 Joules for monophasic defibrillators. Proper
synchronization mode selection is necessary to avoid induction of ventricular fibrillation when
the countershock falls on the T-wave of the electrocardiogram. The general recommendations
for selecting synchronized shocks are as follows: 1) A patient with symptomatic stable
tachycardia; 2) A patient with unstable tachycardia with pulses. Otherwise, the asynchronous
mode should be selected for delivering shocks to prevent delays in therapy. General
recommendation for delivering asynchronous shocks include the following: 1) A patient who is
pulseless; 2) An unstable, rapidly deteriorating patient with rapid polymorphic VT, when delay in
delivery of the shock is not considered safe; 3) When you are unsure whether monomorphic or
polymorphic VT is present in the unstable patient. If VF is induced following an
unsynchronized shock, asynchronized defibrillation should be performed immediately.
[E] Administration of sodium bicarbonate produces carbonic acid which is converted to carbon
dioxide. In the setting of cardiac arrest and CPR, cardiac output is known to be severely
decreased such that efficient elimination of carbon dioxide is absent, even with adequate
ventilation. Therefore, carbon dioxide permeates cell walls and will lead to severe tissue
acidosis that will not be reflected in arterial blood gases. Therefore, the AHA recommends
237
ASCCA Residents Guide to the ICU
4.4.
4.5.
4.6.
avoiding sodium bicarbonate until successful resuscitation has reestablished a perfusing
rhythm. An exception is cardiac arrest secondary to known hyperkalemia.
[C] The effectiveness of epinephrine during cardiac arrest is entirely related to its alphaadrenergic stimulating properties of raising vascular resistance, aortic blood pressure, and
myocardial and cerebral perfusion pressure during closed chest compressions. Direct
stimulation of the myocardium through its beta-adrenergic effects is unnecessary for successful
restoration of spontaneous circulation although these effects may be useful in the immediate
post-resuscitation period to overcome the global myocardial stunning. There is little evidence
that epinephrine improves the success rate or lowers the energy necessary for defibrillation.
[A] Epinephrine is absorbed through the tracheal mucosa when administered through the
endotracheal tube. With an intact circulation, peak blood levels and the time course of
physiologic effect are similar to intravenous administration. However, during CPR with
markedly reduced pulmonary circulation, peak levels tend to significantly reduce and the time
course of effect is delayed and unpredictable.
[A] There is little evidence that administering calcium salts during CPR will improve
resuscitation success. The exceptions include known severe hypocalcemia, severe
hyperkalemia, or calcium channel blocker overdose.
5. Transport of the Critically Ill
5.1.
5.2.
5.3.
5.4.
5.5.
[E]
[B]
[D]
[B]
[B]
6. Sedation of the Critically Ill
6.1.
6.2.
6.3.
6.4.
6.5.
[D] Although midazolam and fentanyl are considered short acting agents, after prolonged
infusion, their kinetics change and active metabolites may accumulate, increasing the time to
recover. Flumazenil and naloxone can be used to reverse the effects of midazolam and
fentanyl; however, they may precipitate acute withdrawal syndromes and cardiovascular
catastrophe. Without localizing findings, an organic cause of coma is less likely. Nonconvulsive status is an uncommon cause of coma, and even less likely in a patient receiving
benzodiazepines.
[E] Breakthrough pain is common with bolus narcotic administration and may cause agitation.
Benzodiazepine withdrawal, which consists of autonomic hyperactivity, usually occurs several
days following cessation, but with bolus administration, withdrawal can occur during drug
administration. Organic causes for agitation should always be sought. Carbon dioxide
accumulation can lead to sympathetic hyperactivity.
[A] Life-threatening status asthmaticus may respond to inhalational anesthesia when other
treatment has failed; however, no controlled studies have determined the superiority of one
agent over the other. Halothane is the best studied direct bronchial dilator; however, its cardiac
depressive effects limit its use in this circumstance. General anesthesia should be given by a
trained individual and scavenging of waste gases should occur. Atmospheric pollution is not
generally monitored in the ICU. Renal failure from prolonged use of methoxyflurane was
frequent in the past. Renal failure from prolonged use of methoxyflurane was frequent in the
past. Elevated fluoride levels may occur with isoflurane; however, renal failure is unlikely.
[C] Failure to provide effective manual ventilation is common in inexperienced hands due to
upper airway obstruction. Foreign bodies can also cause obstruction. Seizures are
occasionally seen with narcotic administration, but the rigid chest syndrome is much more
likely with a large bolus dose of fentanyl.
[B] Unlike the benzodiazepines and other sedative drugs, propofol remains a short-acting
sedative even after days of infusion. Hypotension frequently follows bolus administration but is
rare with continuous infusion. This is believed to be due to vasodilation, although some studies
suggest a mild myocardial depressant effect. Since it is a short acting agent, it must be given
as an infusion to achieve stable effects. Propofol can be combined with narcotics and the
effects are synergistic, often resulting in marked depression of respiratory drive. The costeffectiveness of propofol depends on the clinical situation and what alternatives are available.
238
ASCCA Residents Guide to the ICU
6.6.
[E] Safe sedation requires appropriate monitoring, emergency drugs, and equipment. The
most important monitor is a trained individual whose only responsibility is to watch the patient.
This cannot be the operator, but must be a second individual dedicated to this task and not
necessarily an anesthesiologist. It is not mandatory that an ECG be used; however, pulse
oximetry is more likely to identify problems. Resuscitation equipment must be available;
however, airway equipment is more important than a defibrillator.
7. Analgesia in the ICU
7.1.
7.2.
7.3.
7.4.
7.5.
[B]
[C]
[D]
[B]
[C]
8. Neuromuscular Blockade
8.1.
8.2.
8.3.
8.4.
8.5.
[B] Phenytoin is associated with resistance. The other drugs are all associated with
potentiating the effect of NMB drugs.
[C] Pipecuronium has long lasting effects
[D] Hyperkalemia is a potential complication of the depolarizing muscle relaxant,
succinylcholine, in certain patients
[C] There is increasing recognition of prolonged weakness in the ICU associated with the use
of NMB drugs. However, this entity is most likely multifactorial with concomitant use of
steroids, drug overdose, drug or drug metabolite accumulation, sepsis and other as potential
etiologies. Propofol has not been implicated in damaging the neuromuscular junction.
[A] Although adjustment of the TOF to 2 twitches is recommended while using NMBA, its use
is sometimes challenging in the ICU patient due to diaphoresis and edema.
9. Acid-base Balance
9.1.
9.2.
9.3.
9.4.
9.5.
[D] The most common acid-base abnormality associated with salicylate intoxication is a
respiratory alkalosis caused by direct stimulation of the medullary respiratory center. A pure
metabolic acidosis due to salicylate intoxication is unusual. Even in severe salicylate
intoxication, it is more common to see a mixed disorder secondary to the anion gap metabolic
acidosis due to both salicylic acid and formation of lactate.
[C] Winter’s formula is used to estimate the expected drop in PaCO2 that occurs as a result of
respiratory compensation for a metabolic acidosis. The formula is less accurate for mild
acidosis with a bicarbonate level > 20 mmol/L. The formula is as follows:
Expected PaCO2 (mmHg) = [(1.5 x HCO3-) + 8]
[E] In contrast to acute acidemia, acute alkalemia reduces cerebral blood flow, increases the
affinity of oxygen for hemoglobin and increases calcium binding to proteins.
[E] Depending on the etiology of the acidosis, bicarbonate therapy remains controversial. It is
generally acceptable to administer alkali when the pH falls below 7.2 due to a metabolic
acidosis to prevent arrhythmias and cardiovascular collapse. However, it may have harmful
consequences as listed in the question above and therapy with alkali should be monitored
closely.
[B] The anion gap is the difference between the major cations (Na+ and K+) and major anions
(HCO3- and Cl-) which is normally 12 + or – 4 when potassium is included. The difference
between these measured ions is typically due to the other normal anions in circulation, mainly
albumin and phosphate. To a much lesser extent, sulfates and lactate also contribute to the
anion gap, but their contribution is offset by other proteins and ions. Therefore, the anion gap
is used to narrow the differential diagnosis of a metabolic acidosis, but is limited in the critically
ill patient by changes in albumin and phosphates to the point that it has been recommended
that the “normal” range for the anion gap be adjusted for each patient by the following formula:
“adjusted” anion gap = 2 x [albumin(g/dL)] + 0.5 x [phosphate(mg/dL)]
239
ASCCA Residents Guide to the ICU
10. Metabolic and Endocrine Abnormalities
10.1. [C] Early goal resuscitation is considered the cornerstone of management of septic patients.
Tight glucose control and the use of vasopressin in refractory hypotension despite fluid
resuscitation are also recommended strategies. Steroids have been proven to be effective and
safe in those individuals that have abnormal response to stimulation test.
10.2. [A] Tight glucose control has been proven to improve mortality in the surgical ICU but not in the
medical ICU. (Except in patients staying longer than 3 days). The other options are correct
despite the type of ICU.
10.3. [D] Vasopressin improves renal parameters in at least one study and has been proven effective
and safe in septic patients for 24 hrs. No myocardial ischemia in up to 24 hrs of treatment has
been described and a trial comparing vasopressin vs. norepinephrine is on going.
Norepinephrine should still be consider first line in septic patients requiring vasopressors.
10.4. [D] Euthyroid sick syndrome is defined as abnormal TFT’s without thyroid disease. There are
many presentations with a low T3 being the most common. This particular presentation seems
to be caused by decreased conversion from t4 to T3 in the liver.
10.5. [D] CIRCI seems to be multifactorial with deficiencies beyond the adrenal gland itself. It is
commonly characterized by hypotension. Steroid replacement appears indicated only in
patients who remain pressor dependent despite adequate fluid resuscitation. Etomidate, even
at single doses, has been implicated in the development of CIRCI.
11. Fluid Management
11.1.
11.2.
11.3.
11.4.
11.5.
[B]
[A]
[D]
[D]
[D]
12. Gastrointestinal Bleeding
12.1. [E] The incidence of significant bleeding due to superficial erosive gastritis, or stress gastritis in
decreasing. The disease process is well recognized and a high index of suspicion maintained.
Improvements in resuscitative techniques with early recognition and aggressive correction of
perfusion deficits, use of prophylactic anti-secretory agents, and the early institution of enteral
feeds have all contributed towards the observed reduction in incidence. All of the above are
therefore true.
12.2. [D] The first step in the management of patients with gastrointestinal bleeding is to institute
resuscitative measures. After assessing and securing the airway, IV access is obtained and
fluid resuscitation is begun. Failure to respond to isotonic crystalloids necessitates the use of
blood products.
12.3. [E] Patients with head injury, burns, and need for steroids and those with liver failure are all are
an increased risk of bleeding from the upper gastrointestinal tract. Prophylactic anti-secretory
agents are indicated in all these patients. A complicated soft tissue infection by itself does not
increase the risk of bleeding
12.4. [A] Failure to respond to conventional measures is an indication for surgical intervention. As the
source of the bleed cannot be identified and is presumed by history to be in the lower
gastrointestinal tract, a subtotal colectomy must be performed to eliminate the source of
bleeding.
12.5. [D] The patient has a Mallory-Weiss tear. It is unlikely to cease spontaneously. Anitemetics and
increasing the gastric pH may decrease progression but does not stop the bleeding. EGD with
thermal coagulation is the most definitive method for controlling bleeding. Esophagectomy is
required in the rare instance that endoscopic treatment fails.
13. Thromboembolic Disease
13.1. [B] The primary goal is full anticoagulation for patients in which a high index of suspicion for
PE exists. The diagnostic study chosen will be partly dependent on institutional strengths and
rapid availability of any given study.
13.2. [E] A high probability V/Q scan is accepted as evidence of PE in greater than 90% of patients,
and no further studies are needed. Exceptions to the above rule would include patients who
240
ASCCA Residents Guide to the ICU
have previously experienced a PE and who have not received a V/Q scan or baseline
comparison. With low clinical suspicion, only 56% of patients with a high-probability V/Q scan
will have a PE. Warfarin therapy should be initiated within 24 hours of the diagnosis of DVT.
13.3. [C] Normal lower extremity studies, of course, don’t preclude DVT or PE. Depending on the
patient’s hemodynamic and pulmonary vasculature response, hypoxemia may or may not be
present with clinically significant PE. Hormone replacement is not associated with an
increased risk of DVT, although pharmacologic estrogen therapy (i.e., oral contraceptives) is.
14. Bleeding in the ICU
14.1. [C] In any bleeding, the first priority is volume and/or blood replacement followed by
investigation of the bleeding causes to establish proper therapy
14.2. [D] Hemophilia A or Factor VIII deficiency is a sex-linked recessive disease occurring in the
U.S.A. with an incidence of 1 in 5,000 to 10,000 males. Anti-thrombin III deficiency would
render a patient hypercoagulable. Beta-thalassemia may lead to splenomegaly and splenic
sequestration of platelets but is not a major cause of bleeding disorders. The prevalence of
vWD is believed to be as high as 1-2%, although the incidence of severe (homozygous) vWD
is only one in a million.
14.3. [C] Factor VII is used in the management of hemophiliacs and recently its use have been
investigated for management of bleeding that does not responds to conventional therapy .
14.4. [A] Autologous blood transfusions are advocated by some to minimize allopathic transfusions in
the periperative period
14.5. [C] Blood transfusions are unlikely to cause hypercoagulability.
15. Specialized Nutrition Support in Critically Ill Patients
15.1. [C] Phosphorus regulates the amount of 2,3-diphosphoglycerate (2,3-DPG) in red blood cells.
2,3-DPG promotes the release of oxygen by hemoglobin thus delivering oxygen to tissues.
Thus, reduced RBC 2,3-DPG levels are associated with a shift to the left of the oxyhemoglobin
saturation curve. This shift to the left is associated with a decrease in the release of oxygen to
peripheral tissues and may result in tissue hypoxia.
15.2. [D] The human body is not capable of synthesizing fatty acids with a double bond between the
ninth and terminal carbon of the fatty acid chain. Therefore, linoleic acid and О±-linolenic acid
are essential fatty acids in man. Arachidonic acid may be synthesized from linoleic acid and is
therefore conditionally essential to the human body.
15.3. [D] Most current randomized, prospective studies evaluating enteral vs. parenteral nutrition
after major trauma has concentrated on patients with head injuries, blunt and penetrating
injuries, and burns. Although both enteral and parenteral feeding are commonly used in the
ICU setting, current evidence suggests that enteral delivery of nutrition significantly reduces
subsequent septic complications, presumably because of the preservation of gastrointestinal
barriers and host defenses.
15.4. [C] This patient is currently experiencing minimum stress, therefore her calorie and protein
requirements can be estimated to be 25-30 total kcal/kg/day and 1.5 gm protein/kg/day,
respectively. Only the TPN regimen outlined in answer C would meet these needs. All of the
other TPN regimens provide excessive amounts of calories and protein, placing this patient at
risk for adverse events related to overfeeding.
15.5. [C] Surgical procedures may frequently cause malabsorption requiring nutritional intervention.
Parenteral nutrition may be indicated in the immediate postoperative period following a subtotal
gastrectomy if significant malnutrition is present. Factors leading to malnutrition as a result of
this procedure include impaired mechanical digestion of food by the deformed stomach, more
rapid and less regulated gastric emptying, and decreased intestinal transit time. Parenteral
nutrition is generally not required following an esophagogastrectomy because the
gastrointestinal tract is nearly always functional. Patients undergoing vagotomy/pyloroplasty
can usually be managed with an antidumping diet. This oral diet includes frequent small
meals, avoidance of simple sugars to decrease the osmolality of food, and increased complex
carbohydrate intake.
15.6. [C] A recent analysis of perioperative nutrition support suggests that significant improvements
can be demonstrated when parenteral nutrition is given to severely malnourished patients in
adequate amounts for ≥7-15 days preoperatively. Benefits from parenteral nutrition include an
improvement in anthropometric measurements, reduction in major complications (i.e., intra-
241
ASCCA Residents Guide to the ICU
abdominal abscess, peritonitis, anastomotic leakage), a decrease in postoperative sepsis, and
lower postoperative mortality. Data available from published reports suggests that
administration of parenteral nutrition <7 days in the preoperative period is unlikely to
significantly improve outcome in severely malnourished patients.
15.7. [C] Adults are considered at risk for developing malnutrition if they have inadequate nutrient
intake for ≥7 days or if they have lost ≥5% of usual body weight in 1 month, ≥10% of usual
body weight within 6 months, or are 20% under ideal body weight. The onset or development
of malnutrition should be prevented or slowed by giving appropriate patients optimum nutrition
therapy. If the gastrointestinal tract is compromised, parenteral nutrition should be initiated to
replenish and maintain nutrient stores.
16. Routine Monitoring in the ICU
16.1. [C] See reference 1 in reading list.
16.2. [D] The first line therapy for oliguria is fluid resuscitation. If urine output declines after
adequate fluid resuscitation, a PA catheter may be indicated.
16.3. [D] The opposite is true. The CVP tracing dampens (flattens) with tachycardia.
16.4. [B] The risk of pneumothorax is smaller, but not zero, with the internal jugular approach
compared with the subclavian approach.
16.5. [E] The major drawback of a ventriculostomy is an increased rate of infection.
17. Rational Use of the Pulmonary Artery Catheter (PAC)
17.1. [C] The PAC with special capabilities for ejection fraction and end diastolic volume makes
those measurements only on the right side of the heart by a modification of the thermodilution
technique. Left ventricular end diastolic volume and ejection fraction cannot be monitored with
a right sided PAC.
17.2. [B] The determinants of oxygen delivery are cardiac output, hemoglobin, and arterial oxygen
saturation when looking at total body or global oxygen transport. The determinant of oxygen
demand is oxygen consumption. Sv02 monitors the balance of this supply and demand but
does not reliably correlate with any of the four factors.
17.3. [A] Pulmonary artery rupture is a rare but commonly lethal event with pulmonary artery
monitoring. It occurs most commonly with elevated pulmonary artery pressures as might be
seen in mitral stenosis or primary pulmonary artery hypertension. Wedging the catheter in
these conditions is commonly difficult and the high pressures seem to put the patient at risk for
vessel rupture when the balloon on the tip of the catheter is inflated.
17.4. [B] When floating a PAC from the preferred site of the right internal jugular vein, the catheter
will enter the right atrium at 20-25 cm. Advancing the catheter to 30-35 cm will cause the
catheter to cross the tricuspid valve and enter the right ventricle. A pulmonary artery trace will
be seen on the bedside monitor at 40-45 cm. Finally, a wedge position is commonly obtained
at 50-55 cm. Knowing these distances is essential for the safe passage of a PAC and the
avoidance of looping and knotting in the heart.
17.5. [D] The greatest measurable disturbance in global oxygen supply and demand occurs in
cardiogenic shock. Low cardiac output causes excessive extraction of oxygen from arterial
blood and a low mixed venous oxygen saturation. Thus there is a correlation between the
severity of cardiogenic shock and SvO2. That is not true in septic, anaphylactic, or spinal
shock where cardiac output is high and SvO2 is high and may not correlate with the
hemodynamic disturbance of loss of peripheral resistance that characterizes these other
clinical problems.
18. Echocardiography in the Intensive Care Unit
18.1. [B] The evaluation of the hemodynamically unstable patient is a Class I recommendation for
echocardiography. Pressure parameters obtained from pulmonary artery catheters or central
venous lines are oftentimes misleading or misinterpreted. A focused examination with the
acquisition of 2D images may add to diagnostic process and possibly redirect management
strategies. Although echocardiography is used in the other scenarios of the questions, the
strength of supporting evidence or expert opinion that echocardiography improves outcome is
less.
18.2. [D] The TEE probe has a diameter of around 1 cm and may inadvertently lead to an
esophageal injury, tear or rupture in the setting of esophageal strictures. Rheumatic arthritis
242
ASCCA Residents Guide to the ICU
18.3.
18.4.
18.5.
18.6.
requires careful evaluation for atlantoaxial instability. TEE probe insertion and manipulation
should be performed extremely cautiously. TEE can be performed safely in the liver transplant
recipient even in the presence of esophageal varices and coagulopathy.
[A] Initial resuscitation efforts should be focused around the basic principle of airway, breathing,
and circulation as simple maneuvers may stabilize the patient. TEE may help in the diagnosis
and management of refractory life-threatening instability.
[D] Aortic dissection may lead to aortic regurgitation should the intimal flap tear through the
aortic valve and acute pericardial effusion with tamponade and involvement of the coronaries
may result in acute myocardial ischemia and dysfunction. A D-shape ventricle is the hallmark of
a volume or pressure overloaded right ventricle in which the intraventricular septum is flattened
by the increased right sided pressure pushing the septum across to the left ventricular cavity.
[D] Loculated hematoma or fluid collections within the pericardium or mediastinum in the
postcardiac surgical patient may lead to obstruction of any lower pressure chamber including
the left atrium. Typically, the left ventricle is underfilled and the left ventriulcar enddiastolic
dimensions are decreased. Diastolic collapse may be seen in the right atrium, right ventricle or
left atrium. The inferior vena cava is dilated without any respiratory variation in its diameter.
[C] Acute coronary syndromes may lead to myocardial dysfunction. On echocardiography
decreased myocardial systolic thickening and excursion is noted. Typically, the left ventricle is
involved leading to diastolic failure with increased diastolic area and possible biventricular
failure with an increased right ventricular size. “Kissing papillary muscles” is often observed
during hypovolemia and describes the process of the posteriomedial and anterolateral papillary
muscles coming together very closely during ventricular systole.
19. Airway Management in the Intensive Care Unit
19.1. [C] The neck must be maintained in-line to prevent injury and reduce the potential for further
injury. Nasal or oral intubation is acceptable, depending on other injuries, patient condition,
urgency of intubation, etc. A lateral C-spine film can be used to document injury, but a “normal”
film does not rule out ligamentous injury or instability. While there is controversy as to whether
a multi-slice CT scan with reconstruction images alone or MRI is required to rule out all injury, it
is generally accepted that a more advanced study than plain films is required.
19.2. [D] Positive pressure ventilation can actually be provided by mask, e.g., BiPap. Hypercarbia
can be a patient’s normal status or can be a result of a loss of FRC and can, therefore, be
treated with CPAP. In addition, if due to a medication effect, hypercarbia may be amenable to
a reversal agent such as naloxone for narcotics or flumazenil for benzodiazepines. While
ventilation can be provided to the obtunded patient by other means, intubation may be
considered necessary for airway protection. However none of these are absolute indications
for oral-tracheal intubation.
19.3. [A] Cricoid pressure involves gentle backwards pressure of the cricoid cartilage on the
esophagus to prevent passive aspiration of gastric contents in patients at risk for aspiration,
such as patients with full stomachs or active gastric-esophageal reflux disease (GERD).
However, it will not prevent aspiration with absolute certainty and thus, rapid sequence
intubation involves quickly pushing medications to achieve optimal intubating conditions as
rapidly as possible and performing direct laryngoscopy and placing the ETT as soon as
possible. Aspiration of gastric contents is certainly possible in patients with full stomachs, for
example a patient with a small bowel obstruction. When performing cricoid pressure, it should
be continued until the ETT is confirmed in the trachea by end-tidal CO2 and bilateral breath
sounds rather than releasing the pressure immediately after the ETT is placed.
19.4. [C] The Cook airway exchange catheter is designed to help guide placement of the new ETT.
In addition, one is also capable of using the catheter to jet ventilate the patient or provide
oxygen insufflation with the standard adapter. After dislodgement of the catheter, replacement
is not generally feasible, particularly when direct laryngoscopy is found to be difficult with a
grade IV view. One now has to consider the management of a difficult airway. After calling for
help, the next step is to attempt ventilation and establish an airway by mask ventilation. If this
is successful, then mask ventilation can be continued until the airway is secured with more
sophisticated maneuvers, including but not limited to fiberoptic inbution, intubating laryngeal
mask airway (LMA). If bag mask ventilation is inadequate, then consideration for other airways
must be given, with transtracheal jet ventilation and cricothyroidotomy being the final steps in
the difficult airway algorithm.
243
ASCCA Residents Guide to the ICU
20. Chronic Airway Management
20.1. [D] All other options are indications for tracheotomy
20.2. [D] Although not mandatory, fiberoptic guidance during percutaneous tracheostomies helps to
avoid injuries and assures final tube position.
20.3. [B] Tracheal buttons delay stoma healing
20.4. [B] Acute pneumothorax is an acute complication
20.5. [D] Major complications can occur during tracheotomy tube changes. It should be only
performed after a secure tract has formed
21. Management of Mechanical Ventilation
21.1.
21.2.
21.3.
21.4.
21.5.
21.6.
21.7.
[D]
[B]
[B]
[D]
[A]
[B]
[B]
22. Side- Effects of Mechanical Ventilation
22.1. [B] Large tidal volumes and high airway pressures can result in ventilator-induced lung injury.
Injury is postulated to arise from mechanical and biological factors. Most patients do not have
macroscopic lung damage; the underlying lung damage is worsened by VILI.
22.2. [D] Mean airway pressure is determined by tidal volume, minute ventilation, level of PEEP, and
inspiratory time. Increased inspiratory time results in increased mean airway pressure.
Increasing mean airway pressure increases oxygenation.
22.3. [E]
22.4. [E] Intrinsic PEEP (a.k.a. auto-PEEP) has the same side-effects as extrinsic PEEP. It is,
however, often unrecognized. Additionally, since ventilator triggering usually requires the
generation of either a set pressure or flow drop, intrinsic PEEP, when not accounted for, may
increase the difficulty in triggering the ventilator.
22.5. [B]
22.6. [B] Patients with COPD exacerbations have improved outcomes with noninvasive ventilation
strategies compared with intubation. Noninvasive ventilation is best suited for awake,
cooperative patients. Gastric distension may occur. Patients are also at an increased risk for
aspiration.
22.7. [C] Tracheal injury may occur with limited periods of intubation. However, the risk increases
with increased duration of intubation. Evidence of tracheal dilation on CXR is a relatively late
sign of tracheal injury.
22.8. [A]
23. Lung Protection Strategies
23.1. [B] False. ALI/ARDS is most commonly a syndrome involving heterogeneous lung injury,
despite CXR appearances.
23.2. [C] Diagnosis of ALI/ARDS requires correlation between radiological information and gasexchange characteristics (e.g. paO2/FiO2 ratio of less than 200 for ARDS, and between 200
and 300 for ALI) in addition to absence of increased hydrostatic pressure (e.g. heart failure
from LV-dysfunction, other).
23.3. [D] All of the above. Gender and height of a given patient give rise to an ideal body weight
which is considered for ventilation in ALI/ARDS rather than the actual body weight. This favors
normalization of tidal volumes to lung size for a given patient. Initial arterial blood gas analysis
can be used to estimate minute ventilation requirements and PEEP/FiO2-levels, with
adjustments being made as needed based on follow-up ABG’s (after the initial ventilator
prescription has been instituted)
23.4. [E] All of the above. Best available data suggests improvement in patient outcomes by use of a
coordinated strategy involving lung protective ventilation and negative fluid balance.
244
ASCCA Residents Guide to the ICU
24. Weaning From Mechanical Ventilation
24.1.
24.2.
24.3.
24.4.
24.5.
24.6.
[E]
[D]
[E]
[D]
[A]
[C]
References 6 and 7 emphasize the value of T-piece weaning.
It is important to know how to set the ventilator (Reference 4).
Reference 7 emphasizes protocol-driven techniques in hastening ventilator weaning.
See the failure to wean checklist in the text.
These parameters are consistent with adequate ventilatory reserve for extubation.
References 6 and 7 emphasize the SIMV is an inefficient mode in chronic weaning.
25. Support of the Failing Circulation Shock
25.1.
25.2.
25.3.
25.4.
25.5.
25.6.
[A]
[D]
[A]
[E]
[C]
[C]
26. Diagnosis and Treatment of Dysrhythmias
26.1.
26.2.
26.3.
26.4.
26.5.
26.6.
26.7.
[E]
[A]
[D]
[E]
[A]
[D]
[A]
27. Diagnosis and Treatment of Myocardial Ischemia
27.1.
27.2.
27.3.
27.4.
27.5.
[C]. Most PMI are subendocardial
[B]. ACE inhibitors are contraindicated in patients with renal failure
[A]. B-Blockers should be taken indefinitely after an MI
[C]. By increasing oxygen delivery the myocardial damage might be limited
[C]. Patients with stents have a risk of thrombosis of stents in the perioperative period,
particularly if anticoagulation is stopped.
27.6. [D]. The other factors are related to the possibility of the presence of CAD but have not been
demonstrated that they increase risk of PMI independently.
28. Valvular Heart Disease
28.1. [E] This patient may have aortic stenosis. The extent (area and peak-to-peak gradient) of the
disease process is not known. If the patient has significant aortic stenosis, the hemodynamic
management aims to avoid tachycardia with maintenance or restoration of sinus rhythm. Both
ejection time as well as diastolic filling time is reduced wit tachycardia, increasing the work of
the heart (the stroke volume has to be ejected in a shorter time frame requiring the generation
of higher LV pressures) while decreasing the time for the LV chamber to fill. Maintenance of
afterload is a close second priority to ensure adequate coronary perfusion pressure.
28.2. [C] Patients with severe mitral stenosis have a large left atrial-left ventricular pressure gradient
that permit large atrial volume and pressure, causing left atrial enlargement, left atrial
dysfunction, and atrial fibrillation. TEE is the best tool to identify any clot formation, particularly
in the left atrial appendage. Cardioversion is less successful when the atrium is dilated (>4.5
cm2).
28.3. [B] If a patient has a dysfunctional prosthetic mitral valve, the best diagnostics tool to assess
the integrity and function of the valve is TEE. Transthoracic echocardiography can not visualize
the mitral valve apparatus well because of acoustic shadowing from the mechanical device.
28.4. [C] Intraaortic balloon pulsation is useful in all of the above critical situations except aortic
insufficiency, in which the pathology would be augmented by IABP.
245
ASCCA Residents Guide to the ICU
29. The Systemic Inflammatory Response Syndrome (SIRS), Sepsis and Multiple
Organ Dysfunction Syndrome (MODS)
29.1. [E] SIRS is the starting point of a continuum that extends to Multiple Organ Dysfunction
Syndrome. Given the recent occurrence of the injury, the hyperdynamic state the pt is in is
consistent with this diagnosis. There is no evidence of hypotension and both MH and thyroid
storm would be associated with fevers. Opioid overdose would cause bradypnea.
29.2. [D] The hallmark of the sepsis syndromes is microcirculatory endothelial dysfunction with
massive endocapillary leak leading to edema and hypoperfusion. The patient has an elevated
wedge pressure with persistent hypotension despite massive resuscitiation. At this point, the
patient would not benefit from more fluids and needs vasopressors to restitute peripheral
vascular tone.
29.3. [C] All of the above measures are indicated in conjunction with initiation of broad-spectrum
antibiotic therapy as soon as cultures are taken depending on the most likely pathogens. Once
the organism is identified, antibiotics should be de-escalated and tailored to the particular
pathogen, eliminating those antibiotics not needed.
29.4. [D] In acute tubular necrosis, the kidney loses its ability to concentrate and filter. Sodium is very
efficiently retained with normal kidney function, therefore a high concentration of this ion in the
urine is indicative of intrinsic renal damage. All of the other options are consistent with prerenal
(hypovolemic) failure.
29.5. [A] Although there are subsets of septic patients who are cortisol deficient, this condition is not
always present. A random cortisol level followed, if needed, by cortisol stimulation test helps
identify those patients who are likely to benefit from steroid replacement. The other options are
present in septic shock.
30. Nosocomial Infections
30.1. [E] All of the answers have been implicated in studies to contribute to nosocomial infections in
the ICU. Length of ICU stay has been shown as the predominant risk factor, which is not
surprising given that the less time spent in the ICU usually translates into less time for
exposure to other risk factors such as mechanical ventilation and central venous cannulation.
30.2. [B] Arterial catheters are now believed to have complication rates similar to those for venous
catheters. Thrombotic complications have been reported to occur in 19-38% and infections in
up to 23% of patients with indwelling arterial catheters.
30.3. [C} Alcohol-based hand sanitizers have improved the compliance and thus the reduction of
nosocomial infections throughout the hospital setting. Alcohol-based sanitizers have excellent
germicidal action against gram-postitive and gram-negative bacteria, many viruses including
Human Immunodeficiency virus and influenza virus. However, these products are not effective
against bacterial spores such Clostridium Difficile. Warm soap and water hygiene are therefore
still recommended for patients infected with Clostridium Difficile.
30.4. [E] You should wear gloves any time there is the possibility of exposure to blood or body fluids.
Gloves alone are adequate for drawing venous blood or placing an intravenous line, for
example. More equipment is required if there is a reasonable risk of splash. Fluid-resistant
masks and eyewear should be worn any time there is a splash potential, such as during an
arterial stick, or drainage of an abscess under pressure. Masks should always be used in
conjunction with protective eyewear, such as goggles or masks with eye shields. Fluidresistant gowns are worn any time there is a significant risk of splash of blood or body fluid
during a procedure. If the splash is predictably limited, as in nasotracheal suctioning,
manipulating an intravenous line, or drawing an arterial blood gas, then gloves, mask and
eyewear are adequate.
30.5. [A] Several studies have now demonstrated that routine replacement of central venous
catheters every 7 days did not reduce the rate of catheter-related bloodstream infections.
Furthermore, increased complication rates have been associated with the practice of routine
replacement and it is no longer recommended.
31. Infections and Antibiotic Therapy in the Intensive Care Unit
31.1. [A] Severity of acute and chronic illnesses is probably the most important predictor. After
controlling for other factors, gender by itself is probably not a risk factor for infection.
246
ASCCA Residents Guide to the ICU
31.2. [B] The most common site of colonization is the oropharynx/airway. Differentiating between
organisms causing infection vs. colonization is difficult.
31.3. [C] E. coli and B. fragilis are the most common organisms cultured from intraabdominal
abscesses. S. pneumonia and H. influenzae cause community acquired pneumonias.
Although C. albicans is frequently isolated from the airway, it rarely causes pneumonia in
patients who are not severely immunosuppressed (neutropenic leukemic, bone marrow
transplant recipients).
31.4. [D] If 10,000 CFU/ ml are isolated, repeat culture may be warranted.
31.5. [B] Proven line infections require replacement of the catheter to a new site unless absolutely
impossible. Routine changing of catheters without evidence of infection is not warranted.
31.6. [D] Cefepime, Tobramycin, and Imipenem all have useful anti-Pseudomonas activity.
31.7. [B] Full barrier precautions and meticulous aseptic technique are important actions during
insertion and maintenance of central venous catheters.
31.8. [C] Voriconazole is preferred for aspergillosis.
31.9. [A] Reported to be as high as 4%. The incidence is probably lower if dosing is adequately
adjusted for renal dysfunction. Meropenem does not appear to be nearly as epileptogenic.
32. Antibiotic Prophylaxis
32.1. [C] Common exceptions include evidence of perforation (as in case sited) or possible ongoing
infection (cholecystitis or appendicitis). Nonetheless, even in these cases antibiotics can
generally be given for one day or less.
32.2. [A] In fact, prophylaxis for the actual duration of the operation only (no postoperative doses)
may be adequate, though this is unproven.
32.3. [B] Despite all efforts, there remains a low but real wound infection rate even under ideal
circumstances. These infections are probably due to bacteria found in hair follicles, sweat
glands, etc., that may be out of reach of both skin cleansers and systemic antibiotics.
32.4. [D] For many reasons, including: traumatic wound, perforated viscus, unprepared colon.
32.5. [B] Important, of course, when choosing antibiotics. All others are also found in the colon, but
do not predominate.
32.6. [B] Based on studies of both serum and tissue (colon) levels. This issue may be related to
why longer operations have consistently higher wound infection rates even when other
variables are controlled.
32.7. [A] As a historical note, Köcher reduced his wound infection rate for thyroidectomy to 3%,
without the use of effective skin antiseptics or antibiotics, just by using meticulous technique.
33. Management of the Immunocompromised Patient
33.1. [B]. Hypothyroidism is a delayed complication seen after 90 days post BMT. (refs 4,5)
33.2. [C]. During outbreak of nosocomial fungal infections, dry mopping was found to result in
dispersion of 800 000 particles/m3, whereas wet mopping resulted in dispersion of 30000
particles/m3. [ref 5]
33.3. [A]. Opportunistic infections are less likely early after transplant since the state of
immunosuppression is not developed. (ref. 6)
33.4. [B] Macrolides inhibit RNA-dependent protein synthesis by reversibly binding to the 50S
ribosomal subunit (ref.8)
33.5. [D] In microbiologically documented infection, use of the most narrow-spectrum agent possible
is recommended (ref. 8)
33.6. [A] Fever is infrequent sign of rejection in kidney transplant. (ref. 12)
33.7. [D] Humidification of the inspired air by heated humidifier causes liquid condensate to rain out
in the circuit and to become contaminated with high-level bacterial growth; washing back of the
liquid through the patient’s endotracheal tube, a potent inoculum is delivered directly to the
lower respiratory tract. The greater the manipulation of the ventilator circuit, the greater the risk
for lower respiratory tract infection. It is recommended that the ventilator circuit should not be
changed on schedule. (ref.13)
33.8. [E] Interstitial pneumonitis has been associated with therapeutic as well as toxic levels of
sirolimus. Metastatic pulmonary calcification is frequently associated with renal failure and
renal transplants but it has been documented in patients with liver transplant without renal
insufficiency. Right-sided diaphragmatic dysfunction is a common complication of liver
transplantation as a result of phrenic nerve injury. After heart transplantation, there typically is
247
ASCCA Residents Guide to the ICU
a worsening of the impairment in diffusing capacity that persists for years after transplantation
(ref 14)
34. Neurologic Critical Care
34.1. [B] Symptomatic vasospasm is manifested by a gradual decrease in level of consciousness in
most cases, and in some cases is associated with hemiparesis, mutism, and even apraxia.
Some patients have fluctuation level of consciousness. Clinical signs usually appear between
4 and 14 days following the subarachnoid hemorrhage, and peak at day 7. Commonly
accepted treatment for symptomatic cerebral vasospasm is to increase the cerebral perfusion
pressure by increasing mean arterial pressure through volume expansion and judicious use of
vasoactive agents.
34.2. [D] These symptoms are consistent with bacterial meningitis, but may represent other
illnesses, which should be looked for if the CSF is normal. CSF leukocyte count >1000 is
commonly seen with bacterial meningitis, as is a reduction in CSF glucose and increase in
CSF protein. A traumatic puncture may result in bloody CSF, but the proportion of SBC to RBC
will still allow determination of increased CSF leukocyte count.
34.3. [E] GBS is an acute inflammatory polyneuropathy which usually develops following an acute
infection or mild respiratory syndrome. Its onset is gradual, over 3 - 21 days, with weakness
appearing symmetrically in the legs, and ascending to involve respiratory muscles in about
one-third of patients. Dysautonomia of varying degrees is common in these patients. In
patients where GBS causes a bed-bound state or severe ataxia, plasma exchange or IVIG are
preferred treatment options; steroids are not beneficial.
34.4. [D] Neuroleptic malignant syndrome is a curious and often unrecognized consequence of
administering neuroleptic drugs, as often occurs in the ICU. Like malignant hyperthermia,
NMS leads to sustained muscular contraction, and thermogenesis; rhabdomyolysis has been
reported. Sodium dantrolene has been used for treatment of NMS, but with inconsistent
results. Discontinuation of neuroleptic agents and administration of appropriate supportive
therapies are initial management strategies. Some advocate the administration of
bromocriptine, amantadine, and levodopa/carbidopa to modulate central dopaminergic tone
and alter thermoregulation.
34.5. [A] Waxing and waning, or a changing localized neurologic pattern is not characteristic of
peripheral neuropathy, cerebral ischemic infarct, or expanding mass lesions. Intracranial
hypertension that produces deterioration in neurologic status does not resolve until ICP
deceases. Ischemic episodes, as occur with transient ischemic attacks and with cerebral
vasospasm, are often associated with a waxing and waning neurologic state.
34.6. [E] All choices are obvious concerns for the comatose, head injured patient. In addition,
coagulopathy may be of concern.
34.7. [E] Hypoxia, hyperthermia. Anemia, hyperglycemia and diminished autoregulation may have
deleterious effects on neurologic outcome. Hypothermia may have a protective role.
34.8. [D] Acute ischemic stroke could be embolic or atherothrombotic. BP should not be aggressively
lowered. Echocardiography is required to identify a source of embolus. Thrombolysis within 3
hours of onset is associated with improved neurologic outcome. Antiplatelet therapy is also
recommended.
34.9. [D] ICP monitoring is indicated in patients with severe head injury (GCS < 8) and abnormal
head CT finding or patients with severe head injury (GCS < 8) and normal head CT if more
than 40 years old, posturing, or systolic BP < 90 mmHg.
34.10.[B] Edrophonium test is used to diagnose not treat myasthenia grravis. All others are used to
treat the disease.
35. Traumatic Brain Injury
35.1. [D] Cooper et al (Ref 18) compared patients resuscitated with Hypertonic saline versus
patients resuscitated with LR/NS and demonstrated identical neurological function at 6 months.
35.2. [D] The use of chronic and profound (25 mmHg) hyperventilation is discouraged. (Class I
evidence guideline). (Ref 2) Hyperventilation may decrease CBF and reduce brain
oxygenation.
35.3. [D] Many factors can make airway management in the TBI patient challenging. A cooperative
non intoxicated patient is not one of them
248
ASCCA Residents Guide to the ICU
35.4. [C] Initial surgical decompression is aimed to minimize the initial injury
35.5. [B] Hypocarbia and the use of steroids are listed in the guidelines as “not to do” interventions.
(Ref 2) Early feeding is recommended as head injury is a hypermetabolic state. Seizure
prophylaxis for seven days is recommended.
36. Management of Increased Intracranial Pressure
36.1. [C] All but prophylactic hyperventilation is recommended by the Guidelines for the Management
of Severe Head Injury. Historically patients in whom adequate hemodynamic resuscitation has
not been achieved have experienced worse neurologic outcomes. Muizelaar, et al. have
demonstrated deleterious consequences of prophylactic hyperventilation, presumably
secondary to further vasoconstriction and decreased blood flow to already ischemic brain.
36.2. [A] Head injury patients who are following commands (GCS 9-15) are at low risk for increased
ICP and may be followed with sequential neurologic exams. Exceptions include those patients
with traumatic mass lesion, in which ICP monitoring may be indicated. Certain patients with
severe head injuries but normal head CT scans are at risk for increased ICP. Those with 2 of 3
adverse features should be monitored for ICP: age >40, unilateral or bilateral motor posturing,
or systolic BP <90.
36.3. [D] Mannitol is a very useful diuretic agent in lowering ICP. However, by doing so the patient is
made hypovolemic. This results very commonly in a rise in serum sodium. In addition, renal
function should be monitored closely.
36.4. [A] A young patient who arrives to the emergency room with a suspected severe head injury
and hypertension should never have the blood pressure lowered artificially until the ICP is
known. By using the simple formula for CPP one can see that if the patients ICP is high, a high
MAP may be needed to maintain CPP ≥ 70.
36.5. [B] The Cerebral compliance by definition is the change in volume divided by the change in
ICP. As a mass occupying increases in size, initially the intracranial compartment will
accommodate. However, when a critical size is reached the intracranial components will reach
maximum compliance and the ICP will increase linearly with change in volume. Figure 1.
36.6. [D] Steroids have no role in the management of patients with head injury.
36.7. [B] In most clinical studies performed on hypothermia there are some benefits in the head
injured population. There is reproducible data to suggest it is effective at lowering ICP.
However, there is no clear evidence that it improves functional outcome.
36.8. [A] Autoregulation is defined as the ability of the brain to maintain adequate cerebral blood flow
over a wide range of MAP. Intrinsic to this phenomenon are various metabolites in the blood
that are vasoactive including CO2. In an injured brain autoregulation is ineffective.
36.9. [C] Mild hyperventilation is defined as an arterial CO2 of 30-35. Due to the fact that
hyperventilation causes rapid vasoconstriction of the cerebral blood vessels it may exacerbate
ischemia. Therefore, it should be used for short intervals at a time, particularly for intracranial
hypertension crisis.
37. Renal Protection
37.1. [C] A prerenal state implies oliguria due to hypovolemia (i.e., decreased renal perfusion), in
which damage to the renal tubules has not yet occurred. An appropriate tubular response is
sodium retention and urinary concentration, in an effort to restore the intravascular volume.
Urine sodium will be low (<20 mEq/L) and fractional excretion of sodium (FENa, the sodium
clearance expressed as a percentage of the creatinine clearance) less than 1%. Activation of
tubular concentrating ability results in a high urine specific gravity (>1.020), urine to plasma
osmolar ration (>1.5) and urine to plasma creatinine ration (>30). The converse occurs once
acute tubular necrosis (ATN) is established: the tubules are no longer able to conserve salt and
water, so that the urine has a high sodium concentration and is dilute, despite hypovolemia.
37.2. [E] The urine test for myoglobin is a qualitative, not quantitative. It may be intermittently
negative despite active rhabdomyolysis. Although urine myoglobin and myoglobin clearance
are the most accurate prognostic indicators of the risk of renal damage, they are not routinely
measured. The total CPK serves as a useful guide to the extent and severity of muscle
necrosis; once the CPK level exceeds 10,000 IU/L the risk of renal injury progressively
increases. Animal studies have indicated that acute renal failure is more likely with
myoglobinuria when urine pH is less than 6; in an acidic milieu myoglobin is converted to
249
ASCCA Residents Guide to the ICU
ferrihematin, which is toxic to the tubular cells. Addition of one ampule of sodium bicarbonate
to the maintenance IV is usually sufficient to alkalinize the urine to a pH >6. The most effective
means of renal protection in rhabdomyolysis is to maintain a high tubular and urine flow by
aggressive fluid administration.
37.3. [D] Normally, intraabdominal pressure is close to atomospheric pressure. When it is increased
>20 mmHg, e.g., by hemorrhage, a marked reduction in cardiac output and renal blood flow
(RBF) follows. Fluid administration can restore the cardiac output, but RBF remains low,
because of renal vein compression. High renal venous pressure impedes glomerular filtration.
Administration of furosemide, mannitol and/or low dose dopamine may transiently increase
urine flow rate, but they do not correct the fundamental problem. Indeed, the elevated
intraabdominal pressure is transmitted to the mediastinum to that PAP and PAOP may
overestimate the effective, or transmural, filling pressure of the left ventricle. Fluid challenge
could therefore be considered, but ultimately restoration of RBF and urine flow rate depends on
relief of the intraabdominal hypertension. Surgical exploration should be considered when
oliguria is associated with an intraabdominal pressure >20 mmHg.
37.4. [D] It is quite possible that dopamine administration may have an inotropic effect which
increases cardiac output, renal blood flow, and glomerular flow rate, thereby increasing urine
flow rate. However, in this situation, it is more likely that dopamine has had a direct effect on
the tubules. Studies on the use of dopamine compared with saline administration during infrarenal aortic cross-clamping reveal that it induces a significantly greater diuresis but no greater
protection against a decrease in glomerular flow rate than saline. It is now well known that
dopaminergic receptors exist in the renal tubule which, when stimulated by dopamine, inhibit
active sodium reabsorption and induce diuresis and salureses. Stimulation of presynaptic
dopaminergic (DA-2) receptors inhibits the release of norepinephrine and contributes to renal
vasodilation, but is likely to be less important than inhibition of sodium reabsorption in inducing
diuresis.
38. Renal Replacement Therapy
38.1. [C] Mortality in AKI is around 50% depending on the series. All other alternatives are wrong
38.2. [D] There is not evidence that demonstrate CRRT is better than IHD, despite its theoretical
benefits
38.3. [B] Peritoneal dialysis had a role in the ICU before the introduction of CRRT. It is not very
effective in solute removal but is cheap and does not require anticoagulation
38.4. [D] CVVHDF is probably the most common CCT used in the ICU as it offers good fluid control,
maintenance of hemodynamics and allows both diffusion and ultra filtration.
38.5. The cellulose membrane is thicker and activates the inflammatory response. It is therefore not
used in the critically ill. The synthetic membrane is thinner and is best for UF purposes.
39. Toxicology and Support of Patients with Drug Overdoses
39.1. [D] Cyanide, by blocking the cytochrome c in the electron transport chain, prevents the
utilization of oxygen and consequently the mixed venous saturation increases (Reference 1)
39.2. [C] See Reference 1
39.3. [B] Isopropyl is converted to ketones, not ketoacids, so although the patient’s breath has the
aroma of ketones, he does not exhibit a metabolic acidosis (Reference 7).
39.4. [E] See Reference 12.
39.5. [D] See Reference 6.
39.6. [B] See Reference 10.
39.7. [C] Reference 13
40. Solid Organ Transplantation
40.1. [C]. See References.1, 3.
40.2. [D]. See References 1, 3
40.3. [D]. Coronary artery vasculopathy is a long-term complication of heart transplant with
multifactorial etiology – in addition to the usual risk factors for CAD, complex immune
mechanisms play a role.
40.4. [B]. Bronchiolitis obliterans is a late complication and rarely occurs in the first three months
after lung transplant. See References. 7,8.
250
ASCCA Residents Guide to the ICU
40.5. [E]. Apsergillosis is unlikely to develop early in the course of immunosuppression.
40.6. [A]. Mycophenolate, unlike calcineurin inhibitors, has not been linked to significant kidney
dysfunction.
41. Organ Donation and Procurement in the ICU
41.1.
41.2.
41.3.
41.4.
[E]
[A]
[B]
[C]
42. Trauma Management
42.1. [B]
42.2. [E]
42.3. [C]
42.4. [True]
42.5. [C]
42.6. [True]
42.7. [B]
42.8. [True]
42.9. [D]
42.10.[True]
43. Burn Management
43.1. [D] the cleansing and irrigation of the wound gently debrides the necrotic tissue.
43.2. [C] 4 cc/kg/% burn surface per 24 hours of isotonic crystalloid. Half of the 24-hour calculated
volume is to be administered in the first 8 hours.
43.3. [B] Mafenide acetate causes a hyperchloremic acidosis.
43.4. [E] When applied liberally, silver nitrate can cause all the above side-effects.
43.5. [A] Often there are no physical signs or symptoms indicating eventual respiratory distress in
the first 24 hours following a burn. Given the possibility of a chemical pneumonitis and the
eventual fluid resuscitation required it would be prudent to protect the airway at this time under
controlled conditions.
43.6. [B] Feeding should be accomplished by the enteral route, as soon as possible. 25 kcal/kg
body weight + 40 kcal/% burn is recommended. Daily caloric requirements for adult patients
with burns greater than 20% of TBSA are estimated with nitrogen given in quantities that give
a caloric-to-nitrogen ratio of 150:1 and calories in the form of glucose up to a dose of 5mg/kg/
min, which is supplemented with a fat emulsion. Mineral and vitamin supplements are essential
43.7. [E] Torso escharotomy is the best intervention at this time considering his burns and the
swelling from fluid administration is affecting ventilation.
43.8. [C] urine alkalinization with bicarbonate may facilitate clearance of myoglobin by preventing its
entry into tubular cells. Renal function must be aggressively monitored to prevent acute tubular
necrosis.
44. Obstetric Critical Care
44.1. [A] Most (85%) cases of preeclampsia affect women in their first pregnancy, rather than
multips. Although young patients have a higher incidence of the disease, older parturients may
present with more severe disease. Preeclampsia accounts for 20% of maternal deaths in the
United States. The etiology remains unknown.
44.2. [D] A urine output of less than 400 mL in a 24-hour period is sufficient to diagnose severe
preeclampsia. The diagnosis of severe preeclampsia would be indicated if any ONE of the
following features were present: (1) systolic BP of at least 160 mmHg, or (2) diastolic BP of at
least 110 mmHg, or (3) greater than 6 gm proteinuria in a 24-hour urine collection.
44.3. [D] The colloid oncotic pressure (COP) is reduced in normal pregnancy and is often decreased
further in preeclamptic patients. The COP can be expected to fall slightly, not increase,
postpartum.
44.4. [E] Thrombocytopenia occurs in 15-30% of women with preeclampsia or eclampsia. The
classic manifestations of severe preeclampsia include several headache, visual disturbances,
251
ASCCA Residents Guide to the ICU
CNS hyperexcitability, and hyperreflexia; cerebral hemorrhage is the leading cause of death.
Angiotensinogen, angiotensin I, angiotensin II, and aldosterone levels decrease markedly in
preeclamptic women.
44.5. [A] Hydralazine is not a myocardial depressant; it has direct smooth muscle relaxing
properties. ACE inhibitors are contraindicated in the antepartum period because of potentially
lethal fetal and neonatal sequelae. Sodium nitroprusside indeed results in the release of nitric
oxide and is useful in the antepartum period, primarily for short-term use to control the pressor
response to tracheal intubation.
44.6. [D] Approximately 70% of seizures in preeclamptic patients precede delivery.
252